You are on page 1of 269

TOPIC 1: DIMENSIONS OF A PHYSICAL QUANTITY.

Definitions:
Qn1. Define the following terms

1. Dimension of a physical quantity


2. Fundamental quantity
3. Derived quantities
4. Fundamental units/ basic units
5. Derived units
6. Dimensional quantities
7. Non-dimensional quantities
8. A physical quantity
Factors, Merits, Demerits, Features and Examples
1. Give any two uses of dimensional analysis
2. Mention three dimensionless physical quantities
3. State any two examples of fundamental and derived physical quantities
4. State one disadvantage of dimensional analysis
Derivations and Relations

1.Determinethedimensions of the following;Momentum,Pressure,Weight,Kinetic energy, Work


done,Stress,Impulse,Power,Surface tension, Potential energy.

2. Use dimensional analysis to show that the equations of motion are dimensionally consistent.

3. The velocity, V of a wave along a plucked string is given by v=


√ TL where T- is the tension of the
m
string, m- mass and L- length of the string. Show that this expression is dimensionally consistent.

4. The equation for the volume, V, of a liquid flowing through a pipe in time, t, under steady
4
v πr p
flow is given by = where r- radius of the pipe, P is the pressure difference between the
t 8 ɳL
ends of the pipe; L is the length of the pipe and ɳ -coefficient of viscosity of liquid whose
dimensions are ML-1T-1.

5.The centripetal force required to keep a body of mass, m, moving in a circular path of radius,
m v2
r, is given by F= where V-speed. Show that the formula is dimensionally consistent.
r

1|Page
6. The distance vertically a projectile fired initially at a velocity of ureaches in time, t, is given by
1 2
s=ut sin θ+ g t , where θ -is the angle of projection and g- is acceleration due to gravity. Show that
2
the expression is dimensionally consistent.

7. The period of a conical pendulum whirled in a horizontal circle and of an inextensible string

of length, L and θ is theangle of inclination to the vertical is given byT =2 π

acceleration due to gravity. Show that the formula is dimensionally consistent.


L cos θ
g √
, where g'

8. The period, T of a satellite in a circular orbit of radius, R, about a planet of mass M, is given by

( )
3 1
R 2
T =2 π , where G is universal gravitational constant. If the [G] =M -1L3T-2. Show that this
GM
equation is dimensionally consistent.

9. The period of a body performing S.H.M is given by T =2 π


L
g √
, L, length and g, is acceleration

due to gravity. Show that this formula is dimensionally consistent.

10. The speed of sound in a gas, is given by V =


ρ √
yP , V-speed, y -the ratio of specific heat of the gas,

P-pressure and ρ – density of the gas. Show that this expression is dimensionally consistent.

11. The speed, C, of a transverse progressive wave travelling along a stretched string of mass per unit

length, µ is given by ¿

consistent.
√ T , where T- tension in the string. Show that this expression is dimensionally
μ

1 2
12. The total pressure P= ρ c , where ρ-density of the gas, c 2is the mean square speed of the
3
molecules of the gas. Show that this equation is dimensionally consistent.

13. When a sphere of radius, a, moves slowly with a speed, V, through a fluid of viscosity, ɳ,
the force, F, on the sphere due to viscous drag is given by ⃗
F =6 π aɳV , Show that this equation
is dimensionally consistent.

14. The pressure difference, P, across a spherical surface of radius r, between air and a liquid
2y
when y is surface tension of the liquid is given by P= .Show that this equation is
r
dimensionally consistent.

15. Write down the expression for the pressure at a point depth, h, from the surface of a liquid
of density, ρ and Show that the equation is dimensionally consistent.

2|Page
16. A spring of force constant K, undergoes an elastic change resulting into an extension, x. the work
1 2
done W is given byW = k x .Show that the equation is dimensionally consistent.
2

17. The velocity of a particle whirled in a horizontal circle is given by V = √ rgtanθ , where r, radius, g–
acceleration due to gravity and θ -angle of inclination to the vertical. Show that the equation is
dimensionally consistent.

18. The velocity of sound waves through any material depends on its density, its modulus of elasticity, E,

Use the knowledge of dimension analysis to show that V =


√ E.
ρ

19. The frequency of vibrations of the drop of a liquid depends on surface tension ɤ , of the drop, its

density ρ and radius r of the drop. Show that f =k

dimension of surface tension are MT-2.


√ ɤ
ρr
3
where k -is a non-dimensional coefficient, if

20. A sphere of radius, a, moving through a liquid of density, ρ, with high velocity v experiences a
retarding force F given by F=K a x ρ y v z .Where K-non dimensional coefficient. Use the method of
dimensional analysis to find the value of x, y and z.

21. The formula T =K E x P y ρz Where E is the energy, T is the period, and P-pressure, ρ-density and K is
a non-dimensional constant. Determine the relationship between T, E, P and ρ.

22. The frequency, f of tuning fork depends on the length, L, of the prong, its density, ρ and young’s

()
1
K y 2
modulus y . Use the dimensional theorem to show that f = .where K, is a dimensional constant.
L ρ

23. The pressure at a point in a liquid of density ρ, is given by the expression P=K g x h y ρ z , where h-
depth, gis acceleration due to gravity. Find the expression of pressure at that point.

24. The speed, v, of propagation of a transverse wave along a stretched string is given by … where K is
dimensional constant, T is tension, and µ is mass per unit length of the string. Use dimensions to find x
and y.

25. The period of a simple pendulum, T depends on mass, m, length L and acceleration due to gravity g.

Show that the expression T =K


√ L holds for for dimensional analysis where K=2 π .
g

ɋV
26. The stress, σ between two planes of molecules in a moving liquid is given by σ = where V-
X
velocity difference of the planes, x their distance apart, and ɋ is a constant of liquid. Obtain the
dimensions of ɋ.

3|Page
27. A body moving through air at a high speed, V, experiences a retarding force ⃗
F is given by
⃗ x
F=KAρ V where A- surface area of the body, ρ density of the air and K is a numerical constant.
Deduce the value of x.

28. For one mole of a real gas, the equation of state is given by the expression

( P+ va ) ( v −b)=RT
2 , where P-pressure, T is absolute temperature, V-volume of the gas, a

and b are constants. Find the dimensions of a and b and hence the units ofa and b.

29. For a streamline flow of non-viscous incompressible fluid, the pressure P at any point is related to
1
the height h, and the velocity V, by ( P−a )=ρg ( h−b ) + ρ ( V −d ) where a, b and d are constants. ρ-is
2
2
density and g acceleration due to gravity.

30. The two alternative units for gravitational field strength Nk g−1 are andm s−2 . Use the method of
dimensions to show that the two units are dimensionally equivalent.

31. The two alternative units for force exerted by a body standing on the table are N andkgm s−2. Use
dimensional analysis to show that the two units are dimensionally equivalent.

32. The two units of power developed by a young boy running a wheelbarrow uphill are J s−1and Nm s−2
. Show that the two units are equivalent.

33. The alternative units of linear momentum of a vehicle in a certain direction are and. Use dimensional
analysis to show that the two units are dimensionally equivalent.

34. The two alternative units of potential energy are J andkg m2 s−2.use the theory of dimensions to
show that the two units are equivalent.

35. The two alternative units of universal gravitational constant G , are N m2 k g−2 andm 3 kg−1 s−2. Show
that the two units are dimensionally equivalent.

36. Given that the velocity V of a wave on a stretched string depends on mass per unit length µ of the
string and tension T in the string, derive the relationship between V, µ and T using dimensions.


2
37. Show that the equationV = Ke is dimensionally consistent where e-extension, V-velocity, K-force
m
constant and m-mass.

4|Page
TOPIC 2. MOTION IN A STRAIGHT LINE

DEFINITIONS
1. Define the following terms; displacement, distance, speed, velocity, uniform velocity, non-uniform
velocity, acceleration, uniform acceleration, non-uniform acceleration, uniformly accelerated motion,
retardation, uniform motion

SKETCHES OF THE GRAPHS

1. Sketch the following graphs;

1. Displacement-time graph for uniform velocity.


2. Displacement-time graph for non-uniform velocity.
3. Displacement-time graph for a body at rest.
4. Displacement-time graph for uniform acceleration (increasing velocity).
5. Displacement-time graph for uniform deceleration (decreasing velocity).
6. Velocity-time graph for uniform velocity (constant velocity i.e. zero acceleration)
7. Velocity-time graph for uniform acceleration (increasing velocity).
8. Velocity-time graph for uniform deceleration (decreasing velocity).
9. Velocity-time graph for a body thrown vertically upwards
10. Velocity-time graph for non-uniform acceleration.
11. Acceleration-time graph for a body thrown vertically upwards.
12. Acceleration-time graph for a body falling freely under gravity.
13. Sketch a graph of velocity-time graph for an object thrown vertically upwards.
14. Sketch the speed-time and distance-time graph for a body moving with uniform
acceleration.
15. Draw and explain displacement, speed and velocity-time graph for a body thrown
vertically upwards.
16. Sketch the speed-time graph for a uniformly accelerated body.
17. For a uniformly decelerating motion, sketch;
18. A displacement-time graph.
19. A velocity-time graph.
20. Sketch the displacement-time graph for a body moving with a uniform motion.

5|Page
DERIVATION AND RELATIONS

1. Write down the equations of uniformly accelerated motion.


1. Derive an expression relating the distance, s, initial velocity, u, the time t, and the
acceleration, a, for a particle moving with uniform acceleration.
2. Show that the area under a velocity time graph for uniformly accelerated motion
presents the distance covered during the time of motion.
1 2
3. Derive the expression, s=ut+ a t for the distance, s, moved by a body which is
2
initially travelling with speed, u, and is uniformly accelerated for time, t.
4.

i. A body initially at rest is accelerated uniformly until it attains a velocity V. if


the distance covered during its motion is S, derive an expression for V in
terms of S and acceleration a.

ii. Show that the expression derived in (I) above is dimensionally consistent.

5. Show that the area under a velocity-time graph for a uniformly accelerated motion
presents the distance covered during the time of motion.

6. A particle travelling in a straight line with a constant acceleration covers a distance s 1


and s2 in the third and fourth seconds of its motion respectively. Show that its initial
1
speed, u, is given byu= ( 7 s 1−5 s 2 ).
2
CALCULATIONS

1. A body travelling in a straight line with an acceleration of ams -2 passes a point O with a
velocity of U ms-1. During the first 5s after passing O it travels 45m and during the next one
second, it travels a further distance of 15m.

i. Calculate a and u.
ii. Calculate the velocity of body when its 140m from O and the time to reach this point.

2. A train travels along a straight piece of track between two stations. The train starts from rest at
A and accelerates at 1.25ms-2 until it reaches a speed of 20ms -1. It then travels at this steady
speed for 1.56km and then decelerates at 2ms-2 to come to rest at B. find

a) The total time to move from A to B.


b) The distance from A to B.
c) The average speed for the journey.

6|Page
1 −2
3. A cyclist travelling downhill accelerates uniformly at1 ms . If initial the initial velocity at
2
the top of the hill is 3ms . Find;
-1

(i)How far he travels in 8 seconds.

(ii)How far he travels before reaching a velocity of 7ms-1.

4. A stone slides in a straight line across a horizontal sheet of ice. It passes a point A with
1
velocity 14ms-1 and the point B 2 seconds later. Assuming the retardation is uniform and that
2
AB=30m. Find;
i. The retardation.
ii. Velocity at B.
iii. How long after passing A stone come to rest.

5. A train starts from rest and accelerates uniformly at 1.5ms -2 until it attains a speed of 3ms-1.
Find the distance travelled during this motion and the time taken.

6. A car travelling with a velocity of 10ms -1 accelerates uniformly at 1.0ms-2 until it reaches a
velocity of 15ms-1. Calculate;

i. The time taken


ii. The distance travelled during the acceleration.
iii. The velocity reached 100mfrom the place where the acceleration began.

7. A motorist travelling at 60kmhr-1 brakes steadily to a halt in 100m. Find;

i. Acceleration.
ii. Stopping time.

8. A car at rest is uniformly accelerated for 5s at which time its speed is 10ms -1. The car
maintains this speed for 10s and is then brought to rest in 10s by a steady application of brakes.
Find;

i. The acceleration during the first 5 seconds


ii. The total distance travelled by the car in 25 seconds.

7|Page
9. A car starting from rest with uniform acceleration acquires a speed of 20ms -1 in 20s, it travels
at this speed for 30s. Is then brought by being uniformly retarded, 150s after starting.
i. Sketch the speed-time graph and deduce the acceleration and retardation.
ii. Find the total distance travelled.

9. A motorist travelling at a constant speed of 50kmhr -1 passes a motor cyclist just starting off in
the same direction, if the motor cyclist maintains a constant acceleration of 2.8ms-1, calculate;

i. The time taken by the motor cyclist to catch up with the motorist
ii. The speed with which the motor cyclist overtakes the motorist
iii. The distance travelled by the motor cyclist before overtaking.

10. A car moving with a velocity of 10ms -1 accelerates uniformly at 1.0ms-2 until it reaches a
velocity of 15ms-1. Calculate;

i. The time taken


ii. Distance travelled during acceleration.
iii. Velocity reached 100m from the origin.

11. A train travelling at 72kmhr-1 undergoes uniform deceleration of 2.0ms-1 when brakes are
applied. Find the time taken to come to rest and the distance travelled from the place where
brakes are applied.
12. Two particles are travelling along a straight line AB of length 20m. At the instant when one
particle starts from rest at A and travels towards B with a constant acceleration of 2ms -2, the
other starts from rest from B and travels towards A with a constant acceleration of 5ms -2. Find
the time after which the two bodies collide and how far from A they collide.

13. A motorcar moving with a uniform acceleration covers 5.5m in its 4 th second and 9.5m in its
8th second of its motion. Find its acceleration and its initial velocity.

14. A train stops at two points A and B which are 2km apart. It accelerates uniformly from A at
1.0ms-2 for 15s, and maintains a constant speed for a time before decelerating uniformly to rest at
B. if the deceleration is 0.5ms-2. Find the time for which the train is travelling at a constant speed.

15. A car A travelling at a constant velocity of 25ms-1 overtakes a stationary car B. two seconds
later B sets off in pursuit accelerating at a uniform acceleration 6ms -2. How far does B travel
before catching up with A.

8|Page
16. A car is being driven along a road at a steady speed of 25ms -1 when the driver suddenly
notices that there is a fallen tree blocking the road 65m a head. The driver immediately applies
the brakes giving the car a constant retardation of 5.0ms-2. How far in front of the tree does the
car come to rest. If the driver had not reached immediately and the brakes were applied one
second later, with what speed would the car have hit the tree.

17. A, B and C are three points which lie in that order on a straight road with AB=95m and BC
=80m. A car is travelling along the road in the direction ABC with a constant acceleration ams-2.
The car passes through A with a speed Ums -1 reaches B five seconds later, and C two seconds
later after that. Calculate the value of U and a.

18. A train of mass 100,000kg starts from rest at station P and accelerates uniformly at 1ms -1. It
maintains this speed for further 90s and then brakes are applied, producing a resultant breaking
force of 50kN. If the train comes to rest at Q. find the distance between the two stations.

19. A ship of mass 107kg is travelling at 2ms-1 when its engine is switched off. As a consequence,
the ship’s speed is reduced to 1.5ms-1 in a distance of 100m. Assuming the resistance to the
ship’s motion is uniform; calculate the magnitude of the resistance.

20. A particle is moving in a straight line with a constant acceleration of 6.0ms -2. As it passes a
point A. its speed is 20ms-1, what is its speed 10s after passing point A.

21. A particle is moving in a straight line with a velocity of 15ms -1 accelerates uniformly for 3.0s
increasing its velocity to 45ms-1 what distance does it travel whilst accelerating.

22. A car starts to accelerate at a constant rate of 0.8ms -2 it covers 400m whilst accelerating in
the next 20s. What was the speed of the car when it started to accelerate.

23. A body of mass 3.0kg, initially at rest moves along a smooth horizontal surface under the
effect of a horizontal force of 12N.

i. Find the acceleration of the body


ii. Find the speed of the body after 5.0s

24. A car moving at 30ms-1 is brought to rest with a constant retardation of 3.6ms-2 of 3.6ms-2.
How far does it travel whilst coming to rest.

24. A bus travelling steadily at 30ms-1 along a straight road passes a stationary car w. 5s, later
begins to move with a uniform acceleration of 2ms-2 in the same direction as the bus.

i. How long does it take the car to acquire the same speed as the bus.
ii. How far has the car travelled when it is at the same level with the bus.

25. A particle moving in a straight line with a constant acceleration travels 10m in 2 seconds then
a further 30m in the following 2s find the acceleration of the particle and its velocity at 1000m.

9|Page
26. A body starts from rest, travels for 6.0s with a uniform acceleration of 2ms -2. It then
maintains this speed until it’s brought to rest with a uniform retardation of 3ms-2. If the total
distance travelled is 180m.

i. Calculate the time taken for the journey.


ii. Sketch a velocity-time graph for the motion of the body.

27. A sports car travelling on a straight road has an initial speed of 36kmhr -1. It gains speed steadily and
5s later, the speed is 90kmhr-1. Find;

i. The acceleration
ii. The distance travelled

TOPIC 3: MOTION UNDER GRAVITY

DEFINITIONS.

1. Define the following terms; Free fall, Acceleration due to gravity

DERIVATIONS AND RELATIONS


1. With the aid of a velocity-time graph describe the motion of a body projected vertically
upwards.
2. A ball is thrown vertically upwards with a velocity of 10ms -1 from a point 5.0m above the ground.
Describe with the aid of a velocity-time sketch graph, the subsequent motion of the ball.
3. A ball is thrown straight upwards with a speed of Ums -1 from a point h meters above the ground.

[ ( )]
1
u 2 gh
Show that the time taken to strike the ground is t= 1+ 1+ 2 2
g u
4. Sketch a graph of velocity against time for an object thrown vertically upwards
5. Sketch a graph of
a) Speed
b) Distance fallen as a function of time for a body falling under the influence of gravity.
6. A cricketer throws a ball of mass 0.2kg directly upwards with a velocity of 20ms -1 and catches it
again 4.0s later. Draw labeled sketch graphs to show;
a) The velocity
b) The kinetic energy
c) The height of the ball against time over the stated time 4.0s period. Showing numerical values of
the given quantities.
7. A stone is projected upwards and eventually returns to the point of projection. Ignoring any
effect due to air resistance, sketch graphs to show the variation with time of the following.

a) Velocity d) Momentum f) Speed


b) Kinetic energy e) Distance of
c) Potential energy projection

10 | P a g e
8. An object is projected in air and falls to the ground, after a certain time t, sketch;
a) Distance-time graph for the motion
b) Velocity-time graph for the motion

C) Explain the shape of each graph.

9. A body is projected vertically upwards. Sketch the following graphs to represent the motion;
a) Displacement-time graph
b) Distance-time graph
EXPERIMENTS
1. Describe an experiment to measure the acceleration due to gravity by free fall method
2. Describe an experiment to determine the acceleration due to gravity|
CALCULATIONS ON VERTICAL MOTION
1. A stone is thrown vertically upwards from the top of a building with an initial velocity of10
ms-1 .the stone takes 2.5s to land on the ground
i. Sketch the velocity time graph for the motion of the stone.
ii. Calculate the height of the building.
iii. State the energy changes that occurred during the motion of the stone
2. A brick is thrown vertically downwards from the top of a building and has an initial velocity of
2
1.5ms-1. If the height of the building is 19 m. Find
7
i. The velocity with which the brick hit the ground
ii. The time taken for the brick to fall
3. A stone is dropped from a piston which is 40m above the ground; find the time taken for the
stone to reach the ground.
4. A stone is dropped from the top 0f tower and falls to the ground with a speed of 14ms -1,
find the of the tower.
5. A ball is thrown vertically upwards with the speed 28ms -1 from a point which is 1m above the
ground level .find ;
i. Speed the ball will have when it returns to the level from which it was projected.
ii. Height above the ground level of the highest point reached.
6. A ball is thrown vertically downloads from the top of a tower and has an initial speed of 4ms -1. If
the ball hits the ground 2 seconds later.
i. The height of the tower
ii. The speed with which the ball strikes the ground
7. A ball is thrown vertically upwards from a point A, with an initial speed of 21ms -1 and is later
caught again at A. Find the length of time for which the ball is in air.
8. A ball is kicked vertically upwards from ground level within an initial speed of 14ms -1.find the
height above the ground level of the highest point reached and total time for which the ball is in
air.
9. A stone is projected vertically upwards from ground level at a speed of 24.5ms -1. Find the height
above ground level of the highest point reached and the total time for which the ball is in air.

11 | P a g e
10. A stone is projected vertically upwards from ground level at a speed of 24.5ms -1. Find how long
after projection the stone is at a height of 19.6m above the ground
i. For the 1st time
ii. For the 2nd time
iii. For how long is the stone at least 19.6m above the ground.
11. A stone is thrown vertically upwards with a speed of 20ms -1 from a point of projection at a
height h m above the ground. If the stone hits the ground 5s later find h.
12. A body is thrown upwards with an initial velocity of 20ms -1, calculate;
i. The time taken to return to the tower
ii. The maximum height reached.
13. A stone is thrown vertically upwards with a speed of 14ms -1. 2 seconds later a second ball is
dropped at the same point. Find where the balls meet.
14. A stone is projected vertically upwards with a speed of 50ms -1. On return it passes the point of
projection and falls 78m below.
i. Calculate the total time taken
ii. State the energy changes that occurred during the motion of the ball.
15. A ball is thrown vertically upwards with an initial speed of 20ms -1. After reaching a maximum
height and on the way down, it strikes a bird 10m above the ground;
i. How high does the ball rise
ii. How fast is the ball moving when it strikes the bird.
16. A particle is projected vertically upwards with a speed of 16ms -1. One second later another
particle is fired vertically upwards from the same point. Find the initial velocity of the second
particle in order that the two particles will collide at the greatest height of the first particle.
17. A ball is dropped from a tall building. One second later another ball is thrown with a speed of
30ms-1 vertically downwards. Determine when and where the two balls meet.
18. A man stands on a cliff 10m above the ground and throws an object vertically upwards with an
initial velocity of 10ms-1;
i. Draw a velocity-time graph for the motion
ii. Sketch a distance-time graph for the same motion.
19. A stone is thrown vertically upwards with a speed of 20ms -1 from a point at a height h above the
ground level. If the stone hits the ground 5.0s later find;
i. Velocity with which it hits the ground
ii. The value of h.
20. A stone is thrown vertically upwards from the top of a tower and hits the ground 10s later with a
speed of 51ms-1. Find the initial velocity and height of the tower.
21. A ball is thrown vertically upwards with a speed of 15ms -1 from a point which is 0.7m above the
ground. Find the speed with which the ball hit the ground and the time taken.
22. A stone is dropped vertically from the top of an overlapping cliff and it hits the sea 3s later. The
acceleration due to gravity at this location is 10ms -2.find the speed of the stone as it hits the sea
and the height of the cliff.
23. A ball is projected from a point 2m above the ground with an upward speed of 3ms -1. Assuming
that the acceleration due to gravity is 10ms -2, find;

12 | P a g e
i. Time taken for the ball to reach its greatest height
ii. Maximum height attained
iii. Speed of the ball when it first strikes the ground.
24. Two stones are thrown from the same point at the same time. One vertically upwards with a
speed of 40ms-1and the other vertically downwards at 40ms -1. Find how far the stones are after
2s.
25. A stone dropped from a roof of a high building. A second stone is dropped 1.0s later how far
apart the stones are when the second one has reached the speed of 23ms -1.
26. A ball is thrown vertically upwards with a velocity of 20ms -1, calculate;
i. The maximum height reached
ii. The total time for which the ball is in air (g=10ms -2)
27. A stone is thrown from the top of a cliff which is 80m high. How long does it take to reach the
bottom of the cliff.
28. A particle is projected vertically upwards with a velocity of 30ms -1. Calculate;
i. How long it takes to reach its maximum height
ii. The two times at which it is 40m above the point of projection
iii. The two times at which it is moving with 15ms -1.
29. A hot air balloon is 21m above the ground and is rising at 8.0ms -1 when a sand bag is dropped
from it. How long does it take the sand bag to reach the ground.
30. A stone of mass 8g is released at the top of a vertical cliff. After falling for 3.0s it reaches the foot
of the cliff and penetrates 9cm into the ground, find;
i. The height of the cliff
ii. The average force resisting penetration of the ground by the stone.
31. A stone is thrown vertically upwards from the top of the tower with a velocity of 20ms -1, the
stone takes 5.0s to hit the ground.
i. Sketch the velocity-time graph for the motion
ii. Calculate the velocity of the stone just before it hits the ground
32. A ball is thrown vertically upwards from the top of a building 10.5m tall. The ball takes 9.0s to
reach the maximum height;
i. Sketch a displacement-time graph for the motion of the ball
ii. State the energy changes that take place as the ball moves upwards.
iii. Find the time taken by the ball to hit the ground below from the time it is thrown.
33. An object is dropped from a height of 45m above the ground. Calculate;
i. Time taken to reach the ground
ii. The velocity when it reaches the ground

TOPIC 4: VECTORS AND SCALARS

DEFINITIONS
1. Define the following; vectors, scalars, resultant force, relative velocity, relative
motion, coplanar forces, concurrent forces, resolution of forces
13 | P a g e
FACTORS, EXAMPLES, MERITS, DEMERITS AND FEATURES
1. Identify scalar and vector quantities from the following; density, momentum,
acceleration, impulse and temperature
2. Identify scalar and vector quantities from the following; Momentum, density,
acceleration, impulse, pressure and temperature
3. What is meant by saying that a body is moving with a velocity V- relative to another
4. What is meant by the relative velocity of a body A with respect to body B
5. Explain the meaning of relative velocity of two bodies.

CALCULATIONS

1. Given that there are 5 forces acting on the body.


6N

6N
5N
5N

10N
Find (i) the resultant force
(ii) The mass of the body if it accelerates at a rate of 6.25ms-2.
2. The diagram below shows the three forces acting on body .

6N

7N 300 13N

Find the resultant force in N.


3. A force particle of mass 1.0kg initially at M acted on by forces as shown below;
7N

500
3N

14 | P a g e
4N
(i). Find the resultant force on the particle
(ii).Fine the velocity of the particle after travelling a distance of 4.0m.
4. Three forces of 3N, 4N, and 10N act on a point A as shown below.

4.0N

10N
3N
600
A

(i). Find the resultant force at A


(ii).What will be the velocity at 6m from A if the body is of mass 3.0kg.
5. Find the resultant force, F, of the forces shown below.

10N

45 0
450 B 3N

12N

6N

6. The figure below shows three forces acting on a particular P of mass 5.0kg initially at
rest.

6N 96.870 7N

300 P

3N

4N

15 | P a g e
Determine the magnitude and direction of the resultant force on the particle and its
kinetic energy after moving 10m.
7. The figure below shows three forces acting on a particle P. of mass 3kg initially at rest.

10N

360, 521 P 6N

8N
(i). Determine the magnitude and direction of the resultant force on the particle
(ii).The Kinetic energy of the particle after moving 20m.
8. A free particle of mass 10kg initially at M is acted on by forces as shown below.
7N

M 530
3N

4N

(i). Find the resultant force on the particle


(ii).The velocity of the particle at a point 4m from M.
9.
2.0N 3.5N

0
300
600

4.0N

16 | P a g e
Three forces of 3.5N, 4.0N, and 2.0N act at θ as shown.
Find the resultant force.

10.

2.83N

450

600 300 Forces of 2.83N, 4.00N and 6.00N act on a

Particular O as shown.

4N 6N Find the resultant force on the particle.

1.7N

11. 2.5N

3.5N

310 250 Find the resultant force

12. Fine the resultant of the system of forces shown


20N
Find the magnitude and direction of the resultant of a system of forces
shown.

60N

17 | P a g e
600

30N

13. Find the magnitude and direction of the resultant of a system of forces shown.
5N

10N

8N 300

14. Forces of 3N, 4N and 7N act on a particle of mass 1.0kg initially at rest at a point O as
shown.
7N

530 61
3N
(i) Find the position of the particle after 2.0s

4N

15. Find the resultant of the forces in the figure below;

2N 6N

18 | P a g e
600 600 4N

33N

16.

7N

3N O 530

4N

(i) Find the resultant force on the particles


(ii) Find the velocity of the particle at a point 4m from O.
17. Three forces area applied to the point as shown;

6N Y

450 X

450 O 14N

8N Calculate;

(i) The component in the directions OX and OY respectively

(ii) The resultant force acting on O.

18. The particle is acted upon by three forces value are coplanar
6N Y

Calculate θ to the nearest degree if the resultant is in the west


500
Direction.
19 | P a g e
X
θ

5N 20N

19. Find the resultant of the system of forces shown below

60.0N

600

30.0N

20. N

20. Calculate the magnitude and direction of the resultant of the forces shown in the figure.

600

50.0N 450

3N
100.0N
21. 1N 7N

30 2N

Find the resultant force acting on a body y as shown.

3N

20 | P a g e
22. Four forces of 1.7N, 3.5N, 4.5N, and 5.0N act on the particle P as shown below.
17N
3.5N

4.5N

200

5.0N
Find the resultant force acting in P.
23. Find the resultant of forces 5N, 7N, 6N and 4N acting at a point in directions 0500, 1000,
2000, and 3100 respectively
24. Five forces act on the mass of 6kg as shown below.
15N 8N

30 0

450

600 10N

Find the resultant acceleration of the body

300

20N

25. A free particle of mass 2kg initially at a point A is acted upon by forces shown.

6N 7N

46.90

21 | P a g e
10N 1430

Find the;

(i) Resultant force on the particle

4N (ii) Time taken for the particle to cover a distance of 4.0m

26. A body M of mass 6kg is acted on by forces of 5N, 20N, 25N and 30N as shown.
30N
5N
40 20
0 0

600 25N

Find the acceleration of the mass


20N
27. Three forces of 4.0N, 7.0N and 8.0N act at a point O as shown.
4.0N
7.0N

300

600

8.0N
Find the;
(i). Resultant force
(ii).The acceleration.

CALCULATIONS ON RELATIVE VELOCITY

1. A boy runs at 10ms-1 due to East and at the same time the girl runs at 5ms-1 due to East.
Find (i). Velocity of the boy relative to the girl
(ii) Velocity of the girl relative to the boy.
2. A man walks at 5kmhr-1 due to west and a boy runs at 12kmhr -1 towards the South East.
Find the velocity of the boy relative to the man.
3. A particle A is moving due to North at 30ms-1 and particle B is moving due to south at
10ms-1. Find the relative velocity of A to B.

22 | P a g e
4. A bus travels at 5kmhr-1 due west and A taxi travels at 12kmhr-1 on a bearing of 1500.
Find the velocity of the bus relatives to the taxi
5. A cruiser is moving at 30kmhr-1 due to North and a battleship is moving at 20kmhr-1 due
to North. Find;
(i). The velocity of the cruiser relative to the battleship
(ii).The relative velocity of battleship to the cruiser.
6. (a) A ship A is travelling due to North at 20kmhr-1 and ship B is travelling due East at
15kmhr-1. Find the velocity of A relative to B.
(b) If the ship B in (a) is 10Km due west of A at noon. Find the shortest distance a part
and when it occurs.
7. The resultant of two velocities is a velocity of 10kmhr-1 N 300W. If one of the velocities
is 10kmhr-1 due west. Find the magnitude and direction of the other velocity.
8. The resultant of two velocities is a velocity of 6ms-1 due east if one of the velocities is
5ms-1 N30W. find the magnitude and direction of the other velocity
9. The resultant of two velocities is a velocity of 19ms-1 S600E. If one of the velocities is
10ms-1 due East, find the magnitude and direction of the other velocity.
10. A girl is running at 20ms-1 due east and at the same time boy is running in the opposite
direction at 15ms-1. Find the velocity of a girl relative to the boy.
11. Trains X and Y are approaching 600 rail junctions with velocities of 20ms-1 respectively.
Both trains are 300m from the junction at the instant in question.
(i) Find the velocity of Y relative to X.
(ii) Find the shortest distance apart.
12. A bomber M is flying horizontally at a speed of 250ms-1 due east. A second bomber, N is
flying horizontally at a speed 160ms-1 due N 400E. Determine the position of bomber N.
relative to M after 30minutes. Assume that the two bombers take off from same place at
the same time.
13. A thief vehicle is travelling at 17kmhr-1 due north and a battle truck is travelling at
N260W at 15kmhr-1. Find the magnitude and direction of the resultant velocity.
14. A truck A is travelling at 10kmhr-1 N350W and track B is moving at 15kmhr-1 S400W.
Find the magnitude and direction of the resultant velocity.
15. Henry rides his horse with a velocity 24kmhr-1 while job is riding his horse with a
velocity of 13kmhr-1. Find the velocity of Henry relative to Job.
16. A motorist is running at 8kmhr-1 due south and a cyclist is travelling at 15kmhr-1 due
west. Find the velocity of the motorist relative to the cyclist.
17. Tom walks at 4kmhr-1 due to North and Jane walks at 3Kmhr-1 due East. Find their
relative displacement after 17minutes.
18. What is the velocity of a cruiser moving at 20kmhs-1 due North as seen by an observer on
a line moving at 15kmhr-1 in a direction N300W.
19. A car is being driven at 20ms-1 on a bearing 0400. The wind is blowing from 3300 with a
speed of 10ms-1. Find the velocity of the wind as experienced by the driver of the car.

23 | P a g e
20. An air craft is moving at 250kmhr-1 in a direction N600E. A second air craft is moving at
200kmhr-1 in a direction. N20W. Find the velocity of the first air craft as seen by the pilot
of the second aircraft.
21. Find the velocity of a craw flying at 16ms-1 due North as seen by a black bird flying at
12ms-1 due East. What is the velocity of the black bird as seen by the craw?
22. To the Pilot of air craft A, travelling at 300kmhr-1 due South, it appears that aircraft B is
travelling at 600kmhr-1 in a direction N600W. Find the true speed and direction of air craft
B.
23. David is riding her horse at 5kmhr-1 due North and sees Sarah Riding her horse
apparently with velocity 4kmhr-1 N600E. Find Sarah’s true velocity.
24. To a person walking due to 3kmhr-1, the wind appears to come from the North East at
7kmhr-1. Find the time velocity of the wind.
25. A train is travelling at 80kmhr-1 in a direction N 150E. A passenger on the train observes a
plane apparently moving at 125kmhr-1 in a direction of N500E. Find the true velocity of
the plane.
26. To a driver of a motor boat moving at 6kmhr-1 on a bearing 3450, a yacht appears to be
moving at 18kmhr-1 on a bearing 2200, find the true velocity of the yacht.
27. Rain is falling vertically at 8.0ms-1 relative to the ground. The rain drops make tracks on
the side window of a car at angle of 300 below the horizontal. Calculate the speed of the
car
28. A helicopter heads NE and has an air speed of 120kmhr-1. Simultaneously, a wind of
50kmhr-1 from the North. What is the position of the helicopter 2h after leaving the
aerodrome.
29. A ship A moves due North at 30kmhr-1, a ship B moves N600W at 20kmhr-1.
(i). Find the velocity of B relatives to A
(ii).If ship B is 10km due East of ship A at this instant, find the closest distance of
approach of the two ships.
30. A police car moving at 100kmhr-1 chases a thug is car moving at 120kmhr-1. A police man
inside the car shoots at the thug is car. If the bullet leaves the gun at a speed of 720kmhr-1.
Find the velocity with which the bullet hits the thug’s car.

CALCULATIONS ON CROSSING A RIVER USING A BOAT

1. A Boat crosses a river 3km wide flowing at 4ms-1 to reach a point on the opposite bank
5km upstream. The boat is speed in still water is 12ms-1. Find the direction in which the
boat must be headed.
2. A man can swim at 6kmhr-1 in still water, would like the swim between two directly
opposite points on the bank of the river 300m wide flowing at 3kmhr-1. Find the time he
takes to cross the river and direction.
3. A boats man crosses a river flowing at 5ms-1 with a velocity of 10ms-1. If the river is
400m wide, find the time he takes to cross the river as quickly as possible and the

24 | P a g e
distance downstream.
4. A boat can travel at 3.5ms-1 in still water. A river is 80m wide and the current flows at
2ms-1. Calculate;
(i). The shortest time to cross the river and distance downstream that the boat is
carried.
(ii).The course that must be set to cross the river to a point exactly opposite the
starting point and the time taken for the crossing.

5. A boats man who can swim at 2ms-1 in still water wishes to swim across a river, 150m
wide. If the river flows at 1.5ms-1. Find;
(i). The time the boats man takes for the crossing and how far down stream he travels.
If he is to cross the river as quickly as possible.
(ii).The course that helmets set in order to cross to a point exactly opposite the
starting point and the time taken for the crossing.
6. The banks of a river are parallel and 50m a part and a current flow at 8ms-1. A boat with a
speed of 10ms-1 in still water sails from a point A at one bank to B directly opposite on
the other bank.
(i). Find the directions in which the boat is steered and the time it takes to cross the
river.
(ii).Explain why the shortest time of crossing is achieved when the boat is steered in a
direction perpendicular to the banks, find where the boat reaches on the opposite
bank.
7. A man wishes to row across a river to reach a point on the far bank exactly opposite his
starting point. The river is 100m wide and flows at 3ms-1. Instill water the man can row at
5ms-1. Find at what angle to the bank the man must start the boat in order to complete the
crossing and the time taken by him.
8. A man wishes to cross a river to reach a point on the far bank, exactly opposite his
starting point. The river is 125m wide and flows at 1.0ms-1. If the man can travel at 3ms-
1still water. Find the direction the man must steer in order to complete the crossing and
the time it takes him.
9. A boy wishes to swim across a river 100m wide as quickly as possible. The river flows at
3kmhr-1 and the boy can swim at 4kmhr-1 instill water. Find the time that it takes the boy
to cross the river and how far down stream he travels.
10. A man who can swim at 2ms-1 in still water wishes to swim across a river 120m wide as
quickly as possible. If the river flows at 0.5ms-1. Find the time the man takes for the
crossing and how far down streams he travels.
11. A man swims at 5kmhr-1 instill water. Find the time it takes the man to swim across a
river 250m wide flowing at 3kmhr-1. If he swims so as to cross the river.
(i). By the shortest route
(ii).In the quickest time.

25 | P a g e
12. A man wishes to swim about across a river to reach a point on the far bank that is 35m
downstream from his starting point. The man can row the boat at 2.5ms-1 in still water. If
the river is 50m wide and flows at 3ms-1. Find the two possible courses the man could set
and find the respective crossing times.
13. When swimming in a river a man finds that he has a maximum speed V when swimming
downstream and U when swimming upstream.
(i). Find an expression for his maximum speed when swimming in still water .
(ii). If the river is of width, S, show that the shortest time in which the man can swim
2S s (V −U )
across is and that such a crossing would take him a distance of
V +U V +U
down stream from his starting point.
(iii). If the man wishes to swim as quickly as possible from a point on one bank to a
point on one bank to appoint exactly opposite on the other bank show that he must

swim in a direction that makes an angle of Cos-1 ( V −U


V +U ) with the bank and that the
S
crossing will take a time .
√VU
14. A man who can swim at 5.0ms-1 in still water wants to cross river 20.0m wide flowing at
3.0ms-1 to a point 5m downstream. Find the velocity he should use to achieve this.

TOPIC: 5. PROJECTILE MOTION

DEFINITIONS;

1. Define the following terms as used in projectiles; a projectile, Angle of projection, Angle of
projection, range, time of flight, projectile motion, greatest height

DERIVATIONS AND RELATIONS

1. A projectile is fired in air with a speed 4ms-1 at an angle of  to the horizontal. Find the
expression for the time of flight of the projectile.
2. Derive the expression for the maximum horizontal range by a projectile in terms of the
initial speed, 5ms-1 and the angle of projection to the horizontal.
3. Prove that the time of flight T and the horizontal range, R, are related by gT=2R tanα
where α angle of inclination to the horizontal.
4. A projectile is fired in air with a speed u at angle α to the horizontal. Show that the path
of the projectile is a parabolic.
5. Prove that the time of flight for a particle projected up at angle α with an initial velocity
u is equivalent to twice the time to reach maximum height.
6. An object is projected from an origin o with initial velocity, V, and at angle of
inclination, above the  horizontal. Given that object lands at the same horizontal as the
26 | P a g e
point of projection and that resistance to motion is negligible, show that the range
V 2 sin 2
attained by the object is R where R =
g

7. State the value of  for which R is a maximum


8. Sketch to show the relationship between kinetic energy and height above the ground for
a projectile.
CALCULATIONS
1. A projectile is fired horizontal from the top of a cliff 2.50m high. The projectile lands
1.414x103m from the bottom of the cliff. Find the
i. Initial speed of the projectile
ii. Velocity of the projectile just before it hits the ground.

2. A stone is projected at an angle of 200 to the horizontal and just clears a wall which is 10m
high and 30m from the point of projection. Find the

i. Speed of projection
ii. Angle which the stone makes with the horizontal as it clears the wall.

3. Calculate the range of a projectile which is fired at angle of 450 to the horizontal with a
speed of 20ms-1.

4. A ball is kicked from a spot 30m from the goal posts with a velocity of 20ms-1 at 300 to the
horizontal. The ball just clears the horizontal bar of the goal posts. Find;

i. the height of the goal posts


ii. the time of flight
iii. how far behind the goal posts the ball lands.

5. A bullet is fired from a gun placed at a height of 200m, with a velocity of 159ms-1 at an
angle of 300 to the horizontal, find;

i. the maximum height attained


ii. The time taken for the bullet to hit the ground.

6. A stone of mass 0.5kg is attached to the string of length 0.5m which breaks if the tension in
it exceeds 20N. The stone is whirled in a vertical circle, the axes of rotation being at a vertical
height of 1.0m above the ground. The angular speed is gradually increased until the string
breaks. Find the position where the stone hits the ground when the string breaks.

7 .A particle p is projected from a point A with an initial velocity of 60ms-1 at an angle of 300 to
the horizontal. At the same time a particle Q is projected in opposite direction with initial

27 | P a g e
velocity of 50ms-1 from a point at the same level with A and 100m from B. Given that the
particles collide find,

i. the angle of projection of Q


ii. the time when collision occurs.

8. A particle is projected with speed v at an angle of  above the horizontal. If the particle passes
through the point (x, y)

i. Write down expression of x and y in terms of V, Q, and time t.


2 2
g X se c
ii. show that y=x tan θ – 2 where g –acceleration due to gravity
2V
V 2 sin 2
iii. show that the range on the horizontal through the point of projection R is R=
g

9. A stone thrown upwards at angle of α to the horizontal with a speed u ms-1 just clears a
vertical wall 4m high and 10m from the point of projection when travelling horizontally .find the
angle of projection.

10. A particle is projected from level ground towards a vertical pole 4m high and 30m away
from the point of projection. It just passes the pole in one second. Find.

i. its initial speed and angle projection


ii. The distance beyond the pole where the particle will fall

11. A particle is projected at an angle of 600 to the horizontal with a velocity of 20ms-1
Calculate the greatest height the particle attains (take g =10ms-1)

12. A particle is protected at 200 to the horizontal and it just clears a wall 10m high and30m
away from the point of projection. Find

i. the speed of projection


ii. velocity of the projectile when it strikes the building
iii. time taken to reach the building

13. A short is fired from the top of a vertical cliff 200m high with a velocity of 500ms-1 at an
elevation of 300. Find

i. the distance from the point of here the short strikes the ground to the bottom of the cliff
ii. the time taken
iii. the distance from the ground to the highest point reached.

14 .A particles is projected horizontally at 20ms-1 from a point 78.4m above the horizontal
surface. Find;

28 | P a g e
i. the time taken for the particle to reach the surface
ii. the horizontal distance travelled in that time.

15. An object A is projected upwards from a height of 60m above the ground with a velocity
of 20ms-1 at 300 to the horizontal. At the same time, another object B is projected from the
ground upwards towards A at 300 to the horizontal. A and B collide at a height of 60m above the
horizontal ground, when they are both moving down wards. Find;

i. the speed of the projection of B


ii. the horizontal distance between the points of projection
iii. The kinetic energy of A just after collision with B if mass of A = 0.55kg.

16. A stone is projected at angle of600 to the horizontal with a velocity of 30ms-1. Calculate
the

i. highest point reached


ii. the range
iii. the time taken for the flight

17. A shot is projected at angle of 550 to the horizontal with a velocity of 8ms-1. Calculate

i. the highest point reached


ii. the range
iii. the time taken to return to the ground
iv. the height of the short when its path makes angle of 300 with the horizontal

18. A soft ball is thrown at angle of 600 above the horizontal it lands a distance of 3m from the
edge of a flat roof of height 15m. The edge of the roof is 40m horizontal from the thrower as
shown Roof

3m

600 Ground

40m

i. At what speed was the soft ball thrown


ii. Calculate the velocity with which the soft ball hits the roof.

19. A bullet of mass 10g moving at 60ms-1 hits a block of wood of mass 3.7kg resting on a
smooth table of height 20m as shown below.

29 | P a g e
Bullet Block of wood

20m

If the bullet embeds into the block. Calculate the horizontal distance travelled by the block
after this collision.

20. A particle is projected with a speed of 30ms-1 at an elevation of 300. Find

i. the greatest height reached


ii. the time of flight
iii. the speed of the particle at height of 4m.
iv. the horizontal angle

21. A ball rolls off the edge of a table top 1m above the floor with a horizontal range of 1ms-1
find;

i. the time taken to hit the ground floor


ii. the horizontal distance it covers along the floor
iii. the velocity with which it strikes the floor.

22. A projectile is fired from ground level with a velocity of 500ms-1 at 300 to the horizontal.
Find the horizontal range, greatest height to which it rises and time taken to reach the greatest
height

23. A stone is projected horizontally from the top of a wall 25m high, if the stone falls a
distance 140m from the foot of the wall find

i. the speed of projection of the stone


ii. Velocity of the stone just before it hits the ground.

24. A small bob suspended from a string is set into oscillation when the bob passes through the
lowest point of the motion, the string is cut. If the bob is then moving with a velocity of 0.8ms-1
at a height 5.0m above the ground. Find the horizontal distance travelled by the bob.
25. A projectile is fired with a velocity of 320ms-1 at angle of300 to the horizontal. Find
i. the time taken to reach the greatest height
ii. the horizontal range with the same velocity what is the maximum range possible .

30 | P a g e
26. A ball is thrown forward horizontally from the top of a cliff with velocity of 10ms-1, the
height of the cliff above the ground is 45m. Calculate

i. time to reach the ground


ii. the distance from the cliff of the ball on hitting the ground
iii. the direction of the ball to the horizontal just before it hits the ground.
iv. Calculate the range of a projectile which is fired at angle of 450 to the horizontal with a
speed of 20ms-1.

27. A bomb is dropped from an aero plane when it is directly above a target at a height of
1402.5m. The aero plane is moving horizontally with a speed of 500kmhr-1. Determine whether
the bomb will hit the target.

28. A ball is kicked from the ground at angle of 300 to the horizontal and just clears a wall 12m
high at a distance of 30m from the point where it was kicked from

i. the speed with which the ball was kicked


ii. the velocity of the ball as it clearing the wall.

29. A soft ball is thrown at angle of 600 above the horizontal .it lands a distance 2m from the
edge of a flat roof of height 20m. The edge of the roof is 38m horizontally from the thrower

Roof

2m

20m

600

38m Ground

i. At what speed was the soft ball thrown


ii. Calculate the velocity with which the ball strikes the roof.

30. An athlete executing a long jump leaves at 300 to the horizontal and travels 8.6m. Find the
take –off speed.

31. A bullet is fired from a gun placed at a height of 200m with a velocity of 150ms-1 at angle of
300 to the horizontal. Find

i. the maximum height attained


ii. the time taken for the bullet to hit the ground.

32. A particle is projected horizontally at 36ms-1 from a point 122.5m above a horizontal surface.
Find

31 | P a g e
i. time taken to reach the surface
ii. horizontal distance travelled in that time.

33. A particle is projected horizontally from a point 44.1m above the horizontal plane. The
particle hits the plane at a point which is horizontally 39m from the point of projection, find the
initial speed of the particle.

34. A body is projected horizontally with a velocity of 39.2ms-1 . Find the horizontal and
vertical components of the velocity of the body 3.0s after projection. Find also the speed and
direction of motion of the body at this time.

35. A plane moving at 1500ms-1 drops a bomb when it is at a height 0.5 km from the ground.
How far from the point at which the bomb is dropped does the bomb land.

36. A stone thrown horizontally at a speed of 24 ms-1 from the top of a cliff takes 4.0s to hit the
sea .calculate;

i. the height of the cliff top above the sea


ii. the distance from the base of the cliff to the point of impact.

37. A transport plane travelling at a speed of 50ms-1 at an altitude of 300m releases a parcel
when directly above a point X on a level ground. Find;

i. the time taken for the parcel to hit the ground


ii. the speed of impact of the parcel
iii. the distance from X to the point of impact.

38. A shot is fired from the top of a cliff 250m high with a velocity of 650ms-1 at elevation of
300. Find the distance from the point where the shot strikes the water at the bottom of the cliff.

39. A shell is fired at 400ms-1 at an angle of 250 to the horizontal.

i. calculate the range of the shell


ii. find the maximum initial velocity of the shell required to achieved the above range.

40. A body is projected at such an angle that its range is five times the greatest height attained

i. find the angle of projection


ii. If with this angle of projection, the range is 800m, calculate the speed of projection.

41. An athlete executing a long jump leaves the ground at 300 angle and travel 8.90m. Find the
take- off speed.

42. Two footballers 100m a part stand facing each other one kicks a ball from the ground such
that it takes off at a velocity of 25ms-1 at 450 to the horizontal . Find the speed at which the

32 | P a g e
second man must run towards the first man in order to trap the ball as it touches the ground if he
starts running at the instant the ball is kicked.

43. If a foot baller kicks a ball from a ball line towards the opponent’s goal. If the initial speed
of the ball is 20ms-1 and makes an angle of 300 to the horizontal .calculate the speed at which the
opponent must run in order to trap the ball as it touches the ground Assume that the goal posts
are 100m a part.

44. Two foot ballers 120m a part stand facing each other on of them kicks a ball from the
ground such that the ball takes at a velocity of 30ms-1 at 380 to the horizontal. Find the speed at
which the second foot baller must run towards the first foot baller in order to trap the ball as it
touches the ground, if he starts running at the instant the ball is kicked.

45. A particle is fired at a height of 200m with a velocity of 150ms-1 at an angle of 300 to the
horizontal. Find;

i. the maximum height attained


ii. the time taken for the particle to hit the ground.

46. A projectile is fired at angle of 450 to the horizontal from a point A. The projectile passes
through a point B above the level A having coordinates (600m, 22m) with respect to A.
calculate;

i. the speed of projection


ii. the angle which the projectile which the projectile makes with the horizontal as it passes
through B.

47. A projectile fired at angle of 600 above the horizontal strikes a building 30m a way at a
point 15m above the point of projection.

i. find the speed of projection


ii. find the velocity of the projectile when it strikes the building.

48. A particle is projected from a point on a horizontal plane and initially travels in a
3
direction which makes an angle of tan-1 ( ) with the horizontal. in the sub sequent motion the
4
particle passes through a point above plane which is 20m horizontally and 10m vertically from
the point of projection .find the speed of projection.

49. Show that when the maximum horizontal range is 160km the time of flight is about three
minutes and determine the muzzle speed and maximum height of the trajectory.

50. A body is projected with a velocity of 200ms-1 at angle of 300 above the horizontal.
Calculate;

33 | P a g e
i. the time taken to reach the maximum height
ii. its velocity after 16s.

51. A particle is projected with a speed of 25ms-1 at 300 above the horizontal. Find

i. the time taken to reach the highest point of the trajectory


ii. the magnitude and direction of the velocity after 2.0s.

52. A particle is projected with a velocity of 3oms-1 at angle of 400 above the horizontal plane.
Find;

i. the time for which the particle is in air


ii. the horizontal distance it travels.

53. A pebble is thrown from the top of a cliff at a speed of 10ms-1 and at 300 above the
horizontal. It hits the sea below the cliff 6.0s later. Find

i. the height of the cliff


ii. the distance from the base of the cliff at which the pebble falls into the sea.

54. A pencil is accidently knocked off the edge a desk top .if the height of the desk is 64.8 cm
and the pencil hits the floor a horizontal distance of 32.4 cm from the edge of the desk .what was
the speed of the pencil as it left the desk.

55. A particle is projected from level ground in such a way that it’s horizontal and vertical
components of velocity 20ms-1 and 10ms-1 respectively. Find

i. the maximum height of the particle


ii. its horizontal distance from the point of projection when it returns to the ground
iii. the magnitude and direction of its velocity on landing.

56. An aero plane moving horizontally at 150ms-1 releases a bomb at a height of 500m. The
bomb hits the intended target. What was the horizontal distance of the aero plane from target
when the bomb was released.
57. A stone thrown horizontally from the top of a vertical cliff with velocity 15ms-1 is
observed to strike the ground at a distance of 45m from the base of the cliff. Find
i. the height of the cliff
ii. the angle the path of the stone makes with the ground at the moment of impact.
58. Two objects p and Q are projected with the same velocity. P is projected at an angle of 300
to the horizontal while q is projected vertically upwards but at a distance of 150m from the point
where p is projected if the particles collide when both are at maximum height, find their velocity
of projection.
59. A stone is projected horizontally from a point 150m above ground and lands 70m away
from the point of projection. Calculate;

34 | P a g e
i. velocity of projection
ii. velocity of stone before hitting the ground
60. A particle is projected with an initial velocity 30ms-1 at angle of elevation of 300. find;
i. the greatest height reached
ii. time of flight
iii. Horizontal range
61. A projectile is fired horizontally from the top of a cliff 300m high. The projectile lands
1.50x103m from the bottom of the cliff. Find;
i. Initial speed of the projectile
ii. velocity of the projectile just before it hits the ground
62. The 8g bullet is fired into a 2.5kg block initially at rest at the edge of a friction less table of
height 1m. The bullet remains in the block and after impact the block lands 2.0m from the
bottom of the table
Bullet Block of wood

1m

2m
Determine the initial speed of the bullet.

63. A shell is fired from a gun with a velocity of 600ms-1 at an angle of 400 to the ground level.
Calculate;

i. the time of flight


ii. the range
iii. the maximum height

64. A ball is kicked from a spot with a speed of 8ms-1 inclined at an angle of  to the
horizontal. At the highest point of its path, it just clears a fence which is 2.2m high .Assuming
negligible air resistance

i. show that =550


ii. calculate the time of flight and the horizontal range.

35 | P a g e
TOPIC 6: NEWTON’S LAW OF MOTION

DEFINITIONS

1. Define thefollowingterms;inertia,momentum,linearmomentum,force,anewton,impulse,inelastic
collision, elastic collision , perfectly Elastic Collision , Perfectly inelastic Collision .

LAWS, PRINCIPLES AND THEOREM

1. State the following

(i). Newton’s laws of motion


(ii). Principle of conservation of linear momentum
(iii). Relative velocity rule for a perfectly elastic collision
FACTORS, DEMERITS, MERITS AND FEATURES EXAMPLES
1) State the conditions under which the following will be conserved in a collision
between two bodies.
(i). Linear momentum
(ii).Mechanical energy
2) Give two factors on which the magnitude of linear momentum depends on.
3) Give two practical examples of perfectly in elastic collision
4) State and explain one application of principle of conservation of momentum.
DERIVATIONS AND RELATIONS.
1) Define impulse and derive its relation to linear momentum of a body on which it
acts.
2) A body of mass m1 and velocity u1 collides head on with a body of mass m2 having
velocity u2 in the same direction as u1. Use Newton’s law to show that quantity
m1u1 + m2u2 is conserved.
3) Show that the law of conservation of linear momentum follows Newton’s laws of
motion.
4) A body A of mass m1 moves with velocity u1 and collides head on elastically with
another body B of mass m2 which is at rest. If the velocities of A and B are v1 and
m1
v2 respectively and given that X = show that;
m2

36 | P a g e
U 1 ×+1 U2 2 ×
(i). = (ii) =
V 1 ×−1 V 1 ×−1
5) Use Newton’s laws to derive the law of conservation of linear momentum.
6) Use Newton’s laws of motion to show that when two bodies collide their
momentum is conserved.
7) A body of mass m1 moving with a velocity u1 makes a perfectly elastic collision
with another body of mass m2 moving with a velocity U2 in the same direction as
8) (a) (i) Define Impulse of a force, give its SI unit.
(ii) Use Newton’s second law to derive the expression F = ma.
9) Define linear momentum and state its SI unit
10) Derive the relationship between impulse and change in linear momentum of a
body
11) Distinguish between elastic and inelastic collision and in each case state the
characteristics
12) Define linear momentum and state the law of conservation of linear momentum.

m1 B C
13) A V1 m2 m3
Balls A, B, and C of masses m1 m2 and m3 respectively lie on a straight line on a smooth
surface. The balls are initially at rest. Ball a which is projected with a velocity V 1
towards B makes an elastic collision with B. if B moves and makes a perfectly inelastic
collision with C. show that both B and C move with a common velocity;
2 m1 m2 V 1
V 3=
( m1 +m2 )( m2 +m3 )

14) A body explodes and produces two fragments of masses m and M if the velocities
of the fragments are u and v respectively. Show that the ratio of Kinetic energies
E1 M
of the fragments is = where E1 is the kinetic energy of m and E2 is the
E2 m
kinetic energy of M
15) (i) State Newton’s law of motion

(ii) Use Newton’s law to show that F = ma

16) (i) Distinguish between perfectly elastic collision and perfectly inelastic collision.

(ii) Give one example of each of the above collision.

37 | P a g e
17) A body of mass m1 moving with velocity u1 collides with a stationery body of
mass m2. The bodies move with velocities V1 and V2 respectively after collision. If

(
the collision is perfectly elastic show that U1 =
m1−m 2)
m 1+ m 2
V2

18) A particle of mass m1 lies on a smooth horizontal table and is connected to a


freely hanging particle of mass m2 by a light in extensible string passing over a
smooth. Fixed pulley at the edge of the table initially, the system is at rest with m1
at a distance, d from the edge of the table. Show that the time taken for the mass

m1 to reach the edge of the table is. t =



2 d (m1 + m 2)
m2 g
19) A particle of mass m1 moving with a velocity u1 makes a perfectly elastic collision
with a stationery particle of mass m2. After a velocity V1 while the second particle
moves in the second particle moves in the same direction with a velocity V2.
Show that.
( m1−m2 ) 2 m1 U 1
V 1= And V 2=
m1 +m2 m 1+ m2
20) A particle P of mass m1 moving with a speed u1 collides head on with a stationary
particle q of masses m2. If the collision is elastic and the speeds of the particles
m2
after impact are V1 and V2. Show that for φ = , then.
m1
V2 2 4φ
(i) = (ii) Q gains of the total energy of the system.
U 1 1+φ ( 1+ φ ) 2
21) A particle A of mass m1 travelling with a speed u1 makes a head on collision with
a stationery particle B of mass m2. If the collision is elastic and the speeds A and
m1
B after collision are V1 and V2 respectively, show that for P = then;
m2
V 1 ( P−1 ) V 1 ( P−1 ) ( V 1 +U 1 )
(i) = (ii) = (iii) P =
U 1 ( P+ 1) U1 2P (U ¿ ¿ 1−V 1) ¿

V 1 2 P ( P+1 )
(iv) =
U 1 ( p2−1)

22) Two blocks A1 of mass m, and B of mass, 3m, are side by side and in contact with
each other. they are pushed along a smooth floor under the action of a constant
force F applied to A. as shown.

F A B

38 | P a g e
F
Show that; (i) acceleration of the blocks a =
4m
3F
(ii) The force exerted by B on A, P =
4
23) A particle A of mass ,m moving with an initial velocity U makes a head on
collision with another particle B of mass 2m, B being initially at rest, in terms of
U, calculate the final velocity of A if the collision is
(i). Elastic
(ii).Inelastic
24) P is a smooth fixed pulley, over which passes light in extensible string. Each and
of the string supports pan of mass m kg one scale pan contains a particle of mass
M1 the other contains a particle of mass M2, given that M1 7M2 show that the
( m2−m1 ) g ms−2
acceleration of the particle is a =
2 m+ m 1+m 1
25) A particle of mass, m1 with a velocity U collides head on with a stationery particle
of mass m2. If the collision is elastic and the velocity of 1st and 2nd after collision
2 UM 1
are V2 and V2 respectively. Show that v2 =
m1 +m 2
26) A body of mass 2m moving with a velocity Ums-1 make perfectly elastic collision
with a stationary body of mass M. show that if 2m and m move with velocities V1
V2
and V2 respectively than = 4.
V1
27) A particle A of mass m1 travelling with a velocity u1 makes a perfectly elastic
collision with another body b of mass m2 travelling with a velocity of u2 in the
same direction. Show v1 and v2 are that (v2 – v1) = - (u2 – u1) where the velocities
of A and B respectively after collision.
28) A body of mass m1 moving with a velocity u1 collides head on with a stationery
body of mass, m2. if m1 moves with v1 and m2 with v2 after collision. Show that
m2
for K =
m1
U 1 1+ K
(i). =
V2 2
U 1 1−K
(ii). =
V2 2
29) A particle A of mass m1 moving with a speed u1 collides head on with a stationery
particle b, of mass m2. if the collision as elastic and the speeds of particles are V1
M2 U1 2
and V2 respectively show that for M = ; then =
M1 V 2 1−M

39 | P a g e
30) Use Newton’s Laws of motion to show that impulse is equal to change in linear
momentum
31) A ship is powered by water jet propulsion unit driven by a diesel engine. When
the ship is stationary, the engine is running at full power and the unit takes in
water and expels. It jet of cross sectional area 0.030m2 at a speed V.
(i). Write down an expression for the mass water flowing in the jet in one second.
Take density of water to be 1050kgm-3
(ii).The kinetic energy given to the jet in one second is 1.5 x 106 J. Calculate the
velocity V and the momentum gained by the water in one second. State the thrust
exerted by the jet on the ship.
32) A particle A of mass 4kg is incident with a velocity, V to a stationary particle of
equal mass as shown in the diagram below.
C

Vms-1 600

A B

V -1
After collision A moves in direction BC with a velocityms where BC makes an angle
2
of 600 with AB produced and B moves along BD at an angle θ to AB. Calculate;

(i). The velocity of B after collision


(ii).The angle θ
33) A bullet of mass 20g is fired from a gun of mass 2.5kg. The bullet leaves the gun
with a velocity of 50ms-1. Find the initial velocity of the gun.
34) In a pool game a white ball is projected at a speed of 10ms-1 towards a stationary
yellow ball. After impact they move with a velocity of U1 and U2 in the direction
as shown respectively; Yellow
ball U2
White ball
300
R φ
10ms -1

White ball U1
Assuming the two balls have the same mass and the collision is
Perfectly elastic find;
(i). The values of U1 and U2
(ii).Angle θ.

40 | P a g e
35) The two balls shown below collide and bounce off each other.

30ms-1 θ 50ms-1

300 5kg

8kg 15ms-1

(i). What is the final velocity of the 5kg ball of the 8kg ball has a speed of 15ms-1
(ii).Is the collision perfectly elastic
36) A truck of mass 1.0tonnes moving at 3.0ms-1 catches up and collide with another
truck of mass 2.0 tones moving at 3.0ms-1 in the same direction. If the trucks
become coupled together, calculate;
(i). The common velocity
(ii).Percentage loss on their Kinetic energy.

EXPLANATIONS
1. Explain why when catching a fast moving ball the hands are drawn while the ball is
being brought to rest.
2. Explain why a long jumper should normally land on sand.
3. Explain why a passenger in a bus jerks forward when the bus is suddenly stopped.
4. Explain how a rocket is kept in motion
5. Explain why passengers in a bus are thrown back wards when the bus suddenly starts
moving.
6. Explain why a high jumper has to bend his knees when landing on the ground.
7. Explain the cause of the difference in Kinetic energy during collision of bodies
8. Explain why a person executing long jump should flex his knees when landing on the
ground.
9. Explain why the tension in a cable of a lift is different when the lift is ascending from
when it is descending
10. The weight of body is measured using a spring balance which is suspended from the
roof of a lift.
11. Explain the reading obtained when the lift is accelerated down wards and when it is
accelerating upwards.
12. Explain why martial arts player can break a pile of bricks with a lot of ease.

CALCULATIONS

41 | P a g e
1) A block of mass 2kg is pushed a long a table with a constant velocity by a force of 5N.
When the push is increased to 9N. What is.
(i). The resultant force
(ii).The acceleration.
2) Two blocks of masses M1 = 3kg and M2 = 2kg are in contact on a horizontal force F =
5.0N is applied to the block M1 in the directions shown

F M1 M2
There is a constant frictional force of 2N between the table and the block of mass M2 but
no frictional force between the table and the block of mass M1.
Calculate; (i) the acceleration of the two blocks
(ii) The force of contact between the two blocks.
3) A truck of mass 2000kg is moving at 20ms-1 and is acted on by two forces, a forward
force of 800N due to the engine and a 300N retarding due to friction, both forces being
constant.
Find; (i) the rate at which the truck gains speed
(ii) The distance the truck travel in 8.0s
4) A 4.0 x 103 kg truck pulls a tailor of mass 8.0 x 103kg from rest through a distance of 10m
in 4s. Using a cable if the total resistance offered by the whole system is 3600N,
calculate;
(i). The tractive pull of the truck
(ii).The tension in the cable
5) An engine of mass 5.0 x 103kg pulls a train of 5 trucks each of mass 2.0 103kg along a
horizontal track. If the engine exerts a frictional force of 5.0104N and given that the
frictional resistance is 5000N. Calculate;
(i). Acceleration
(ii).Tension in the rope used for pulling.
6) In the figure below, two blocks A and B of masses 8kg and 4kg respectively are in
contact on a horizontal table. A constant horizontal force of 10N is applied on block A as
shown below. There is a constant frictional resistance of 3N between table and block B
but there is no frictional force between A and the table.

F
A B

Calculate the;
(i). Acceleration of the blocks

42 | P a g e
(ii).Constant force between the two blocks.

7) A car of mass 900kg tows a trailer of mass 600kg by means of a rigid tow bar. The car
experiences a resistance of 200N and the trailer a resistance of 300N. if the car engine
exerts a forward force of 3KN. Find the tension in the tow bar and acceleration of the
system. The engine is now switched off and brakes applied to produce a retarding force
of 500N. Assuming that the same resistance apply. Calculate the retardation of the
system. Also comment on the nature and magnitude of the force on the tow bar for this
case.
8) A lorry of mass 3000kg tows a trailer of mass 100kg along a level road and accelerates
uniformly form rest to 18ms-1 in 24.0s. The resistances on the lorry and trailer are
proportional to their masses and total to 1200N. find the
(i). Driving force exerted by the engine of the lorry
(ii).Tension in the tow bar.
9) A vehicle has a mass of 600kg. Its engine exerts a attractive force of 1500N but motion is
resisted by a constant frictional force 300N. find;
(i). The acceleration of the vehicle
(ii). The momentum 10s after starting to move
(iii). Its kinetic energy 15.0s after starting to move.
10) A jet planes sets off from the ground by chemical combustion producing exhaust gases
through the pipe of diameter 10.0cm. If the mass of the jet at take- off is 200kg and
accelerates at 3gms-2 density of exhaust gases is 1.80kgm-3. Calculate the speed of escape
of exhaust gases.
11) A ball of mass 0.5kg travelling horizontally with a speed of 9.90ms-1 strikes and bounces
from the vertical wall. If the collision lasts 0.02seconds. Find the average force exerted
on the wall.
12) Two rectangular blocks of M1 = 4.0kg and M2 = 1.0kg are placed in contact on a
horizontal surface as shown.

F M1 M2

A contact horizontal force, F = 6.4g where g is acceleration due to gravity is applied


on the block of mass M1. The coefficient of sliding friction between the horizontal
surfaces and the masses M1 and M2 are 0.45 and 0.40 respectively if the blocks stay
in contact with each other, as they move along the surface, determine;

(i). The acceleration of the two blocks


(ii).The time taken for the e blocks to come to rest if the force is suddenly
removed after the blocks have moved through 2.0m.

43 | P a g e
CALCULATIONS ON F = ma ON AN INCLINED PLANE.
1) A car of mass 100kg climbs a track which is inclined at 300 to the horizontal. The speed
of the car at the bottom of the incline is 36kmhr-1.
2) A body of mass 5.0kg is pulled up a smooth plane inclined at 300 to the horizontal by a
force of 40N acting parallel to the plane. Find.
(i). The acceleration of the body
(ii).The force exerted on the body by the plane.
3) A 30kg block rests on a smooth plane inclined at an angle of 300 to the horizontal. The
block is pulled up the plane with a force of 30g N parallel to the plane. Find;
(i). The acceleration of the block
(ii).The force exerted by the block on the plane.
4) A car of mass 1.5 x 103kg and tractive pull of 3.5 x 103 climbs a track which is inclined at
angle of 300 to the horizontal. The velocity of the car at the bottom of the incline is 20ms-
1
and the frictional force is 3.2 x 103N. find;
(i). The distance travelled along the incline before the car comes to a stop.
(ii).The time taken travelling along the incline before the car comes to rest.
5) A car of mass 1000kg tows a caravan of mass 600kg up a road which rises 1m vertically
for every 20m of its length. There are constant frictional resistance of 200N and 100N to
the motion of the car and to the motion of the caravan respectively. The combination has
an acceleration of 1.2ms-2 with the engine exerting a constant driving force. Find
(i). The driving force
(ii).The tension in the tows bar.
6) A car of mass 1500kg rolls from rest down a road inclined to the horizontal at an angle of
350, through 50m. The car collides with another car of identical mass at the bottom of the
inclined if the two vehicles interlock on collision and the coefficient of kinetic friction
between the block and the flour is 0.20. find the common velocity.
7) A body of mass 5.0kg is pulled up a smooth plane inclined at 300 to the horizontal by a
force of 40N acting parallel to the plane. Calculate the acceleration of the body and the
force exerted on it by the plane.
8) A body of mass 3.0kg slides down a plane which is inclined at 300 to the horizontal. Find
the acceleration of the body.
(i). If the plane is smooth
(ii).If there is a frictional resistance of 9.0N.
9) A railway truck of mass 6.0tonnes moves with an acceleration of 0.050ms-2 down a track
1
which is inclined to the horizontal at angle of  where sin = . Find the resistance to
120
motion.
10) A railway engine of mass 1.0 x 105kg is attached to a line of trucks of total mass 8.0 x
104kg. Assuming there is no resistance to motion find the tension in the coupling between
the engineered the leading truck when the train;
(i). Has an acceleration of 0.020ms-2

44 | P a g e
(ii).Is moving at constant velocity.

CALCULATIONS ON LIFTS ACCELERATING


1) A boy whose mass is 36kg stands on a spring weighing machine inside a lift. When the
lift starts to ascend, its acceleration is 0.25ms-2. Find
(i). The reading of the weighing machine
(ii).The reading of weighing machine if deceleration of the lift is 2ms-2.
2) A body hangs from a screen balance supported from the roof a lift. The lift has an upward
acceleration 2.5ms-2 and the balance reads 60N.
(i). What is the weight of the body.
(ii). Under what circumstances will be balance read 40N.
(iii). What would the balance read if the cable of the lift breaks.

3) A lift of mass 450kg is carrying a woman of mass 50kg


a) The lift is ascending at a uniform speed. Calculate the
(i). Tension in the lift cable.
(ii).Vertical force exerted on the woman by the flour of the lift
(iii). If the lift later ascends with a down ward acceleration of 1.2ms-2.

Calculate;

(i). Tension in the lift cable


(ii).Vertical force exerted on the woman by the flour of the lift.
4) A man of mass 85kg stands on a lift of mass 30kg when he pulls on the rope he exerts a
force of 400N on the floor of the lift. Calculate the acceleration of the lift.
5) A woman stands in a lift. Explain how her weight on the reaction at the floor of the lift
will vary when the lift is
(i). At rest
(ii).Moving upwards with a constant velocity
(iii). Accelerating upwards
(iv). Freely falling.

6) A body of mass 7.0kg rests on the floor of a lift. Calculate the force R1 exerted on the
body by the floor when the lift;
(i). Has an upward acceleration of 2.0ms-2.
(ii).Has a downward acceleration of 3.0ms-2
(iii). Is moving down with a constant velocity.
7) A train is moving along a straight horizontal track. A pendulum suspended from the roof
of one of the carriage of the train is inclined at 40 to the vertical. Calculate accelerating of
the train.
45 | P a g e
8) A body hangs from a spring balance which is suspended from ceiling of a lift. What is the
mass of the body if the balance registers ready of 70N when the lift has an upward
acceleration of 40ms-2.
9) A light cord connects two objects of masses 10kg and 6kg respectively, over a light
frictionless pulley. Find the acceleration of the system and the tension in the cord.
10) A body brings from a spring balance supported from the roof of a lift.
(i). What is the weight of the body if the balance reads 50N when the lift has an
upward acceleration of 2.45ms-2.
(ii). Under what circumstances wills the balance read 30N.
(iii). What will the balance read if the cable of the lift breaks.
11) Two particles of masses 5kg and 10kg are connected by an in extensible string which
passes over a smooth fixed pulley. Find the acceleration of the system and the tension in
the sting.
12) Bodies of mass 6kg and 2kg are connected by a light in extensible string which passes
over a smooth fixed pulley with the masses vertically, the system is released from rest.
Find the acceleration of the system and the distance moved by the 6kg mass in the first
two seconds of its motion.

13) Particles of mass 0.6kg and 0.4kg are connected by a light inextensible string passing
over a smooth pulley. Initially both masses are hanging vertically, 30cm above the
ground. If the system is released from rest, find the greatest height reached above the
ground by the 0.4kg mass.
14) Two particles of masses 6.0kg and 2.0kg are connected by a light inextensible string
passing over a smooth pulley. The system is released from rest with the string tent. Find
the speed of the particles of the particles when the heavier one has descended 2.0m.
15) A man of mass 50kg rests on the floor of a lift when standing calculate the reaction at the
man’s feet when;
(i). The lift is stationary
(ii).The lift accelerates upwards at 2ms-2.
16) A man of mass 80kg stands in a stationary lift on Earth. Calculate his apparent weight
when the lift
(i). Accelerates upwards at a rate of 2ms-2
(ii).Falls freely under gravity.
17) An object A of mass 2kg moving with a velocity 3ms-1 collides head on with another
object B of mass 1kg moving in opposite direction with velocity of 4ms-1. After collision
both objects coalesce so that they move with common velocity V. calculate V.
18) A ball A of mass 0.1kg moving with 6ms-1 collides directly with the ball B of mass 0.2kg
at rest. Calculate the common velocity if both balls move together. If A had rebounded
with a velocity of 2ms-1 in the opposite direction after collision what would be the new
velocity of B.

46 | P a g e
19) Two trolleys are involved in a collision. Trolley A mass 2kg and initially at 70ms-1
collides with B which was initially at rest. Trolley B eventually reaches a velocity of 6ms-
1
in the same direction in which trolley A was travelling while trolley A eventually
rebounds at 2ms-1. If the time spent in a collision while trolley B travels 0.015m.
Calculate;
(i). The mass of trolley B
(ii). The average force exerted on both trolleys during collision
(iii). K.e change of both trolleys.
20) Two particles of masses 3kg 5kg travelling in opposite direction on a horizontal surface
collide. Their respective velocities before collision are 6ms-1 and 5ms-1. They separate
after collision and move in the same direction in which the 5kg block was moving before
collision. Given that the velocity of the 5kg body after collision is 1ms-1, find the speed of
the 3kg after impact and the percentage loss in energy.
21) A ball A of mass 10kg moving with a speed of 8ms-1 collides with another ball B of mass
20kg initially at rest. After the collision, A and B move in a direction making angles of
300 and 4500 respectively with the initial direction of motion of A. calculate the speeds of
A and B after collision.
22) A sphere of mass 3kg moving with a velocity 4ms-1 collides head on with a stationary
sphere of mass 2kg and imparts to it a velocity of 4.5ms-1. Calculate the velocity of the
3kg sphere after collision and the amount of energy lost by the moving bodies in the
collision.
23) A railway truck of 4.0x 104 kg moving at a velocity of 3ms-1 collides with another truck
of mass 2.0 x 104 kg which is at rest. The couplings join and the trucks move off together.
Find the fraction of the first truck’s initial Kinetic Energy remains as kinetic energy of
the two trucks after the collision.
24) A proton of mass 1.6 x 10-27kg travelling with a velocity of 3.0 x 107ms-1 collides with a
nucleus of an oxygen a tom of mass 2.56 x 10-26kg initially at rest and rebounds in a
directions at 900 to the incident path. Calculate the velocity and direction of the recoil
oxygen nucleus. Assuming the collision was perfectly elastic.
25) Two trolleys P and Q of masses 0.50kg and 0.30kg respectively are held together on a
horizontal track against a spring which is in a state of compression. When the spring is
released the trolleys separate freely and P moves to the left with an initial velocity of
6ms-1.Calculate.
(i). The initial velocity of Q
(ii).The initial total kinetic energy of the system
26) A bullet of mass 2.0 x 10-3 kg is fired horizontally into a free standing block of mass 4.98
x 10-1kg, which it knocks forward with an initial speed of 1.2ms-1. Calculate;
(i). The speed of the bullet
(ii).Kinetic energy lost after impact.
27) A block of wood of mass 1.0kg is suspended freely by a thread a bullet of mass 10g is
fired horizontally at the block and becomes embedded in it. The block swings to one side

47 | P a g e
rising a vertical distance of 0.5m. With what speed did the bullet hit the block.
28) A particle A of mass 2kg and particle B of mass 1kg are connected by alight elastic string
C, and initially held at rest 0.9m apart on a smooth horizontal table with the string in
tension. They are then simultaneously released. The string releases 12J of energy as it
contracts to its natural length
C
A B

Calculate the velocity acquired by each of the particles and where the particles do collide
(VA=2ms-1, VB = 4ms-1, x=0.3m from A)

29) A particle of 3kg and another of mass 1kg are connected by a light elastic string and
initially held at rest on a smooth horizontal table with the string in tension. The string
gives up 24J of energy as it contracts to its natural length. Calculate the velocity acquired
by each of the particles assuming no energy is lost.
30) A helicopter of mass 810kg support itself in a stationary position by importing a down
ward velocity V1 to all the air in a circle of area 30m2. Given that the density of air is
1.20kgm-3. Calculate;
(i). The value of V
(ii).The power needed to support the helicopter.
31) A block of wood of mass 950g rests on a horizontal table of height 3.0m at a distance of
2m from the edge of the table. A bullet of mass 50g moving with a horizontal velocity of
750ms-1 hits and gets imbedded in the block as shown.
Bullet Block of wood
24.8m

Table
3.0m

If the coefficient of dynamic friction between the block and the table is 0.3, find;
(i). The initial velocity of the block and the bullet
(ii).The horizontal distance from the table to the point where block hits the ground.

CALCULATIONS ON MOMENTUM AND COLLISION OF BODIES


1) A car of mass 1000kg travelling at a uniform velocity of 20ms-1 collides perfectly in
elastically with a stationary car of mass 1500kg. Calculate;

48 | P a g e
(i). The common velocity.
(ii).The loss in Kinetic energy.
2) A bullet of mass 0.1kg moving horizontally with a speed of 420ms-1 strikes a block of
mass 2.0kg at rest on a smooth table and becomes embedded in it. Find the Kinetic
energy lost if they move together.
3) An object X of mass M moving with a velocity 10ms-1 collides with a stationary object Y
of equal mass. After collision X moves with speed U at angle of 300 to its initial direction
while Y moves with a speed of V at angle of 900 to the new direction.
(i). Calculate the speeds U and V.
(ii).Determine whether the collision is elastic or in elastic.
4) Two balls P and Q travelling in the same line in opposite directions with speeds 6ms-1 and
15ms-1 respectively make a perfectly inelastic collision. If the masses of P and Q are 8kg
and 5kg respectively. Find the;
(i). Final velocity of P
(ii).Change in Kinetic energy.
5) A bullet of mass 300g travelling horizontally at a speed of 8ms-1 hits the body of mass
450g moving in the same direction as the bullet at 1.5ms-1. The bullet and the body move
together after collision. Find the loss in Kinetic energy.
6) A body of mass 2kg moving on a smooth horizontal surface at 3ms-1 collides with a
second body of mass 1kg which is at rest. After the collision the bodies coalesce. Find the
common velocity.
7) Two spheres A and B of masses 150g and 350g are travelling towards each other along
the same horizontal line with speeds of 4ms-1 and 2ms-1 respectively. After the collision,
the direction of motion of B has been reversed and it is travelling at a speed of 1ms-1.
Find.
(i). The speed of A after collision
(ii).The loss in Kinetic energy due to collision.
8) A bullet is fired from a gun with a horizontal velocity of 400ms-1. The mass of the gun is
3kg and the mass of the bullet is 60g. find;
(i). The recoil speed of the gun
(ii).The gain in kinetic energy of the system.
9) A bullet of mass 8g is fired from a gun of mass 500g. If the muzzle velocity of the bullet
is 500ms-1. Calculate;
(i). The recoil velocity of the gun
(ii).Gain in kinetic energy of the system.
10) A trolley X of mass 50g travelling at 2ms-1 collides with another trolley Y. of twice its
mass moving in the opposite direction with a velocity of 0.5ms-1. If the trollies stuck
together on collision, calculate the common velocity with which they move.
11) A particle A of mass 150g lies at rest on a smooth horizontal surface. A second particle B
of mass 100g is projected along the surface with a speed of 4ms-1. Find;
(i). The common velocity if the collision was inelastic
49 | P a g e
(ii).The loss in kinetic energy of the system during collision.
12) When two bodies having masses of 2kg and 4kg move with velocities 6ms-1 and 3ms-1
respectively in the same direction collide. find their respective velocities after an elastic
collision.
13) A 2kg object moving with a velocity of 6ms-1 collides with a stationary object of mass
1.0kg. Assuming that the collision is perfectly elastic calculate the velocity of each of the
object after collision.
14) A car of mass 600kg travelling at 20ms-1 collides with a stationary truck of mass 1400kg.
the two vehicles are locked together after collision. Find
(i). The common velocity of the vehicles.
(ii).Loss in kinetic energy.
15) A 200g block moves to the right at a speed of 1000ms-1 and meets a 400g block moving
to the left with a speed of 800ms-1. Find final velocity of each block. If the collision is
elastic.
16) A flat track of mass 400kg is moving freely along a horizontal track at 3.0ms-1. A man
moving at right angles to the track jumps on the truck causing its speed to decrease by
0.50ms-1. What is the mass of the man.
17) A block of mass 2.0kg slides from a point 5m high down a plan. At the bottom it strikes a
block of mass 6kg which is at rest on a smooth surface. If the collision is inelastic and
friction is ignored. Calculate;
(i). The speeds of the two blocks after collision
(ii).How far back the incline will the smaller mass go.
18) A truck of mass 104kg moving at 10ms-1 rams into a truck of mass 4 x 103kg which is
stationery. The trucks stick together and skid to stop along the horizontal surface.
Calculate the distance through which the trucks skid of the coefficient of Kinetic friction
is 0.25.
19) A truck of mass 200kg is moving at 25ms-1 down a plane inclined at 300 to the horizontal.
The truck collides with another mass of 300kg moving in the same direction with a
velocity of 10ms-1. If the collision is perfectly inelastic, find;
(i). The velocity of the trucks immediately after collision
(ii).The kinetic energy lost during collision
(iii). Velocity of the trucks 10 s after collision given that the coefficient of
kinetic friction is 0.30.
20) A ball of mass M1 moving with a velocity of 6ms-1 hits a stationary ball q at a glancing
angle as shown below.

Q 300

P 600

50 | P a g e
P

Q moves off at angle of 300 to the initial direction of P and P moves off at angle of 600 to
its direction. Calculate the velocity of Q and P after collision. Is the collision elastic or
inelastic .
21) A car of mass 600kg travelling at 20ms-1 collides with a stationary truck of mass 1400kg.
the two vehicles are lacked together after collision.
Find (i) the common velocity of the vehicles
(ii) The loss of kinetic energy.
22) A bullet of mass 300g travelling at a speed of 8ms-1 bits a body of mass 450g moving in
the same direction as the bullet at 1.5ms-1. The bullet and body move together after
collision. Find the loss in kinetic energy.
23) A car of mass 1000kg travelling at uniform velocity of 20ms-1 collides perfectly in
elastically with a stationary car of mass 1500kg. Calculate the loss in kinetic energy of
the car as a result of the collision.
24) A particle P of mass 3kg and a particle Q of mass 1kg are connected by a light elastic
string initially held at a rest on a smooth horizontal table with the string in tension. They
are then simultaneously released. The string give up 24J of energy as it contracts to its
natural length.
(i). Calculate the velocity acquired by each of the particular assuming no energy is
lost.
(ii).If the collision on impact is perfectly elastic, find the velocity of each particle
after collision.

CALCULATIONS ON MOMENTUM AND IMPLUSE OF A BODY


1) A ball of mass 500g travelling at a speed of 10ms-1at 600 to the horizontal strikes a
vertical wall and rebounds with the same speed at 1200 from the original direction. If the
ball is in connect with the wall for 8.0 x 10-3s. Calculate the average force exerted on the
wall by the ball.
2) Water issues horizontally from a pipe and strikes a nearby wall without rebounding. The
cross sectional area of the pipe is 6cm2 and the water travels at 4.0ms-1. Find the
magnitude of the force acting on the wall (Take density of water of water = 1000kg-3).
3) A gardener directs a jet of water at right angles to a wall at a speed of 3ms-1. If the pipe
has a cross sectioned area of 0.02m2. Calculate the force on the wall assuming that the
water does not rebound. (Density of water = 1000kgm-3)
4) Water from a horizontal pipe ejects at 10ms from the horse of cross area 0.1m 2 at a speed
of 8ms-1. The water hits a vertical wall without rebounding. Find the magnitude of the
force acting on the wall.

51 | P a g e
5) Water from a horizontal pipe ejects at 10ms-1 from the horse of cross sectional area 5cm2.
The water hits a vertical wall at right angles without rebounding. Find the magnitude of
the force acting on the wall.
6) A Water cannon ejects water horizontally at a speed of 16ms-1 through a pipe of cross
sectional area 0.015m2. If the water hits a nearby vertical surface, assuming that the water
does not rebound after impact. (Take density of water = 1000kgm-3).
7) A horizontal pipe ejects water at a speed of 8ms-1 horizontally from a hose pipe of cross
sectional area 1.0mm2. Find the magnitude of force on the vertical wall besides. If the
water hits the wall at right angles without rebounding.
8) A factory waste pipe emits fluid of density 1200kgm-3 through a pipe of cross sectional
area 25cm2. The fluid leaves the pipe horizontally with a speed of 5ms-1 and strikes a
vertical wall without rebounding. Find the magnitude of the force acting on the wall.
9) A helicopter of mass 20000kg remains in air at one place by imparting a downward
velocity, V1 to the air displaced by its rotating blades the area swept out by the blades is
60m2. Calculate the velocity V1 given that the density of air is 1.3kgm-3, and g = 10ms-2.
10) A ball of mass 900g travelling at a speed of 15ms-1 at 600 to the horizontal strikes a
vertical wall and rebounds with the same speed at 1200 from the original directions. If the
ball is in contact with the wall for 5.0 x 10-1s. Calculate the average force exerted by the
ball.
11) Water leaves a horse at a rate of 5.0kgs-1 with a speed of 20ms-1 and is directed
horizontally on a vertical wall which stops it. Calculate the force exerted by the water on
the wall.
12) Sand is deposited at a uniform rate of 20kgs-1 and with negligible Kinetic energy onto an
empty convey or belt moving horizontally at a constant speed of 10meters per minutes
Find;
(i). Force required to maintain the constant velocity
(ii).The rate of change of kinetic energy of the moving sand.
13) A fire hose sends 20kg of water each second onto a burning building. If the water leaves
the nozzle at 60ms-1 and does not bounce back from the wall. What is the force on the
wall of the building.
14) A tennis ball has a mass of 0.07kg. It approaches a racket with a speed of 5ms-1 and
bounces off it and returns the way it come from with a speed of 4ms-1. The ball is in
contact with the racket for 0.2 s calculate:
(i). The impulse given to the ball
(ii).The average force exerted on the ball by the racket.
15) A horse directs a jet of water, moving with a velocity of 20ms-1 onto a vertical wall. The
cross sectional area of the jet is 5.0 x 104 m2. If the density of water is 1000kgm-3. Find
the force on the wall assuming the water is dropped to rest there.
16) A ball of mass 0.5kg is projected from the horizontal ground towards a vertical wall with
a speed of 18.0ms-1 at an angle of 300 to the horizontal. The ball falls to a sport on the

52 | P a g e
ground 8.0m away from the wall after bouncing off. If the wall is 14.5m away from the
point of projection it stays in contact with the wall for a period of 5.0 x 10-2S find;
(i). The time it takes the ball to hit the wall from the point it is projected
(ii).The force that the ball exerts on the wall.
17) A hose pipe has a hole of cross sectional area 50cm2 and ejects water a horizontally at a
speed of 0.3ms-1. If the water is incident on a vertical wall and its horizontal velocity
becomes zero, find the force the water exerts on the wall.
18) A steady stream of balls each of mass 200g hits a vertical wall at right angles with equal
intervals of time between the impacts of each ball on the wall. The speed of the balls is
15ms-1 and 600balls hit the wall in 12S and rebounds at the same speed. Find;
(i). the averages force acting on the wall
(ii).The force exerted on the wall during each collision if the duration of each
collision is 0.01S, and sketch a graph to show how the actual force on the wall
varies with time over a period of 0.10S.
19) A ball of mass 0.2kg falls from a height of 45m on striking the ground it rebounds in 0.1S
2
with of velocity with which it strike the ground. Calculate;
3
(i). The momentum change on hitting the ground
(ii).The force on the ball due to the impact.
20) A body of mass 2kg initially moving with a velocity of 1.0ms-1 is acted upon by a
horizontal force of 6N for 2s. Find
(i). Impulse given to the body
(ii).Final speed of the body.
21) Jinja factory waste pipe emits fluid of density 1200kgm-3 through a pipe of cross
sectional area 25 x 10-4 m2. The fluid leaves the pipe horizontally with a speed of 8ms-1
and strikes a vertical wall without rebounding. Find the magnitude of the force acting on
the wall.
22) Water emerges at 2ms-1 from a hose pipe and hits a wall at right angles. The pipe has a
cross-sectional area of 0.03m2. Calculate the force on the wall assuming that the water
does not rebound. (Density of water = 1000kgm-3)
23) A helicopter of mass 1.0 x 103kg hovers by imparting a down ward velocity V to the air
displaced by its blades. The area swept out by the blades is 80m2. Calculate Value of V.
(Density of air = 1.3kgm-3, g = 10ms-2)
24) Sand falls on a conveyor belt at a constant rate of 2kgs-1 the belt is moving horizontally at
3ms-1. Find;
(i). The extra force required to maintain the speed of the belt
(ii). The rate at which this force is doing work
(iii). The rate at which the kinetic energy of sand increases.
25) Water is squirting horizontally at 40ms-1 from a burst pipe at a rate of 3.0kgs-1. The water
strikes the vertical wall at right angles and runs down it without rebounding. Calculate

53 | P a g e
the force the water exerts on the wall.
26) A machine gun fires 300 bullets per minute horizontally with a velocity of 500ms-1. Find
the force needed to prevent the gun moving backwards if the mass of each is 80 x 10-3kg .
27) Coal is falling onto a conveyor belt at a rate of 540tonnes every hour. The belt is moving
horizontally at 2.0ms-1. Find the extra force required to maintain the speed of the belt.
28) The rotating blades of a hovering helicopter sweep out an area of radius 4.0m importing a
downward velocity of 12ms-1 to the air displaced. Find the mass of the helicopter. (g =
10ms-2, and density of air = 1.3kgm-3).
29) Find the force exerted on each square meters of a wall which is at right angles to wind
blowing at 20ms-1. Assume that the air does not rebound. (Density of air = 1.3kgm-3).
30) Hail stones with an average mass of 4.0gfall vertically and strikes a flat roof at 12ms-1. In
a period of 5.0miuntes, six thousands hailstones fall on each square meters of roof and
rebound vertically 3.0ms-1. Calculate the force on the roof if it has an area of 30m2.
31) The speed of rotation of the blades of the helicopter from 12ms-1 so that the air now has a
downward velocity of 13ms-1. Find the upward acceleration of the helicopter.
32) A ball of mass 6.0 x 10-2kg moving at 15ms-1 hits a wall at right angles and bounces off
along the same line at 10ms-1.
(i). What is the magnitude of the impulse of the wall on the ball
(ii).The ball is estimated to be in contact with the wall for 3.0 x 10-2S what the
average force on the ball.
33) A body of mass 2.0kg and which is at rest is subjected to a force of 200N for 0.20S
followed by a force of 400N for 0.30s acting in the same direction. Find;
(i). The total impulse on the body
(ii).The final speed of the body .
34) A hose with a nozzle 80mm in diameter ejects a horizontal stream of water at a rate of
0.044m3s-1.with what speed will the water leave the nozzle. What will be the force
exerted on a vertical wall if after hitting the wall;
(i). The water falls vertically to the ground
(ii).The water rebounds horizontally.
(Density of water = 1000kgm-3).
35) A helicopter of total mass 103kg is able to remain in a stationary position by imparting a
uniform down ward velocity to a cylinder of air below it of effective diameter 6m.
Assuming the density of the air to be 1.2kgm-3. Calculate the downward velocity given to
the air.
36) A bullet of mass 0.020kg is fired horizontally at 150ms-1 at a wooden block of mass 2.0kg
resting on a smooth horizontal plane. The bullet passes through the block and emerges un
deviated with a velocity of 90ms-1. Calculate;
(i). The Velocity acquired by the block
(ii).The total kinetic energy before and after penetration and account for their
difference.

54 | P a g e
37) A space craft of 2.0 x 104 kg is travelling at 1500ms-1. It rocket ejects hot gases at a speed
of 1200ms-1 relative to the space craft. During one burn, the rackets are fired for 5.0S
period. In this time the speed of the space craft increases by 3.0ms-1. Find;
(i). The acceleration of the space craft
(ii).The distance travelled during the burn
(iii). The thrust produced by the rocket.
(iv). The mass of gas ejected by the rocket during the burn.
38) A top class tennis player conserve the ball of mass 57g at an initial horizontal speed of
50ms-1. The ball remains in contact with the racket for 0.050seconds. Calculate the
average force exerted on the ball during the serve.

TOPIC 7. SOLID FRICTION


DEFINITIONS
1. Define the following terms; friction, static friction, kinetic friction, limiting friction,
coefficient of friction.
FACTORS, MERITS, DEMERITS AND EXAMPLES
1. State four instances where increasing friction is useful
2. Give two methods of reducing friction between solid surfaces
3. State two disadvantages of friction
4. Give two factors on which frictional force between two surfaces depend on.

PRINCIPLE, LAWS AND THEOREMS


1. State the laws of solid friction.
2. State the laws of solid friction and explain them using the molecular theory.
3. State the laws of kinetic friction.
4. State laws of static friction

EXPERIMENTS
1. Describe an experiment to determine the coefficient of kinetic friction between two solid
surfaces
2. Describe an experiment to determine the coefficient of static friction for an interface
between a rectangular block and a plane surface.
3. Describe briefly how you would measure the coefficient of static friction between two
solid surfaces.
4. Describe how the coefficient of static friction for an interface between a rectangular
block of wood and a plane surface can be determined
5. With the aid of a labeled diagram, describe how the coefficient of static friction for an
interface between a rectangular block of wood and a plane surface can be determined

EXPLANATIONS
1. Explain the laws of solid friction using the molecular theory.
55 | P a g e
2. Explain the origin of friction force between two solid surfaces in contact.
3. Using the molecular theory, explain the laws of friction between solid surfaces
4. Explain why a body can be able to slide downwards when its angle of inclination is
increased
5. Explain the tyres of a vehicle may burst driven on a rough murram road

CALCULATIONS ON FRICTION ON A SRAIGHT HORIZONTAL PLANE.


1. A block of mass 5.0kg resting on the floor is given a horizontal velocity of 5.0ms-1 and
comes to rest in a distance of 7.0m. Find the coefficient of kinetic friction between the
block and the floor.
2. A truck of mass 104 kg moving at 10ms-1 rolls into the other truck of mass 1 tone which
is stationary. The trucks stick together and skid along the horizontal surface. Show that
the distance through which the trucks skid is 5.0mm if the coefficient of kinetic friction is
0.2.
3. A bullet of mass 5g is fired horizontally into a 2.995kg of a wooden block resting on a
horizontal surface. The coefficient of sliding friction between the block and the surface
is0.2. The bullet remains embedded into the block. it is observed to have slid 25cm along
the surface .What was the velocity of the bullet before collision.
4. A truck ox mass 200kg moving a long a straight road at a speed of 96kmhr-1 is brought to
rest by a steady application of brakes in a distance of 80m. find the coefficient of kinetic
friction between the tyres and the road.

12.0kg

5. 6kg
2kg

The diagram above shows three masses connected by in extensible strings which
pass over friction less pulleys. The coefficient of friction between the bench and
the 12kg mass is 0.25. If the system is released from rest determine the
a. acceleration of 12kg mass
b. Tensions in each string.
6. A truck of mass 103 kg moving at 10ms-1 rams into a truck of mass 4x103kg which is
stationary. The trucks stick together and skid to stop a long a horizontal surface.
Calculate the distance through which the trucks skid if the coefficient of kinetic friction is
0.25.
7. A car of mass 1000kg moves a long a straight surface with a speed of 20ms-1.When
brakes are applied steadily the car comes forest after travelling 50m. Calculate the coefficient
of friction between the surface and the tyre.

56 | P a g e
8. 12.0kg

Bench 6.7kg
1.3kg

The diagram above shows three masses connected by inextensible strings which
pass over friction less pulley. The coefficient of friction between the bench
and the 12.0kg mass is 0.25. If the system is released from rest, determine
i. the acceleration of the 12.0kg mass
ii. the tension in each string.
9. 7.0kg

a. 4kg 2.5kg

Three blocks of mass 7kg, 4kg and 2.5kg respectively are arranged as shown above and the
system is released from rest. if the coefficient of sliding friction between 7.0kg and the plane
is 0.55. Calculate;

i. the acceleration of the system


ii. the tension in the strings
10. A 5g bullet is fired horizontally into a 2.995kg wooden block resting on a horizontal
surface. The coefficient of sliding friction between the block and the surface is0.2. The bullet
remains embedded in the block which is observed to slide 25cm along the surface before
coming forest. Find the velocity of the bullet before collision.
11. A tractor of mass 2.0x103kg is used to pull a car of mass 1.0x103kg to which its
connected by a chain whose mass is negligible. The tractor pulling steadily moves the car
from rest a long a horizontal road through a distance of 12.5m in 5.0s. The coefficient of
kinetic friction between the tyres of the tractor and the road is 0.4 while that between the
tyres of the car and the road is 0.2. Find
i. tractive pull exerted by the tractors ‘ engine
ii. power developed by the tractor s engine.
12. A bullet of mass 10g fired horizontally with a speed of 200ms-1 into a 4kg wood block at
rest on a horizontal surface. The bullet remains embedded in the block which is observed to
move a distance of 20m before stopping .calculate the coefficient of friction between the block
and the surface.
57 | P a g e
13. A truck of mass 104kg moving at 10ms-1 rams into a truck of mass 4.0x103kg which is
stationary. The trucks stick together and skid to a stop a long a horizontal surface. Calculate the
distance through which the trucks skid if the coefficient of kinetic friction is 0.25.
14. Three blocks A, B, and C of masses 5kg, 3kg and 2kg respectively are arranged as
shown.

C
B

The system is released from rest. if the coefficient of sliding friction between
1
block A and the table is
7
Calculate;
i. the acceleration of the system
ii. the tension in the strings.
15. A tractor of mass 2.0x103 kg is used to pull a car of mass 1.0x103kg to which its
connected by a chain whose mass is negligible. The tractor pulling steadily moves the car from
rest a long horizontal road through a distance of 12.5m in5.0 seconds. The coefficient of kinetic
friction between the tyres of the car and the road is 0.2. Find the
i. tractive pull exerted by the tractor’s engine
ii. power developed by the tractor’s engine
17. A bullet of mass 10g fired horizontally with a speed of 200ms-1 into a 4kg wooden block at
rest on a horizontal surface. The bullet remains embedded in the block which is observed to
move a distance of 20m before stopping. Calculate the coefficient of friction between the block
and the surface.
18. In the figure below, masses A and B are 6kg and 1kg respectively and the coefficient of
friction between body A and the table is 0.2

B
If the system is released from rest, find the frictional experienced by A and state
whether motion will occur or not.
19. If for the figure in question (18) above A and B are respectively 1kg and 500g and that the
1
coefficient of friction between A and table is and the system released from rest with A
3
3m away from the pulley and B, 2.5m above the floor, find

58 | P a g e
i. Initial acceleration of the system
ii. speed with which B hits the floor
iii. speed with which A hits the pulley.
20. Two cartons of masses 80kg and 120kg respectively are in contact and at rest on a
horizontal surface. A 700N horizontal force is exerted on 80kg carton. If the coefficient of
kinetic friction is 0.25.Calculate;
i. the acceleration of the system
ii. the force that each carton exerts on the other.
21. Two rectangular blocks of masses m1=4kg and m2=1kg are placed in contact on a horizontal
surface as shown below

F m1 m2

A constant force F=6.4g N where g- acceleration due to gravity is applied to the block of mass
m1. The coefficients of sliding friction between the horizontal surface and masses m1 and m2
are 0.25 and 0.40 respectively. If the blocks stay in contact with each other as they move
along the surface. Determine;

i. the acceleration of the blocks


ii. the time taken for the blocks to come to rest if the force F is suddenly removed after the
blocks moving through 4.0m.

22. A body of mass 1200kg collides with a stationary body of mass 1450kg. The two stick
together and move a distance of 6.2m before coming to rest. if the two bodies experiences a
frictional force equal to one –fifth of their total weight . Calculate the speed with which the first
body was moving when it hit the stationary one.

23. A car of mass 1000kg pulls a trailer of mass 3000kg a long a rough horizontal road . If the
frictional force on the ground is 500N, and the car exerts a driving force of 1000N.
Calculate

i. the acceleration of the car


ii. tension in the cable connecting the car and the trailer.

24. The diagram below shows three masses connected by in extensible strings which pass over
smooth pulleys. The coefficient between the table and the 12.0kg mass is 0.25

59 | P a g e
300
120kg
6.7kg

9.4kg table

If the system is released from rest, determine the;


i. acceleration of the 12.0kg mass
ii. tension in each string.

CALCULATIONS ON FRICTION ON AN INCLINED PLANE


1) A block of mass 3kg slides down a plane inclined at an angle of 300 to the horizontal.
6
Find the distance moved through 5.0s if the coefficient of friction is .
6 √3
2) A body of mass 2kg is moved up an inclined by means of a force parallel to the plane. if
the coefficient of kinetic friction is 0.3 and the plane has an inclination of 450. Find the
magnitude of that force if the acceleration of the body is 2.5ms-2.
3) Two blocks of masses 4kg and 8kg are connected by a rigid weight less rod and placed
on a plane inclined at angle of 300 to the horizontal. The block slide down the plane. if the
coefficient of sliding friction between the 4kg mass and the 8kg masses are 0.25 and 0.5
respectively calculate;
(i). the acceleration of the block
(ii). the tension in the rod.
4) A car of mass 3 tones climbs a truck which is inclined at angle of 200 to the
horizontal .The speed of the car at the bottom of the incline is 15ms-1. If the coefficient of
kinetic friction is 0.3 and the engine exerts a force of 600N, how far up the incline does
the car move in 10 second.
5) A block of wood of mass 150g rests on an inclined plane. If the coefficient of static
friction between the surfaces in contact is 0.3. find
(i). the greatest angle which the plane may be tilted without the block slipping
(ii). the force parallel to the plane necessary to prevent slipping when the angle of the
plane with horizontal is 300.

60 | P a g e
6) A car of mass 500kg moves from rest with the engine switched off down a road which is
inclined at an angle of 490 to the horizontal.
(i). calculate the normal reaction
(ii). if the coefficient of friction between the tyres and surface of the road is 0.32.
Find the acceleration.
7) A car of mass 1500kg rolls rest down a road inclined to the horizontal at angle of 350
through 50m. The car collides with another car of identical mass at the bottom of the
incline. if the vehicles interlock on collision and the coefficient of kinetic friction is 0.20,
find the common velocity of the vehicles.
8) A block of mass 6.0kg is projected with a velocity of 12ms-1up a rough plane inclined
at 450 to the horizontal if it travels 5.0m up the plane , find the frictional force.

9) A car of mass 1000kg climbs a track which is inclined at angle of 300 to the horizontal.
The speed of the car at the bottom of the incline is 0.3 and the engine is 36kmhr-1 if the
coefficient of kinetic is 0.3 and the engine exerts a force of 4000N, how far up the
incline does the car move in 10s.

10) In the figure below three forces 90N, 40N, and 24.5N act on a block placed on a smooth
plane inclined at angle of 600 to the horizontal. calculate
90N

24.5N

40N 600

(i). acceleration of the block


(ii).gain in kinetic energy 5.0s after moving from rest.
11) A body A of mass 13kg lies on a rough plane inclined at angle  to the horizontal where
5
Sin = .The coefficient of friction between the plane and the body is 0.1. from A
13
a light in extensible string passes up the line of greatest slope and over a smooth pulley
to a body B of mass 1kg . if the system is released from rest , find the
(i). acceleration of the system
(ii).tension in the string
(iii). distance moved by A after 3.0 seconds.

61 | P a g e
12) An old car of mass 2.0x103kg and tractive pull of 4.0x103N climbs a track which is
inclined at angle of 300 to the horizontal. The velocity of the car at the bottom after
1
incline is 30ms-1 and the coefficient of sliding friction is . calculate the
4
(i). distance travelled along the incline before the car comes to rest
(ii).time taken.
13) A body of mass 2kg is held at rest on a rough slopping roof at a distance of 2m from the
3
edge of the roof. The roof is inclined at an angle  =sin-1 ( ) to the horizontal and
5
the edge of the roof is 14m above the horizontal ground floor. The body is then given an
initial velocity of 1ms-1 down the line of greatest slope, it falls to the ground vertically
and rebounds to a height of 5m. If the coefficient of friction between the roof and the
body is 0.25. find the speed of the body
(i). at the edge of the roof
(ii).just after the first collision with the ground floor
(iii). if the collision lasts 0.02 seconds find the average force which the floor exerts on
the body.
14) A mass of 4kg lies on a rough plane which is inclined at 300 to the horizontal. A light
inelastic string has one end attached to this mass, passes up the line of greatest slope over
a smooth fixed pulley at the top of the plane and carries a freely hanging mass of 1kg at
1
its other end. The system is released from rest with coefficient of friction being
5
calculate
(i). acceleration of the system
(ii). tension in the string
(iii). Kinetic energy of the 1kg after 3.0s.
15) In the figure below b, the 80kg block is placed on the inclined plane and the block rests
against a spring. The system is released from rest.
80k
g

200

If the coefficient of static friction between the block and the incline is 0.25
(i). Determine the maximum and minimum values of the initial compression force in the
spring for which the block will not slip upon release
(ii).Calculate the magnitude and direction of the frictional force acting on the block if the
spring compression force is 200N.

62 | P a g e
16) A body of mass 0.1kg rests on a rough horizontal surface and has a light string inclined at
200 above the horizontal attached to it when the tension in the string is 5.0x10-1 N, the
body is found to be in limiting equilibrium. Find the coefficient of friction between the
body and the surface. What would the tension in the string have to be for the body to
accelerate along the surface at 1.5ms-2.
17) A force F acting parallel to and up a rough plane inclined at θ, is just sufficient to prevent
a body of mass m from sliding down the plane. A force 4F acting parallel to and up the
same rough plane. if µis the coefficient of friction between the mass and the plane , show
that 5µ=3tanθ .
18) A particle of mass 250g is released from rest at the top of a rough plane which is inclined
3
at sin-1 ( ) to the horizontal .The coefficient of friction between the particle and the
5
11
plane is and the plane is of length 2.5m. find whether the particle will slide down the
18
plane and if it does , find its speed on reaching the bottom.
19) A block of mass2.0kg is moved up an inclined plane of a force parallel to the plane. if the
coefficient of kinetic friction is 0.3, the inclination of the plane is 450 and acceleration up
the plane is 1.0ms-2. find the magnitude of the force.
20) In an experiment to determine the coefficient of static and kinetic friction between a
block of metal and a plane wooden surface a student placed the metal block on the
wood and tilted the wooden surface until the block began to move . He observed that this
happened at an angle of inclination of the plane with the horizontal of 200 and that the
block slide 100cm down the plane is 2.0 seconds. calculate the coefficients of static
and kinetic friction from these result.

21) Two blocks with masses of 4kg and 8kg are connected by a rigid mass less rod and
placed on a plane inclined to the horizontal as shown;

8kg

4kg

300

The block slide down the plane. If the coefficient of sliding friction between the 4kg
block and the plane is 0.25 and between the 8kg block and the plane is 0.50. Calculate;
(i). the acceleration of each block
(ii). the tension in the rod.
22) A loaded box is being pulled uniformly up a rough h inclined plane by means of a light
inextensible rope. The inclined plane makes an angle  with the horizontal

Rope

63 | P a g e
Box

Inclined plane

(i). Draw a force –diagram to indicate the forces on the box.


(ii).If the mass of the box is 200kg and the coefficient of sliding friction between the box and
the plane is 0.4 and  =250. what work must be done to pull the box a distance 10m up
the plane
(iii). if the rope breaks ,what will be its velocity after travelling 10m down the plane.
23) A car of mass 1000kg increases its speed from 10ms-1 to 20ms-1 whilst moving 500m up
1
a road inclined at an angle  to the horizontal where  = .This is a constant resistance
20
to motion of 300N. calculate the driving force exerted by the engine assuming that it is
constant.
24) A truck of mass 1500kg moves with uniform velocity of 5.0 ms-1 up a straight truck
inclined at an angle of 300 to the horizontal. The total frictional resistance to the motion
of the truck is 580N. Calculate the power developed by the engine. If the engine of the
truck cannot develop a power greater than 75kw, calculate the maximum speed attainable
by the truck.
25) A car of mass 1500kg and tractive pull 4000N climbs a track which is inclined at an
angle of 300 to the horizontal. The velocity the car at the bottom of the incline is
108kmhr-1 and the coefficient of sliding friction is 0.350 calculate
(i). the distance travelled along the incline before the comes to a halt
(ii). the time taken travelling along the incline before the car comes to a halt.
26) A car of mass 2 tones moves up a straight road of inclination 600 to the horizontal at a
constant speed of 60kmhr-1. calculate
(i). the engine tractive force if the friction along the road is equal to half the weight of
the train
(ii). the power output of the engine.
27) A truck of mass 100kg is travelling in a straight line on a level ground with an initial
speed of 20ms-1. After a distance of 30m, the ground slopes at an angle of 300 the
horizontal. The frictional resistance of the ground is 5N per kilogram. Find how far up
the slope the truck will travel before coming to rest.
5
28) A rough surface is inclined at tan-1 ( ) to the horizontal. A body of mass 130kg is
12
pulled at a uniform speed a distance 50m up the surface by a force acting a long a line of

64 | P a g e
2
greatest slope. The coefficient of friction between the body and the surface is . Find the
7
work done.
29) An old car of mass 1.5x10 3kg and tractive pull of 3.5x103N climbs a track which is
inclined at an angle of 300 to the horizontal. The velocity of the car at the bottom of the
incline is 20ms-1 and the coefficient of sliding friction is 0.25 calculate;
(i). the distance travelled along the incline before the car comes to a halt
(ii).the time taken travelling along the incline before the car comes to rest.

TOPIC: 8 STATICS
DEFINITIONS;
1. Define the following terms; moment of a force,acouple,translational equilibrium,rotational
equilibrium,stability,centre of gravity ,Centre of mass, unstable ,stable, neutral
equilibrium,uniform body.

LAWS, PRINCIPLES AND THEOREM.


1. State the principle of moments.

DERIVATIONS AND RELATIONS


1) Write down the expression for the work done by a couple.
2) A uniform ladder of length 2L and weight W rest on a rough ground and upper end on a
smooth wall at 300 to the vertical as shown

R
W

A man of weight wishes to climb the ladder. Explain why it is safer for the man to ascend
when another person of weight, w, is standing on the foot of the ladder.

3) Show that the average kinetic energy of translation of a molecule is proportional to the
absolute temperature of the gas.
65 | P a g e
FACTORS, MERITS, DEMERITS AND CONDITIONS.
1. State the conditions for equilibrium of a rigid body under the action of coplanar forces
2. state the factors which affect the magnitude of moment of a couple
3. State the factors on which the magnitude of moment of a force depends
4. State any four properties of a couple of forces
5. Give two applications of a couple of forces
6. How can the moment of any force be made larger
7. Give two examples where the moment of a force is increased considerably in practical
life
8. The handle of a door is near its outside edge. Explain why it is so.
9. State two ways of increasing the stability of a bus during manufacturing
EXPERIMENTS
1. Describe an experiment to determine the Centre of gravity of a plane sheet of material
having an irregular
2. Describe an experiment to find the Centre of gravity of a flat irregular piece of a card
board
3. Describe an experiment to determine the mass of a meter rule.
4. Describe an experiment to verify the principle of moments.

CALCULATIONS ON STATICS
1) A mass of 5.0kg is suspended from the end A of a uniform beam of mass 1.0kg and
length 1.0m. The end B of the beam is hinged in a wall. The beam is kept horizontal by a
rope attached to A and to point C in the wall at a height 0.75m above B.
(i). Draw a sketch diagram to show the forces acting on beam
(ii).Calculate the tension in the rope
(iii). What is the force exerted by the hinge on beam.
2) A 3.0m long ladder rests at an angle of 600 to the horizontal against a smooth vertical
wall on a rough ground. The ladder weighs 5kg and its Centre of gravity is one third from
the bottom of the ladder.
(i). Draw a sketch diagram to show the forces acting on the ladder
(ii).Find the reaction of the ground on the ladder.
3) A uniform ladder of mass 40kg and length 5m, rest with its upper end against a smooth
vertical wall and with its lower end at 3m from the wall on a rough ground. Find the
magnitude and direction of the force exerted at the bottom of the ladder.

66 | P a g e
4) One end of a plank of length 4m and weight 100N is hinged to a vertical wall. An
inelastic rope tied to the other end of the plank is fixed at a point 4m above the hinge so
that the plank is horizontal. A weight of 300N is suspended from the plank at a distance
of 3m from the hinge; find
(i). tension in the rope
(ii).the reaction of the wall on the plank.
5) A sphere of weight 20N and radius 15cm rests against a smooth vertical wall. The sphere
is supported in this position by a string of length 10cm attached to a point on the sphere
and to a point on the wall as shown below
10cm

15cm
Sphere

(i). copy the diagram and show the forces acting on the sphere
(ii).Calculate the reaction the sphere due to the wall
(iii). Find the tension in the string.
6) An oil drum of diameter 75cm and mass 90kg rests against as shown below

Stone
75cm
Ground

15cm

Find the least horizontal force applied through the center of the drum, which will cause
the drum to roll up the stone of height b15cm.

7) A mass of 20kg is hanged from the mid –point p of a wire as shown below, calculate the
tension in the wire

Support Support

200 200

1100 1100

(20g)N

67 | P a g e
8) A uniform ladder AB, 12m long and of weight 500N rests in equilibrium with end B
against a friction less wall and A on the horizontal ground. if the ladder makes an angle
of 600 to the horizontal. Calculate the reaction at A.
9) A uniform beam AB of mass 40kg is hang to a wall at end A and held horizontally by a
wire joining B to a point C which is on the wall vertically above A. if a stone of mass
10kg is hinged at B and given that ABC is 300, find the force
(i). In the wire connecting B to C
(ii).exerted by the beam on to the hinge.
10) A uniform ladder 3m long and of mass 17.5kg rest with its upper end A against a
smooth vertical wall and the lower end B on a rough ground . What must be the
coefficient of friction between the ground and the ladder be for the ladder to be inclined
at 600 to the horizontal without slipping. what is the reaction at B.
11) A beam of mass 20kg and length 2.4m is hinged at A which is a point on a vertical
wall. The beam is maintained in a horizontal position by means of a chain attached to a
point in a wall 1.5m vertically above A. if the beam carries a load of 10kg at a point 1.8m
from point A calculate;
(i). tension in the chain
(ii).magnitude and direction of the reaction at A.
12) A uniform rod AB of length 4x and weight 30N is smoothly hinged at its upper end A,
The rod is held at 300 to the horizontal by a string which is at 900 to the rod, and attached
to it at C where AC=3X, Find the;
(i). tension in the string
(ii).vertical component of the reaction at A
(iii). horizontal component of the reaction at B
13) A sphere of weight 40N and radius 30cm rests against a smooth vertical wall. The sphere
is suspended in this position by a string of length 20cm attached to a point on the sphere
and to a point on the string
(i). tension in the string
(ii).reaction due to the wall.
14) A uniform beam AB, 3.0m long and weighing 6.0kg is hinged to a wall at A, and is held
horizontally by a rope attached to B and joined to a point C in the wall, 4.0m vertically
above A. Fin the tension in the rope and reaction R of the hinge.
15) A uniform ladder is 13m long weighing 50Nrests with its upper end against a smooth
wall and its lower end on the rough ground at a point 5m from the wall. What must be
least coefficient of friction between the ground and the ladder to prevent slipping.
16) A uniform ladder 40m long of mass 25kg rests with its upper end against a smooth
vertical wall and with its lower end on the ground. What must be the least coefficient of
friction between the ground and the ladder for it to be inclined at 600 with the horizontal
without slipping. find the resultant reaction at the ground.

68 | P a g e
17) A uniform ladder 5.0m long having mass of 40g rests with its upper end against a
smooth vertical wall and with its lower end 3m from the wall on the rough ground. find
the magnitude and direction of the force exerted at the bottom of the ladder.
18) A uniform ladder 4m long, mass 50kg rests with its upper end against a smooth
vertical wall and with its lower end on a rough ground. What must be the least
coefficient of friction between the ground and the ladder for it to be inclined at 600 to the
horizontal without slipping.
19) A uniform plank of mass 5kg and length 3m has one end hinged to a vertical tree trunk.
The plank is kept horizontal by a rope tied to the other end, a fixed 3m above the hinge if
a monkey of mass 10kg suspended at a point 2.5m from the hinge
(i). draw a sketch diagram to show the forces acting on the plank
(ii).calculate the tension in the plank
(iii). What is the reaction of the trunk to the plank.
20) A sign post in the form of a uniform rectangular board 1.0m wide and0.6m high weighs
200N.The sign post is hung from a horizontal pole, AC, 2m long that weighs 300N, and
wire CD which makes an angle of 300 with the horizontal as shown

300 300 C
A B

0.6m

Find; 1.0m

(i). the tension in the wire


(ii).the horizontal and vertical component of the before exerted by the wall at A.
21) A uniform rod AB of mass 10kg is smoothly hinged at A and rests in a vertical plane
with the end B against a smooth vertical wall. if the rod makes angle of 400 with the wall
find the thrust on the wall and the magnitude and direction of the reaction.
22) A uniform ladder 5m long and weight 400N rests against a friction less wall with its
lower end 3m from the wall. The coefficient of static friction of the ladder with the
ground is 0.40. A man of weight 800N stands on the ladder at its mid-point
(i). find the frictional force
(ii).What is the limiting frictional force between the ground and the ladder.

69 | P a g e
23) A uniform plank AB which is 6m long and has a weight of 300N is suspended
horizontally by two vertical ropes at A and B. A weight of 150N rests on the plank at C
where AC =2m .Find the tension in each rope.
24) A uniform ladder which is 5m long and has a mass of 20kg leans with its upper end
against a smooth vertical wall and its lower end on rough ground. The bottom of the
ladder is 3m from the wall. calculate the frictional force between the ladder and the
ground.
25) A particle whose weight is 50.0N is suspended by a light string which is at 350 to the
vertical under the action of a horizontal force F find;
(i). the tension in the string
(ii).force F.
26) A particle of weight, w, rests on a smooth plane which is inclined at 400 to the horizontal.
The particle is prevented from slipping by a force of 50.0N acting parallel to the plane
and up a line of greatest slope. find
(i). the value of w
(ii).the reaction due to the plane.
27) A sphere of weight 40N and radius 30cm rests against a smooth vertical wall. The sphere
is supported in this position by a string of length 20cm attached to a point on the sphere
and to a point on the wall. find
(i). the tension in the string
(ii).the reaction due to the wall.
28) AB is a uniform rod of length 1.4m. it is pivoted at C, where AC=0.5m, and rests in
horizontal equilibrium when weight of 16N and 8N are applied at A and B respectively
calculate;
(i). the weight of the rod
(ii).the magnitude of the reaction at the pivot.
29) A uniform rod AB of length 4a and weight w is smoothly hinged at its upper end A. The
rod is held at 300 to the horizontal by a string which is at 900 to the rod and attached to it
at C where AC=3a, find;
(i). the tension in the string
(ii).the vertical component of the reaction at A
(iii). the horizontal component of the reaction at A.
30) A non- uniform rod AB of weight 40N and length 20cm is supported by a pivot at C
where AC =14cm. The rod rests in horizontal equilibrium when a weight of 30N is
attached to it at B. Find the distance of the Centre of gravity of the rod from A.
31) A uniform beam AB of length 5m and which weighs 200N is supported horizontally by
two vertical ropes x and y at A and B respectively . calculate the tensions in the ropes if a
man weighing 700N stands on the beam at a distance of 2m from
32) A uniform ladder which is 10m long and weight 300N leans with its upper end against a
smooth vertical wall and its lower end on rough horizontal ground. The bottom of ladder

70 | P a g e
is6m from the base of the wall. A man weighing 700N stands on the ladder at a point 6m
above the ground. calculate the magnitudes and directions of the forces exerted on;
(i). the wall
(ii).the ground.
TOPIC 9: WORK, POWER AND ENERGY
DEFINITIONS

1. Define the following terms;work,ajoule,power,awatt,energy,kinetic energy,potentential


energy,conseavation of energy, mechanical energy,dissipatve forces, conservative forces, non-
conservative forces, elastic twisting ,internal energy.

LAWS, PRINCIPLES AND THEOREM

1. State law of conservation of energy.


2. State the principle of conservation of mechanical energy
3. State the work – energy theorem

DERIVATIONS AND RELATIONS.


1) Show that a stone thrown vertically upwards obeys the principle of conservation
of mechanical energy throughout its upward motion.

2) (i) A wind turbine made of a blade of radius r is driven by a wind of speed V. if µ


is the density of air derive an expression for the maximum power, P which can be
developed by the turbine in terms of µ, r and V

(ii) Explain why the power attained is less than the maximum values in (i) above.

3) Explain how conservation of energy applies to an object falling from rest in a


vacuum.

4) Two pendula of equal length, L have bobs P and Q are of masses 3M and M
respectively. The pendula are bung with bobs in contact as shown.

71 | P a g e
P Q

The bob P is displace such that the string makes an angle θ of with the vertical and
released.
If P makes a perfectly inelastic collision with Q. find the height to which Q rises.
5) A car of total mass m1 is driven at maximum power with constant velocity v1 up a
hill of constant slope. if m is the coefficient of friction between the tyres and the
road. find an expression for the effective power of the car’s engine in terms of v1,
m1, 1, g and θ the angle to the horizontal of the hills slope. neglect air resistance.
6) Describe the energy changes which occur when;
(i). A ball is thrown upwards in air
(ii).A loud speed is vibrating.
7) Sketch a graph to show the relationship between kinetic energy and height above
the ground in a projectile.
8) Prove the work energy theorem for a body moving with constant acceleration.
9) Show that the mechanical energy of a falling ball is conserved.
10) A rough surface is inclined at angle of θ to the horizontal a body of mass m is
pulled at a uniform speed a distance x up the surface by a force acting a long a
line of greatest slope. The coefficient of friction between the body and the plane is
. If the only resistances to motion are those due to gravity and friction show that
the total work done on the body is mgx (Sinθ +  cosθ) where g acceleration due
to gravity.
11) An object is projected with a velocity U at a height H, vertically upwards
(i). Explain how its Kinetic energy vanes with height above the ground.
(ii).Sketch graph to show the relationship in (i) above.

12) Two pendula of equal length 4m have bobs A and B of masses 3m and m
respectively. The pendula are hung with the bobs in contact as shown below.

72 | P a g e A B
Bob A is displaced such that the string on to which it’s attached makes an angle 60 0 with
the vertical and released. If A makes a perfectly inelastic collision with B, find the height
to which B rises.

13) A light elastic string of natural length L1 metres has one end fixed to hang
vertically. To the free end of the string is attached to a particle of mass m kg.
When the particle hangs freely under gravity, the length of the string is L2 meters.
The particle is then held at the point of suspension and released to fall.
(i). Derive an expression for the elastic constant of the string in terms of L1, L2
and m
(ii).Use the principle of conservation of energy to show that the speed of the
particle at point 2L1 metres below the point of suspension is given by.

V=
√ ( 4 L2−5 L1 ) gL1
L2−L1
14) When is work done positive or negative.

15) A bullet of mass 15g is fired from a gun placed horizontally with a velocity of
250ms-1. If it hits and become embedded in a block of wood of mass 3kg
suspended freely by two vertical strings each of length 1.0m as shown.

Θ θ

250ms-1 Block h

73 | P a g e
Bullet

Calculate the;

(i). Velocity of the bullet after collision


(ii). The height, h
(iii). The maximum angle θ through which the string is deflected from the
vertical.
16) A bullet of mass M1, is fired with a velocity V into a massive wood of mass M2
which is suspended by parallel chords originally at rest

m1 m2

When the bullet is embedded into the block of wood and the bullet swings up to a
maximum height h above show that;

V= ( m1+ m2
m1 )
( 2 gh )
1
2

17) A bullet of mass 10.0g is fired at close range into a block of mass 4.99kg
suspended from a rigid support by an inelastic string and becomes embedded in
the block. The block rises through a height of 2.50cm before momentarily coming
to rest. Calculate the initial speed of the bullet.
18) A bullet of mass 0.01kg is fired into a block of wood of mass 0.3kg lying on a
smooth surface. After impact the wood moves with a speed of 10ms-1. Find the;
(i). Velocity of the bullet before collision
(ii).Total kinetic energy before and after collision deduces the loss in kinetic
energy of the system.
19) A simple pendulum of length 1m has a bob of mass 100g. It is displaced from its
mean position A to a position B so that the sting makes an angle of 450 with the
vertical. Calculate the;

74 | P a g e
(i). The maximum potential energy of the bob
(ii).Velocity of the bob when the string makes angle of 300 with the vertical.

FACTORS, MERITS, DEMERITS, EXAMPLES AND CONDITIONS

1. State a factor on which the angle of swing a simple pendulum depends.

EXPLANATIONS

1. Explain why more energy is required to push a wheel barrow uphill than on a
level ground.
2. Explain how conservation of energy applies to an object falling from rest in a
vacuum.
3. Explain whether a person carrying a bucket of water does any work on the bucket
while walking on a level road.
4. Explain briefly what is meant by internal energy of a substance
5. Explain why it is easier to pull down a wheel barrow than that to push it
6. Explain how it is possible to walk.

CALCULATIONS
1) A body is released from a rough plane inclined at 300 to the horizontal and slides a
distance of 20m along the plane. If the coefficient of friction of the plane is 0.25,
calculate the velocity with which it reaches the bottom of the plane.
2) A block of mass 6.0kg is projected with a velocity of 12ms-1 up a rough plane inclined at
450 to the horizontal. If it travels 5.0m up the plane. Find the frictional force.
3) A mass of 500g is released from rest so that it falls vertically through a distance of 20cm
on to a scale pan of negligible mass, hung from a spring of force constant 100Nm-1. Find
the position of the scale pan when it first comes to rest.

4) A car of mass 100kg moving along a straight road with a speed of 72kmh-1 is brought to
rest by a steady application of the brakes in a distance of 50m. Find the coefficient of
kinetic friction between the tyre and the road.
5) A bullet of mass 10g is fired at a short range into a block resting on a smooth horizontal
surface and attached to a spring of force constant 100Nm-1. The bullet remains embedded
in the block while the spring is compressed by a distance of 5.0m. find the elastic energy
of the compressed spring and the speed of the bullet just before collision with the block.
6) Calculate the work down when a force of 5N acts on a block of mass 2kg which moves
for 3m.
7) A car of mass 1000kg moving on a rough horizontal surface is brought to rest in a
distance of 10m by a steady application of the brakes. If the coefficient of slide friction
between the surface and tyre is 0.4. Calculate the work done by the frictional force.

75 | P a g e
8) A block of mass 5.0kg is reached from rest on a smooth plane inclined at an angle of 300
to the horizontal and slides through 10m. Find the work done by gravitational force.
9) A 5kg mass is released from rest at the top of a smooth inclined plane. The angle of
elevation of the incline is 300 what is the speed of the mass when it passes a point 10m
down the incline.
10) Sand is deposited at a uniform rate of 20kgs-1 and with negligible K.e onto an empty
conveyor belt moving horizontally at a constant speed of 10metres per minute. Calculate;
(i). The force required to maintain the constant velocity
(ii). Power required to maintain the constant velocity
(iii). The rate of change of K.e of the moving sand. Why are the two quantities un
equal.
11) A car of mass 750kg starts from rest on a level road and is uniformly accelerated for 10s
until its speed is 18kmhr-1. If the resistance to motion is 49.0N. find the power of the car
10s after the start. What is the average power during the 10 s.
12) A car of mass 100kg travelling at 20ms-1 on a horizontal road is brought to rest by the
action of its brakes in a distance of 25m when time is 2.0s. Find the average retarding
force and the power developed by the engine.
13) A car of mass 200kg travelling at 10ms-1 on horizontal surface is brought to rest in a
distance of 12.5m by the action of its brakes. Calculate the average retarding force. What
power must the engine develop in order to take the car up in an incline in 20 at a constant
speed of 10ms-1. If the frictional resistance is 200N.
14) A body of mass 0.1kg is attached to the free end of a spring of force constant 15Nm-1
whose other end is fixed to a wall. The mass rests on a smooth horizontal surface as
shown.

0.1kg

The mass is pulled through a distance of 0.2m from equilibrium position and the
released. Find the speed of the mass as it goes through the equilibrium position.
15) An ideal mass less spring can be compressed 2.0cm by a force of 200N. The same spring
is placed at the bottom of a friction less inclined plane which makes an angle of 450 with
the horizontal as shown.

76 | P a g e
450

A mass M = 2.0kg is released from rest at the top of the incline and is brought to rest
momentarily after compressing the spring by 3.0cm. find;
(i). The elastic potential energy stored in the string.
(ii). The distance through which the mass slides before it reaches the spring.
(iii). The speed of the mass just before it reaches the spring
(iv). Time taken by the mass to reach the spring.
16) A block of mass 0.1kg is released from rest at the top of a smooth plan inclined at 300 to
the horizontal. The block compresses a spring S at the bottom of the incline by 5.0cm
before it is brought momentarily to rest. If the force constant of the spring is 100Nm-1.
Calculate;
(i). Speed of the block just before it reaches the spring
(ii).The distance travelled by the block before it reaches the spring.
17) A force of 15N is applied to the body of mass 3kg initially at rest on a smooth horizontal
surface for a time of 3.0s. calculate:
(i). The change in the Kinetic energy
(ii).The work done by the force .
18) A train of mass 2.0 x 105kg moves at a constant speed of 72kmhr-1 up a straight incline
against frictional force of 1.28 x 104 N the incline is such that the angle of the incline sin
1
= it travels a distance of 100m along the machine. Calculate;
100
(i). The rate of increase per second of the potential energy
(ii).The necessary power developed.
19) A car of mass 1000kg travelling at 72kmhr-1 by action of brakes and frictional forces.
Find;
(i). The average stopping force
(ii).The time taken to stop the car.

20) For a smooth surface inclined at tan-1 () 3


4
to the horizontal a body of mass 50kg lies on it

and is pulled of a uniform speed a distance of 5m up a line of greatest slope against


resistance is totaling 50N. find;
(i). Work done against gravity
(ii).Work done against resistance.
21) A body of mass 2kg falls vertically passing through 2 points A and B with speeds 1ms-1
and 4ms-1 respectively. The resistance against which the body falls is 9.6N. find the
distance AB.
22) A block of mass 2.0kg is placed on a plane inclined at 300 to the horizontal. The block is
then released from rest. The coefficient of sliding friction for the two surfaces is 0.25.
calculate;
(i). The distance moved by the block after 2.0s

77 | P a g e
(ii).The K.e of the block 2.0s, after the block is released.
23) A 10.0kg block is released from rest on a rough plane inclined at 300 to the horizontal
from a point 0.8m from the base. The coefficient of kinetic friction between the block and
the inclined plane is 0.2.
(i). With what speed will the block reach the bottom of the incline
(ii).If the block is projected up the incline with a speed of 10.0ms-1 how far will the
block travel up the incline.
24) A block of mass 2.0kg is released from rest at a point A on a track which is one of.
Quadrant of a circle of radius 1 m as shown.
A

1.0m B C

3.0m

The block slides down the track and reaches point B with a velocity of 4ms-1. From
point B it slides on a level surface through a distance of 3m to point C where it comes
to rest.

(i). What is the coefficient of sliding friction on the horizontal surface


(ii).How much work was done against friction as the body slides down the plane
from A to B.

25) A particle slides along a track with elevated ends and a flat central part as shown.

A D

B C

The flat part has a length L equal to 2.0m. The curved positions of the track are frictionless. For
the flat part, the coefficient of kinetic friction is 0.2. the particle is released at A which is a height

78 | P a g e
h = 1.0m above the flat part of the track. Determine the position of the particle when it finally
comes to rest.
26) A car of mass 1000kg moves with uniform velocity of 10.0ms -1 up a straight track inclined at an
angle of 200 to the horizontal. The total frictional resistance to motion of the car is 248N;
calculate the power developed in the engine.
27) A 1.0kg block of wood is attached to a spring of force constant 200Nm -1 and rest on a smooth
surface as shown.
K = 200Nm-1

20g 1kg

A 20g bullet as shown into the block and the spring compresses 13.3cm. if the bullet
remains embedded in the block.
(i). Find the velocity of the bullet before the collision
(ii).What fraction of the original mechanical energy is lost in the collision.
28) A car of mass 500kg accelerates steadily from rest down a steady incline at 200 to the
horizontal then on level road. If the car takes 15s to travel 100m from rest to the bottom
of the incline, find
(i). The speed and K.e at the bottom
(ii). Loss in potential energy
(iii). Work done against friction
(iv). Average frictional force.
29) A block of mass 6.0kg is projected with a velocity of 12ms-1 up a rough plane inclined at
450 to the horizontal. If it travels 5.0m up the plane, find the frictional force.
30) A car of mass 100kg moves along a straight surface with a speed of 20ms-1. When brakes
are applied steadily, the car comes to rest after travelling 50m. Calculate the coefficient
of friction between the surface and the tyre.
31) A body slides down a rough plane inclined at 300 to the horizontal if the coefficient of
friction between the body and the plane is 0.4. Find the velocity after it has travelled 6m
along the plane.
32) A pump discharges water through a nozzle of diameter 4.5cm with a speed of 62ms-1 into
a tank 16m above the intake.
(i). Calculate the work done personal by the pump in raising the water if the pump is
ideal
(ii). Find the power wasted if the efficiency of the pump is 73%
(iii). Account for the power lost in (ii).
33) A mass of 20g falls from a height of 2.0m to the top of spring of force constant 100Nm-1.
Calculate the resulting compression in the spring if;
(i). The mass sticks on top of the spring

79 | P a g e
(ii).The mass bounces off the spring with a speed of 2.0ms-1.
34) A block of mass 2.0kg is dropped from a height of 0.8m on a scale pan resting on a
spring of force constant 1900Nm-1. Find the maximum distance the spring will be
compressed.
35) A power of 10kw is required to drive a 1200kg car at 60kmhr-1 on a level road. What is
the total resistance on the vehicle. What power will be necessary to drive the car at
60kmhr-1 up a plane inclined at 300 to the horizontal?
36) A car of mass 2 tones move up a straight road of inclination 600 to the horizontal at a
constant speed of 60kmhr-1. Find;
(i). The engine tractive force if the friction along the road is equal to half the weight
of the train.
(ii).The power output of the engine.
37) (i) A car of mass 1000kg travelling at 20ms-1 on a horizontal is brought to rest by the
action of its brakes in the distance of 25m. Find the average retarding force.
(ii) If the same car travel up an incline of 1 in 20 at a constant speed of 20ms-1. What
power does the engine develop if the frictional resistance is 100N.
38) A vehicle of mass 200kg travelling at 10ms-1 on horizontal surface is brought to rest in a
distance of 12.5m by the action of its brakes. Calculate the average retarding force what
up an incline in 1 in 10 at a constant speed of 10ms-1. If the frictional resistance is 200N.
39) A train of mass 500tonnes has a maximum speed of 90kmhr-1 while moving up in an

incline of sin-1( ) 1
50
against frictional resistance of 1.0 x 105N. Find the maximum

power of engine.
40) A car of mass 750kg resting on a level road is uniformly accelerated for 10s. Until the
speed is 18kmhr-1. If the resistance to motion is 5gN. Find the power developed in this
time.
41) A body of mass 10kg is pulled a distance of 20m across a horizontal surface against
resistance totaling 40N. if the body moves with a uniform velocity. Find the work done
against resistance

42) A car of mass 800kg working at a constant rate of 15000w ascends a hill of sin ( 951 )
against a constant resistance as a motion of 420N. find
(i). The acceleration of the car up the hill when travelling with a speed of 10ms-1
(ii). The maximum speed of the car up hill
(iii). The acceleration of the car down the same hill at the instant when its speed is
20ms-1, resistance being constant.
43) A cycle of mass 200kg traveling at 144kmhr-1 on a horizontal road is brought to rest in a
distance of 80m by action of brakes and frictional force. Find the;
(i). Average stopping force

80 | P a g e
(ii).Time taken to stop the cycle.
44) A body of mass 6kg, initially moving with a speed of 12ms-1 experiences a constant
retarding force of 10N for 3s. find the kinetic energy of the body at the end of this time.
45) A block of mass 0.2kg is released from rest at the top of a smooth plane inclined at 300 to
the horizontal. The block compresses a spring placed at the bottom of the plane by 100m
before momentarily brought to rest. If the force constant of the spring is 20Nm-1.
Determine the distance the block has travelled down the incline before it comes to rest
and its speed just before it reaches the spring.
46) A 12kg block is released from rest from a rough plane inclined at 300 to the horizontal
from a point 0.9m from the base. The coefficient of kinetic friction between the block and
the inclined plane is 0.25.
(i). With what speed will the block reach the bottom of the incline
(ii).If the block is projected up the incline with a speed of 20ms-1. How far up the
incline will the block travel.
47) A car of mass 800kg and moving at 30ms-1 along a horizontal road is brought to rest by a
constant retarding force of 5000N. Calculate the distance the car moves whilst coming to
rest.
48) A car of mass 1.0 x 103kg increases its speed from 10ms-1 to 20ms-1 whilst moving

0.500m up a road inclined at an angle of  to the horizontal where sin  = ( )1


20
. There is

a constant resistance to motion of 300N. find the driving force exerted by the engine
assuming that it is constant.
49) A car of mass 1.2 x 103kg moves 300m up a road which is inclined to the horizontal at an
1
angle of  where sin  = . By how much does the gravitational potential energy of the
15
car increases.
50) A car of mass 800kg moving at 20ms-1 is brought to rest by the application of the brakes
in a distance of 100m. Calculate the work done by the brakes and the force they exert
assuming that it is constant and that there is no other resistance to motion.
51) The speed of a dog –sleigh of mass 80kg and moving along horizontal ground is
increased from 3.0ms-1 to 9.0ms-1 over a distance of 90m. find;
(i). The increase in the kinetic energy of the sleigh
(ii).The force exerted on the sleigh by the dogs, assuming that it constant and that
there is no resistance to motion.
52) A simple pendulum consisting of a small heavy bob attached to a light string of length
40cm is released from rest with the string at 600 to the down ward vertical. Find the speed
of the pendulum bob as it passes through its lower point.
53) A car of mass 900kg accelerates from rest to a speed of 20ms-1 whilst moving 80m along
a horizontal road. Find the tractive force exerted by the engine, assuming that it is
constant and that there is a constant resistance to motion is 250N.

81 | P a g e
54) A child of mass 20kg starts from rest at the top of a playground slide and reaches the
bottom with a speed of 5.0ms-1. The slide is 5.0m long and there is a difference in height
of 1.6m between the top and the bottom. Find ;
(i). The work done against friction.
(ii).The average frictional force.
55) A pump raises water through a height of 3.0m at a rate of 300kg per minute and delivers
it with a height of 8.0ms-1. Calculate the power output of the pump. (Assume g = 10ms-2).
56) A pump with a power output of 600W raises water from a lake through a height of 3.0m
and delivers it will a velocity of 6.0ms-1 what mass of water is removed from the latke in
one minute.
57) A man of mass 70kg rides a bicycle of mass 115kg at a steady speed of 4.0ms-1 up a road
which rises 1.0m for every 20m of its length. What power is the cyclist developing if
there is a constant resistance to motion is 20N.
58) A stone of mass 20g was released from rest at a point 5m above the surface of water in
the container. The stone made a perfectly inelastic collision with an object of mass 5g
which as on the surface of the water and they hit the bottom of the container with kinetic
energy of 3.2J against water resistance of 0.21N. Calculate the depth of water.

CALCULATIONS ON CONSERVATION OF ENERGY AND IMPULSES.


1) A ball of mass 0.5kg is allowed to drop from rest from a point a distance of 5.0m above
the horizontal concrete floor. When the ball first hits the floor, it rebounds to a height of
3.0m.
(i). What is the speed of the ball just after the first collision with the floor.
(ii).If the collision lasts 0.01s, find the average force which the floor exerts on the
ball.
2) A ball of mass 250g is released to fall from a point 20m above the ground and rebounds
to a height of 14m. If the ball is in contact with the ground for 0.015s. Calculate the
average force exerted to the ground.
3) A bullet of mass 40g is fired from a gun at 200ms-1 and hits a block of wood of mass 2kg
which is suspended by a light vertical string 2m long. If the bullet gets imbedded in the
wooden block.
(i). Calculate the maximum angle that the string makes with vertical.

4) A bullet of mass 10g travelling horizontally at a speed of 100ms-1 strikes a block of


wood of mass 990g suspended by a alight vertical string and is embedded in the block
which subsequently swings freely. Find the;
(i). Vertical height through which the block rises
(ii).Kinetic energy lost by the bullet.
5) A bullet of mass 10g travelling horizontally at a speed of 1.0 x 102ms-1 embeds its self in
the block of wood of mass 9.9×102 g suspended by string so that it can swing freely find;
(i). The vertical height through which the block rises

82 | P a g e
(ii).How much of the bullets energy becomes the internal energy.
6) Ball A is released from the point shown in the diagram below.
A

1.8m

Frictionless wire

The ball slides along the frictionless wires and collides with ball B. if the collision is
elastic, find how high ball B will rise after collision.
If 2MA = MB where MA and MB are the masses of A and B respectively.
7) A block of mass 0.5kg is released from rest so that it falls vertically through a distance of
20cm on a scale P on of negligible mass, hang from a spring of force constant 100Nm-1.
Find the position of the scale P on when it first comes to rest.
8) A steel ball of mass (3m) kg is suspended by a light in extensible string 2.5m long, the
mass is held out at 600 from the vertical as shown.

2.5m

600

A B

2m

A second steel ball of mass m kg rests just at the edge of the table which is 2m high.
Ball A is released and strikes ball B head on in a perfectly elastic collision. Find;
(i). How fast A is moving just before it strikes B.
(ii).Speeds of A and B just after collision
(iii). Height of A with respect to the ground when it momentarily comes to rest.
9) (a) A wooden block of mass 3.98kg rests on a smooth horizontal surface. The block is
attached to a light spring of force constant 100Nm-1 whose other end is fixed. A bullet of
mass 0.020kg fired into the block embeds its self-there and the spring is compressed by
0.4m. Find the velocity of the bullet just before it hits the block.
(b) Suppose the bullet in(a) above was fired into a wooden block of mass 3.98kg placed
on a rough horizontal surface and the block was not attached to the spring through what

83 | P a g e
distance would the composite mass have moved before coming to rest. If the coefficient
of kinetic friction between the block and the surface is 0.3.
10) A bullet of mass 10.0g is fired at close range into a block of mass 9.99kg suspended from
a rigid support by an inelastic string and becomes embedded in the block. The block rises
through a height of 2.0m before momentarily coming to rest. Calculate the initial speed of
the bullet.
11) A ball of mass 3kg slides down a friction less surface and then strikes a stationary 5kg
block on a horizontal surface as shown below.

Ball

1.5m Smooth surface

5kg

Block
The coefficient of Kinetic friction between the block and table is 0.2. If the ball and the
block stick together, how far do they slide before coming to rest.
12) A bullet of mass 10g is fired at short range into a block of wood of mass 990g resting on
a smooth horizontal surface and attached to a spring of force constant 100Nm-1. The
bullet remains embedded in the block while the spring is compressed by a distance of
5.0cm. Find the elastic energy of the compressed spring and the speed of the bullet just
before collision with the block.
13) A bullet of mass 40g is fired from a gun at 200ms-1 and hits a block of wood of mass 2kg
which is suspended by a light vertical string 2m long. If the bullet gets embedded in the
wooden block. Calculate the maximum angle the string makes with the vertical. State a
factor on which the angle of swing depends.
14) A bullet of mass 10g travelling horizontally at a speed of 100ms-1 strikes a block of wood
of mass 900g suspended by a light vertical string and is embedded in the block which
subsequently swings freely. Find the;
(i). Vertical height through which the block rises
(ii).Kinetic energy lost by the bullet.
15) A tennis ball of mass 100g is dropped from rest through a height of 10m above a concrete
floor. On hitting the floor, the ball rebounds to a height of 4.5m. Find the impulse of
force exerted by the floor on the ball.
16) A bullet of mass 0.012kg and horizontal speed 70ms-1 strikes a block of wood of mass
0.4kg and instantly comes to rest with respect to the block. The block is suspended from

84 | P a g e
the ceiling by means of a thin string.
(i). Calculate the height to which the block rises
(ii).Estimate the amount of heat produced in the block.
17) A bullet of mass 10g is fired at short range into a block of wood of mass 990g resting on
a smooth horizontal surface and attached to a spring of force constant 100Nm-1. The
bullet remains embedded in the block while the spring is compressed by a distance of
5.0cm. find;
(i). Elastic potential energy of the compressed spring.
(ii).Speed of the bullet just before collision with the block.
18) A constant force pulls a body of mass 500g a long a straight smooth horizontal surface.
The body passes through points A and B with speeds 3ms-1 and 5ms-1 respectively. Given
that A is at a distance of 3m from B, find the magnitude of the constant force acting on
the body.
19) A pump draws 3.6m3 of water of density 100kgm-3 from a well 5m below the ground in
every minute and issues it at ground level through a pipe of cross sectional area 40cm2.
Find;
(i). The speed with which water leaves the pipe
(ii). The rate at which the pump is working
(iii). If the pump is only 80% efficient, find the rate at which it must work.
20) A water pump working at constant rate of 900W draws 0.3m3 of water from a deep well
and issues it through a nozzle situated 10m above the level from which the water was
drawn after every minute. If the pump is 75% efficient, find the velocity with which the
water is ejected and the cross sectional area of the nozzle.
21) A bullet of mass 20g travelling horizontally at a speed of 200ms-1 embeds itself in a block
of mass 980g suspended by a string such that it swings freely find;
(i). The vertical height through which the block rises
(ii).How much of the bullet’s energy becomes internal energy.
22) A bullet of mass 20g is fired horizontally with a speed of 200ms-1 into a block of wood
suspended by a string 1m long. The bullet is embedded in the block. Calculate the
maximum inclination of the string to the vertical.

23) A block of mass 0.50kg is released from rest so that it falls vertically through a distance
of 20cm onto a scale pan of negligible mass hung from a spring of force constant
1000Nm-1. Find the position of the scale pan when it first comes to rest.
24) A mass of 800g is released from rest so that it falls vertically through a distance of 30cm
onto a scale pan of negligible mass hung from a spring of force constant 200Nm-1. Find
the position of the scale Pan when it first comes to rest.
25) A ball of mass 50g falls from a height of 2.0m and rebounds to a height of 2.0m and
rebounds to a height of 1.2m. How much energy is lost on impact.
26) The blade of a large wind turbine, designed to generate electricity sweep out an area of
1400m2 and rotate about a horizontal axis which points directly into a wind of speed
85 | P a g e
15ms-1.
(i). Calculate the mass of air passing per second through the area swept out by the
blades.
(ii). The mean speed of the air on the far side of the blades is reduced to 13ms-1. How
much kinetic energy is lost by the air per second
(iii). How many turbines, operating with 70% efficiency would be needed to equal the
power output of a single conventional 1000MW power station.
27) A block of mass 2.0kg is dropped from a height 0.4m on a vertical spring of force
constant 1960Nm-1. Find the maximum distance the spring will be compressed.
28) A ball of mass 2kg falls from the top of a building of height 5.0m and hit the floor. If the
collision between the floor and the ball lasts 0.01seconds. Find the force exerted on the
floor.
29) A body of mass 2kg is releases from a certain height above the ground; it bounces to a
height of 2.0m above the ground if 80% of its energy is retained as kinetic energy on
collision with the ground find.
(i). The height from which the body was released
(ii).The force that the body exerts on the floor if it stays in contact with the floor for a
time of 2.0 x 102 seconds.
30) A bullet of mass 10g moving horizontally with a speed of 120ms-1 collides with a
stationary block of 8kg held by a string gets embedded in it. If the length of the string is
130cm find;
(i). The vertical height through which the block rises
(ii).The angle of swing.
31) A bullet of mass, m1 is projected horizontally towards a wooden block hanging by a
string of length 1m. If the block and the bullet are deflected through angle of 600 to the
vertical find the initial velocity of the bullet.
32) A rough slope of length 5m is inclined at angle of 300 to the horizontal. A body of mass
2kg is released from the top of the slope and travels. Down the slope against constant
resistance. The body reaches the bottom of the slope with a speed of 20ms-1. Find the
work done against resistance and find the magnitude of resistance.

33) A simple pendulum of length 1.5m has a bob of mass 130g. It is displaced from its mean
position A to a position B so that the string makes angle of 600 with the vertical.
Calculate the;
(i). Maximum potential energy of the bob
(ii).Velocity of the bob when the string makes an angle of 450 with vertical.

TOPIC 10. CIRCULAR MOTION


DEFINITIONS

86 | P a g e
1. Define the following terms;angular displacement,aradian,angular velocity,period,
frequency,centripetal acceleration,radial acceleration,centripetal acceleration,centripetal
force,centrifugal force,banking of atrack, angle of banking,circular motion

EXAMPLES, CONDITION, FACTORS, MERITS, AND DEMERITS


1. Give the conditions under which a car travelling in a circular path will topple/
overturn
2. State the conditions under which a car moving round a curve is least likely to skid
3. State on advantage and disadvantage of banking of a track
4. Give two examples of uniform motion in a circle
5. State three factors which affect the centripetal force
EXPLANATIONS
1. Explain why a force is necessary to maintain a body moving with constant speed in a
circular
2. Explain briefly the action of a centrifuge
3. Explain why a racing car can travel faster on a banked track than on a flat track of the
same radius of curvature
4. Explain why a maximum speed of a car on a banked road is higher than that on un
banked road
5. Explain why a cyclist leans at an angle  to the vertical towards the centre of the curve
when negotiating a corner.
6. Explain why a mass attached to a string rotating at a constant speed in a horizontal circle
will fly off along at tangent if the string breaks.
7. Explain why an aero plane has to bank its wings in order to move in a circular path
8. Explain briefly why a body moving with uniform motion in a circle has an acceleration
9. Explain two applications of centripetal force
10. Explain briefly what is meant by a geo-synchronous Earth satellite
11. Explain how honey is separated from honey combs using centrifugal machine
DERIVATIONS AND RELATIONS
1) Derive an expression for the speed of a body moving uniformly in a circular path
2) Derive an expression for the acceleration of a body moving uniformly in a circular path.
3) Show that the speed of a particle moving in a circle of radius with angular w is v=wr
4) Derive an expression for the centripetal force acting on a body of mass m moving in a
circular path of radius r.
5) A circular road of radius r is banked at an angle of show that the car will move along
1
the road without slide friction if its speed does not exceed (rgtan) Derive an expression
2
for the speed with which a car can negotiate a bend on a banked track without skidding.
6) Discuss the variation in magnitude and direction of the force which a string exerts on a
body moving in a vertical circle of radius R.
7) A small mass attached to a string suspended from a fixed point moves in a circular path at
a constant speed in a horizontal plane.
(i). Draw a diagram showing the force acting on the mass

87 | P a g e
(ii).Derive an expression showing how the angle of inclination of the of the string depends on
the speed of the mass and the radius on the speed of the mass and the radius of the
circular path.
8) A car of mass travels round a circular track of radius with a velocity V
a) sketch the diagram to show the forces acting on the car
arg
b) show that the car does not over turn if v2 where a is distance between the wheels b
2b
is the centre of gravity above the ground and g-acceleration due to gravity
9) A car goes round a level un banked curve of radius Ro. The coefficient of the kinetic
friction is µ. Derive a formula for the maximum speed in terms of µ, g, and Ro which the
car will go round without skidding.
10) A small mass attached to a string suspended from a fixed point moves in a circular path
at a constant speed in a horizontal plane show that
2
R g 1/2
(i). V= ( )
h
mg 2 2 1/2
(ii).T= (r +h ) where the symbols carry their usual meanings
h
11) A bob of mass, m, is tied to an inelastic thread of length L, and whirled with constant
speed in a vertical circle
(i). with the aid of diagram, explain the variation of tension in the string along the circle
(ii).if the string breaks at one point along the circle, state the most likely position and explain
the subsequent motion of the bob.
12) A car whose Centre of gravity is, h, meters above the ground and its width is 2a, meters,
move round a horizontal circular track of radius , R, with velocity, V
(i). Draw a sketch diagram to show the forces acting on the car
(ii).Derive an expression showing the velocity for which the car can move safely round the
track without over turning
13) Derive the relation between the instantaneous speed, V, and the angular velocity, w, of a
body moving uniform in a circle of radius r.
14) A student proposes to determine the acceleration due to gravity, g , by measuring the
period, T, of rotation of such a pendulum for various values of 
(i). Find the expression for T in terms of L, , and g
(ii). Sketch graph that could be used to obtain the value of g.
(iii). Discuss critically whether this is a good way of determination of g.
15) A light inelastic string of length, L, has one end fixed at a point A, and a particle of
mass, m, attached to the free end. The particle describes a horizontal circle such that the
string makes an angle of 450 with the vertical at the point of suspension. if the period of
oscillation is 3s find
(i). height of A above the circle
(ii). Velocity of the mass.
16) Consider a car of mass, m, moving around a banked track of radius, r, with a uniform
speed, v, which is inclined at angle  to the horizontal. show that;
(i). V= (r g tan )1/2
tan+µ
17) for skidding to take place, then the maximum speed is given by Vmax= [r g ( ¿
1−µ tan ¿
)1/2 A pendulum bob of mass m k g is attached to one end of a string of length, L, it is

88 | P a g e
then slightly displaced through a small angle , and its motion describes a circular path of


radius, r, show that the period , T, of the motion is given by T= 2 π
L cos
g

CALCULATIONS IN CIRCULAR MOTION ON CONICAL PENDULUM.


1) A particle moving in a circular path of radius 0.5m has a speed of 12ms-1
Find;
(i). angular speed
(ii).Period of revolution.
2) Calculate the velocity of a planet whose radius is 6.4x106m and period of 1 day.
3) A particle moves along a circular path of radius 3m with an angular velocity of 20 rads-
1
find;
(i). linear speed of the particle
(ii).time taken for one revolution
4) A particle of mass 0.2kg moves in a circular path with an angular velocity 5 rads-1 under
the action of a centripetal of 4N. Find the radius of the path.
5) n What force is required to cause a body of mass 3 g to move in a circular path of radius
2m at a constant rate of 4 revolutions per second.
6) A small bob of mass 0.20kg is suspended by an inextensible string of length 0.80m.
The bob is then rotated in a horizontal circle of radius 0.40m. find;
(i). the linear speed of the bob
(ii).tension in the string.
7) A pendulum of mass 0.2kg is attached to one end of an inelastic string of length 1.2m.
The bob moves in a horizontal circle with the string inclined at 300 to the vertical.
calculate;
(i). the tension in the string
(ii).the period of motion.
8) The period of oscillation of a conical pendulum is 2.0s. if the string makes an angle of
600 to the vertical at the point of suspension .calculate,
(i). the vertical height of the point of suspension above the circle
(ii).Length of the string.
(iii). velocity of the mass attached to the string
9) A 30kg body is whirled in a horizontal circle by a conical pendulum by means of an
inelastic string that has breaking strength of 392N. when the speed of the body is 8.0ms-1
the string breaks; calculate;
(i). the angle the string makes with the vertical at that in the string
(ii).the length of the string.
10) An air force plane makes a loop of radius 5.0x102m. If the centripetal acceleration down
the plane is 40ms-2. What is the magnitude of the angular velocity and the tangential
speed of the plane.
11) A steel ball of mass 0.50kg is suspended from inelastic string of length 1.0m. The ball is
whirled in a horizontal circle. If the maximum tension in the string is 24N. calculate;
(i). the maximum angle of inclination
(ii).Angular velocity
(iii). the centripetal force.

89 | P a g e
12) A ball of mass 0.5kg is suspended from a light inelastic string of length 1.0m. The ball
is whirled in a horizontal circle of radius 0.5m. find
(i). the centripetal force and the tension in the string
(ii).the angular speed of the ball
(iii). the angle between the string to such a value that the tension in the string is 10N
13) A mass of 0.6kg is whirled in a horizontal circle of radius 40cm by a string inclined at
600 to the vertical. calculate;
(i). the speed of the mass in the horizontal circle
(ii).length of the string
(iii). Vertical height from the point of suspension to the centre of the circle
(iv). Angular speed
(v).the period of the mass
14) A particle of mass 0.20kg is attached to one end of light inelastic string of length 0.5m.
The particle moves in a horizontal circle with angular velocity of 5.0rads-1 with the string
inclined at angle of  to the vertical. Find the value of .
A body of mass 0.45kg is attached to one end of a light inelastic string of length 2m. The bob
moves in a horizontal circle and the string sweeps out the surface. The maximum tension
that the string can sustain is 21N. Find the maximum angular velocity of the particle
15) An astronaut banks the winds of a space craft so as to travel at a speed of 720kmhr-1
along a horizontal circular path of radius 1.6x104m . calculate;
(i). centripetal force
(ii).Angle the astronaut should bank the space craft
16) A pilot on training describes a horizontal circle of radius 50cm if he can with stand a
maximum acceleration of 65.6ms-2 .What is the maximum angular velocity at which the
pilot can remain operating consciously.
17) A body moving a circular path of radius 0.5m makes 40 revolutions per second. find the
centripetal force if the mass is 1kg
18) A body of mass 2kg moves with a constant angular speed of 5 rads-1 around a circle of
radius 10cm. find the magnitude of the force that must be acting on the body towards the
Centre of the circle.
19) A steel ball of mass 0.5kg is suspended from a light inelastic string of length 1.0m. The
ball is whirled in a horizontal, circle of radius 0.5m. Find;
(i). the centripetal force and the tension in the string
(ii).the angular speed of the ball
(iii). the angle between the string and the circle if the angular speed is increased to
such a value that the tension in the string is 10N.
20) A stone of 0.50kg is tied to one end of the string is 2m above the ground. The stone is
whirled in a horizontal circle with the increasing angular velocity. The string will break
when the tension in it is 12.5N and the semi vertical angle is maximum. find;
(i). the value of the maximum semi vertical angle
(ii).the angular velocity of the stone when the string breaks
(iii). when does the stone hit the ground.
21) An inelastic string of length 2m is fastened at one end to a fixed point 2m above the
horizontal ground. At its other end is attached a small circle which describes a horizontal
circle in plane 1m above below the point of suspension.
(i). show that the angular velocity of the mass is 3.13 rads-1

90 | P a g e
(ii).if the string snaps, find the distance of the point where the mass strikes the ground from
the vertical through.
22) A pendulum of mass 0.2kg is attached to one end of an inelastic string of length 1.2m.
The bob moves in a horizontal circle with the string inclined at 300 to the vertical.
calculate;
(i). the tension in the string
(ii).the period of motion.
23) An inextensible string 1.2m of negligible mass is used to whirl a mass of 0.5kg in a
horizontal circle. if the maximum tension in the string is 25N, calculate;
(i). the maximum angle of inclination
(ii).Angular velocity
(iii). the centripetal force.
24) A body moving in a circular path of radius 0.6m makes 48 revs-1. Find the centripetal
force if the mass is 1kg.
25) An object of mass 0.5kg on the end of a string is whirled round a horizontal circle of
radius 2m with a constant speed of 10ms-1. find
(i). the angular velocity and the speed
(ii).If the same object is whirled in a vertical circle of the same radius and speed, find the
maximum and minimum tension.
26) The earth’s radius is 6.0x103km and it rotates on its axis once every 24hrs
(i). what is the centripetal acceleration at the equator
(ii).if a man weighs 700N at the North Pole, what is his effective weight at the equator.
27) A particle moving a circular path of radius 50cm has a speed of 12ms-1. find the angular
speed of the particle in
(i). radians per second
(ii).revolution per minute.
28) A particle moves along a circular path of radius 3m with an angular velocity 20 rads-1.
calculate;
(i). linear speed of the particle
(ii).angular velocity in revolution per second
(iii). time taken for one revolution
29) A mass of 0.2kg is whirled in a horizontal circle of radius 0.5m by a string inclined at
300 to the vertical. calculate;
(i). speed of the mass in the horizontal circle
(ii).length of the string
(iii). vertical height from the point of suspension to the centre of the circle
(iv). angular speed
(v).period.
30) A pilot banks the wing of an air craft so as to travel at a speed of 540kmhr-1 along a
horizontal circle paths of radius 8km. calculate;
(i). centripetal force if its m=2kg
(ii).angle the pilot should bank the air craft.
31) A particle of mass 0.25kg is attached to one end of a light inextensible string of length
50cm. The particle moves in a horizontal circle and the string sweeps out the surface of
cone. The maximum tension that the string can sustain is 12N. Find the maximum
angular velocity of the particle.

91 | P a g e
32) An astronaut, as part of her training, is spun in a horizontal circle of radius 5m. if she can
with stand a maximum acceleration of 78.5ms-2. What is the maximum angular velocity at
which the astronaut can remain conscious.
33) A pendulum bob of mass 2.0kg is attached to one end of a string of length 1.2m. The bob
moves in a horizontal circle in such a way that that string is inclined at 300 to the vertical.
calculate;
(i). the tension in the string
(ii).the period of the motion.
34) An air craft of mass 1.0x104kg is travelling at a constant speed of 0.2kms-1 in a horizontal
circle of radius 1.5km. find
(i). angular velocity
(ii).centripetal force and tension.
35) What force is necessary to keep a mass of 0.80kg revolving in a horizontal circle of
radius 0.7m with a period of 0.5s. Find the direction of this force.
36) A small mass of 5g is attached to one end of a light in extensible string of length 20cm
and the other end of the string is fixed. The string is taut and horizontal and the mass is
released. when the string reaches the vertical position, what are the magnitudes of
(i). the kinetic energy of the mass
(ii).the velocity of the mass
(iii). the acceleration of the mass
(iv). the tension in the string.
37) A force of 140N is used to drive a turbine making it determine 20 revolutions in 20
seconds. if the radius of the path described is 20cm, find the power developed
38) A particle p, of mass 5kg is suspended from a fixed point o by a light inextensible string
of length 1m. The particle is projected from the lowest point A, with a horizontal speed of
4ms-1. When the angle Aop is 600. find;
(i). speed of p
(ii).tension in the string
39) A level railway is in form of a circular arc of radius 200m. find the horizontal force
acting on the rails when the train of mass 15,000kg, travels around the bend with a speed
of 20ms-1.
40) A piston in a car engine performs sin m of the frequency 13.0Hz. If the mass of the piston
is 0.80kg and its amplitude of vibration is 50mm, find the maximum force on the piston.
41) A body moving in a circular path of radius 0.6m makes 48 revolutions per second. find
the centripetal force if the mass is 1kg
42) A particle moves in a circular path of radius 3.0m with an angular velocity of 20 rads-1.
find
(i). the linear speed of the particle
(ii).the angular velocity in revolution per second
(iii). the period
(iv). the centripetal acceleration
43) A satellite of mass 6000kg is in an orbit a distance 5.5R above the surface of the Earth
(R- is the radius of the earth). if its period is 756kmhr-1 determine the satellite’s
(i). angular speed (ii) angular acceleration (iii) period (iv) Angular displacement
in 2 days
(v) Centripetal acceleration

92 | P a g e
44) A weather satellite of mass 1500kg revolving in its orbit is always at a point above the
earth mete logical station. Determine (i) speed with which the satellite’s moving if it is at
a height of 3.6x103 km above the earth station
(ii) Centripetal force of the satellite
45) An object of mass 10kg moves round a circle of radius 10m at a constant speed of 24ms-
1
calculation the centripetal force acting on the mass in its circular path.
46) Calculate the mass of a toy which experiences a centripetal force of 303.2N when
whirled in a circle of radius 1.6m making 2.5 revolution per second
47) (a) Determine the angular speed of the wheels of a car of diameter 60cm when the car is
moving at 72kmhr-1
(b) Determine the angular displacement of such a wheel in 0.05 seconds
48) A fun fair ride of diameter 12m makes 0.5 revolutions per second. Determine;
(i). its angular velocity
(ii).the linear speed
(iii). Angular acceleration
(iv). centripetal force of the child if she has a mass of 30kg

CALCULATIONS ON BANKING AND VERTICAL CIRCLE


1) A car goes round on un banked curve at 15ms-1. The radius of the curve is60m. Find the
least coefficient of kinetic friction that will the car negotiate the curve without skidding.
2) A car of mass 3000kg travel at 108kmhr-1 around un banked curve of radius 200m.
(i). what is the minimum coefficient of friction between the car tyres and the rod that will
permit the car to negotiate the curve without skidding
(ii). At what angle would the rod have to be banked if there were to be no friction at force
between the tyres and the road surface.
3) A 1200kg car rounds a curve of radius 65m banked at angle of 140. if the car is travelling
at 80kmhr-1 will frictional force be required. if so how much and in which direction.
4) A bend of 200m radius on a level road is banked at the correct angle for a speed of 15ms-
1
. if a vehicle rounds the bend at 30ms-1. what is the minimum coefficient of kinetic
friction between the tyres and the road so that the vehicle will not skid
5) A car travel round a bend banked at an angle of 22.6 if the radius of the curvature of the
bend is 62.5m and the coefficient of friction is 0.3. calculate the maximum speed at
which the car negotiate the bend without skidding.
6) A cyclist moving at a speed of 120kmhr-1 takes a curve of radius 150m. At what angle to
the vertical must he lean if he is to take the curve without skidding.
7) If a body of mass 540g is rotated by a string at a constant speed v in a vertical circle of
radius 1.2m. if the minimum tension is 4.5N .calculate
(i). the constant speed
(ii).the maximum tension, T
(iii). T when the string is horizontal.
8) A stone of mass 0.5kg is whirled round on a 0.8m long string in a vertical circle. if the
speed of the stone is 4ms-1.
(i). At what point in the circle is the tension in the string a maximum and what is its value
(ii).At which point in the circle is the tension in the string a maximum and what is its value.
9) A stone of mass 0.5kg is attached to a string of length 0.5m which breaks if the tension
in it exceeds 20N. The stone is whirled in a vertical circle, the axis of rotation being at a

93 | P a g e
height of 1m above the ground. The angular velocity is gradually increased until the
string breaks. In what position is the break most likely to occur and at what angular
speed. where will the stone hit the ground.
10) A rope is tied to a pail containing water. The pail is swung in a vertical circle of radius
1.0m. What must be the minimum speed of the pail of the highest point of the circle of no
water is to spill from the pail.
11) A particle of mass 4kg is suspended from a fixed point o by a light inextensible string of
length 0.20m. The particle is projected from the lowest point x with a horizontal speed of
6ms-1 and describes a vertical circle. When the particle is at point y, then the tension in
the string. is T N where O Y makes angle of 450 with the down ward vertical.
Determine the speed of the particle an d the tension in the string at y.
12) An aero plane loops the loop in a vertical circle of radius 2x102m with a speed of 40ms-1
at the top of the loop. The pilot has a mass of 80kg. what is the tension in the strap
holding him into his seat when he is at the top of the loop.
13) A bucket of water is swung in a vertical circle of radius 64.0m in such a way that the
bucket is upside down when it is at the top of the circle. what is the minimum speed that
the bucket may have at this point if the water is to remain it.
14) A bend on a level road forms a circular path of radius 295m. The greatest speed at which
a car can travel round the bend a round the bend without slipping occurring is 198kmhr-1.
find the coefficient of friction between the tyres of the car and the road surface.
5
15) A car moves around a circular arc of radius 75m which is banked at tan-1 ( ) to the
12
horizontal. At what speed should the car be driven if it’s to have no tendency of slipping.
16) A car travels around a bend on a rough road which is a circular arc of radius 62.5m. The
5
road is banked at angle tan-1 ( ) to the horizontal. If the coefficient of friction between
12
the tyres of the car and the road surface is 0.4. find
(i). the greatest speed at which the car can be driven around the bend without
slipping occurring
(ii).least speed at which the car can be driven around the bend without slipping
occurring.
17) A truck moves in a horizontal circular path of radius 1.4m around a banked corner of
race track. The greatest speed with the car can be driven around the corner without
slipping occurring is 42ms-1. if the coefficient of friction between the tyres of the car and
1
the surface of the track is , find the angle of banking.
3
18) A car travels around a band of radius 4.0×102m on a road which is banked at angle of 
to the horizontal. if the car has no tendency to skid when travelling at 35ms-1 find the
value of .
19) A car travels around a bend of radius 105m banked 450 to the horizontal. The car finds
that at least 21ms-1 the speed was used by the drive. if he to a void slipping sideways. find
the coefficient of friction between the tyres of the car and the road surface.
20) A stone of mass 0.5kg is attached to a string of length 0.5m which will break if the
tension in it exceeds 20N. The stone is whirled in a vertical circle, the axis of rotation
being at a vertical height of 1.0m above the ground. The angular speed is gradually
increased until the string breaks

94 | P a g e
(i). in what position is the string most likely to break explain
(ii). At what angular speed will the string break
(iii). find the position where the stone hits the ground when the string breaks.
21) A small bob of mass 0.05kg is tied on an inextensible string of length 0.8m. the bob is
whirled in a vertical circle of radius 0.8m at a speed of 5.0ms-1
(i). Draw a diagram to show the position where the string is most likely to break and
give the reason why.
(ii).Calculate the period of motion of the bob
(iii) If the string breaks when the tension in it is 2.10N is it safe to move the bob at
5.0ms-1.
22) A car travels round a bend banked at angle of 250. If the radius of curvature of the bend is
65m and the coefficient of friction between the tyres of the car and the road surface is
0.35. Calculate the maximum speed at which the car negotiates the bend without
skidding.
23) A Pilot banks the wings of his space craft 50 as to travel at a speed of 360kmhr-1 in a
circular path of radius 5.0m. Find the angle at which he should bank the space craft.
24) A high way is 13.6m. Calculate the difference in level between the external and internal
edges of the road in order for a car to be able travel at 6ms-1 without skidding round a
curve without skidding round a curve whose radius is 600m.
25) A cyclist rounds a curve of 30m radius on around which is banked at an angle of 200 to
the horizontal. If the coefficient of sliding friction between the tyres and the road is 0.5.
Find the greatest speed at which the cyclist can ride without skidding and find his
inclination to the horizontal at this speed.
26) A bend of 200m radius on a level road is banked at the correct angle for a speed of 15ms-
1
, if the vehicle rounds the bend at 30ms-1, what is the minimum coefficient of kinetic
friction between the tyres and the road so that the vehicle will not skid.
27) A small coin is placed at a distance of 1.2m from the centre of a circular plat form. The
coefficient of kinetic friction between the oil and the plat from is 0.2. Find the maximum
angular speed with the plat from can rotate without causing the coin to slide off the
platform.
28) A particle of mass 2kg is suspended from a fixed point O by alight inextensible string of
length 20cm. the particle is project acted from the lowest point A with a horizontal speed
of 5ms-1 and describes a vertical circle. When the particle is at point B, then the tension in
the string is TN where OB makes an angle of 450 with the downward vertical. Determine
the speed of the particle and the tension in the string at B.
29) A bend on a level road forms a circular path of radius 75m, the greatest speed at which a
car travel around the bend without slipping occurring is 63kmhr-1. Find the coefficient of
friction between the tyres of the car and the road surface.
30) A girl whirled a toy of mass 0.9kg at a speed of 8ms-1 in a vertical circle using a string of
length 1.2m. determine;
(i). Maximum tension and minimum tension
(ii).At what positions of the toy is maximum tension and minimum tension
experienced in the string

TOPIC 11: GRAVITATION

95 | P a g e
DEFINITIONS

1. Define the following terms; universal gravitational constant,asatellite,passive satellite,active


satellite, parking orbit, gravitational potential, escape velocity, gravitational field strength,
weightlessness of a satellite.

LAWS, PRINCIPLES, AND THEOREMS;

1. State Kepler’s laws of gravitation/planetary motion


2. State Newton’s law of gravitation

EXPLANATIONS
1. Explain why an astronaut may feel weightless in orbit round the Earth although the
gravitational attraction of the earth still acts on the astronaut
2. Explain why a comet spends a large part of the time for one orbit a large distance from
the sun.
3. Explain why the geostationary satellites must be places above the Equator.
4. Explain why the objects at the equator do not get flung off into space.
5. Explain how global communication can be enhanced using artificial satellites.
6. Explain the term weightlessness as used in satellite motion.
7. Explain why the gravitational force of attraction between two bodies of ordinary is
negligible in everyday life
8. Explain the fact that sun has got an atmosphere.
9. Explain why the moon has no atmosphere and its cold
10. Explain and sketch the variation of acceleration due to gravity with distance from the
centre of the Earth
11. Explain briefly how worldwide radio/television communication can be achieved with the
help of satellites
12. Explain what would happen if the mechanical energy of a satellite was decreased.
13. The period of a simple pendulum is measured at different locations along a given
longitude. Explain what is observed.
14. Explain why any resistance to the forward motion of an artificial satellite results into an
increase its speed
15. Explain why does acceleration due to gravity vary with location on the surface of the
earth
16. Explain the meaning of a parking orbit as applied in a communication via a satellite.
17. Explain why any resistance to forward motion of a space satellite results in an increase in
Kinetic energy.
18. Account for the following;
19. Acceleration due to gravity varies with latitude

96 | P a g e
20. The moon has no atmosphere
21. Explain briefly the effects of frictional forces on the motion of an earth satellite
22. Explain the effect of friction between such a satellite and the atmosphere
23. Explain why the mechanical energy of a satellite is always negative

EXPERIMENTS
1. With the aid of a diagram describe an experiment to determine the universal
gravitational constant, G

DERIVATIONS AND RELATION.


1) Show that a period of satellite in a circular orbit of radius r about the earth is given by

( )
2
4π 3
T= r where G is universal constant and Me is Mass of the earth.
GMe 2
2) Show how to estimate the mass of the sun if its period and orbit radius of the planet
are unknown.
3) The gravitational potential, U, at the surface of a planet of mass M and radius R is
GM
given by U= where G is the gravitational constant. Derive an expression for the
R
lowest velocity V which an object of mass M must have all the surface of the planet if
it is to escape from the planet.
4) Draw a sketch graph to show how the gravitational field strength varies with length h,
above the earth surface.
5) Derive the expression for the escape velocity of a rocket fired from the earth.
6) A satellite moves in a circular orbit of radius R, about the planet of mass M with a
2
3 GMT
period t .Show that ; R = 2 Where G is the universal gravitational constant.

7) Find an expression for the period of a satellite which moves in a circular of orbit of
radius r, about a planet of mass M.
8) Derive from Newton’s law of gravitation, the dimensions of Universal gravitational
constant .
9) State Newton’s law of gravitation and use it to show that the square of the period of a
satellite in orbit is proportional to the cube of the radius of orbit.
10) Derive the expression for the potential energy due to the Earth at the any point
outside the Earth.
11) Sketch a graph to show how the potential energy varies distance from the centre of
the earth for points outside the Earth.
12) Derive the expression for the period of a satellite moving in a circular orbit about the
Earth in terms of the radius of the orbit, the acceleration due to gravity at the
Earth’s surface and radius of the Earth.
13) Show that Newton’s law of gravitation is consistent with Kepler’s 3rd law

97 | P a g e
14) (i) Derive the expression for the potential energy due to the Earth at any point outside
the Earth.

(ii) Sketch a graph to show how the potential energy varies with distance from the
centre of the Earth for points outside the Earth.

15) Draw a sketch graph to show how the gravitational field strength varies with distance
from the Earth’s surface for points external to it.
16) The moon moves in a circular orbit of radius R, about the Earth of mass, Me with
gre 2 T 2
period T. show that R3 = 2 where re = radius of the Earth, g – acceleration

due to gravity 4Л2 on the earth’s surface.
17) Show that the frequency of revolution of a planet in a circular orbit of radius R about

the sun is given by∫ ¿

gravitational constant
1
2π √ GMs
R
3 where Ms – mass of the sun and G is the

18) Show that the gravitational P.e of a satellite around the earth is twice its K.e.
19) Show that the total energy of a satellite of mass, m in a circular orbit about the Earth
−mgR 2
is given by E = where R is the radius of the Earth.
2r
20) (i) A satellite is in a circular orbit about a planet of radius R, if the altitude of the
satellite is h and its period is T, show that the density of the planet is given by;

ρ = 2 1+ 3
¿ ( )
h
R

(ii) Calculate the average density of the planet if the planet if the period is 200
minutes and the satellite’s orbit is close to the planet’s surface.

CALCULATIONS ON GRAVITATIONAL FORCE


1) Calculate the gravitational attraction of two cars 5m apart if the masses of the cars are
100kg and 1200kg.
2) Calculate the force between the sun and Jupiter. Assuming that the mass of the sun = 2.0
x 1030kg, the mass of Jupiter = 1.89 x 1027kg and the radius of Jupiter’s orbit = 7.73 x
101m.
3) Calculate the force of attraction between two masses; one of 5.0kg and one of 8kg whose
centers are 10cm apart.
4) A mass of 5kg is first weight on a balance at the top of a tower 20m high. The mass is
then suspended from a fine wire 20m long and re weighed. Find the difference in weight.
Assuming that the radius of the earth is 6400km, the mass of the Earth 6.0 x 1024kg ad G
is 6.67 x 1011Nm2kg-2 .

98 | P a g e
5) A binary star consisting of two dense spherical masses of 1030kg and 2.0 x 1030kg whose
centers are 107 km a part. Find the gravitational force about an axis intersecting aline
joining the centre.
6) A space ship travels from earth directly toward. At what distance from the centre of the
Earth will the gravitational forces of the sun and of the Earth on the ship exactly cancel.
7) The moon moves in a circular orbit of radius 6.0 x 1026kg around the earth with a period
of 2.36 x106s. Calculate the gravitational field of the earth at the moon.
8) Calculate the gravitational force on a mass of 2kg at the earth’s surface. If the earth were
a perfect sphere of radius 6.3 x 106m, mass 5.98 x 1024kg, G = 6.67 x 10-11m3kg-1s2, g =
9.81ms-2.
9) (i) Calculate the force on a mass of 10kg at a height of 800km above the earth’s surface
assuming the gravitational field strength is 9.8Nkg-1 at the earth’s surface.
(ii). what is the speed of a space craft in that orbit and its period.
CALCULATION ON VARIATION OF ACCELERATION DUE TO GRAVITY
1) The moon moves in a circular orbit of radius 3.84 x 108m around the Earth with a period
of 2.36 x 106S. Calculate the gravitational field of the Earth to that of the moon.
2) The period of the moon around the Earth is 27.3 days. If the distance of the moon from
the Earth is 3.83 x108km. Calculate the acceleration due to gravity at the face of the
Earth.
3) If the moon moves round the Earth in a circular orbit of radius 4.0 x 108m and takes
exactly 27.3days to go round once, calculate the value of acceleration due to gravity 1g at
the surface.
4) Calculate the ratio of acceleration due to gravity on the surface of mercury to that on the
surface of the Earth given that the radius of mercury is 0.38times that radius of the Earth
and the mean density of mercury if 0.68 times that of the Earth.
5) Calculate ratio of the mass of the sun to that of the Earth, given that the moon moves
round the Earth in a circular orbit of radius 4.0 x 105km with a period of 2.3days and
orbital radius of the earth round the sun is 1.5 x 108km and its period is 365days.
6) A body has weight of 10N on earth. What will be the weight of the body on the moon if
the ratio of the moon’s radius to the Earth is radius is 0.27 and that of moon’s mass to
that of earth’s mass is 1.2 x 10-2 .

7) A body weighs 68.6N on the Earth’s surface where acceleration due to gravity is 9.81ms-
2
. How much will it weigh at a height of 2.6 x 106m.
8) Given that the radius of the Earth is 6400km and that the value of acceleration due to
gravity at its surface is 9.81ms-2. Calculate its value for the acceleration due to gravity at
a point 400km above the earth’s surface.
9) The period of the moon around the Earth is 65.2hours if the distance of the moon from
the Earth is 3.83 x 108m. Calculate the acceleration due to gravity at the surface of the
Earth.

99 | P a g e
10) Obtain the value of g from the motion of the moon assuming that its period of revolution
around the earth is 27.3days and the radius of its orbit about the earth is 3.85 x 105km.
11) Assuming that mars is a sphere of radius 3400km, and density 3900kgm-3 and earth is a
sphere of radius 6400km and density 5500kgm-3. Calculate the value of the acceleration
due to gravity of the surface of mars. (Neglect effects due to rotation of the mars and
Earth).
12) At what distance away from the earth is surface will the acceleration be one eight of its
value at the earth’s surface.
13) The moon moves in a circular orbit of radius 3.84 x 108m around the earth with a period
of 2.36 x 106s, calculate the gravitational field strength of the earth at the moon.
14) A body weighs 63N on the Earth’s surface. How much will it weigh at a height equal to
half the radius of the Earth above the Earth’s surface.
15) Assuming the Earth to be a sphere of radius 6.0 x 106m, estimate the mass of the Earth,
given that the acceleration of free fall is 9.81ms-2 and that the gravitational constant G is
7.0 x 10-11 Nm2 kg-2.
16) The period of the moon around the Earth is 27.3 days if the distance of the moon from the
earth is 3.83 x 105km. calculate the acceleration due to gravity at the surface of the Earth.

CALCULATION ON SATELLITE MOTION’S SPEED, PERIOD, PARKING ORBIT.

1) A communication satellite orbit the Earth in a synchronous orbit. Calculate the height of
a communication satellite above the Earth.
2) An artificial satellite is launched such that it revolves in the parking orbit. Find the
velocity and height above the Earth’s surface to which the satellite is launched.
3) A small artificial satellite revolves in an equatorial circular orbit. Find the height of the
satellite above the Earth is surface of its period is 64minutes.
4) An artificial satellite is lunched in a circular orbit at a height of 3.6 x 107m above the
earth surface;
(i). Determine the speed with which the satellite must be launched to maintain it in
the orbit.
(ii).Determine the period of the satellite
(iii). What conclusion can be made about the period of the satellite.
5) A communication satellite is launched in a circular orbit about the earth’s equator so that
it appears to remain in the same position as viewed from the Earth. Find the height of the
satellite above the equator (radius of the Earth = 6.4 x 106m).
6) Calculate the radius of a parking orbit for an earth satellite
7) A 103kg satellite is launched in a parking orbit about the Earth. Calculate the height of the
satellite above the Earth surface.
8) A satellite is revolving around the Earth in a circular orbit at an altitude of 6.0 x 105m
where the acceleration due to gravity is 9.4ms-2, Assuming that the earth is spherical.
Calculate the period of the satellite.

100 | P a g e
9) The period of a satellite is 7.75hours and the mean height of its orbit about the planet
mars is 5840km. if the diameter of mars is 6730km, calculate the density rising of mass.
10) A certain weather satellite is in a circular orbit with time period of two hrs. Calculate its
height above the earth is surface given that g = 9.8ms-2 and earth’s radius is 6400km.
11) A communication satellite has a mass of 800kg. Find its speed in its 0rbit.
12) Given that the radius of the Earth is 6.37 x103km and period of revolution of the moon
round the Earth is 27.3days. Calculate the average distance from the earth to the moon.
13) An artificial satellite is launched at a height of 3.6 x 107m above the earth’s surface
(assume radius of the earth = 6400km), acceleration due to gravity at the earth’s surface =
9.81ms-2)
(i). Determine from first principles, the speed with which the satellite must be
launched to maintain it in the orbit.
(ii). Determine the periodic time of the satellite.
(iii). What deduction can be made from the result obtained in (ii) above with reference
to the satellite.
(iv). To what application can such a satellite be put to.
14) The satellite describes a nearly circular orbit of radius 9.7 x 106m round the planet mars.
With a period 2.75 x 104S.
(i) Calculate the mass of mars
(ii) The period of revolution of the other Martian satellite is 1.09 x 105S, what is the
radius of the orbit.
15) An artificial satellite in circles the earth in a circular orbit in the plane of the equator at a
height of 30,000km above the earth’s surface (mass of earth = 6.0 x 1024kg, Radius of
earth = 6.4 x 106m G = 6.67 x 10-11 Nm2 kg-2)
(i). Calculate its speed,
(ii).What is the time between successive appearances over a point on the equator if it
appears to come from the west
(iii). How much higher would it have to be in order to appear stationary.
16) (a) (i) If the acceleration due to gravity, at the moon’s surface is 1.70ms-2 and its radius
is 1.74 x 106m, calculate the mass of the moon.
(ii). Find the period of a satellite at 400km above the earth .
(iii). The velocity with which a body must be projected horizontally so that it may
revolve as a satellite just above the earth’s surface
(b) The minimum velocity with which it must be projected vertically upwards in order
that it may not return to the Earth.
17) A communication satellite is launched in a circular orbit about the equator at a height of
3.6 x 104m above the surface of the earth. Find;
(i). The speed with which the satellite is launched to the orbit
(ii). The period of the satellite. What deduction can be made about this period.
(iii). If another satellite moves in a circular orbit concentrate with that above at a height
2.496 x 104km above the earth’s surface what will be the period in this case.
101 | P a g e
18) A communication satellite is set in a circular orbit at a certain height above the earth’s
surface. If the orbital velocity is 3.5 x 104ms-1. Determine the height above the earth’s
surface.
19) A communication satellite is launched in a circular orbit about the earth’s equator so that
it appears to remain in the same position as viewed from the earth. Find the height of the
satellite above the equator.
20) A satellite is launched in a circular orbit about the equator at a height of 3.6 x 104km
above the earth’s surface find;
(i). The speed with which the satellite is launched into the orbit
(ii).The period of the satellite.
21) An artificial satellite revolves on an equatorial circular orbit.
(i). Calculate the height of the satellite above the earth’s surface if its period is 94
minutes
(ii).Find its velocity (Assume g = 9.8ms-2, RE = 6.4 x 106m.)
22) When a space shuttle is in an orbit at a mean height of 0.38 x 106m above the surface of
the earth, it requires 91minutes to complete one orbit. Find the mass of the earth.
23) The Earth of earth, Me, describe a circle of radius Re about the sun of mass Ms and radius
R se
Rs at an angular velocity 2.0 x 107 rads-1 due to gravitational attraction. Given that =
Rs
200. Find the density of the sun.
24) The distance of Jupiter from the sun is 5.2 times the distance of the earth from the sun.
Assuming that both orbits are circular, find the time Jupiter takes to complete its orbit. If
the earth takes one year.
25) Calculate the speed of the satellite and the period of the orbit given that g = 9.8ms-2, and
R = 6.4 x 103km.

CALCULATION ON SATELLITES ENERGY.


1) A satellite of mass 1000kg in a circular orbit at a height of 3.59 x 107m above the Earth’s
surface. Find the mechanical energy of the satellite.
2) A body of mass 15kg is moved from the Earth’s surface to appoint 1.8 x 106m above the
Earth. If the radius of the earth is 6.4 x 106m and its mass is 6.0 x 1024kg. Calculate the
work done in taking the body to the point.
3) A mass is released from a point at a distance 10R from the centre of the Earth, where R is
the radius of the Earth. Find the speed of the mass at a point a distance of 7R from the
center of the Earth.
4) A satellite of 103kg is launched is a circular orbit of radius 7.2 x 106m about the Earth.
Calculate the mechanical energy of the satellite.
5) A 103kg satellite launched in a parking orbit about the Earth. Calculate the mechanical
energy of a satellite.

102 | P a g e
6) A body of mass 15kg is moved from the earth’s surface to a point 1.8 x 106m above the
earth. If the radius of the earth is 6.4 x 106m and its mass is 6.0 x 1024kg, calculate the
work done in taking the body to that point.
7) A satellite of mass 500kg is in a circular orbit of height 3.6 x 104km above the earth’s
surface.
(i). Find the kinetic energy of the satellite
(ii).Explain what happens if the mechanical energy of the satellite is increased.
8) Calculate the gravitational potential at a point on the earth is surface due to the sun.
9) A rocket is launched from the earth’s surface of M and Radius R with a velocity Vo it
V
reaches a distance ^R from the centre of the Earth with a velocity 0 . Express V0 in
10
terms of acceleration due to gravity g, and R.
10) Calculate the velocity of escape of a satellite launched from a planet where the
acceleration due to gravity is 5.0ms-2 and radius of the planet is 2.0x107m.
11) A satellite of mass 600kg is in a circular orbit at a height. 2000km above the earth’s
surface .find;
(i). Kinetic Energy
(ii).Gravitational potential energy.
12) A satellite of mass 1000kg moves in a circular of radius 7.0 x106m round the earth. At
this height the acceleration due to gravity is 8.2ms-2. Calculate the total energy of the
satellite.
13) A projectile is fired vertically from the surface of the earth. Calculate the minimum speed
required vertically from the surface of the earth. Calculate the minimum speed required
for the projectile to escape the earth’s gravitational force.
14) A satellite of mass 102kg is launched in a circular orbit at a height of 6.4 x 106m above
the surface of the earth find;
(i). Period
(ii).Mechanical energy of the satellite.
15) A satellite of mass 250kg moves in a circular equatorial orbit at a distance of 500km
above the surface of the earth. Find;
(i). The radius of its orbit
(ii). Period
(iii). Total energy of the satellite.
16) A 1.0 tone satellite is placed in a circular orbit so that it goes round the earth 14 times a
day.
(i). Calculate the radius of the orbit and hence its altitude
(ii).How much energy is required to lift the satellite to this altitude.
(iii). Calculate the kinetic energy of the satellite in its orbit.
17) A space craft of mass 100kg is launched in a circular orbit at a height of 128000km about
the surface of the Earth.

103 | P a g e
(i). Find the period of the satellite
(ii). Calculate the mechanical energy of the satellite
(iii). What would happen if the satellite’s speed were halved while it’s in orbit.
18) A rocked is launched from the Earth’s surface with a velocity V. if h is the maximum
V 2re
height above the Earth’s surface it would go. Show that h = , re radius of the
2 gre V 2
earth.
19) A satellite of mass 66kg is in orbit round the earth at a distance of 5.7R above the Earth’s
surface, where R is the radius of the Earth. If the gravitational field strength at the Earth’s
surface is 9.8Nkg-1. Calculate the centripetal force on the satellite. Assuming that the
earth’s radius is 6400km calculate the period of the satellite in orbit.
20) A radio astronomy research satellite of mass 200kg circles the Earth in an orbit of radius
3
R where R is the radius of the Earth. If the gravitational pull on a mass of 1kg at the
2
Earth’s surface is 10N, calculate the pull on the satellites.
21) Calculate the least kinetic energy which must be given to a mass of 2000kg at the Earth’s
surface for the mass to reach a point a distance R0 from the centre of the Earth.
22) A space craft of mass 4.0 x 104kg has its motors switched off. It slows down as it moves
away from 10 x 106m above the earth’s centre to 15 x 106m. Find the loss of kinetic
energy of the craft and the average force acting on the space craft.
23) Calculate the gravitational potential difference between a point on the Earth’s surface and
a point 1600km above the Earth’s surface.
24) Calculate the minimum energy required to project a space craft of mass 2.0 x 106kg from
the surface of the earth so that it escapes completely from the influence of the Earth’s
gravitational field.

TOPIC 12: SIMPLE HARMONIC MOTION


DEFINITIONS

1. Define the following terms; simple harmonic motion,period,amplitude,periodic


motion,frequency,forced oscillation, damped oscillation, under damped oscillation, over
damped oscillation, critically damped oscillation, free oscillation, resonance energy.

FACTORS, MERITS, DEMERITS, EXAMPLES AND CONDITION.


1. State two practical example of objects which executes oscillatory motion
2. Give two example of oscillatory motion which approximate to s.h.m and state the
assumptions made in each case.
3. State four characteristics of simple harmonic motion.
EXPERIMENTS

104 | P a g e
1. Describe how the acceleration due to gravity can be obtained using a simple pendulum;
2. Describe an experiment to determine the force constant of a spring using different masses
3. Describe an experiment to determine acceleration due to gravity using the following
apparatus; a spiral spring, a retort stand with a clamp, a pointer, seven 50g masses, a
meter rule, and a stop clock.
4. Suppose you are provided with the following terms; a spiral spring, a stock clock, a set of
masses, a mass hanger, a paper pointer, a meter rule and retort stand and clumps.
Describe how you would determine the acceleration due to gravity.
5. You are provided with the following items only; A meter rule, one half meter rule, a
spring (helical) of force constant K, one wedge, one 100g mass, a piece of inelastic
thread, a piece of manila paper, one retort stand and two clamps.
6. Design an experiment that can be carried out using all these items to determine the mass
of the meter rule, illustrate your answer with a diagram.
7. Comment on the accuracy of the results obtained in experiments above.
8. Describe how the acceleration due to gravity can be measured using a helical spring of un
known force constant and other relevant apparatus.
9. Describe an experiment to determine the acceleration due to gravity g using a spiral
spring of known force constant.

DERIVATIONS AND RELATIONS

1) Show that the speed of a body moving with simple harmonic motion of angular
frequency, w is given by V= W (A2 – X2)1/2 where A is the Amplitude and x is the
displacement from equilibrium position.
2) Sketch graphs to show the variation with displacement of the kinetic and potential
energies of a body moving with simple harmonic motion.
3) Sketch the following graphs for a body performing simple harmonic motion;
(i). Velocity against displacement
(ii).Displacement against time.
4) Sketch displacement time graph for under damped and over damped oscillations.
5) A mass hanging on a spring is given a small vertical displacement and the released.
(i). Show that the mass performs simple harmonic motion
Discuss briefly the energy changes which occur as the mass oscillates
(ii).Explain why the oscillations ultimately die down.

6) Two springs of force constants K1 and K2 are suspended from a horizontal support. A
mass m hangs from the lower ends of the spring as shown.

105 | P a g e
k1 k2

If both springs have negligible mass, show that when m is displaced from its
equilibrium position, it describes simple harmonic motion of frequency f given by.

f= 1
2π √ K 1+ K 2
m

7) Show with the aid of a suitable sketch graph how kinetic energy of a mass attached to the
and of an oscillatory light spring changes with distance from the equilibrium position
8) A cylinder of length, L, cross sectional area A, and density σ , floats in a liquid of density
ρ, the cylinder is pushed down slightly and released;
(i). Show that it performs simple harmonic motion
(ii).Derive the expression for the period of oscillation.
9) A cylindrical vessel of cross sectional area A, contains air of volume, V at a pressure P
trapped by a frictionless air tight piston of mass, M. the piston is pushed down and
released.
(i). If the piston oscillates with simple harmonic motion, show that the frequency is

given by. f =
A

2 π MV
p

(ii).Show that the expression for f in (i) is dimensionally consistent.


10) A particle of mass M, executes simple harmonic motion between two points A and B
about equilibrium position O. sketch a graph of the restoring force acting on the particle
as a function of a distance r, moved by the particle.

11)

A B

106 | P a g e m
Two springs A and B of springs constants KA and KB respectively are connected to a mass m as
shown above. The surface on which the mass slides is frictionless

(i). Show that when the mass is displaced slightly it oscillates with the simple
harmonic motion of frequency f, given by

f=
2π√
1 K A+ K B
M
(ii).If the two springs are identical such that KA = KB = 5.0 Nm-1 and mass m = 50g.
Calculate the period of the oscillation.
12) Sketch a graph of;
(i). Velocity against displacement
(ii).Acceleration against displacement for a body executing s.h.m.
13) A glass U – tube containing a liquid is tilted slightly and then released.
(i). Show that the liquid oscillates with s.h.m
(ii).Explain why the oscillation ultimately comes to rest.
14) A hydrometer is made to float in a liquid and there after given a small vertical
displacement after which it’s released. Show that the hydrometer. Executes simple
harmonic motion.
15) A volume V of air, pressure P, is contained in a cylindrical vessel of cross sectional area
A by a frictionless air tight piston of mass m. the piston is slightly forced downwards and
then released. Shown that the Piston oscillates with simple harmonic motion of period T

given by T =
A√
2 π MV
P
16) A platform moves up and down with s.h.m of period T, and Amplitude A. A small
particle of mass m, is placed on the platform.
(i). Find an expression in terms of g, T, and x for the reaction of the platform on the
particle when the platform is at a displacement x from its mean position.
(ii).If the plat form vibrates in s.h.m with a period of 0.5s. Calculate the maximum
amplitude for the motion, which allows the particle to remain in contact with the
platform throughout the motion.
17) A light elastic sting hangs vertically with its upper and fixed and a body attached to its
lower and. The body initially rests in equilibrium with sting stretched 5cm beyond its un
stretched length. The body is pulled down a further distance 10cm and released from rest.
1
Show that the period of the ensuring motion as given by T = ( 2 π +3 √ 3 )
2L
18) A particle s moves about O in a circular path of radius r and with angular, velocity w in a
horizontal plane as shown.
S

107 | P a g e
r

P O Q

(i). Show that S executes shm along diameter PQ


π
(ii).If the radius is 100cm and the angular velocity is rads-1. Calculate the velocity
4
of S when its projection on PQ is 50cm form O.
19) A mass m is suspended from a rigid support by a string of length L, the mass is pulled a
side so that the string makes an angle θ with the vertical and then released
(i). Show that the mass executes s.h.m with a period.

T = 2π

L
g
(ii).Explain why this mass comes to a stop after a short time.
20) A glass U –tube contains a liquid. Air is gently blown into one of the limbs and on
releasing the pressure, the liquid oscillates in the tube.
(i). Show that the liquid oscillates with s.h.m
(ii).Explain why the oscillations eventually die out.

CALCULATIONS ON SIMPLE HARMONIC MOTION

1) A mass of 0.1kg is suspended from a light spring of force constant 24.5Nm-1. Calculate
the potential energy of the mass.
2) A mass of 0.1kg suspended from a spring of force constant 24.5Nm-1 is pulled vertically
down wards through a distance of 5.00m and released. Find the;
(i). Period of oscillation
(ii).Position of the mass 0.3seconds after release.
3) A uniform cylinder rod of length 0.08m, cross sectional area 0.02m2 and density 900kgm-
3 floats vertically in a liquid of density 100kgm-3. The rod is depressed through a distance
of 0.005m and then released.
(i). Show that the rod performs simple harmonic motion
(ii).Find the velocity of the rod when it is at a distance of 0.04m above the
equilibrium position.
4) A mass of 0.5kg is suspended from the free ends of two springs of force constants
100Nm-1 and 50Nm-1. Respectively as shown

108 | P a g e
100Nm-1 50Nm-1

Calculate the;
(i). Extension produced
(ii). Tension in each spring
(iii). Energy stored in the spring
(iv). The frequency of small oscillation when the mass is given a small vertical
displaced.

5)

Spring Horizontal surface

1.0kg

A spring of force constant 100Nm-1 and normal length 20.0cn is fixed to the vertical wall
as shown above. A wooden block of mass 1.0kg presses against the spring through a
compression of 10.0cm when released the block moves along a horizontal surface of
coefficient of friction is 0.2. Find the;
(i). Energy stored in the compressed spring
(ii).Speed with which the block moves when it has just lost contact with the spring
(iii). Distance moved by the block from the time it loses contact with the spring to
when it comes to rest.

6)

1.2m 4.0kg
1.2m

3.0m

109 | P a g e
A body of mass 4kg rests on a smooth horizontal surface. Attached to the body are two
pieces of light elastic strings each of length 1.2m and force constant 6.25Nm-1. The ends
of the strings are fixed to two points A and B 3.0m apart as shown above.
Calculations
The body is the pulled through 0.1m towards and then released
(i). Show that the body will execute simple harmonic motion
(ii).Find the period of oscillation of the body
(iii). Calculate the speed of the body when it is 0.03m from the equilibrium position.
7) A mass of 0.2kg is put on a scale pan of negligible mass hanging on a spring of constant
40Nm-1. The mass is then depressed 30m. below the equilibrium position and then
released; Calculate the;
(i). Frequency of oscillation
(ii).Velocity of the mass when its 1cm above the equilibrium position
(iii). The maximum amplitude of oscillation for the 0.2kg mass to stay in contact
with the pan throughout.
8) A wave is described by the equation, y = 0.2 sin (6x - 100πt) meters. Find;
(i). The wave length
(ii).The speed of the wave
9)

S1

S2

1kg

A mass of 1kg is hung from two springs S1 and S2 connected in series where the force
constant of the springs are 100Nm-1 and 200Nm-1 respectively. Find;
(i). The extension produced in the combination
(ii).The frequency of oscillations of the mass if it is pulled downwards through a
small distance and then released.
(iii). Explain with the aid of a sketch graph, what would happen to the oscillations
above if the mass was immersed in a liquid such as water.
10) A uniform cylindrical rod of length 0.08m, cross sectional area 0.02m3 and density floats
vertically in a liquid of density 1000kgm-3. The rod is depressed through a distance of

110 | P a g e
0.005m and then released.
(i). Show that the rod of performs simple harmonic motion
(ii).Find the frequency of the resultant oscillation
(iii). Find the velocity of the rod when it is a distance of 0.004m above the
equilibrium position.
11) A spring of force constant 40Nm-1 is suspended vertically. A mass of 0.1kg suspended
from the spring is pulled down a distance of 5mm and released. Find the;
(i). Period of oscillation
(ii).Maximum oscillation of the mass
(iii). Net force acting on the mass when it is 2mm below the entre of oscillation.
12) A particle executing simple harmonic motion vibrates in a straight line. Given that the
speeds of the particles are 4ms-1 and 2ms-1 when the particle is 3cm and 6cm respectively
from the equilibrium calculate;
(i). Amplitude of oscillation
(ii).Frequency of the particle
π
13) Given that X = 16cos (0.5 π t+ , describe the displacement of a particle from rest
3
position. Find the;
(i). Amplitude
(ii).Frequency
(iii). Period
(iv). Maximum velocity
(v).Maximum acceleration
(vi). Phase angle.
14) A particle is moving with s.h.m of period 8.0s and amplitude 5.0m find;
(i). Speed of the particle when it is 3.0m from the centre of its motion.
(ii).Maximum speed of the particle
(iii). Maximum acceleration.
15) A particle moves with simple harmonic motion about a mean position O when passing
through two points which are 2m and 2.4m from O, the particle has speeds of 3ms-1 and
1.4ms-1. Respectively find the;
(i). Amplitude of the motion
(ii).Greatest speed attained by the particle
(iii). Maximum acceleration
(iv). Velocity of the particle m from O
16) A particle is moving with s.h.m of period of 16s and amplitude 10m. Find the speed of
the particle when it is 6.0m from its equilibrium position. Also find how far the particle is
from the equilibrium position 1.5s after passing through it and its speed at the instant.
17) The displacement of a body moving with simple harmonic motion is given by x = 3.5sin
(4 πt + 0.2). Find the amplitude w, period, T, velocity V at a time t = 1.5s.

111 | P a g e
18) If a pendulum has a period of 1.5s at the Earth’s surface. What would be its period at a
height of 6000km above the Earth’s surface assuming that the radius of the Earth is
6000km.
19) A helical spring gives a displacement of 5.0cm for a load of 500g. calculate;
(i). Period of small vertical oscillation
(ii).Find the maximum displacement produced when a mass of 80g is dropped from a
height of 10cm onto a light pan attached to the spring.
20) A ball of mass 20g is attached to an elastic spring of nature length 80cm with one end
fixed to a ridge support. The spring is stretched to 120cm along a horizontal frictionless
table and then released. The spring is of force constant 400Nm-1 for an instant when the
ball is 100cm from the fixed point, calculate the;
(i). Kinetic energy
(ii).Potential energy
(iii). Velocity of the ball.
21) A light spring is suspended from a rigid support and its free end carries a mass of 0.40kg
which produces on extension of 0.060m on the spring. The mass is slightly pulled down,
an additional 0.060m and released such that the mass oscillates with s.h.m. calculate the
Kinetic energy of the mass as it passes through the midpoint of its motion.
22) Two springs each of spring constants 200Nm-1 and 100Nm-1 respectively are suspended
side on a rigid support. A mass of 2.0kg is suspended from the free ends of the two
springs as shown in the figure below;

2.0kg

Calculate the;
(i). Extension produced
(ii). Tension in each spy
(iii). Energy stand in the springs
(iv). Frequency of the small oscillations that would result when the mass if given
a small vertical displacement.
23) A particle describes shm about a point O as centre and the amplitude of motion is a
π
metres. Given that the period of the motion s. and that maximum speed of the particle is
4

112 | P a g e
16ms-1. Find;
(i). The speed of the particle at a point a distance of ½a from O.
(ii).The time taken to travel directly from O to the point in (i) above .
24) A particle P describes shm making 3 complete oscillations per second. At a certain
instant P is at the point O and is moving at its maximum speed of 5ms-1.
(i). Find the speed of P 0.05s. after it passes through O
(ii).If after passing through O, P first comes to instantaneous rest at a point A, find the
average speed of P as it moves from O to A.
25) A particle is released from rest at a point A at a time t = O and performs shm about a
mean position B. The particle just returns to A during each oscillation. Given that
AB=2 √ 2m and tat the particle passes through B with a speed of π √2 ms-1. Find the value
of t when the particle is first travelling with a speed of π ms-1. How far from B is the
particle then .
26) A body of mass 0.50kg hangs from a spiral spring when pulled down 10cm below the
equilibrium position O and released. It makes 50 vertical oscillations in 25s. Assuming
the body oscillates with shm, find;
(i). The body’s speed as it passes through O
(ii).The acceleration of the body when it is 10.0cm below O
(iii). The Kinetic energy of the body when it is 5.0cm below O
(iv). The potential energy when the body is 2.5cm below O.
27) The figure below shows a block of mass 0.10kg resting on a smooth horizontal surface
and attached to two springs S1 and S2 of force constants 60Nm-1 and 100Nm-1
respectively.

S1 S2

0.10kg

The block as pulled a distance of 4.0 x 10-2m to the right and released.

(i). Show that the mass oscillates with shm and frequency of oscillation
(ii). Find the new amplitude of oscillation when a mass of 0.06kg is dropped vertically on to
the block as the block passes the equilibrium position. Assume that the mass sticks to
the block.
28) A body of mass 0.10kg moves with shm of amplitude 10cm and frequency 10Hz. The
displacement is 3.0cm when, time is 0.05s.
(i). Find an expression for the displacement as a function of time.
(ii). Calculate the K.e of the mass when the displacement is 5.0cm.
29) A spring elongates 2.0cm for a load of 15.0g. If a body of mass 294g is attached to the spring and
set into vibration with amplitude of 10.0cm. find;

113 | P a g e
(i). The force constant
(ii). The period of oscillation
(iii). The maximum speed of the vibrating body.
30) A 8.0kg body performs shm of amplitude 30cm. the restoring force is 60N when the displacement
is 30cm. find;
(i). The period
(ii). The acceleration when the displacement is 12cm
(iii). The kinetic energy and potential energy when the displacement is 12cm.
31) A horizontal platform is executing shm along a horizontal line with a frequency of 2.0Hz and
amplitude of 3.0cm.
(i). Calculate the maximum acceleration
(ii). Find the speed when the displacement from the equilibrium position is 1.5cm.
(iii). A small block is placed on the platform without changing the motion. Find the minimum
value of the coefficient of Kinetic friction so that the block will not slip with respect to
the platform.
32) A mass of 0.1kg moves with shm with period of 4.0s and amplitude 25cm. the mass itself is at
25cm initially at time t = 0.
(i). Calculate the frequency of oscillation of the mass
(ii). Find the displacement of the mass at time t = 2,.5s.
(iii). Find the magnitude and direction of the acceleration when t = 2.5s.
33) A ball of mass 20g is attached to an elastic spring of natural length 80cm with one end fixed to
support. The spring is stretched to 120cm along the horizontal frictionless table and then released.
If the ball is 100cm from the fixed point and the elastic constant of the spring is 400Nm -1. Find;
(i). The Kinetic energy of the ball
(ii). Velocity of the ball
(iii). Sketch energy displacement graph of the system above.
34) A mass of 0.1kg is placed on a frictionless horizontal surface and connected to identical springs
of negligible mass and of spring constant 33.5Nm-1. The springs are then attached to the fixed
points P and Q on the surfaces as shown.

0.3 0.3

35) A body of mass 1.0kg moving with s.h.m has speeds of 5ms-1 and 3ms-1 when it is at
distances of 0.10m and 0.20m respectively from the equilibrium point. Find the
amplitude of motion.
36) A particle executing s.h.m vibrates in a straight line. Given that the speeds of the particle
are 4ms-1 and 2ms-1 when the particle is 3cm and 6cm resp. from the equilibrium.
Calculate the;
(i). Amplitude of oscillation
(ii).Frequency of the particle

114 | P a g e
37) A horizontal spring of force constant 200Nm-1 fixed at one end has a mass of 2kg
attached to the free end and resting on a smooth horizontal surface. The mass is pulled
through a distance of 4.0cm and released. Calculate the;
(i). Angular speed
(ii). Maximum velocity attained by the vibrating body
(iii). Acceleration when the body is half way towards the centre from its initial
position.
38) A piston in a car engine performs s.h.m of frequency 12.5Hz if the mass of the piston is
0.50kg and its amplitude of vibration is 45mm; find the maximum force on the piston.
39) A body executing s.h.m has a velocity of 0.03ms-1, when its displacement is 0.04m, and a
velocity of 0.04ms-1 when its displacement is 0.03m. find;
(i). The amplitude
(ii). The period of oscillation
(iii). If the mass of the body is 50g. calculate the total energy of oscillation
40) A test tube weighs 10.0g; it is floated vertically in water by placing a mass of 10g at the
bottom. The tube is depressed slightly into the water and released.
(i). Show that the test tube performs s.h.m
(ii). If the diameter of the test tube is 20cm, find the period of the resulting oscillation.
41) A particle is moving with shm of period 8.0s and amplitude 5.0m. find;
(i). The speed of the particle when it is 3.0m from the centre of its motion
(ii). The maximum speed
(iii). The maximum acceleration.
42) A particle is shm of period 16.0s and amplitude 10m. find the speed of the particle when
it is 6.0m from the equilibrium position.
43) A particle moving with shm has a speed of 8.0ms-1 and an acceleration of 12ms-2 when it
is 3.0m from its equilibrium position. Find;
(i). The amplitude of the motion
(ii).The maximum velocity
(iii). The maximum acceleration.
44) A particle moves with shm of period of 4.0s and amplitude 4.0m. its displacement from
the equilibrium position is X. find the time taken f or x
(i). From x = 4.0m to x = 3.0m
(ii). From x = -4.0m to x = 3.0m
(iii). From x = 1.0m to x = 3.0m
45) A light string carrying a small bob of mass 5.0 x 10-2kg hangs from the roof of moving
vehicle. If the acceleration is 2.0ms-2 which is horizontal from left to right.
(i). Sketch the forces acting on the bob
(ii).Determine the angle which the string makes with the vertical.
46) A body of mass 200g is executing shm with amplitude of 20mm. the maximum force
which acts upon it is 0.064N. find;
(i). Its maximum velocity

115 | P a g e
(ii).Its period of oscillation.
47) A body of mass 0.30kg executes shm with a period of 2.5s and amplitude of 4.0 x 10-2m.
determine the;
(i). Maximum velocity of the body
(ii).Maximum acceleration of the body
(iii). The energy associated with motion.
48) A particle moves with shm in a straight line with amplitude 0.05m and period 12.0s. find;
(i). Maximum speed
(ii).Maximum acceleration of the particle.
49) The bob of a simple pendulum moves with shm with amplitudes 8.0cm and period 2.00s.
its mass is 0.50kg. find the;
(i). Speed of the bob
(ii).Kinetic energy of the bob
50) The displacement y in metres from the equilibrium position of a particle moving shm is
given by y = 0.05sin 6πt where t – time in seconds
(i). State the amplitude of the oscillations
(ii).Find the period of oscillation
(iii). Calculate the maximum acceleration of the particle.
51) A light spring is suspended from a rigid support and its free end carries a mass of 0.40kg
which produces an extension of 0.060m in the sting. The mass is then pulled down a
further 0.060m and then released causing the mass to oscillate. Calculate the kinetic
energy as it passes through the midpoint of its motion.
52) A piston in a car engine performs shm. The piston has a mass of 0.50kg and its amplitude
of vibration is 45mm. the revolution counter in the car roads 750 revolution per minute.
Find the maximum force on the piston.
53) A 100g mass is suspended vertically from a light spring and the extension in the
equilibrium position is found to be 10cm. the mass is now pulled down a further 0.5cm
and is then released from rest.
(i). Show that motion is shm
(ii). Find the period of oscillation
(iii). What is the maximum kinetic energy of the mass.
54) A light platform is suspended by two identical springs each having constant 20Nm-1, as
shown.
Plat form

Spring

116 | P a g e
Calculate the weight which must be placed on the centre of the platform in order to produce
a displacement of 3.0cm.
55) A mass hangs from a light spring. This mass is pulled down 30mm from its equilibrium position
and then released from rest. The frequency of oscillation is 0.50Hz. calculate;
(i). The angular frequency of the oscillation
(ii). The magnitude of the acceleration at instant is released from rest.
56) (a) When a metal cylinder of mass 0.2kg is attached to the lower end of a light helical spring the
upper end of which is fixed, the spring extends by 0.16m the metal cylinder is then pulled down a
further 0.08m. Find the force that must be exerted to keep it there is Hooke’s law is obeyed.
(b) The Cylinder is then released. Find the period of vertical oscillations and the kinetic energy
the cylinder possess when it passes through the mean position.
57) A particle of mass 1.0kg is exciting shm of amplitude 3.6 x 10 -2m between two points A and B
about point O as the centre of oscillation. The maximum restoring force on the particle has a
magnitude 3.52N. calculate;
(i). Period of the motion
(ii). The kinetic energy of the particle a point in the path of motion a distance 4.5 x 10 -2m fromA.
(iii). The total energy of the particle .
58) A spring of natural length 30.0cm is suspended vertically with its upper fixed. A mass of 0.150kg
suspended at rest from the lower end increases the length of the spring to 35.5cm. The mass is
then pulled down a further distance of 3.0cm and released. Find the maximum kinetic energy of
the mass.
59) A wooden block of mass 2kg is attached to a spring of natural length 25.0cm and placed on a
horizontal surface. The free end of the spring is fixed to rigid support as shown.

3.0N

10cm

A horizontal force of 3.0N acts on the block and causes the spring to extend by 10cm and the
force is removed.
(i). Show that the motion is shm
(ii). What is the total energy of the vibrating system
(iii). What is the velocity of the block when its 5cm from its equilibrium

TOPIC 13: SURFACE TENSION


DEFINITIONS
1. Define the following terms; surface tension, angleofcontact,capillarity,cohesion,adhesion,free
surface energy,coefficient of surface tension.

117 | P a g e
FACTORS, FEATURES, MERITS, DEMERITS, EXAMPLES
1. State two factors which affect the surface tension of a liquid
2. State the relationship between surface energy and surface tension
3. State the factors that can affect the magnitude of angle of contact of a liquid
4. What does the angle of contact between a liquid and a solid surface depend on
5. State any four practical applications of surface tension
6. Give two observable effects of surface tension
7. Give one application of spreading
8. State three advantages of jaegers’ method over others in determination of surface tension

DERIVATIONS AND RELATIONS


1) Sketch a graph of potential energy against separation in the two molecules in a
substance. Explain the main features of the graph above.
2) Account for the temperature dependence of surface tension.
3) Draw a graph to show the relationship between surface tension with temperature.
4) Surface tension may be defined in terms of force per unit length of in terms of energy per
unit area. Show by consideration, an increase in surface area of a liquid that these
definitions are equivalent. State any necessary condition.
5) What are the dimensions of surface tension
6) A uniform capillary tube of radius r is held vertically and lowered in a liquid of density, ρ
2 ρcos
and surface tension. show that the liquid rises to a height h, given by h = where -
rpg
is the angle of contact of the liquid with the tube and g is the acceleration due to gravity

7) Show that the excess pressure in an air bubble is given by ∆ P= where r is radius and γ
r
- surface tension

8) Show that the excess pressure in a soap bubble of radius r is given by ∆ P= , ρ -
r
surface tension
9) Derive an expression relating p1, the pressure inside a soap bubble of diameter ,d, to
the external pressure p2 and the coefficient of surface tension , ρ , of the soap solution.
10) A soap bubble of radius r1 is attached to another bubble of radius r2. If r1 is less than r2.
r1r 2
show that the radius of curvature of the common interface is
r 2−r 1
11) Define surface tension and state its dimensions
12) Deduce the dimensions of surface tension
13) Define surface tension in terms of surface energy
EXPLANATIONS
1. Explain using the molecular theory the occurrence of surface tension
2. Use the molecular theory to account for the surface tension of a liquid
3. Explain the effect of increasing temperature of a liquid on its surface tension

118 | P a g e
4. Account for the temperature dependence of surface tension
5. Give a concise explanation of the origin of surface tension in terms of inter molecular
forces
6. Explain why small drops of mercury are spherical where large drops are flat
7. Explain why large mercury drops tend to flatten out where as small drops assume
spherical shapes.
8. Account for the following;
9. A small needle may be placed on the surface of water in a beaker so that it floats
10. If a small quantity of detergent is added to the water, the needle sinks
11. Explain using simple molecular theory, where surface of a liquid behaves in different
manner from the bulk of the liquid.
12. Explain why small liquid drops are spherical while large liquid drops tend to flatten at
the top.
13. Explain why a soap film can be supported in a vertical rectangular wire frame for
some time but a film of water cannot.
14. Explain why water dripping out of a tap forms spherical drop lets
15. Explain the following;
16. why liquid drops are spherical.
17. Why a steel needle can be made to float on water even though steel is denser than
water.
18. Why water rises up a narrow capillary tube.
19. why a water proof tent will not let in a rain
20. why insects called pond- skaters can walk on water
21. Explain why the meniscus of mercury curves upwards and that of alcohol down
wards.
EXPERIMENTS
1. Describe using molecular theory the origin of surface tension
2. Describe briefly an experiment to show that surface tension decreases with increase in
temperature
3. Describe an experiment to determine the angle of contact of a particular liquid and solid
surface
4. Describe an experiment to measure the surface tension of a liquid by capillary tube
method.
5. Describe briefly how surface tension of a liquid can be determined by jaegers’ method.
6. Describe the capillary tube method of measuring surface tension of a liquid.
CALCULATIONS ON SURFACE TENSION AND CAPILLARITY
1) Calculate the work done to break up a droplet of mercury of radius 2.0mm into drops
each of radius 0.5mm.Given that surface tension of mercury is 0.52Nm-1.
2) A spherical drop of mercury of radius 2mm falls to the ground and breaks into 10 smaller
drops of equal size. Calculate the amount of work that has to be down. Assuming the
surface tension of mercury is 4.72x10-1Nm-1.
119 | P a g e
3) Calculate the radius of the capillary tube if water rises to a height of 12.5cm with it
assuming that the angle of contact between glass and water is 00 and γ = 72.7x10-3Nm-1,
g=10ms-2, and density of water =1000kgm-3
4) Water rises to a height, h, inside a clean capillary tube of radius 0.2mm when the tube is
placed vertically inside the water .calculate the height , h, if ϒ=7.0x10-2Nm-1, and the
angle of contact is zero. The tube is now pushed into the water such that 4.0cm of it is
above the surface. Describe and explain what happened.
(Given that density of water is 1000kgm-2, and g= 10ms-2).
5) A soap bubble of radius 3cm is below under isothermal condition from
(i). A soap film of very small area.
(ii).a bubble of radius 2cm. calculate the work done in each case
( γ =3.0x10 -2Nm-1 for soap solution).
6) A droplet of mercury of radius 2.00mm falls vertically and on hitting the ground its splits
into two droplets of radius 0.50mm.
(i). Calculate the change in surface energy
(ii).Account for the change in (i) above (Assume γ =0.52Nm-1).
7) A clean glass capillary tube of internal diameter 0.04cm is held with its lower end
dipping in water in a beaker and with 12cm of the tube above the surface
(i). To what height will the water rise in the tube
(ii).What will happen if the tube is now depressed until only 4cm of its length is above the
surface. Given that the surface tension of water =7.0x10-2Nm-1 and angle of contact is
zero.
8) A capillary tube is immersed in water of surface tension 7.0x10-2Nm-1 and water rises
6.2cm in the capillary tube. What will be the difference in the mercury levels if the same
capillary tube is immersed in mercury. Surface tension of mercury =0.84Nm-1, angle of
contact between mercury and glass 1400, density of mercury is 13.6x103 kgm-3.
9) Calculate the work done against surface tension forces in blowing a soap bubble of
diameter 15mm, if the surface tension of the soap solution is 3.0x10-2 Nm-1.
10) A clean glass capillary tube of internal diameter 0.60mm is held vertically with its end
lowered in water and with 80mm of the tube above the water surface. if the surface
tension of water is 7.0x10-2 Nm-1 calculate;
(i). the length of water column that rises in the tube
(ii).the new angle of contact if the tube is lowered until only 30mm of its length is above the
surface
11) A clean open ended glass u- tube has vertical limbs, one of which a uniform diameter of
4.0mm and the other of 20mm. clean water is introduced into the tube. It is observed that
the height of the water meniscus is different for the two limbs.
20mm 4mm

120 | P a g e
Calculate the difference. h, in the level of water in the two limbs of the u- tube . Assuming
that angle of contact = 00, and surface tension of water = 7.0x10-2 Nm-1
12) A spherical drop of mercury of radius 2mm falls to the ground and breaks into 10
smaller drops of equal size. Calculate the amount of work that has to be done (surface
tension = 4.72x10-1Nm-1).
13) The two vertical arms of a u- tube containing water have different internal radii of
1.0x10-3m and 2.0x10-3m respectively Determine the difference in height of the two liquid
levels when the arms are open to the atmosphere. (Given that the surface tension and
density of water are 7.0x10-2Nm-1 and 103kgm-3 respectively).
14) A soap bubble of radius 3.0cm is blown at the end of vertical tube which is connected to
a u- tube containing water ;
(i). what difference in water levels would be produced
(ii).if another soap bubble is now allowed to make contact with the first so that the radius of
curvature of the common surface is 8cm. calculate radius of the second bubble.
15) A soap bubble whose radius is 12mm becomes attached to one of radius 20mm. calculate
the radius of curvature of the common interface.
16) The end of a clean glass capillary tube having internal diameter 0.6mm is dipped into a
beaker containing water, which rises up the tube to a vertical height of 5.0cm above the
water surface in the beaker. Calculate the surface tension of water. What would be the
difference if the tube were not perfectly clean so that the water did not wet but had an
angle of contact of 300 with the tube surface.
17) Two soap vertical arms of a manometer, containing water have different internal radii of
10-3 and 2.0x10-3m respectively. Determine the difference in height of the two liquid
levels when the arms are open to the atmosphere.
(Surface tension of water =7.0x10-2Nm-1, and density =103kgm-3).
18) Calculate the work done against surface tension force in blowing a soap bubble of
diameter 15mm, if the surface tension of a soap solution is 3.0x10-2Nm-1

CALCULATIONS OF EXCESS PRESSURE IN SOAP AND WATER BUBBLE


1) Calculate the total pressure within a bubble of air of radius 0.1mm in water if the bubble
is formed 10cm below the water surface and surface tension of water is 7.27x10-2 Nm-1
and atmospheric pressure is 1.01x105 pa.
2) A bubble of steam, 1mm in diameter is formed at a depth of 10cm below the surface e of
water in a beaker. The barometric pressure is 75.1cm of mercury. What is the pressure
inside the bubble .Assume density of water =0.96x103 kgm-3, surface tension of water is
60x10-3 Nm-1, and density of mercury =13.6x103kgm-3.
121 | P a g e
3) Calculate the pressure inside a spherical air bubble of diameter 0.1cm blown at a depth of
10cm below the surface of a liquid of density 1.26x103kgm-3 ,and γ = 0.064Nm-1, density
of mercury =1.36x104kgm-3 and height of the mercury barometer is 0.67m.
4) Calculate the excess pressure inside a soap bubble of diameter 3cm if surface tension is
2.5x10-2 Nm-1.
5) The soap bubble has a diameter of 4cm. calculate the pressure inside it if the atmospheric
pressure is 1.0x105 pa and surface tension is 2.8x10-2Nm-1.
6) Two soap bubbles of radii 2.0cm and 4.0cm respectively coalesce under isothermal
conditions. If the surface tension of the soap solution is 2.5x10-2Nm-1. calculate the excess
pressure inside the resultant soap bubble.
7) A soap bubble in a vacuum has a radius of 3cm an d another bubble in a vacuum has a
radius of 6cm. if the two bubbles coalesce under isothermal conditions, calculate the
radius of the bubble formed.
8) Two soap bubbles of radii 2.0cm and 4.0cm respectively coalesce under isothermal
conditions. if the surface tension of the soap solution is 2.5x10-2Nm-1 and atmospheric
pressure is 2.6pa.Calculate the pressure inside the resulting soap bubble
9) A soap bubble of radius 3.0cm is blown at the end of a vertical tube which is connected
to u- tube containing water
(i). what difference in water levels would be produced
(ii).if another soap bubble is now allowed to make constant with the first so that the radius of
curvature of the common surface is 8cm. calculate the radius of the second bubble.

TOPIC 14: HYDRODYNAMICS


DEFINITIONS
1. Define the following terms;streamline flow,laminar flow,turbulent
flow,streamlines,viscosity,coefficient of viscosity,pressure gradient,viscous drag,terminal
velocity,velocity gradient,static pressure,dynamic pressure,equation of continuity,poiseill’s
equation,viscous drag.
LAWS, THEOREM AND PRINCIPLES
1. State Bernoulli’s principle
2. State Newton’s law of viscosity
3. State stokes’s law of viscosity

FACTORS, EXAMPLES, MERITS AND DEMERITS, FEATURES.

1. State the differences between laminar and turbulent flow in fluids


2. Give factors that determine the rate of a liquid through a pipe under steady state
3. State four applications of Bernoulli’s Principle
4. Distinguish between static pressure and dynamic pressure
5. What are the origins of viscosity in liquids
6. Give the differences between solid and liquid friction.

122 | P a g e
7. State four factors that determine the rate of volume flow of a liquid through a horizontal
capillary tube.

EXPLANATIONS
1. Explain the terms laminar and turbulent flow.
2. Explain why velocity of a liquid at a wide part of the tube is less than at a narrow part.
3. Explain why a vehicle passing near a person draws time into the vehicle
4. Explain why a car may be dragged into a bus during over take
5. Explain why two vehicles may collide in the process of overtaking each other
6. Explain with the aid of a diagram why air flow over the wings of an aircraft at take- off
causes a lift.
7. Explain qualitatively the origin of the lift force on an aero plane at take off
8. Explain the effect of temperature on the viscosity of a liquid
9. Explain the occurrence of viscosity in gasses
10. Explain the temperature dependence of viscosity in a liquid
11. Explain the origins of viscosity in liquids
12. Explain the origin of viscosity in air and account for the effect of temperature on it
13. Account for the variation of pressure of velocity of a liquid for a horizontal pipe varying
diameter
14. Briefly explain using kinetic theory of matter, the effect of temperature on viscosities of
fluids
15. Explain using kinetic theory the effect in the viscosity of a liquid of increasing the
temperature
16. Explain the variation of viscosity of a liquid with temperature
17. Explain the effects of temperature on the viscosity of liquids and gases.
18. Explain what happens when a small steel ball is dropped centrally in a tall jar containing
oil.
19. Explain why rain drops hit the ground with less force than they should.
20. Explain giving appropriate equations, the fall of a rain drop in air.
21. Explain why an aero plane has to bank its wings in order to move in circular
22. Explain why a person standing near a railway line is sucked towards the railway line
when a fast moving train passes
23. Explain the term terminal velocity
24. Explain why the acceleration of a ball bearing falling through a liquid decreases
continuously until it becomes zero
25. Explain the effect of temperature on the viscosity of gas
26. Explain why a car moving alongside a trailer at a high speed is likely to collide with the
trailer.
27. Explain why a person standing near a railway line is sucked towards the railway line
when a fast moving train passes.

EXPERIMENTS
1. Describe a simple experiment to demonstrate the two kinds of fluid flow. Illustrate your answer
with a diagram.

123 | P a g e
2. Describe stating the necessary persecutions an experiment to measure the coefficient of viscosity
of a liquid using stokes’s law
3. Describe an experiment to determine the coefficient of viscosity using poiseuille’s method.
4. Describe an experiment to determine the ration of coefficient of viscosity of a liquid at say 25 0C
and 400C
5. Describe briefly the action of the filler pump
6. Describe an experiment to measure the coefficient of viscosity of water using poiseuille’s formula
7. Describe briefly an experiment to measure the viscosity of motor oil of known density
8. Describe how terminal velocity can be measured in a liquid.

DERIVATIONS AND RELATIONS

1) Write down Bernoulli’s equation for fluid flow defining all symbols used.
2) Show that flow velocity of a liquid from a tank open to the atmosphere is given V =√ 2 gh
, where h – height above a given reference point. Show that equation is dimensionally
consistent.
3) Water stands at depth H in a large open tank whose side walls are vertical. A hole is
punched in the wall at a depth h below the water emerging stream of water strike the
floor.
4) Define coefficient of viscosity and determine its dimensions.
5) State Newton law of viscosity and hence deduce the dimensions of coefficient of
viscosity
dv
6) The rate of volume flow, of a liquid of viscosity of, through a pipe of internal radius r,
at
dv π . Pr 4
and length L is given by = where P - is pressure difference between the ends of
dt 8 δL
the pipe. Show that this equation is dimensionally consistent.
7) Define the term coefficient of viscosity of a liquid stating its SI units and dimensions.
8) The resistive force on a steel ball bearing of radius r, falling with speed V. in a liquid of
viscosity f is given by F = kŋrv where K is a constant. Show that K is dimensionless.
9) Derive an expression for the terminal velocity of a steel ball bearing of radius r, and
density ρ falling through a liquid of density σ .
10) Derive an expression for the terminal velocity of a sphere of radius a falling in a liquid of
viscosity.
11) (i) A liquid of density p is maintained at a depth h, in a tank which has a tap at the
bottom. Derive an expression for the velocity at which the liquid escapes from the tank

(ii) If the tank is mounted on a stand of height 6m above the ground and the height h is
1.2m. Calculate how far from the foot of the stand a jet of water from a tap would reach
the ground.

12) What are the origin of viscosity in liquids


13) Sketch the velocity time graph for the motion of an oil drop in air.

124 | P a g e
CALCULATIONS

1) A gardener sprinkler has 150 small holes each 2,0mm2 area. If the water is supplied at a
rate of 3.0 x 10-3 m3s-1. What is the average velocity of the spray.
2) A known sprinkler has 20holes each of cross sectional area 2.0 x 10-2cm2. The sprinkler is
connected to a horse pipe of cross sectional area 24cm2. If the speed of water in the horse
pipe is 1.5ms-1, estimate the speed of water in the horse pipe 1.5ms-1, estimate the speed
of the water as it emerges from the holes.
3) Water flow along a horizontal pipe of cross sectional area 45cm2 which has a constriction
of area 12cm2 at one place. If the speed of the water at the constriction is 4ms-1, calculate
the speed of water in the wider section.
4) A fluid of density 103kgm-3 flows in a horizontal tube. If the pressure at the entry and
exist are 105 pa and 103pa respectively and given that the velocity at the entry is 8ms-1.
Calculate the velocity of the fluid at the exist.
5) Water enters a house through a pipe of inner diameter 2.0cm at the pressure of 4.0 x 105
Nm-2. The pipe is leading to the 2nd floor bathroom 5.0m above and has a diameter 1.0cm.
when the flow velocity at the inlet pipe is 4ms-1; find the flow velocity and pressure in the
bathroom.
6) Water of negligible viscosity flows steadily through a horizontal pipe of varying cross
sectional area. At point A of cross sectional area 10cm2 the velocity is 0.2ms-1. What is
the pressure difference between A and B if the cross sectional area of B is 2.5cm2. Given
that density of water is 1000kgm-3.
7) In the diagram below; the diameter of the main pipe is 5times the diameter of the
construction. Given that the water flows through the pipe and the pressure difference
between the main pipe and the construction is 2.0 x 105Nm-2. Find the velocities V1 and
V2 at the main pipe and the construction respectively (Density of water = 103kgm-3)

V1
V2
P1
P2

8) Water flowing in a pipe on the ground with a velocity of 8ms-1and a gauge pressure of
2.0 x 105pa is pumped into water tank 10m above the ground. The water ends the tank at
pressure of 1.0 x 105 pa. Calculate the velocity with which the water enters the tank.
9) A certain section of a horizontal water pipe, the static pressure is 1.96 x 105pa, the total
pressure is 2.0 x 105pa and area of cross section is 20cm2. If the density of water is

125 | P a g e
103kgm-3, find the volume flow rate in the pipe.
10) Water flows steadily through a horizontal tube which consists of two parts joined end,
one part is 21cm long and has a diameter of 0.075cm if the pressure difference between
the ends of the tube is 14cm of water, find the pressure difference between the ends of the
first part.
11) An open tank holds water 1.25m deep. A small hole of cross sectional area 3cm2 is made
at the bottom of the tank. Assuming that the density of water is 103kgm-3. Calculate the
mass of water per second initially flowing out of the hole.
12) A particular air craft design calls for a dynamic lift of 2.4 x 104N on each square metre of
the wing when the speed of the air craft through the air is 80ms-1. Assuming that the air
flow past the lower surface is equal to the speed of the air craft. What is the required
speed of air over the upper surface of the wings.
13) In attest experiment on a model aero plane in a wind funnel, it was found out that the
speeds on the upper and lower surfaces of the wings are 81ms-1 and 57ms-1 respectively.
If the wings surface is 3.2m2 and density of air is 1.3kgm-3. Find the dynamic force on the
wing.
14) A pitot -static tube fitted with a pressure gauge is used to measure the speed of a boat at
sea. Given that the density of sea water is 1050kgm-3 and speed of the boat does not
exceed 10ms-1. Calculate the maximum pressure on the gauge.
15) Water flows steadily along a uniform flow tube of cross sectional area 3.0cm2. The static
pressure is 1.20 x 105pa and the total pressure is 1.28 x 105Pa. assuming that the density
of water is 103kgm-3. Calculate;
(i). Flow Velocity
(ii).Volume flow
(iii). Mass of water through a section of the tube per second.
16) The static pressure in a horizontal pipe line is s4.3 x 104Pa. the total pressure is 4.7 x
104pa and the area of the cross sectional is 20cm2. If the liquid is considered in
compressible and non- viscous and has density of 1.0 x 103kgm-3. Calculate the velocity
in the pipeline.
17) Aventuri meter has a horizontal tube with a constriction which replaces part of the piping
system as shown below. If the cross sectional area of the main pipe is 5.81 x 10-3m2 and
that of the constriction is 2.58 x 10-3m2, find the velocity of the liquid in the main pipe.
18) The figure below shows a horizontal tube of diameter 5cm and a constriction of diameter
2cm.

H1 = 10cm

H2 = 4cm

126 | P a g e
Find;
(i). Fluid velocity
(ii). Velocity of the fluid at the constriction
(iii). The rate of mass flow if the fluid has a density of 700kgm-3.
19) An aero plane has a wing are of 40m2. At take -off, the speed of air above and below the
wings are 120ms-1 and 100ms-1 respectively. Find the lifts on the aero plane if the density
of air is 1.3kgm-3.
20) Air flows over the upper surfaces of the wings of an aero plane at a speed of 120ms-1 and
past the lower surfaces of the wing at 110ms-1. Calculate the lift force on the areoplane if
it has a total wing area of 20.0m2. (Take density of air = 1.2kgm-3).
21) Water flows steadily through a non -uniform pipe at a rate of 400cm3s-1. If the cross
sectional are at one point is 4cm2 and at another point is 1cm2, find the pressure
difference between these two points in the pipe.
22) Water flows through a horizontal pipe of varying cross sectional if the pressure of water
is 8cm of mercury where the velocity of flow is 0.3ms-1; what is the pressure at another
point where the velocity of flow is 0.8ms-1.
23) A horizontal pipe of cross sectional area 0.4m2, tapers to cross sectional are of 0.2m2. The
pressure at the large section of the pipe is 8.0 x 104 Nm-2 and the velocity of water
through the pipe is 1.2ms-1. If atmospheric pressure is 1.01 x 105Nm-2 find the pressure at
the small section of the pipe.

h1 = 30.0cm
24)

h2 = 10.0cm

A venturi meter consists of a horizontal tube with a constriction which replaces part of
the piping system as shown in the figure.

127 | P a g e
If the cross sectional area of the main pipe is 5.81 x 10-3m2 and that of the construction is
2.58 x10-3m2, find the velocity of the liquid in the main pipe.
25) Calculate the velocity with which a liquid emerges from a small hole in the side of tank
of large cross section area if the hole is 0.2m below the surface (assume g = 10ms-2).
26) The depth of water in a tank of large cross – sectional area is maintained at 20cm and
water comes out in continuous stream out of a whole 5mm in diameter in the base.
Calculate;
(i). The speed of effect of water from the hole
(ii).The rate of mass flow of water from the hole. (Density of water is 1.0 x 103kgm-3)
27) The static pressure in a horizontal pipe line is 4.3 x 104 Pa the total pressure is 4.7 x 104
Pa and the area of cross sectional is 20cm2. The fluid may be considered to be in
compressible and non- viscous and has density 103kgm-3. Calculate;
(i). The flow velocity in the pipeline
(ii).The volume flow rate in the pipeline.
28) A large tank contains water to a depth of 1.0m. Water emerges from a small hole in the
side of the tank 20cm below the level of the surface. Calculate;
(i). The speed at which the water emerges from the hole d
(ii).The distance from the base of the tank at which the water strikes the floor on
which the tank is standing.
29) Calculate the maximum reduction in pressure that could be achieved using this pump if
the jet diameter is 2.0mm and volume rate of flow of water is 1.3 x 10-4 m3 s-1.
(Density of air = 1.3kgm-3)

30)

A horizontal pipe of diameter 36.0cm tapers to a diameter of 18.0cm at P. an ideal gas at


a pressure of 2.00 x 105Pa is moving along the wider part of the pipe of the gas at P is
1.80 x 105Pa. assuming that the temperature of the gas remains constant calculate the
speed of the gas at P.
128 | P a g e
31) A simple gardener syringe used to produce a set of water consists of a piston of area
4.00cm2 which moves in a horizontal cylinder which has a small hole of area 4.00cm2 at
its end. If the force on the piston is 50.0N. Calculate a value for the speed at which the
water is forced out of the small hole assuming the speed of the piston is negligible.
32) An empty vessel which is open at the top has a horizontal tube of length 20cm and
internal radius 1.0mm protruding from one of its side walls immediately above the base
water flows into the vessel at a constant rate of 1.5cm3s-1. At what depth does the water
level stop rising.(Coefficient of viscosity of water = 1.0 x 10-3NSm-2, density of water 1.0
x 103kgm-3, acceleration due to gravity = 10ms-2)
33) A liquid flows steadily through two pipes, A and B, which are joined end to end and
whose informed radii are 1 and 2 respectively. If B is 8times longer than A and the
pressure difference between the ends of the composites pipe is 9000Nm-2, what is the
pressure difference across A.
34) A liquid of density 1gcm-3 flowing through a horizontal tube has a velocity of 500cm-1s at
a section where the pressure is 2.5 x 105 Pa. find the velocity at a section where the
pressure is 0.5 x 105Pa.

CALCULATIONS

1) If the coefficient of water of a particular temperature is 1.3 x 10-3Nm-1. Calculate the


frictional force on the plate of area 2 x 10-3m2 moving at a velocity of 2 x 10-2ms-1 with
respect to a fluid plate and separated from it by a layer of water 2.0 x 103m thick.
2) A liquid flows through a horizontal tube of length 0.25m and of internal diameter 10 x 10-
4
m across which the pressure difference if 1000Nm-2. If the flow rate is 6.0 x 10-11m3s-1.
Calculate the coefficient of viscosity of the liquid.
3) Water flows steadily through a horizontal tube which consists of two parts joined end to
end, one part is 21cm long and has a diameter of 0.225cm and the other 7.0cm long has a
diameter of 0.075cm. If the pressure difference between the ends of the tube is 14cm of
water, find the pressure between the ends of the first part.
4) A boiling tube which stands vertically has sealed into its lower end, a short length of a
capillary tube which is horizontal it is filled with glycerin of density 1.2 x 103kgm-3 and
viscosity 8.5 x 10-1Pa to a certain mark, and it takes 45s to fall to a second mark by
escaping through the capillary tube. With castor oil of density 0.97 x 103kgm-3, the time
taken to fall from one mark to the other is 68s. Find the Viscosity of the castor oil.
5) A student performed experiment to find the coefficient of viscosity of water with the
apparatus shown.

Liquid of density

129 | P a g e
Capillary tube

The capillary tube has a length of 25cm and an inner diameter of 1.0mm.

a) If the height his 6.0cm, find the pressure gradient between the ends of the
capillary tube.
b) If 17cm3 of water flows out of the ends of the formula to calculate the
coefficient of viscosity of water
c) How much liquid will flow out of the end of the capillary tube 5minutes if
only the radius of the capillary tube is halved.
6) The figure below shows a tank containing light lubricating oil the oil flows out of the
tank through a horizontal pipe of length 0.1m and internal diameter 4.0mm.

Pipe, 4.0mm internal diameter


1.2m

0.10m

(i). Calculate the volume of oil which flows through the pipe in one minute when
the level of oil in the tank is 1.2m above the pipe and does not significantly
alter during this time.
(Density of oil = 920kgm-3, coefficient of viscosity of oil = 8.4 x 10-2Pa)
(ii).It is found that the volume flow is greater at a higher temperature. Assuming
that density changes can be ignored, suggested an explanation for this effect
in terms of the nature of the viscous force.
7) An empty vessel which is open at the top has a horizontal capillary tube of length 20cm
and internal radius 1.0mm protruding from one of its side walls immediately above the
base. Water flows into the vessel at a constant rate of 1.5cm3s-1. At what depth does the
water level stop rising. Assuming the flow is steady.
(Coefficient of viscosity of water = 1.0 x 10-3 Nsm-2, density of water is 1.0 x 103 kgm-3).

130 | P a g e
8) A metal sphere of radius 2.0 x 10-3m and mass 3.0 x 10-4 kg falls under gravity centrally
down a wide tube filled with a liquid at 350C. The density of the liquid is 700kgm-3. The
sphere attains a terminal velocity of magnitude 40.0 x 10-2 ms-1. The tube is emptied and
filled with another liquid at the same temperature of density 900kgm-3. When the metal
sphere falls centrally down the tube. It is found to attain a terminal velocity of magnitude
25.0 x 10-2ms-1. Determine at 350C, the ratio of the coefficients of viscosity of the 2nd
liquid to that of the first.
9) A metal sphere of radius 2.0 x 10-3 and mass 3.0 x 10-4 kg falls under gravity centrally
down a wide tube filled with a liquid at 250C. The density of the liquid is 8.0 x 102kgm-3.
The sphere attains terminal velocity of magnitude 0.3ms-1. The tube is emptied and filled
with another at the same temperature and of density of 9.0 x 102kgm-3. When a metal
sphere falls centrally down the tube it’s found to attain a terminal velocity of magnitude
0.2ms-1. Determine at 250C, the ratio of coefficient of viscosity of first liquid to that of the
second liquid.
10) A liquid whose coefficient of viscosity of is 2.2Nsm-2 has a density of 760kgm-3.Calculate
the terminal velocity of the steal ball of density 7800kgm-3if the radius is 2.0 x 10-3m.
11) Find the terminal velocity of oil drop of radius 2.5 x 10-6m which falls through air
(Neglect the density of air and take coefficient of Viscosity as 1.8 x 10-5 Nsm-2, density of
oil = 900kgm-3 )
12) Calculate the viscous drag on the oil drop of radius 0.1mm falling through air at its
terminal velocity (Viscosity of air 1.8 x 10-5Nsm-2, density of oil is 850kgm-3, density of
air is 1.3kgm-3).
13) 27 spherical rain droplets of the same mass and radius are falling down with a terminal
velocity of 15ms-1 if they coalesce to form a big drop what will be its terminal velocity
(Neglect air buoyancy).

14) A spherical rain drop of radius 2.0 x 10-4m falls vertically in air at 200C. If the densities of
air and water are 1.2kgm-3 and 1000kgm-3 respectively and the viscosity of air at 200C is
1.8 x 10-5Pas. Calculate the terminal velocity of the drop.
15) A Spherical oil drop of density 900kgm-3 and radius 2.5 x 10-6m a charge of 1.6 x 10-19C.
the drop falls under gravity between two metal plates.
(i). Calculate the terminal velocity attained by the drop
(ii).What electric field intensity must be applied between the plates in order to
keep the drop stationary (Density of air 1.3kgm-3, coefficient of viscosity of
air = 1.8 x 10-3Nsm-2).
16) Castor oil at 200C has coefficient of 2.42 pas and a density of 9.40 x 102kgm-3, calculate
the terminal velocity of a steel ball of radius 2.0mm falling under gravity in the oil, taking
the density of steel as 7.8 x 103kgm-3.
17) A steel ball bearing of diameter 8.0mm is timed as it falls through oil at steady speed over
a vertical distance of 0.20m, it takes 0.56s. assuming the density of steel is 7.8 x 103kgm-3
and that of the oil 9.0 x 102kgm-3, calculate;
131 | P a g e
(i). The weight of the ball
(ii).The up thrust on the ball
(iii). The viscosity of the oil
18) A steel ball bearing of diameter 8.0mm falls steadily through oil and covers a vertical
height of 20.0cm in 0.56s. If the density of steel is 7500kgm-3 and that of oil is 900kgm-3.
Calculate;
(i). Up thrust on the ball
(ii).Viscosity of the oil.
19) A metal ball of diameter 10mm is timed as it falls through oil at a steady speed. It takes
0.5s to fall through a vertical distance of 0.30m. assuming that the density of the metal is
7500kgm-3 and that of oil is 900kgm-3, find;
(i). The weight of the ball
(ii).The up thrust on the ball
(iii). The coefficient of viscosity of the oil.
20) A liquid flows steadily through a horizontal pipe of length 3.2cm. If the amount of liquid
collected at one end is 6gs-1, what is the pressure difference between the two ends of the
pipe, hence calculate the pressure gradient. (Given density of liquid = 1.2 x 103 kgm-3,
coefficient of liquid = 0.92Nsm-2).
21) A metal ball of diameter 12mm is timed as it falls through oil at a steady. It takes 0.6
seconds to fall through a vertical distance of 0.48m. Assuming that the density of the
metal is 8000kgm-3 and that of oil is 900kgm-3, find;
(i). The weight of the ball
(ii).The Up thrust on the ball
(iii). The Coefficient of viscosity of the oil.

TOPIC 11: MECHANICAL PROPERTIES OF MATTER


DEFINITIONS
1. Define the following terms;elasticity,strength,stiffness,ductility,brittle materials,elastic
materials,plastic materials,elastic deformation,plastic deformation,tensile stress,tensile
strain,young’s modulus,proportional limit,elastic limit,yield point,breaking stress,work
hardening,mealing,toughness,force constant.

PRINCIPLES, LAWS AND THEOREM


1. state Hooke’s law of elasticity
EXPERIMENTS
1. With the aid of a labeled diagram, describe an experiment to determine the young’s
modulus of a steel wire
2. Describe an experiment to verify the Hooke’s law.
3. With the aid of a labeled diagram describe an experiment to investigate the relationship
between tensile stress and tensile strain of a steel wire

132 | P a g e
4. Describe an experiment to determine the young’s modulus for a metal wire. indicate
possible sources of errors and precautions taken to minimize them
5. Describe an experiment to verify Hooke’s law and state any precautions required
EXPLANATIONS

1) Explain why the following precautions are taken during an experiment to determine the
young’s modulus of a metal wire;
(i). Two long, thin wires of the same material are suspended from a common support
(ii).The readings of the vernier scale are also taken when the loads are gradually
removed in steps.
2) Explain the energy changes which occur during plastic deformation
3) Explain the factors that determine the extension produced in an elastic material.
4) Explain the term plastic deformation in metals .
5) Explain briefly the physical process involved in plastic deformation and work hardening
of metals
6) In an experiment to measure the young’s modulus for steal, a wire is suspended vertically
and loaded at the free end.
(i). Why is the wire long and thin
(ii).Why is a second steel wire suspended adjacent to the first
7) Explain the energy transformations that occur when a wire is stretched.
(i). Elastically
(ii).Plastically
8) Explain why does an iron roof make cracking sound at night
9) Explain why rubber does not obey Hooke’s law.

CALCULATIONS.

1) A force of 10N causes an extension of 15cm. what extension will be produced by a force
of 12N
2) A spring is stretched 0.02m by a weight of 4.0N. calculate the;
(i). Force constant K
(ii).Weight of an object which can cause an extension of 0.08m
3) A mass of 0.5kg causes a spiral spring to extend by 4cm. calculate the force that would
cause extension of 6cm.
4) A load of 4N stretches a spring by 0.5cm. calculate the extension when a load of 8N is
applied
5) A force of 20N extends a spring by 10mm. find the extension in mm caused by a mass of
0.5kg

133 | P a g e
FACTORS, EXAMPLES, MERITS AND DEMERITS, FEATURES.
1. State the measurements necessary in the determination of Young’s modulus of a metal
wire
2. State any two examples of ductile materials as in elasticity
3. Give any three factors that determine the extension produced in an elastic material
4. Classify the following materials as ductile or brittle; chalk, glass road, iron rod, clay cup,
aluminum aid a concrete bricks.
5. Give two examples of plastic materials.

DERIVATIONS AND RELATIONS


1) Determine the dimensions of the following;
(i). Young’s modulus
(ii).Longitudinal stress
2) Sketch a graph of stress versus strain for a ductile material and explain its feature
3) The velocity V, of a wave in a material of Young’s modulus E, and density p is given

by V =
√ E
p
, show that the relationship is dimensionally consistent

4) Derive an expression for the energy released in a unit volume of a stretched wires in
terms of stress and strain.
5) A copper wire is stretched until it breaks
(i). Sketch a stress strain graph for the wire and explain the main features of the
graph
(ii).Explain what happens to the energy used to stretch the wire at each stages
6) Determine the dimensions of the Young’s modulus
7) Derive the expression for the work done to stretch the spring of force constant K by a
distance e.
8) Sketch a graph of stress versus strain for a ductile material and explain its main
features.
9) Sketch using the same axes the stress strain curves for a glass wire a metal wire and a
rubber band. Discuss briefly the main features of the curves
10) Sketch curves showing the variation of stress with strain for a ductile material and
explain the features.
11) Compare the elastic properties of steel and glass. Draw using the same axes, the
stress, strain curves for the two materials
12) Sketch the graph of force against extension and show how it is possible to use the
graph to determine; Young’s modulus for the wire and work done in stretching
the wire.
13) The figure below shows the stress strain curves for a metal wire, rubber and glass.
P Q
R

134 | P a g e
Stress

Strain
(i). Identify each of the curves
(ii).Explain the characteristic features of the curve R.
14) A rod of mild steel of uniform cross sectional area 3.0 x 10-2 m2 and length 1.0m is
stretched steadily until it breaks.
(i). Sketch a graph to show the relationship between the force and extension.
Explain the shape of the graph.
(ii).When the applied force on the rod is 1.20 x 105N, the strain is 4.0 x 10-4.
Calculate the Young’s Modulus for steel.
(iii). The rod is found to break at stretchy force of 2.4 x 105N explain why the
stress at the section of the rod where the break occurs is likely to be much
greater than 8.0 x 107Nm-2
(iv). Just before the rod breaks its extension is about 4.0cm estimate the work
done in stretching the rod by this amount suggest why the calculated value of
the work is likely to be less than the actual value.
15) Show that work done W in stretching an elastic cord through distance x is W = ½ Fx2
and show that the equation is dimensionally consistent.
16) A wire of length L and radius r is made of a material whose Young’s modulus is E.
obtain an expression for the force constant K of the wire in terms of L, r, and E and
show that your expression is dimensionally consistent.
17) Derive in terms of stress and strain an expression for the energy stored in a unit
volume of a stretched wire.
18) A wire of length l and cross sectional area A, has a force constant K the wire is
stretched to a length (l + x) by a constant force F.
(i). Assuming Hooke’s law, find an expression for K in terms of l, A and
Young’s Modulus E for the wire

(ii).Show that the energy stored in a unit volume of the wire is equal to ½ E () x 2
l
19) Show that the energy stored per unit volume in a stretched wire is equal half the
product of the stress and strain.
20) (i) Sketch the stress strain curve for a metal wire and explain the characteristics
features of the curve for what region of the curve can Young’s modulus be defined
(ii) How does the curve in (i) above compare with that of a material like glass.
21) Sketch graph of tension against extension for;

135 | P a g e
(i). A ductile material
(ii).A brittle material
22) Show that energy stored in a unit volume of a stretched wire is equal to ½ (Stress x
strain)
23) A copper wire is stretched until it breaks
(i). Sketch a stress strain graph for the wire and explain the features of the graph
(ii).Explain what happens to the energy used to stretch the wire at each stage.
(iii). Derive the expression for the work done to stretch a spring of force
constant K, by a distance e.
24) Two wires of length L1 and L2 cross sectional area A1 and A2 and Young’s
modulus E1 and E2 respectively are joined in series. Show that the force F causing
A 1 e E1 A 2 E2
an extension e in the composite wire is given by. F =
A 2 E 2 L + A 1 E 1 L2
1

25) In an experiment to determine the Young’s modulus


(i). Explain why two wires of the same material are used
(ii).Outline the measurements to be made
26) Sketch a stress strain graph for glass ,cast iron and explain its features
27) Describe the behavior of a wire that is gradually loaded to breaking point.
28) Derive an expression for the potential energy stored in a stretched wire
29) (i) Sketch a graph of variation of stress to strain for aluminum and label the main
points on it.
(ii) Describe the salient features of the graph in (i) above.

CALCULATION ON STRESS, STRAIN AND YOUNG’S MODULUS


1) A steel wire 10m long and with a cross sectional area of 0.01cm2 is hung from a
support and a mass of 5kg is hung from the end. Calculate the new length of the
wire if the Young’s modulus of steel is 210 GPa.
2) A gymnast of mass 70kg hangs by one arm from a high bar if the gymnast’s
whole weight is assumed to be taken by the humorous bone. Calculate the stress
in the humorous if it has a radius of 1.5cm.
3) A hammer thrower swings a 7.25kg hammer in a horizontal circle at one
revolution per second. If the harmer wire is 1.20m long, 1.5mm in diameter and
made of steel of Young’s modulus 2 x 1011Nm-2. Calculate the extension produced
in it. Neglect the mass of the wire.
4) A copper wire 200cm long and 1.22mm in diameter is fixed horizontally between
two supports 200cm apart. Find the mass in grams of the load which when

136 | P a g e
suspended at the mid-point of the wire produces sag of 2cm at the midpoint at that
point. (The Young’s modulus of copper is 1.2 x 1011Nm-2)

5) Estimate the compression of the femur during running stride for a sprinter of mass
70kg.if the femur is 0.40m long and has a mean diameter of 2.0cm. (Young’s
modulus of the femur is 0.2 x 1010Pa).
6) The ends of a uniform wire of length 2.00m are fixed to points A and B which are
2.00m apart in the same horizontal line. When a 5.0kg mass is attached to the
midpoint C of the wire the equilibrium position of C is 7.5cm below the line AB.
Given that Young’s Modulus for the material of the wire is 2.0 x1011Pa find;
(i). The strain in the wire
(ii).Stress in the wire and the energy stored in the wire .State any assumptions
made.
7) A load of 1.5kg attached to the end of a wire 3.0cm long of diameter 0.46mm
stretched it by 2.0mm. Find the Young’s modulus.
8) A length of copper of square cross sectional measuring 1.0mm by 1.0mm is
stretched by a tension of 40N. Find the stress.
9) A brass wire 2.5m long of cross section area 1.0 x 10-3 cm2 is stretched 1.0mm by
a load of 0.40kg. Calculate the Young’s modulus.
10) A 0.50kg mass is hung from the end of a wire of 1.5m long and diameter 0.30mm.
If the Young’s modulus of the material is 1.0 x 1011Pa. Calculate the extension
produced.
11) Two wires A and B are made of the same material by A is twice as long as B and
has twice the radius of B. Calculate the ratio of the extension of wire B to that of
wire A when they are both subjected to the same tensile force.
12) A uniform steel wire of density 7800kgm-3 weigh 16g and is 250cm long. It
lengthens by 1.2mm when stretched by a force of 50N. calculate;
(i). Young’s modulus of elasticity of steel.
(ii).Energy stored in the wire.
13) Find the maximum load which may be placed on a steel wire of diameter 1.0mm
1
if the permitted strain must not exceed and given that the Young’s Modulus
1000
of steel if 2.0 x 1011Nm-2.
14) A steel wire of length 2.4m extends by 0.06m when a force of 900N is applied if
the cross sectional area of the steel wire is 3.0 x 10-4m2. Determine the Young’s
modulus of the wire.
15) A uniform steel wire of density 7.8 x 103kgm-3 weighs 1.6 x 10-2kg and is 2.50m
long. It lengthens by 1.2mm when stretched by a load of 8kg. calculate;
(i). Young’s modulus for steel
(ii).The energy stored in the wire

137 | P a g e
16) A steel wire of length 30m and cross sectional area 0.5mm2. Given that Young’s
modulus for steel is 2.0 x 1011pa. Calculate the force constant of the wire.
17) A uniform steel rod weighs 1.6 x 10-2 and it is 2.5m long. The rod lengthens by
1.2 x 10-3m when stretched by a force of 80N. Calculate;
(i). The young’s modulus of the rod.
(ii).Energy stored in the rod per unit volume if the density of the steel rod is
7.8 x 103kgm-3.
18) .A thin steel wire initially 1.5m long and of diameter 0.50mm is suspended from a
rigid support. Calculate;
(i). The final extension
(ii).The energy stored in the when a mass of 3.0kg is attached to the lower
end. Assume that the material obey Hooke’s law (Young’s Modulus for
steel = 2.0 x 1011Nm-2).
19) A metal wire of length 2.75m and mass 2.0 x 10-3kg stretches by 1.0 x 10-3m when
a force of 750N is applied. If the density of the metal is 8.0 x 103kgm-3. Calculate;
(i). Young’s modulus for the metal
(ii).The energy stored in the wire
20) A uniform steel wire of density 7800kgm-3 weighs 16g and is 250cm long. It
lengthens by 1.2mm when stretched by a force of 80N. calculate the;
(i). Young’s modulus of elasticity for steel
(ii).Energy stored in the wire
21) A vertical wire 350cm long diameter 0.1cm has a load of 8.50kg applied at its
lower end. If its Young’s modules is 2.0 x 1011Pa. calculate;
(i). Extension of the wire
(ii).Energy stored in the wire.

22) An aluminum wire of length 0.35m and radius 0.20mm is stretched by 1.4mm.
Young’s modules of aluminum is 7.0 x 1011Pa.
Find;
(i). The strain in the wire
(ii). Stress
(iii). Cross sectional area
(iv). Tension in the wire.
23) A mass of 11kg is suspended from the ceiling by an aluminum wire of length 2m
and diameter 2mm. find;
(i). The extension produced
(ii).The elastic energy stored in the wire.
(Young’s modulus of aluminum is 7 X 1010Pa).
24) The maximum upward acceleration of a lift of total mass 2500kg is 0.5ms-2. The
lift is supported by a steel cable, which has a maximum safe working stress of 1.0
x 108Nm-2. What minimum are of cross section of cable should be used.
138 | P a g e
25) An elastic string of cross sectional area 4mm2 requires a force of 2.8N to increase
its length by one tenth. Find the Young’s modulus for the sting. If the original
length of the string was 1m. Find the energy stored in the string when it so
extended.

CALCULATIONS
1) A copper wire and steel wire each of length 1.0m and diameter 1.0mm is joined end to
form a composite wire 2.0m long. Find the strain in each wire when the composite
stretches by 2.0 x 10-3m. (Young’s modulus for copper and steel are 1.2 x 1011Pa and
2.0 x 1011Pa resp.)
2) Two wires, one of steel and one of bronze each 1.5m long and of diameter 0.20cm are
joined end to end to form a composite wire of length 3.0m. Find the tension in this wire
which produces a total extension of 0.064cm. given that Es = 2.0 x 1011Pa, E6 = 1.2 x
1011Pa)
3) A vertical steel wire 350cm long, diameter 0.100cm has a load of 8.50kg applied at its
lower end. Find;
(i). The extension (1.86 x 10-3m)
(ii).The energy stored in the wire given that E = 2.00 x 1011Pa g = 9.81ms-2).
4) Find the maximum load to which a brass wire of radius 10-3m may be subjected if the
strain must not exceed 10-3 given that the Young’s modulus of brass is 1.0 x 1011Pa.
5) A copper wire 200cm long and 1.22cm in diameter is fixed horizontally between two
supports 200cm apart. Find the mass in gram of the load which when suspended at the
midpoint of the wire produces a vertical depression of 2cm at the point. (Young’s
modules of copper is 1.2 x 1011Nm-2).
6) A metal wire of diameter 2.0 x 10-4m and length 2m is fixed horizontally between two
points 2m apart. Young’s modulus for the wire is 2.0 x 1011Nm-2
(i). What force should be applied at the midpoint of the wire to depress it by 1.0 x
10-1m and find the work done in (i) above.
7) A uniform wire of un stretched length 2.49m is attached to two points A and B which
are 2.0m a part and in the same horizontal line. When 95kg mass is attached to the
midpoint C of the wire the equilibrium point of C is 0.75m below the line AB.
Neglecting the weight of the wire and taking Young’s Modulus for its material to be 2.0
x 1011Nm-2 find;
(i). Strain in the wire
(ii). Stress in the wire
(iii). Cross sectional area
(iv). Energy stored in the wire.
8) A and B are two fixed points on the same horizontal level and distance 48cm a part. A
light elastic string of natural length 40cm has one end attached to A and the other at B.
A body of mass 200g is attached at the midpoint of the string and hangs in equilibrium
at a point 7cm below the level of A and B. find the modulus of the string .

139 | P a g e
9) A steel wire of length 0.6m and cross sectional area 1.5 x 10-6m2 is attached at B to a
copper wire BC of length 0.39m and cross sectional area 3.0 x 10-6m2. The combination
is suspended vertically from a fixed point A and support a weight of 250N at C. find the
extension in each of the wires given that young’s modulus for steel is 2.0 x 1011 Nm-2
and that of copper is 1.3 x 1011Nm-2.
10) A uniform wire of un stretched length 2.51m is attached to two points A and B which
are 2.4m apart and in the horizontal line when 6.0 kg mass is attached at the midpoint C
of the wire the equilibrium position C is 0.52m below the line AB. Neglecting the
weight of the wire and taking its modulus of elasticity. 2.0 x 1011Pa; find;
(i). Strain in the wire
(ii).Stress in the wire and energy stored in the wire.
11) One end of a copper wire is welded to a steel wire of length 1.6m and diameter 1.0mm
while the other end is fixed. The length of the copper wire is 0.80m while its diameter
is 0.5mm. A load of 10kg is suspended from the free end of the steel wire. Find the;
(i). Extension which results
(ii).Energy stored in the compound wire (Young’s modulus for copper = 1.0 x
1011Pa, Young’s modulus for steel = 2.0 x 1011Pa).
12) Two wires, one of steel and one of bronze each 1.5m long and of diameter 0.20cm are
joined end to end to form a composite wire of length 3.0m. What tension in the wire
will produce a total extension of 0.064cm.
(Young’s Modulus for steel = 2.0 x 1011Pa, and for bronze is 1.2 x 1011Pa).
13) A steel wire AB length 0.6m and cross sectional area 1.5 x 106m2 is attached at B to a
copper wire BC of length 0.39m and cross sectional 3.0 x 106m2. The combination is
suspended vertically from a fixed point A and supports a weight of 150N at C.
(i). Find the extension in each wire
(ii). Find the energy stored per unit volume in the combination
(iii). What force stretches the combinations by 2.475mm.
14) The ends of the uniform wire of cross sectional are 3.0 x 10-6m and negligible mass are
attached to two fixed points A and B which are 1m apart in the same horizontal plane.
The wire is initially straight and an un stretched. A mass of 0.5kg is attached to the
midpoint of the wire and hangs in equilibrium with the midpoint at a distance of 10mm
below AB. Calculate the value of the Young’s Modulus of the wire.
15) A copper wire and a steel wire each 1.0m long and 1.0mm2 in cross sectional area are
laid side by side and are joined together at the ends. The composite wire is then placed
vertically with the upper end dumped and the lower end supporting a mass of 800g.
Calculate the tension in each wire and the elastic potential energy of the system, stating
any assumptions made. (Young’s modules for copper and steel are 1.2 x 1011Pa and 2.0
x 1011 Pa respectively).
16) A copper wire and steel wire each 2.0cm long and 3.0mm2 in cross sectional area are
laid side by side from a rigid support. A composite wire is placed vertically with the
lower end supporting a mass of 1.2kg Assuming that Young’s modulus for steel is 2.0 x
140 | P a g e
1011pa and that for copper = 1.2 x 1011Pa, calculate the tension in each wire, potential
energy of the whole system and state any assumption made.
Support
17)

Steel Brass
T1 T2

0.1m 0.1m
10g
A light rigid bar is suspended vertically from two wires; one made of steel and the other
made of brass as shown. Each wire is 2m long. The diameter the steel is 0.6mm and the
length of the light rigid bar is 0.2m. When a mass of 10kg is suspended from the centre
of the Light rod, the bar remains horizontal. If the Young’s modulus of elasticity of
steel Es = 2.0 x 1011Pa and that of brass E6 = 1.0 x 1011Pa Calculate; the;
(i). Tension in each wire
(ii). Extension in the steel wire and the energy stored in it.
(iii). Diameter of the brass wire
(iv). If the brass wire was replaced by another brass wire of diameter 1.0mm, where
the mass would be suspended so that the rod remains horizontal.

18) The ends of a uniform wire of length 2.00m are fixed to points A and B which are
2.0m a part in the same horizontal line. When a 5kg mass is attached to the midpoint C
of the wire, the equilibrium position of C is 7.5cm below the line AB. Give that
young’s modulus for the material of the wire is 2.0 x 1011Pa. find;
(i). The strain in the wire
(ii). The stress in the wire
(iii). The energy stored in the wire
(iv). State any assumption made
19) Two wires each 1m long and 1mm2 cross sectional one of steel and the other of brass
are connected end to end. What tensile force would be required to extend the whole
wire by 1mm. (Young’s modulus for steel is 2.0 x 1011Nm-2, and brass is 1.0 x 1011Nm-
2
).
20) A copper wire and steel wire each of length 1.0m and diameter 1.0mm are joined end
to end to form a composite wire 2.0m long find the strain in each wire when the
composite wire stretches by 2.0 x 10-3m. (Young’s modulus for copper and steel are
1.2 x 1011Pa and 2.0 x 1011Pa resp)

141 | P a g e
21) A steel wire of length 0.6m and cross sectional area 1.5 x 10-6m2 is attached at B to a
copper wire BC of length 0.39m and cross sectional area 3.0 x 10-6m2. The
combination is suspended vertically from a fixed point at A and supports a weight of
250N at C. find the extension in each of the wires given that Young’s Modulus for
steel is 2.0 x 1011Nm-2and that of copper is 1.3x1011Nm-2..
22) A copper wire and steel wire of the same diameter and of length 1.0m and 2.0m
respectively are connected end to end. A force is applied which stretches their
combined length by 1.0cm. find;
(i). The extension in each wire
(ii).The stress in the composite wire
23) The end of a copper wire of diameter 0.30mm and 1.50m if fused to one end of a steel
wire of the same diameter and 1.20m in length. The wire is then hung vertically by
attaching the free end to a fixed point and attaching a small weight to the lower end of
the steel wire. A load of 30N is then hung from the lower end of the steel wire.
Calculate;
(i). The stress in each wire
(ii).Strain in each wire and the total extension of the two wires (Es = 2.0 x 1011Nm-
2
Ec = 1.3 x 1011Nm-2).
24) A steel wire AB of length 0.60m and cross –sectional are 1.5 x 10-6m2 is attached at B
to a copper wire, BC of length 0.39m and cross sectional area 3.0 x 10-6m2. The
combination is suspended vertically from a fixed point at A and supports a weight of
250N at C. find the extension of each section of the wire. (Es = 2.0 x 1011Pa, Ec = 1.3
x 1011Pa).
25) A cylindrical copper wire and a cylindrical steel wire, each of length 1.0m and having
equal diameters are joined at one end to form a composite wire 2.0m long. This
composite wire is subjected to a tensile stress until its length becomes 2.002m.
Calculate the tensile stress applied to the wire.
(Ec = 1.2 x 1011Pa, Es = 2.0 x 1011PA).
26) A heavy rigid bar is supported horizontally from a fixed support by two vertical wires,
A and B of the same initial length and which experiences the same extension. If the
ratio of Young’s modulus of A to that of B is 2, and the ratio of the diameter of A to B
is 2. Calculate the ratio of the tension in A to that in B.
27)

Steel Brass

A B
A light rigid bar is suspended horizontally from two vertical wires one of the steel and
one of brass, as shown in the diagram. Each wire is 2,00m long. The diameter of the

142 | P a g e
steel wire is 0.60mm 10 and length of the bar AB is 0.20m. When a mass of 10.0kg is
suspended from the centre of AB the bar remains horizontal. Find;
(i). Tension in each wire
(ii). Extension of steel wire and energy stored in at
(iii). Diameter of the brass wire
(iv). If the brass wire of diameter 1.00mm replaces the former one where the mass
should be suspended so that AB would remain horizontal. (Es = 2.0 x 1011Pa, Eb
= 1.0 x 1011Pa)
CALCULATION ON ENERGY STORED IN A STRETCHED WIRE
1) Calculate the work done in stretching a steel wire 1m in length and of cross sectional area
0.030cm2 when a load of 100N is slowly applied, the elastic limit not to be exceeded.
Given that Young’s modulus of steel is 2.00 x 1011Nm-2.
2) Calculate the energy stored in a 2m long copper wire of cross sectional area 0.5mm2 if a
force of 50N is applied to it.
3) A steel wire of length 2m and radius 5.0 x 10-4 m is stretched by 4 x 10-3m. Calculate the
energy stored per unit volume as a result of this stretching force given that the Young’s
modulus for steel = 2.0 x 1011Pa.
4) A rod of original length 1.2m and area of cross sectional 1.5 x 10-4m2 is extended by
3.0mm when the stretching force is 6N. Calculate the Young’s modulus for the material
of the rod and the energy of the stretched materials.
5) A steel 1.5m is stretched by a force of 50N. Calculate the potential energy stored in the
wire. Es = 2.0 x 1011Pa).

6) A nylon guitar string 62.8cm long and 1mm diameter is tuned by stretching is 2.0cm.
Calculate;
(i). The tension
(ii).The elastic energy stored in the sting .(Young’s modulus of nylon = 2.0 x 109Pa)
7) A thin steel wire initially 1.5m long and of diameter 0.50mm is suspended from a rigid
support. Calculate;
(i). The final extension
(ii).The energy stored in the wire when a mass of 3.0kg is attached to the lower end.
Assume that the material obeys Hooke’s law. (Es = 2.0 x 1011 Nm-2)
8) Two wires, one of steel and one of bronze each 1.5m, long and diameter 0.20cm are
joined end to end to form a composite wire of length 3.0m. Find the tension in the wire
which produces a total extension of 0.064cm. (Es = 2.0 x 1011Pa, Eb = 1.2 x 1011Pa)
What is the energy stored in the wire.

CALCULATIONS ON FORCE IN A METAL BAR DUE TO


EXPANSION/CONTACT
1) A steel bar with cross sectional area of 2cm2 is heated, raising its temperature by 1200C
and prevented from expanding. Calculate.
143 | P a g e
(i). The resulting force in the bar
(ii).Heat gained by the bar.
2) A steel wire of cross sectional area 1mm2 is cooled from a temperature of 600C to 150C.
find the; Strain and the force needed to prevent it from contracting .(Young’s modulus =
2.0 x 1011Pa, coefficient of linear expansivity for steel = 1.1 x 10-5 k-1)
3) Calculate the thermal stress developed in a steel rod of area 10-4m2 which is rigidly being
damped whilst being cooled by 100K. Assume Young’s Modulus for steel is 2.0 x 1011Pa
and the linear expansivity of steel is 12 x 10-6K-1.
4) A cylindrical copper rod of length 0.5m and of diameter 4.0 x 10-2m is fixed between two
rigid supports at a temperature of 200C. The temperature of the rod is raised to 700C.
(i). Calculate the force exerted on the rigid supports at this temperature
(ii).What is the energy stored in the rod at 70.00C.
(Ec = 1.2 x 1011Nm-2, mean coefficient of linear expansivity of copper between
200C and 700C = 1.7 x 10-5 K-1.
5) A cylindrical steel rod of length 1.0m and of cross sectional area 1.5cm2 is fixed between
two rigid supports at temperature of 100C. The temperature of the rod is raised to 600C.
(i). Calculate the force exerted on the rigid supports at this temperature
(ii).What is the energy stored in the rod at 60.00C (C0efficieint of linear expansion of
steel = 1.2 x 10-5K-1, Es = 2.0 x 1011Pa).
6) A uniform steel bar of cross sectional area 1.5cm2 and length 1.00m at 100C is heated to
600C. At this temperature the ends of the bar are fixed to rigid supports. The bar is then
allowed to cool 100C. find
(i). The tension of the bar
(ii).The additional potential energy which is now stored in it.
(Coefficient of linear expansion of steel = 1.2 x 10-5/0C, Young’s modulus for
steel = 2.0 x 1011Nm-2).
7) A 20m length of continuous steel railway line of cross sectional area 8.0 x 10-3m2 is
welded into place after heating to a uniform temperature of 400C. Calculate for normal
operating conditions of 200C.
(i). The tensile strain
(ii). The tensile stress
(iii). The elastic energy in the rail
(iv). How much heat would be required to return the rail to 400C? Explain briefly why
your answer is not the same as that of (iii).
(Es = 2.0 x 1011Pa, linear expansivity of steel = 1.2 x 10-5 K-1, Density of steel = 7.8 x
103kgm-3. E of steel = 5.0 x 102Jkg-1K-1)
8) A steel bar is heated to a temperature of 5000C, its ends are then damped to two points
1m a part and the bar is then allowed to cool. Find at what temperature it will break given
coefficient of thermal expansion of steel is 11 x 10-5k-1, Young’s modulus = 2.0 x 1011Nm-
2
, breaking stress for steel = 8 x 108Nm-2.

144 | P a g e
9) A uniform metal bar of length 1.0m and of diameter 2.0cm is fixed between two rigid
supports at 250C. If the temperature of the rod is raised 750C. find;
(i). The force exerted on the supports
(ii).Energy stored in the rod at 750C. (Young’s modulus for the metals 2.0 x 1011Pa,
coefficient of linear expansion is 1.0 x 10-5 k-1).
10) A steel wire of cross sectional area 1mm2 is cooled from a temperature of 600C to 150C.
find the;
(i). Strain
(ii).Force needed to prevent it from contracting (Young’s modulus is 2.0 x 1011Pa,
coefficient of linear expansivity for steel = 1.1 x 10-5 k-1)
11) (i) A load of 60N is applied to a steel wire of length 2.5m and cross sectional area of
0.22mm2. If young’s modulus for steel is 210Gpa, find the expansion produced.
(ii). If the temperature rise of 1k causes a fractional increase of 0.0001%. Find the change
in the length of a steel wire of length 2.5m when the temperature increases by 4k.
12) Two identical steel bars A and B of radius 2.0mm are suspended from the ceiling. A mass
of 2.0kg is attached to free and of bar A. Calculate the temperature to which B should be
raised so that the bars are again of equal length. (Young’s modulus of steel = 1.0 1011Nm-
2
, linear expansivity of steel = 1.2 x 10-5k-1).
13) Two identical cylindrical steel bar each of radius 2.0mm rest in a vertical position with
their lower ends on a rigid horizontal surface. A mass of 2.0kg is placed on top of one bar
by how much must the temperature of the other bar be altered so that they are once again
equal length.
(Young’s modulus for steel = 2.0 x 1011Nm-2; coefficient of linear expansion for steel =
1.2 x 10-5 K-1).
14) A metal wire of diameter 0.2mm is cooled from a temperature of 500C to 100C. Find the
longitudinal tension set up in the wire when allowed to contract.
15) A steel rod of length 0.60m and cross sectional area 2.5 x 10-5m2 at 1000C is damped so
that when it cools. It is unable to contract. Find the tension in the road when it is cooled
to 200C. (Young’s modulus for steel = 2.0 x 1011Pa, linear expansivity of steel= 1.6 x 10-
70C-1).
16) A 20m length of continuous steel railway line of cross sectional area 8.0 x 10-3 m2 is weld
into place after heating to a uniform temperature of 400C.
(Es = 2.0 x 1011 Pa, Linear expansivity = 12 x 10-6K-1, ρ = 7800kg m-3 and Cs = 500 Jkg-
k ). Calculate, for normal operating conditions at 150C.
1 -1

(i). The tensile strain


(ii).Tensile stress
(iii). The elastic strain energy in the rail
(iv). How much heat would be required to return the rail to 400C.

TOPIC 12: HYDRO STATICS


DEFINITIONS

145 | P a g e
1. Define the following terms;pressure,apascal,compressible fluid, incompressible
fluid,upthrust,buoyancy,density,relative density,afluid,hydrometer,apparent loss in weight.
LAWS, PRINCIPLES AND THEOREMS
1. State Pascal principle of transmission of pressure in fluids
2. state Archimedes’ principle
3. state the law of floatation

FACTORS, MERITS, DEMERITS, EXAMPLES AND TEATURES


1. State any three properties of pressure exerted by a fluid on a body
2. state two factors that affect pressure in solids
3. Give three factors which affect pressure in liquids

DERIVATIONS AND RELATIONS


1) Derive an expression for the pressure at a point depth h from the surface of a liquid of
density p
2) Show that the weight of fluid displaced by an object is equal to the up thrust on the object
3) Use Archimedes’ principle to derive an expression for the resultant force on a body of
weight ,w, and density σ totally immersed in a fluid of density ρ ,
4) State Archimedes’ principle and give one application of this principle.

EXPLANATIONS
1. Explain the origin of up thrust on a body immersed in a fluid
2. Explain why a balloon filled with helium rises up to a certain height in still air and then
stops.
3. Explain why a hippotamus can easily walk on mud without sinking while a goat will
sink.
4. Explain why large water reservoirs are much wider at the base than at the top.
5. Explain the principle of operation of a hydrometer.
6. Explain why an elephant is able to walk on a soft river bank where a cow sinks on the
same bank

EXPERIMENTS
1. State Archimedes’ principle and use a rectangular block immersed in a liquid to verify
it.
2. Describe an experiment to verify Archimedes’ principle.
3. Describe an experiment to determine the relative density of an irregular solid which floats
in water
4. With the aid of a diagram describe how to measure the relative density of a liquid using
Archimedes’ principle and principle of moments.
5. Describe how you would measure the density of an irregular solid which floats in water.
6. Describe the construction and working of a hydrometer.
146 | P a g e
CALCULATIONS ON PRESSURE

1) A metal cylindrical contains a liquid of density 1100kgm-3.The area of the base of the
cylinder is 0.005m2 and the height of liquid is 5m. Calculate the force exerted by the
liquid on the base of the cylinder.
2) In the diagram, a fixed mass of dry air is trapped in bulb A. if the atmospheric pressure is
76cm of mercury, calculate the total pressure of the air in A, in

Pa

68cm

38cm Mercury

Given that the atmospheric pressure =101325 pa , density of mercury is 1.36x104 kgm-3.
3) The diagram below shows a gas trapped by mercury Colum in J- tube

26cm Gas

Mercury

If the atmospheric pressure is 1.0x105 Nm-2 and the density is 1.36x104 kgm-3, find the pressure
of the gas

4) An open, u- tube contains columns of water and kerosene over mercury as shown in the
diagram.

Water

147 | P a g e
Kerosene

8cm 10cm

A B

Calculate the relative density of kerosene.

5) A u- tube whose ends are open to the atmosphere contains water and oil as shown

h Oil

10cm

Water

Given that the density of oil is 800kgm-3


Find the value of h.
6) Calculate the pressure of the gas in the bulb (Atmospheric pressure =1.01x105 pa,
density of mercury = 1.36x104 kgm-3)

A Tube open to atmosphere

0.120m

Bulb contains gas C B


Under pressure

7) Calculate the pressure of the gas in the bulb given that atmospheric pressure is
760mmHg.

148 | P a g e
Gas under reduced 3.0mm C

Pressure B A

8) An open u- tube manometer containing an oil of density 897kgm-3 is used to measure


the pressure of gas. The oil level in the open tube is 25.0cm higher than that in the limb
connected to the gas. find
(i). the gauge pressure
(ii).the absolute pressure of the gas (atmospheric pressure =9.98x104 pa).
9) The pressure on the upper surface of a sub merged sub marine is 1.20x106 pa is the
pressure on the base of the hull is 1.40x106 pa. calculate the height of the sub marine
(Density of sea water is 1.04x103kgm-3).
10) An open –u- tube manometer containing mercury is used to measure the pressure of a
gas .The mercury level in the open tube is in 600mm higher than that in the limb which in
contact with the gas. What is the gas pressure. (Density of mercury =1.36x104kgm-3,
atmospheric pressure =1.01x105 pa)
11) The figure below shows a gas trapped by a column of mercury in a J –tube

Gas 25cm

The atmospheric pressure is 1.0x105Nm-2 and the density of mercury is 1.36x104kgm-3. Find
the pressure of the gas.

CALCULATIONS
1) A glass block weighs 25N in air when wholly immersed in water the block appears to
weigh 15N.Calculate the

149 | P a g e
(i). up thrust
(ii).density of glass block
2) A metal weighs 1N in air and 0.8N when totally immersed in water .Calculate the weight
of displaced water.
3) A metallic ball weighs 20N in air and 15N when fully immersed in water .calculate the;
(i). up thrust
(ii).weight of the displaced water
(iii). volume of the displaced water given that the density of water is 1000kgm-3
(iv). volume of the metallic ball
(v).density of the metallic ball.
4) An iron cube of volume 800cm3 is totally immersed in
(i). water
(ii).Oil of density 0.8gcm-3. calculate the up thrust in each case, given that density of water is
1000kgm-3
5) An iron cube of volume 400cm3 is totally immersed in water of density 1 gcm-3. Find the
up thrust.
6) An iron cube, mass 480g and density 8gcm -3 is suspended by a string so that it is
immersed in oil of density 0.9gcm-3. find the tension in the string.
7) A piece of cork of volume 100cm3 is floating on water. if the density of the cork is
0.25gcm-3.calculate the volume of the cork immersed in water
8) A piece of metal of mass 2.60x10-3 kg and density 8.4x103 kgm-3 is attached to a block of
wax 1.0x10-2 kg and density 9.2x102 kgm-3 when the system is placed in a liquid, it floats
with wax just submerged . find the density of the liquid.
9) A cross sectional area of a ferry at its water line is 720m2. if sixteen cars of average mass
1100kg are placed on board to what extra depth will the boat sink in the water.
10) A block of wood floats at an interface between water and oil with 0.25 of its volume
submerged in the oil. if the density of the wood is 7.3x102kgm-3, find the density of the
oil.
11) A tube of uniform cross sectional area of 4.0x10-3m2 and mass 0.2kg is separately floated
vertically in water of density 1.0x103 kgm-3 and in oil of density 8.0x102 kgm-3, calculate
the difference in the length this immersed.
12) A string supports solid iron object of mass 180 kg totally immersed in a liquid of density
0.8gcm-3. calculate the tension in the string if the density of the iron is 8000kgm-3.
13) A cube of rubber of volume 10-3 m3 floats with half of its volume submerged in a liquid
of density 1200kgm-3. find the depth to which the cube will be submerged in the fluid of
density 103 kgm-3.
14) A solid weighs 237.5g in air and 12.5g when totally immersed in a fluid of density
9.0x102 kgm-3.calculate;
(i). the density of the solid
(ii).the density of the liquid in which the solid would float with one fifth of its volume
exposed above the liquid surface.
150 | P a g e
15) A string supports a solid iron object of mass 0.18kg totally immersed in a liquid of
density 800kgm-3.calculate the tension in the string if the density of iron is 8000kgm-3.
16) A block of wood of density, ρ , floats at the interface between immiscible liquids of
density ρ 1 , and ρ 2 as shown below

P1 Liquid of density ρ1
V1

Block of wood of density ρ


V2

P2 Liquid of density ρ 2

v1
(i). Show that the ratio of the volume V1 to V2 of the block in the two liquids is given by
v2
ρ2−ρ
=
ρ−ρ 1
(ii).What happens when the block of wood is replaced with denser one
3 ρ2−ρ
(iii). if of the blocks volume is in the liquid of density, ρ , show that =3
4 ρ−ρ 1
17) A block of mass 0.1kg is suspended from a spring balance. When the block is immersed
in water of density 1.0x103kgm-3, the spring balance reads 0.63N. When the block is
immersed in a liquid of un known density the balance reads 0.70N. find
(i). the density of the solid
(ii).the density of the liquid
18) A string supports a metal block which completely immersed in a liquid of density
8.8x102kgm-3. if the density of the metal is 9.0x103kgm-3, calculate the tension in the
string .
19) A tube of uniform cross sectional area of 4.0x10-3m2 and mass 0.2kg is separately floated
vertically in water of density 1.0x103 kgm-3 and in oil of density 800kgm-3. calculate the
difference in the lengths immersed.
20) A balloon is inflated with air to a volume of 1.5 litres and is sealed. The density of the
surrounding air is 1.3g per litre given the mass of air in the balloon is 2.15g and the
apparent loss in weight of the inflated balloon is 42.18x10-3N. Calculate the mass of the
balloon fabric
21) A piece of brass weighs 12.9g in air. When fully immersed in water it weighs 11.3g what
is the mass of copper contained in the alloy given that the relative densities of copper and
zinc are 8.9 and 7.1 respectively.

151 | P a g e
22) A body has a weight of 160N when weighed in air and weight of 120N when totally
immersed in a liquid of relative density 0.8. Find the relative density of the body.
23) A tank contains a liquid of density 1.2x103 kgm-3.A body of volume 5.0x10-3 m3 and
density 9.0x102 kgm-3 totally immersed in the liquid and is attached by a thread to the
bottom of the tank what is the tension in the thread.
24) A hot air balloon has a volume of 500m3. The balloon moves upwards at a constant
speed in air of density 1.2kgm-3 when the density of the hot air inside it is 0.80kgm-3.
(i). find the combined mass of the balloon and the air inside it
(ii).what is the upward acceleration of the balloon when the temperature of the air
inside it has been increased so that its density is 0.7kgm-3.
25) An object with a volume of 1.0x10-5 m3 and density 4.0x102kgm-3 floats on water in
tank of cross sectional area 1.0x10-3m2 .By how much does the water level drop when the
object is removed.
26) A hydrometer of mass 30.0g has an upper stem of cross sectional area 0.20cm2.By what
length will it sink when transferred from liquid x of density 1.25 gcm-3 to a liquid y of
density 1.20 gcm-3.
27) A string supports a small iron ball of mass 180 g totally immersed in a liquid of density
800kgm-3. calculate the tension if the density of iron is 8000kgm-3
28) A solid weighs 240.5g in air and 15.5 g when totally immersed in a fluid of density
900kgm-3. calculate the density of the liquid in which the solid would float with one fifth
of its volume exposed above the liquid surface
29) An empty metal can of uniform cross sectional area 72 cm2 is made to float vertically in
water. if 5 litres of cooking oil of density 800kgm-3 is poured into the can, find the extra
depth to which the can will sink

CALCULATIONS ON RELATIVE DENSITY

1) A solid weighs 237.5g in air and 12.5g when totally immersed in a liquid of relative
density 0.9.calculate the relative density of the solid.
2) A solid weighs 20.0g in air, 15.0g in water and 16.0 g in a liquid R. find the relative
density of R.
3) A block of mass 0.1 kg is suspended from a spring balance when the block is immersed
in water of density 103 kgm-3, the spring balance reads 0.63N. When the block is
immersed in a liquid of an un known density, the spring balance reads 0.70N. find
(i). density of the block 2.79x103 kgm-3)
(ii).density of the liquid (800.6kgm-3)
4) A piece of brass of density 8.4x103kgm-3 is attached to a block of w ax of density
9.2x102kgm-3 as a sinker. When the system is placed in a liquid of density 1.15x103kgm-3.
it floats with wax just submerged. if the mass of wax is 1.0x10-2kg find the mass of the
piece of brass.

152 | P a g e
5) An alloy of mass 588g and volume 100cm3 is made of iron of density 8.0gcm-3 and a
aluminum of density 2.7gcm-3. calculate the proportion
(i). by volume
(ii).by mass of the constituents of the alloys.
6) A solid metal cube of side 8.0cm floats vertically at the interface between water and
mercury as shown below.

Water

Solid

1.0cm

Mercury

The lower surface of the tube is 1.0cm the interface. Calculate the density of
metal given that density of mercury is 1.360x104 kgm-3 and density of water is
1x103kgm-3.
7) A specimen of an alloy of silver and gold whose densities are 10.5gcm-3 and 18.9cm-3
respectively weigh 35.2g in air and 33.13g in water. Find the composition by mass of the
alloy assuming that there has been no volume change in the process of producing the
alloy. Assume that density of water is 1 gcm-3.
8) A hydrometer floats in water with 72% of its volume submerged .The hydrometer floats
in another liquid with 80% of its volume submerged. find the relative density of the
liquid.
9) A cylindrical cork of cross sectional area 20cm2 and length 30cm is covered at one end
with a layer of brass 2cm thick if the cork floats in water vertically with the metal while
below water surface, show that the height, h, of the cork above the surface is given by
600 (1− pc ) + 40(1− pb)
h= where ρc and ρb are densities of cork and brass respectively
20
density of water is 1 gcm-3)

10) A hydrometer floats in water with 6.0cm of its graduated stem un immersed and oil
density 0.8cm-3 with 4.0cm un immersed what length of stem is un immersed when the
same hydrometer is placed in a liquid of density 0.9gcm-3
11) A simple hydrometer consisting of a loaded glass stem of uniform cross section sink in
water so that a certain man x on its stem is 4.0cm below the water surface. when placed
in a liquid of density 0.9gcm-3, the hydrometer floats with the man x, 6.0cm the liquid.
How far below the surface will the man x be if the hydrometer is placed in a liquid of
density 1.1gcm-3

153 | P a g e
12) A 400kg wooden block is floating with 0.2 of its volume above the water level. what is
the volume of an object of density 7.9x103kgm-3 which must be attached to the underside
of the block of completely submerge it.
13) A block of wood of volume 1000cm3 floats half immersed in a liquid of relative density
1.2. calculate the volume of brass of relative density 8.7 which must be attached to the
wood in order that the combination just floats in a liquid of relative density 2.2.
14) A hydrometer consists of a spherical bulb and a cylindrical stem of cross section area
0.4cm2. The total volume of the bulb and stem is 13.2cm3. when immersed in water, the
hydrometer floats with 8.0cm of the stem above the water surface in alcohol if floats with
1.0cm of the stem above the surface .calculate the density of alcohol.
15) A block of metal of dimensions 8.0cmx8.0cmx8.0cm floats vertically at the interface
between water and mercury as shown

7.0cm

1.0cm

The lower surface of the block and 1.0cm below the interface .calculate the density of the
metal given that the densities of water and mercury are 1x103kgm-3 and 13.6x103 kgm-3
respectively.
16) A string supports a metal block of 2kg which is completely immersed in a liquid of
density 8.8x102kgm-3. If the density of the metal is 9.0x103kgm-3. calculate the tension in
the string.
17) Liquid y of a volume 0.40m3 and density 900kgm-3 is mixed with liquid Z of volume
0.35m3 and density 800kgm-3. calculate the density of the mixture.
18) The mass of an empty density bottle is 46.00g. When fully filled with water it weighs
96g. And when full of a liquid of the un known road it weighs 86g. calculate
(i). the relative density of the liquid
(ii).the density of the liquid
19) A piece of aluminum weighs 80N in air and 50.37N when completely immersed in
water. calculate the density of glass.
20) A hydrometer of mass 48g floats with 6cm of its stem out of water. The cross sectional
area of the stem is 0.80 cm2. calculate the
(i). total volume of the hydrometer
(ii).length of the stem above the surface when it is made to float in a liquid of density
1.4gcm-3.
21) A solid weighs 30g when totally immersed in a liquid of density 800kgm-3. if it weighs
334g in air, calculate the relative density of the liquid in which the solid float with one

154 | P a g e
quarter of its volume above the liquid surface.
22) An alloy contains two metals X and Y of densities 3.0x103kgm-3 and 5.0x103kgm-3
respectively. calculate the density of the alloy
(i). if the volume of X is twice that of Y
(ii).if the mass of X is twice that of Y.
23) An alloy of two metals, X and Y has a volume of 5.0x10-4m3 and a density of
5.6x103kgm-3. The densities of X and Y are 8.0x103 and 4.0x103kgm-3 respectively. find
the mass of X and the mass of Y.

TOPIC 14: THERMOMETRY


DEFINITIONS
1. Define the following terms;temperature,heat,scale of temperature,akelvin,fixed point,ice
point,steam point,fundamental interval,triple point of water,thermometric
property,thermometry,thermometric liquid,athermometer,bsolute zero
temperature,pyrometery,neutral point of athermocouple.

FACTORS, MERITS, DEMERITS, EXAMPLES AND FEATURES


1. When is the temperature zero Kelvin attained.
2. Give two examples of fixed points.
3. Mention four thermometric properties commonly used in measuring temperature in
named thermometers.
4. State the desired properties a material must have to be used as a thermometric property.
5. State two qualities of a good thermometric liquid substance.
6. State three corrections that need to be made when using a constant volume thermometer
7. State one advantage and disadvantage of the constant volume gas thermometer
8. State and explain the source of inaccuracies in using mercury glass thermometer
9. Mention any two advantages of mercury in glass thermometer
10. Give one disadvantage of a liquid in glass thermometer
11. State two properties of a material to be used in the resistance thermometer
12. What are the advantages of a thermocouple over a constant volume gas thermometer in
measuring temperature
13. Ex plain two main disadvantages of resistance thermometer
14. What are the advantages and disadvantages of platinum resistance thermometer
15. State two advantages and disadvantages of thermocouples
16. State the requirements for establishing the thermodynamic scale of temperature
17. Give disadvantages and advantages of gas thermometer.

DERIVATIONS AND RELATIONS


1. Outline the steps necessary in setting up a Celsius scale of temperature
2. Describe the steps involved in setting up a thermodynamic scale

155 | P a g e
3. How is temperature on a Celsius scale defined on platinum resistance thermometer
4. With reference to a constant volume gas thermometer define Celsius
5. With reference to a liquid in glass thermometer, describe the steps involving in setting up
a Kelvin scale of temperature
6. How is temperature on Celsius scale defined on platinum resistance thermometer
7. With reference to a platinum resistance thermometer, describe the steps involved in
setting up a Kelvin scale of temperature
8. Describe how you would measure the temperature of a body on the Celsius scale of
temperature using a thermocouple
9. Describe how you would measure the temperature of a body on thermodynamic scale
using a thermo couple
10. Describe how you would estimate absolute zero temperature
11. Describe how un known temperature of a liquid can be measured by liquid in glass
thermometer on a Celsius scale
12. Describe the structure of platinum resistance thermometer
13. Describe the steps taken to establish scales of temperature.
14. Describe briefly how to determine absolute temperature of a body using a constant
volume gas thermometer.

EXPLANATIONS
1. Explain why the constant volume gas thermometer is used to calibrate other thermometer
2. Explain why two different thermometers may give different readings for the same un
known temperature
3. Explain why scale of temperature based on different properties may not agree
4. Explain the extent to which two thermometers based on different properties but
calibrated using the same fixed point are likely to agree when used to measure a
temperature
5. near one of the fixed points
6. mid way between the two fixed points
7. Use kinetic theory of gases to explain the existence of absolute zero temperature
8. Explain why constant pressure thermometer may give a slow response
9. Explain the following in relation to liquid in glass thermometer
10. mercury is the best liquid for measuring high temperatures
11. Alcohol is the best liquid for measuring small temperatures
12. Explain the following observations as applied to gas thermometer constant
13. volume gas thermometer is preferred to a constant pressure gas thermometer
14. Gas thermometer are used to calibrate other thermometers
15. Explain why it is possible for two different bodies at different temperatures may have the
same amount of heat
16. Explain two main disadvantages of resistance thermometer
17. Explain why a thermocouple can be used to measure rapidly fluctuating temperatures
156 | P a g e
EXPERIMENTS
1. Draw a labeled diagram to show the structure of a simple constant volume gas
thermometer.
2. With the aid of a labeled diagram, describe the structure and mode of operations of a
constant volume gas thermometer.
3. Describe with the aid of a diagram how a constant volume gas thermometer may be used
to measure temperature on the Celsius scale.
4. With the aid of a labeled diagram, describe how the room temperature can be measured
using un calibrated resistance thermometer.
5. Describe with aid of a labeled diagram how to measure high temperature using an optical
pyrometer.
6. With aid of a labeled diagram describe how the temperature of a hot body can be
measured using a total radiation pyrometer.
7. Describe with the aid of a diagram how you would calibrate a thermo couple
thermometer.
8. Describe an experiment to determine the lower and upper fixed points.
CALCULATIONS
1) The resistance of the element of platinum resistance thermometer is 4.00 at the ice
point and 5.36 at the steam point. What temperature on the platinum resistance scale
would correspond to a resistance of 9.84.
2) The resistance of the element of platinum thermometer is 2 at ice point and 2.73 at the
steam point.
(i). What temperature on platinum resistance scale would correspond to a resistance
of 8.34
(ii).Measured on the constant pressure gas scale, the same temperature corresponds to
the value of 10200C. Explain the discrepancy.
3) The resistance Rθ of platinum varies with temperature θ 0C as measured by the constant
volume gas thermometer according to the equation.
Rθ = 50.0 + 0.17θ + 3.0 x 10-4 θ2
(i). Calculate the temperature on the platinum scale corresponding to 600C on the gas
scale
(ii).Account for the difference between the two values and state the temperature at
which they agree.
4) A resistance thermometer has a resistance of 21.42 at the ice point, 29.10 at some
temperatures of 1000C. Calculate the temperature on the scale of this thermometer if its
resistance at unknown temperature is 28.11.
5) A particular resistance thermometer has a resistance of 30.00 at the ice point, 41.58 at
the steam point, and 34.59 when immersed in a boiling liquid. Calculate the
temperature at which, the liquid is boiling on the scale on resistance thermometer.

157 | P a g e
6) A particular constant volume gas thermometer gives reading of 1.333 x 105Pa, 1.821 x
105Pa, and 1.528 x 105Pa at the ice point, steam point and un known temperature.
Calculate the temperature at which the liquid is boiling on the scale of gas thermometer
above.
7) The resistance Rθ of platinum resistance thermometer at a temperature θ on gas scale
thermometer is given by; Rθ = R0 (1 +  θ + bθ2), where  = 1.3 10-2K-1, and b =1.33 x
10-6 k-2, Ro – is the resistance at O0C.
Calculate the temperature of resistance thermometer when the temperature on the gas
thermometer is 3000C.
8) The value of the property X of a certain substance is given by Xt = X0 – 0.5t + (2.0 x 10-4)
t where t – temperature in 0C on gas thermometer. What would be the Celsius temperature
by the property X which corresponds to a temperature of 500C on this gas thermometer
scale
9) A mercury thread has a length of 25cm at a triple point and 35cm at un known
temperature T. calculate the value of T.
10) The resistance of platinum wire at the triple point of water is 5.16, what will be its
value at 1000C.
11) The resistance of platinum thermometer is 5.7 5.5 and 5.2 at the boiling point of
water, at un known temperature and at a freezing point of water respectively. Determine
the unknown temperature on thermodynamic scale.
12) If the mercury thread has a length of 0.5cm and 20cm at ice point and steam point
respectively and it is 8cm long at any other temperature. Find the value of the unknown
temperatures.
13) A particular constant volume gas thermometer registers a pressure of 1.937 x 104Pa at the
triple point of water and 2.618Pa at the boiling point of liquid. What is the boiling point
of the liquid according to this thermometer.
14) The length of mercury column is 5cm at ice point and 8cm at steam point. What is its
reading when the mercury column is 50 cm.
15) The length of the mercury column of a given thermometer is 25cm at ice point and 95cm
at steam point. Given that the temperature reading is 600C at given value of length of the
mercury column. Determine the value of length.
16) A thermometer is constructed with a liquid which expands according to the relation Vt =
Vo (1 +  t + t2) where Vo is the volume at 00C, Vt is volume at t 0c, on the gas scale and
1 1 are constants. Given that  = 1000, what will the liquid thermometer read when
the gas thermometer reads 500C.
17) The resistance of platinum wire at a temperature t0c measured on the gas scale is Rt = Ro
(1 + at +bt2) where a = 3.80 x 10-3 k-1 and b = -55.6 x 10-7 k-2. What temperature will the
platinum thermometer indicate when the temperature on the gas scale is 2000C.
18) The resistance Rt of an aluminum wire at a temperature t0c measured on the scale of a
mercury thermometer is given by;

158 | P a g e
Rt = Ro (1 + at + bt2) where a = 4.46 x 10-3 K-1, and b = 1.8 x 10-6 k-2. The wire is used in
a resistance thermometer when the mercury thermometer obtained from this thermometer
when the mercury thermometer reads 2500C.

19) A specific type of thermometer of fixed masses of a gas has the following values of
pressure for a given volume of a gas

Temperature Pressure (cmHg) Volume (Cm3)


Ice point 81.6 100
Steam point 90.0 124
Temperature θ 85.0 120
Determine the unknown temperature θ as measured from the thermometer.
20) The resistance of an element in platinum resistance thermometer is 6.750 at the triple
point of water and 7.166 at room temperature. Determine the temperature of the room
on resistance thermometer scale given that the triple point of water is at 273.16K. State
any assumptions you have made.

21)

Temperature Resistance () Pressure (Nm-2)


Steam point 75.000 1.10 x 10+7
Ice point 63.000 8.00 x 10+6
Room Temperature 64.990 8.51 x 10+6
Using the above data with reference to the observation of a particular room temperature.
Using two types of scales of temperature. Calculate the temperature of room. Why do
these values differ slightly.
22) A liquid in glass thermometer was a liquid of which the volume varies with temperature
as Vθ = Vo (1 + aθ + bθ2),Vθ –volume at θ0C and Vo is volume at 00C. What temperature
will be indicated on the liquid in glass thermometer if temperature of glass thermometer
is 600C.
23) A temperature t 0c of a liquid is determined by using a constant pressure gas thermometer
and the results where Vo = 4m3, V100 = 5.5m3, Vt = 4.25m3. Determine the temperature of
t of the thermometer.
24) The resistance of the platinum resistance thermometer varies according to the equation Rt
= Ro (1 + 800αt - α t2) where α is a constant. Calculate the temperature scale on the
platinum resistance thermometer that corresponds to 4000C on the constant volume gas
thermometer.
25) A temperature t0C is determined using a resistance thermometer and the following results
were obtained Ro = 2, R100 = 2.5 and Rt = 2.09. Determine the value of unknown
temperature.

159 | P a g e
26) The resistance of an electrical resistance thermometer is 3.44 at 400C and 3.55 at
500C. The temperature is measured using the resistance thermometer.
(i). What is the coefficient of resistance of temperature of the metal of the resistance
thermometer
(ii).What is the temperature of a liquid in which thermometer has a resistance of
3.79.
27) The resistance of platinum wire at triple point of water is 5.16. What will its resistance
be at 100 0C.
28) The resistance of the element in a platinum resistance thermometer is 6.750 at the triple
point of water and 7.166 at room temperature. Find the temperature of the room on the
scale of the resistance thermometer in 0C.
29) The Electrical resistance in ohms of a certain thermometer varies with temperature, T
according to the equation, (R = Ro (1 + 4 x 10-3(T – To)). The resistance is 112 at the
triple point of water and 150 at 600K. What is the temperature when the resistance is
120.
30) The resistance of a platinum resistance thermometer is 5.2 at the triple point of water
and 9.1 at an unknown temperature θ. Find the value of θ
31) At a temperature t0C where t is measured on the gas thermometer scale, the resistance of
the pure metal is given by Rt = Ro (1 + 4 x 10-3 t + 10-5 t 2) where Ro resistance at 00C.
Calculate the temperature on resistance scale which corresponds to the temperature at
800C measured on the gas thermometer scale.
32) A resistance thermometer has a resistance of 21.42 at the ice point. 29.10 at the
steam point and 28.11 at some unknown temperature θ, calculate value of θ on the scale
of the thermometer.
33) The value of the property X of a substance is given by Xt = Xo + 50t + (2.0 x 10-4) t2.
Where temperature in degrees Celsius temperature. Define by the property X which
corresponds to a temperature of 500C of this gas thermometer scale.

TOPIC 15: CALORIMETRY


DEFINITIONS

1. Define the following terms; heat capacity, specific heat capacity, latent heat, latent heat of
fuion,latent heat of vaporization, specific latent heat of fusion and vaporization, cooling
correction.

FACTORS, MERITS, DEMERITS EXAMPLES AND FEATURES

1. What are the principle advantages of continuous flow method compared to method
of mixtures.

160 | P a g e
2. State one disadvantage of using continuous flow method in determination of shc of
liquids
3. State how heat losses are minimized in a calorimetry
4. State the precautions taken when using continuous flow method to determine the
specific heat capacity of a liquid.
5. Give two ways in which a body can lost heat to the surrounding
6. Mention any three advantages and two disadvantages of continuous flow method to
determine the specific heat capacity of a liquid
7. State factors which affect the body’s rate of cooling
8. State the assumptions made in the method of determination of specific heat capacity
of a metal
9. State two advantage of the continuous flow method over the method of mixtures for
determination of specific heat capacity of liquids.
10. State two advantages of the electrical method over the method of mixtures.

EXPERIMENTS

1. Describe an electrical method for determination of the specific heat capacity of a metal.
2. With the aid of a labeled diagram describe an electrical method of determining the
specific heat capacity of solid.
3. Describe how specific heat capacity of a liquid can be obtained by continuous flow
method
4. Describe with the aid of a labeled diagram an electrical method for determination of
specific latent heat of vaporization of a liquid
5. Describe how you would determine the specific latent heat of vaporization of water by
using a method of mixtures
6. Describe an experiment to determine the specific latent heat of fusion of ice using the
method of mixtures
7. Describe an electrical method for determination of specific heat capacity of a metal
8. Describe an experiment to determine the specific heat capacity of water using the
continuous flow method
9. Describe how you would determine the specific latent heat of vaporization of water using
a method of mixtures
10. Describe an experiment to determine the specific heat capacity of a metal using a method
of mixtures
11. Describe giving relevant theory, an electrical method of determining the specific latent
heat of vaporization of alcohol
12. Outline the steps taken in determination of the specific latent heat of vaporization of a
liquid by the method of mixtures.
13. Describe how you would determine the specific heat capacity of a liquid by the
continuous flow method.

161 | P a g e
EXPLANATIONS

1) Explain how the cooling correction may be estimated in the determination of heat
capacity of a poor conductor of heat by the method of mixtures.
2) Explain why a small body cools faster than a larger one of the same material.
3) Explain why temperature remains constant during change of phase
4) Explain the significance of latent heat in regulation of body temperature
5) Explain why latent heat of vaporization is always greater than that of fusion
6) The specific latent heat of fusion of a substance is significantly different from its specific
latent heat of vaporization at the same pressure. Explain how the difference a rises.
7) Explain in terms of specific heat capacity why water is used in a car radiator than any
other liquid.
8) Explain the changes that take place in the molecular structure of a substances during
fusion and vaporization
9) Explain how the cooling correction of a good conductor of heat can be estimated
10) Explain in principles what steps you would take in to increase the accuracy in the
measurement of value of;
(i). The temperature differences between the inflow and out flow
(ii).The P.d across and the current in the heating element in the continuous method of
determination of shc of a liquid.
11) Explain the changes that take place in the molecular structure of a substance during
fusion and vaporization.
12) Explain the following;
(i). The effect of increasing the supply of heat to boiling water
(ii).The chilling effect on the body of wet clothes.
13) Explain why the distinction between the specific heat capacity at constant pressure and
specific heat capacity at constant volume is important for gases and not for solids and
liquids.
14) Explain why the specific latent heat of vaporization is much higher than the specific
latent heat of fusion for the same substances
15) Use the kinetic theory of matter to explain why the specific latent heat of vaporization of
water is higher at 200C than it is at its boiling point.
16) Explain why the specific latent heat of fusion and specific latent heat of vaporization of a
substance at the same pressure are different.

DERIVATIONS AND RELATIONS

1. Derive the relationship between temperature fall of a body and the excess temperature of
the surrounding.

LAWS, PRINCIPLE AND THEOREM

162 | P a g e
1. State Newton’s law of cooling.

CALCULATIONS ON SPECIFIC HEAT CAPACITY

1) Calculate the quantity of heat required to raise the temperature of a metal block with a
heat capacity of 23.1J0C-1 by 30.00C.
2) An electrical heater supplies 500J of heat energy to a copper cylinder of mass 32.4g. find
the increase in temperature of the cylinder of the cylinder (s.h.c of copper = 385Kkg-1 c-1)
3) How much heat must be removed from an object with a heat capacity of 150J0C-1 in order
to reduce its temperature from 800C to 200C.
4) In an experiment using the continuous flow calorimeter, the following results were
obtained.

Results Experiment 1 Experiment 2


p.d (v) a cross the heating 10 15
Current, I through the coil 0.60 0.80
Temp(0C) of the incoming liquid 25.5 22.5
Temp( C) of liquid collected
0
30.5 30.5
Time (s) taken to collect the liquid 30.0 120
Mass (kg) of liquid collected 0.86 0.69
Calculate the specific heat capacity of the liquid.

5) An experiment using continuous flow calorimeter obtains the following results.


Using water which enters at 180C and leaves at 220C, the rate of flow is 20gmin-1, the
current in the heating element is 2.3A and p.d a cross it 3.3V.
Using oil which flows in and out at the same temperature as water the rate of flow is
70gmin-1, the current is 2.7A, and the p.d is 3.9V.
a) Taking the s.h.c of water to be 4200Jkg-1k-1, calculate explaining your method
clearly,
(i). The rate of heat loss from the apparatus
(ii).The specific heat capacity of the oil.
b) (i) Explain carefully how using this same method, the s.h.c of the oil could be
obtained without acknowledge of the s.h.c of water.
(ii) Explain why readings should only be taken when steadily state exists. How
would you ensure that such a condition has been attained
c) Explain in principle what steps you would take to increase the accuracy in the
measurement of the value of;
(i). The temperature difference between the inflow and outflow
(ii).The p.d across and the current in the heating element
6) A heating coil is placed in a thermo flask containing 0.6kg of water for 600s. The
temperature of water rose by 250C during this time. The water is replaced by 0.40kg of
another liquid and the same temperature rise occurs in 180s. Calculate the specific heat

163 | P a g e
capacity the liquid given that the s.hc. Of water 4200Jkg-1K-1. State any assumptions
made.
7) A piece of copper of mass 100g is heated to 1000C and is then transferred to a well
lagged copper can of mass 50.0g containing 200g of water at 100C. Neglecting heat loss,
calculate the final steady temperature of the water after it has been well stirred. Take shc
of copper = 400Jkg-1k-1 and shc of water is 4200Jkg-1k-1.
8) In a continuous flow method, 50g of water is collected in minute the voltmeter and
ammeter readings are 12.0Vand 2.50A respectively while the inflow and outflow
temperatures are 200C and 280C respectively. When the flow rate is reduced to 25gmin-1
the voltmeter and ammeter reads 8.5V and 1.85A respectively while the temperature
remains constant. Calculate the specific heat capacity of water.
9) In a continuous flow experiment a steady difference of temperature of 1.50C is
maintained when the rate of liquid flow is 45gs-1 and the rate of electrical heating is
60.5W. On reducing the liquid flow rate to 15gs-1, 36.5W is required to maintain the same
temperature difference. Calculate;
(i). Specific heat capacity of the liquid
(ii).Rate of heat loss to the surrounding.
10) Hot water at 850C and cold water at 100C are run into a bathroom vessel at a rate of 3.0 x
10-2m3 min-1 and V respectively. At the point of filling the bath the temperature of water
400C. Calculate the time taken to fill the bath if its capacity is 1.5m3.
11) An electric kettle rated 1000W 240V is used on 220V mains to boil 0.52kg of water. If
the heat capacity of the kettle is 400Jk-1 and its initial temperature of water is 200C, how
long will the water take to boil.
12) An electrical heater rated 48W, 12V is placed in a well-insulated metal of mass 1.0kg at a
temperature of the metal rises to 340C. Find the specific heat capacity of the metal.
13) Water flowing at a speed of 5ms-1 falls over 50m high water fall into a still pool below.
Calculate the approximate rise in temperature of the water.
14) A silver bullet with speed of 500ms-1 initially at temperature of 200C stops suddenly and
all its mechanical energy is converted into thermal energy. Calculate its temperature rise.
15) In an experiment 30g of granite are contained in a test tube that is heated by standing it in
boiling water at 100C. The granite is transferred into a calorimeter weighing 50g in which
there is 35g of water at 100C. The temperature of the water rises to 250C. find the specific
heat capacity of granite (Specific heat capacity of copper is 390Jkg-1k-1, s.h.c of water =
4200Jkg-1k-1).
16) The temperature of 50g of a liquid contained in a calorimeter is raised from 150C to 450C
in 530s by an electrical heater dissipating 10.0w. When 100g of liquid is used and the
same change in temperature occurs in the same time, the power of the heater is 16.1W.
Calculate the specific heat capacity of the liquid.
17) The tangential frictional force extended by a bond brake on a rotating metal drum of
circumference 0.25m is 0.40kg and its specific heat capacity is 0.35Jkg-1k-1. Calculate the
no. of complete revolutions of the drum required to raise the temperature by 5.0k.
164 | P a g e
18) A bullet of mass 30g travelling at 0.20kms-1 becomes embedded in a fixed target.
(i). Describe the energy changes that take place
(ii). What is the increase in internal energy of the target and bullet.
(iii). Calculate the temperature increase of the bullet if it absorbs 75% of this internal
energy. The specific heat capacity of the metal is 0.13Jkg-1k-1.
19) A car of weight 15KN is moving uniformly at 12ms-1 down a l in 6 hill with the engine
switched off. If the brake drums have mass 30kg and specific heat capacity 0.040Jkg-1k-1.
Calculate the rate of increase of temperature. What assumptions have you made.
20) A drill, using a current of 2.0A when connected to a 240mains supply, makes a hole in a
piece of iron of mass 0.80kg. Calculate the temperature rise in 20s if 60% of the electrical
energy is converted into the iron’s internal energy (Iron has a specific heat capacity of
460Jkg-1k-1). Discuss any assumptions you have made.
21) An accurate thermometer of heat capacity 20Jk-1 reads 18.00C it is then placed in 0.25kg
of water and both reach the same final temperature of 50.00C as recorded by the
thermometer. Assuming that there are no other heat exchanges. Calculate the temperature
of the water before the thermometer was place in it.
22) The temperature of 0.45kg of water in a vessel of heat capacity 80Jk-1 is increased from
288k to 352k to 352k in 480s by an electric heater. Neglecting heat losses, calculate the
power of the heater.
23) When this heater in (22) is placed in 0.50kg of paraffin in a similar vessel at the same
temperature, the temperature reaches 314K in 240s. Calculate the shc of paraffin.
24) A 10W immersion heater is placed in 0.25kg of a liquid in a calorimeter of heat capacity
50Jkg-1 and is switched on. After a time the temperature of the liquid reaches a constant
value. The heater is now removed and the initial rate of fall of temperature is measured as
15Ks-1. Find the specific heat capacity of the liquid.
25) Two identical containers each of heat capacity 12JK-1 cools from 340k. and holds 8 x10-
5
m3 of water and takes 150s to cool from 325K to 320K and the other holds an equal
volume of an unknown liquid which takes 50s to cool over the same range if the density
of the liquid is 8.0 x 102kgm-3, what is its average specific heat capacity of this liquid
over the range 325K to 320K.
26) In a calorimeter mass of the solid was 0.30kg and the temperature rise was 1.2k when a
current of 3.0A with a p.d of 2.0V flowed for 22.5. When current of 2.7A with a p.d of
1.8A flowed for the same time, the rise was 0.97K. calculate for this temperature;
(i). The specific heat capacity of the solid
(ii).The power loss.
27) Water flows at a steady rate of 6.0gs-1 through a continuous flow calorimeter when the
p.d across the coil is 11V and the current is 5.0A. The differences between the inflow and
outflow are 2.0k. When the flow changes to 2.0gs-1. The current is adjusted to 3.1A to
produce the same temperature rise. Calculate.
(i). The new p.d a cross the heating coil and hence the new power input and
(ii).The specific heat capacity of the water.
165 | P a g e
28) The steady rate of flow of liquid in a continuous flow calorimeter is 20gs-1 and the
electric heating element dissipates 0.15KW under these conditions, the out flow
temperature of the liquid is 3.0k higher than the inflow temperature. When the rate of
flow is halved, the power required to maintain the same temperature difference is 81W.
Calculate;
(i). The specific heat capacity of the liquid
(ii).The power loss to the surrounding.
29) In a continuous flow experiment, a steady difference of temperature of 150C is
maintained when the rate of liquid flow is 0.045kgs-1 and the rate of electrical heating is
60.5W. when the liquid flow rate if reduced to 0.015kgs-1, 36.5W are necessary to
maintain the same temperature difference;
Calculate;
(i). The specific heat capacity of the liquid
(ii).The rate at which heat is lost to the surroundings.
30) In a continuous flow experiment it was found that, when the applied p.d was 12.0V the
current was 1.5A the rate of flow of liquid of 50.0g per minute caused the temperature of
the inflow and outflow to differ by 100C. When the p.d was increased to 16.0V with a
current of 1.6A, a rate of flow of 90.0kg per minute was required to produce the same
temperature difference as before. Find the specific heat capacity of the liquid and the rate
of heat loss.
31) An electric drill takes 300s to make a hole in a price of brass of mass 9.5kg and average
power delivered from the main is 450W.
(i). Calculate the amount of energy used in drilling the hole
(ii).If 80% of the energy supplied to the drill is used to raise the temperature of the
brass. Calculate the average temperature rise state any assumptions made. (s.h.c
of brass = 390Jkg-1k-1)
32) In a continuous flow calorimeter for measurement of specific heat capacity of a liquid 3.6
x 10-3m3 of the liquid floor through the apparatus in 10minutes. When electrical energy is
supplied to the heating coil at a rate of 44W, a steady difference of 4K is obtained
between the temperature of the out flowing and in flowing liquid. When the flow rate is
increased to 4.8 x 10-3m3 of the liquid in 10minutes the electrical power required to
maintain the same temperature difference is 58W.
Find the;
(i). Specific heat capacity of the liquid
(ii).The rate of heat loss to the surroundings.(Density of liquid = 800kgm-3)
33) An electrical heater rated 500W is immersed in a liquid of 2.0kg contained in a large
thermos flask of heat capacity of heat capacity 840Jkg-1 at 280C. Electrical power is
supplied to the heater for 10minutes. If the specific heat capacity of the liquid is 2.5 x
103Jkg-1kg-1. Its specific heat capacity of vaporization is 8.54 x 103Jkg-1 and its boiling
point is 780C, estimate the amount of liquid which boils off.
State any assumptions made in your calculations.
166 | P a g e
34) A copper cylinder is mounted along the axis of a wooden pulley of calorimeter 0.25m.
the pulley rubs against the cylinder when it turns. When a steady force of 50N is applied
tangentially to the pulley, the temperature of the cylinder rises by 100C after the pulley
has turned through 20 revolutions.
(i). If the mass of the cylinder is 0.20kg, find the specific heat capacity of copper.
(ii). How would you measure the temperature rise
(iii). Give a reason why this is not an accurate method of determining the specific heat
capacity of copper.
35) In such a method, 50g of water is collected in 1minute. The voltmeter and ammeter
readings are 12.0V and 2.50A respectively. While the inflow and outflow temperature are
200C and 280C respectively. When the flow rate is reduced to 25g per minute the
voltmeter and ammeter reads 8.8V and 1.85A respectively while the temperature remains
constant. Calculate the specific heat capacity of water.
36) In an experiment to determine the specific capacity of a liquid. Using the continuous flow
method, a steady temperature of 200C is maintained when the liquid flow rate is
3000gmin-1 and the rate of electrical heating is 80.0W. On increasing the liquid flow rate
to 5000gmin-1 140.0W is required to maintain the same temperature difference. Calculate;
(i). Specific heat capacity of the liquid
(ii).Rate of heat loss to the surroundings.
37) A warm water tap and a cold water are opened at the same time into a bath tab. Water
from the warm tap flows out at a rate of 3.0kgmin-1 at a temperature of 600C while cold
water flows out at a rate of 4.0kgmin-1. When the taps have been opened for 50.0s the
temperature of the water in the tab is found to be 32.00C. If the water in the tab loses heat
at an average rate of 100W. Find
(i). The mass of water in the tab after 50.0seconds
(ii).The temperature of the water flowing out of the cold water tap
38) In a constant flow tube, experiment the flow of liquid is 2.0 x 10-3kgs-1. The heat supply is
24W and the temperature rise is 3K. Neglecting heat loss, calculate a value for specific
heat capacity of liquid. The true shc is 3500Jkg-1k-1. Calculate the percentage heat loss in
the experiment.
39) A current of 2.5A passing through a heat coil immersed in paraffin of shc 2.0Jg-1k-1
contained in a 100g calorimeter of s.h.c 0.04Jg-1k-1 raises the temperature from 50C below
room temperature to 50C above room temperature in 100s. What should be the reading of
a voltmeter connected a cross the heating coil.

CALCULATIONS ON COOLING CORRECTIONS

1) The temperature of a body falls from 300C to 200C in 5minutes the air temperature is
130C. Find the temperature after 10minutes assume Newton’s law of cooling.

167 | P a g e
2) A calorimeter of heat capacity 500Jk-1 is filled with 1.5kg of a molten substance. A
cooling curve of the liquid is then obtained. The rate of cooling just before solidification
starts is 8.4ks-1 and complete solidification takes 2.5mnutes.
(i). Sketch and explain the cooling curve
(ii).If the specific heat capacity of the liquid is 210 Jkg-1 k-1, calculate the specific
latent heat of fusion of the substance.
3) 75g of a liquid is placed in copper calorimeter of mass 50g at 17.20C. A heater of
negligible thermal capacity is immersed in a liquid and operated at 1.8A and 6.3V for
4minutes such that temperature is raised to 250C. Subsequently temperature falls to
24.70C after 2minutes since heating was stopped. Calculate the specific heat capacity of
the liquid given that s.h.c of copper is 0.42Jg-1k-1)

168 | P a g e
CALCULATIONS ON LATENT HEAT
1) Steam at 100 C is passed into a copper calorimeter of mass 150g containing 340g of
0

water at 150C. This is done until the temperature of the calorimeter and its contents is
found to be 525g. Calculate the specific latent heat of vaporization of water.
2) An electrical appliance rated 240V, 200W evaporates 20g of water in the 5minutes. Find
the heat loss if the specific latent heat of vaporization is 2.22 x 106Jkg-1.
3) In an experiment to determine the specific latent heat of vaporization of a liquid using
continuous flow calorimeter, the following results were obtained.

Voltage (V) Current I (A) Mass called in 300s(g)


7.4 2.6 5.8
10.0 3.6 11.3
Calculate the power of the heat required to evaporate 3.0g of water in 2minutes.
4) Water in a vacuum flask is boiled steadily by a coil of wire immersed in the water. When
the p.d across the coil is 5.25V and the current through it is 2.58A, 6.85g of water
evaporates in 20min. when the p.d and current are maintained at 3.20V and 1.57A
respectively 2.38g of water evaporates in 20minutes all other conditions being the same.
Calculate the specific latent heat of vaporization of water.
5) An aluminium pail of mass 1.5kg contains 1kg of water and 2kg of ice at 00C. If 3kg of
water at 700C is poured into this pail. Calculate the final temperature of the pail plus
water. (s.h.c of aluminium = 910Jkg-1k-1, water 4200Jkg-1k-1, and specific latent heat of
fusion is 3.36 x 105Jkg-1).
6) A vessel whose walls are thermally insulated contains 2.10kg of water and 0.20kg of ice,
all at 00C. The outlet of a tube leading from a boiler in which water is boiling at 1
atmospheric pressure is inserted into the water. Find the amount of steam which must
condense to raise the temperature of the system to 200C. Assume that the heat capacity of
the container = 200Jk-1, the specific heat capacity of water = 4.20 x103Jkg-1k-1, and SLh of
ice is 3.36 x 105Jkg-1.
7) When electrical energy is supplied at a rate of 12.0w to a boiling liquid, 1.0 x10-2kg of
liquid evaporates in 30min. on reducing the electrical power to 7.0W, 5.0 x 10-3kg of the
liquid evaporates in the same time find;
(i). The specific latent heat of vaporization of the liquid
(ii).The power loss to the surroundings.
8) When electrical power is supplied at a rate of 12.0W to a boiling liquid, a mass of liquid
of 8.6 x 10-3kg evaporates in 30minutes on reducing the power to 70W, 5.0 x 10-3kg of
the liquid evaporates in the same time. Calculate the;
(i). Specific latent heat of vaporization of the liquid
(ii).Power loss to the surrounding.
9) An electrical heater is rated 240V, 2000W. If it’s used to evaporate 25g of water in
5minutes, find the heat loss if the specific latent heat of vaporization is 2.26 x 106Jkg-1.

169 | P a g e
10) In a factory heating system water enters the radiators at 600C and leaves at 380C. The
system is replaced by one in which steam at 1000C is condensed in the radiators, the
condensed steam leaving at 820C. Find the mass of steam that will supply the same heat
as 1kg of hot water in the first instance. Take specific latent heat of vaporization of steam
to be 2.26 x 106Jkg-1.
11) Calculate the heat given out when 60g of water cools from 800C to -200C. shc of water is
4.2Jg-1k-1, latent heat of fusion of ice = 336Jg-1. Specific heat capacity of ice is 2.1Jg-1k-1.
12) When electrical energy is supplied at rate of 12W to a boiling liquid of 10-2kg. The liquid
evaporates in 30minutes on reducing the power to 7W, 5.0 x 10-3kg of the liquid
evaporates in the same time. Calculate;
(i). The specific latent heat of vaporization of the liquid
(ii).The power loss to the surrounding.
13) A brass calorimeter of mass 0.2kg and s.h.c 380Jkg-1k-1 contains 0.5kg of water at 400C.
60g of dry stain is passed into the water and the temperature rises to 900C. Find specific
latent heat of the vaporization of steam.
14) A calorimeter with a heat capacity of 80J0C-1 contains 50g of water at 400C. What mass of
ice at 00C needs to be added in order to reduce the temperature to 100C. ( specific latent
heat of ice is 3.4 x 105Jkg-1).
15) Calculate the heat required to melt 200g of ice at 00C. (Specific latent heat of ice = 3.4 x
105Jkg-1).
16) Calculate the heat required to turn 500g of ice at 00C into the water at 1000C. (Specific
latent heat of ice = 3.4 x 103Jkg-1, shc of water = 4.2 x 103Jkg-10c-1).
17) Calculate the heat given out when 600g of steam at 1000C condenses to water at 200C.
(specific latent of steam = 2.26 x 106Jkg-1, shc of water = 4.2 x 103Jkg-1k-1).

TOPIC 16: GAS LAWS AND EXPLANSION OF GASES


DEFINITIONS
1. Define the following terms; molar mass, molar gas constant, relative atomic mass, equation
of state of a gas, Avogadro constant, ideal gas equation, absolute temperature, Boyle’s
temperature.

LAWS, PRINCIPLES AND THEOREMS.


1. State Boyle’s law
2. State Charles’s law of ideal gases
3. State pressure law of ideal gases
FACTORS, MERITS, DEMERITS, EXAMPLES AND FEATURES
EXPLANATIONS
1. Use kinetic theory of ideal gases to account for Charles’s law
2. Explain how pressure of the surrounding affects the rate of cooling of the body
3. Use kinetic theory to account for pressure law of ideal gases
4. Use kinetic theory to account for Boyle’s law of ideal gases

170 | P a g e
EXPERIMENTS
1. Describe an experiment to verify pressure law of ideal gases
2. Describe an experiment to verify Boyle’s law of ideal gases
3. Describe an experiment to verify Charles’s law of ideal gases
DERIVATION AND RELATIONS

PV
1. Show that for ideal gas, = constant, where p is the gas pressure, V, the volume and
T
T, absolute temperature
CALCULATIONS ON GAS LAWS AND EXPANSION OF GASES
1) What is the temperature of 19m3 of an ideal gas at 600mmHg if the same gas occupied
12m3 at 760mmHg at 270C
2) A gas has a volume of 60cm3 at 200C and 900mmHg. What would be its volume at s. t. p
3) A vessel containing 400m3 of a gas at a pressure of 8.7x10-2 pa and temperature 200C is
compressed isobaric ally to half its volume. Find the new temperature.
4) A gas which can be considered ideal has a volume of 100cm3at 2.00x105 pa and
27C0.What is its volume at 5.00x105 pa and 600C.
5) What is the temperature of 19.0m3 of an ideal gas at a pressure of 600mmHg if the same
gas occupies 12.0m3 at 760mmHg and 27 0C.
6) A cylinder contains 0.25m3 of gas at a pressure of 2.0x106 pa and a temperature of 170 C.
Calculate
(i). the number of moles of gas in the cylinder
(ii).Pressure of the gas if the temperature of the gas is raised to 370C.
7) The volume of air in a tyre is 24.61 when the pressure is 2.0x105 pa
(i). What volume will this air occupy at a pressure of 1.0x105 pa
(ii).How much air will escape from the tyre when the valve is removed.
8) Initially A contains 3.00m3 of an ideal gas at a temperature of 250k and a pressure of
5.00x104 pa, whilst B contains 7.20m3 of the same gas at 400k and 2.00x104 pa. Find the
pressure after the connecting tap has been opened and the system has reached
equilibrium, assuming that A is kept at 250k and B is kept at 400k.

A B
3
7.2cm
300cm3
2.00 x 104Pa
5.00x 104Pa
400k.
250K

Connecting tap

9) A cylinder contains 2.40x10-3 m3 of hydrogen at 170 C and 2.32x106 pa. The relative
molecular mass of hydrogen =2, R=8.31Jk-1mol-1 and the Avogadro constant, NA
=6.02x1023 mol-1. calculate;
(i). the number of moles of hydrogen in the cylinder

171 | P a g e
(ii).the number of molecules of hydrogen in the cylinder
(iii). the mass of the hydrogen
(iv). the density of hydrogen under these condition.
10) Two glass bulbs of volume 200cm3 and 100cm3 respectively, are connected by a
capillary tube. The apparatus which is hermetically sealed contains air at a pressure of
76cm of mercury at 150C. The 200cm3 bulb is immersed in steam at 100oC , the other
remaining at 150 C. Find the pressure of the air in the tube.
11) The density of oxygen at s t p is 1.43kgm3. if a 20 L cylinder is filled with oxygen at a
pressure of 25 atmospheres and temperature 270 C what is the mass of oxygen in the
cylinder.
12) A balloon of weight 0.8N is filled with the helium at 00C and 1 atmosphere pressure.
The balloon contains 20 moles of helium
(i). find the volume of the balloon
(ii).Calculate the weight that the balloon can have at the earth’s surface molar masses of
helium and air are 4.0 and 29.0 respectively.
13) An ideal gas of volume 2.0litres at s t p expands at constant pressure to a volume of
5.0litres. calculate
(i). the final temperature of the gas
(ii).the work done by the gas.
14) A carbon dioxide fire extinguisher has an interior volume of 2.8x10-3m3. The
extinguisher has a mass of 5.9 kg when empty and a mass of 8.2kg when fully loaded
with Co2. At a temperature of 270 C, what is the pressure of Co2 in the extinguisher.
15) The volume of a car tyre is 2.5x10-2m3. The pressure of the air inside the tyre is 3.0
atmospheres and temperature is 170C.What is the mass of air in the tyre (molecular mass
of air =29g).
16) An empty barometer tube of length 90cm is lowered vertically with its mouth
downwards into a swimming pool full of water. Calculate the depth of top of the tube
when the water has risen 15cm inside the tube, given the atmospheric pressure is 10m
head of water.
A tank containing 0.1m3 of Nitrogen at room temperature and a pressure of 8.0x106 Nm-2 is
connected through a valve to an empty tank of volume 0.4m3. The valve is opened and
Nitrogen allowed to expand. What is the pressure in the tank after the system has returned
to room temperature.

17)

2V V

Two vessels A and B of volumes 2v and V respectively are connected by a tube of


negligible volume shown

172 | P a g e
The vessels contain air of total mass 3.0x10-3kg at 300C and pressure of 1.0x105 pa.
Vessel A is cooled to 00Cand vessel B is heated 1000C.calculate the new equilibrium
pressure inside the vessel and mass of the gas in each vessel at this new state.
18) A vessel of volume 1.0x10-3m3 contains helium at a pressure of 2.0x105 pa and
temperature of 300k
(i). what is the mass of helium in the vessel
(ii).How many helium atoms are in the vessel
(iii). Calculate the root mean speed helium atoms
(Molar mass of helium is 4.0g)
19) Two bulbs A and B of volume 100cm3 and 80cm3 respectively are connected as shown
below

A B

The connecting tube is of negligible volume initially A is filled with ideal gas at 200C at
3.5x105 pa and B is filled with an ideal gas at 1000C at 1.2x105 pa

(i). Calculate the equilibrium pressure inside the bulbs when temperature of A is maintained
at 200C and that of B is reduced to 650C
(ii). Calculate the equilibrium pressure inside the bulbs when temperature of both A and B
maintained at 540C.
20. Pressure of a fixed volume of an ideal gas at 24C0 is 1.0x105 Nm-2, calculate the pressure
of the gas when the temperature changes to 1000C.

TOPIC: 17: VAPOURS AND GASES


DEFINITIONS

1. Define the following terms; Brownian motion,evaporation,boiling,boilingpoint,a gas,avapour,


criticaltemperature,criticalpressure,critical volume, saturated vapour, un saturated vapour, supper
saturated vapour,thermaldynamic equilibrium, vapour pressure, saturated vapour pressure, un
saturated vapour pressure, supper saturated vapour pressure ,dewpoint,humidification,
condensation, partial pressure.

LAWES, PRINCIPLES AND THEOREMS

1. State Dalton’s law of partial pressures

MERITS, DEMERITS, EXAMPLES, FEATURES AND EXAMPLES

1. Give any two factors which affect the rate of evaporation of a liquid
2. State four differences between boiling and Evaporation

173 | P a g e
3. Distinguish between un saturated and saturated vapours
4. State two ways of increasing the rate of evaporation of a liquid
5. State the conditions which favour the formation of dew
6. Outline the kinetic theory explanation for the occurrence of saturated vapours

EXPLANATIONS

1) Explain the kinetic theory of matter


2) Explain why evaporation causes cooling
3) Use the kinetic theory of matter to explain the following observations
(i). Saturated vapour pressure of a liquid with temperature
(ii).Saturated vapour pressure is not affected by decrease in volume at constant temperature
4) Explain why a liquid cools when it evaporates
5) Explain why the pressure of a gas increase when its temperature is increased
6) Using kinetic theory ,why saturated vapour pressure of a liquid increase with temperature
7) Why saturated vapour pressure is not affected by a decrease in volume at constant
temperature
8) Explain using kinetic theory, how boiling occur
9) Using kinetic theory, explaining boiling of a liquid
10) Use the kinetic theory of matter to explain the elevation of boiling point of water by
dissolving salt in it
11) Explain the following observations;
(i). Saturated vapour pressure does not obey Boyle’s law of perfect gases
(ii).Vapour pressure increases with increase in temperature
12) Explain how increasing vapour volume of saturated vapours would affect the pressure
they exert
13) Use kinetic theory to explain;
(i). The occurrence of saturated vapour pressure
(ii).The increase in vapour pressure when temperature of a liquid is increased
14) Explain the following;
(i). the effect of increasing the supply of heat to boiling water
(ii).the chilling effect on the body of wet clothes
15) Explain why vapour from ethanol whose boiling point at atmospheric pressure is 78C0
may give a more serious burn than steam
16) Explain why it is possible to make water boil below its normal boiling point
17) Explain qualitatively using kinetic theory how the saturation vapour pressure varies with
(i). volume, temperature remaining constant
(ii).the temperature, volume remaining constant
18) Explain, with the aid of a volume versus temperature. sketch graph, what happens to a
gas cooled at constant pressure from room temperature to zero Kelvin

174 | P a g e
19) Explain how cooking at a pressure of 76cm of mercury and a temperature of 100C 0 may
achieved on top of high mountain
20) Two similar cylinders P and Q contain different gases at the same pressure. When the
gas is released from p, the pressure remains constant for some time before it starts falling
when gas is released from Q, the pressure continuously drops. Explain the observations
above.
21) Use the kinetic theory of matter to explain the effect of increasing temperature on
saturated vapour pressure
22) Use the kinetic theory of matter to explain the following observations
(i). Saturated vapour pressure of a liquid increases with temperature
(ii).Saturated vapour pressure is not affected by a decrease in volume at a constant
temperature
23) Explain the formation of dew on grass
24) Explain why at a given external pressure a liquid boils at constant temperature
25) Explain why the temperature of a liquid does not change when the liquid is boiling
26) Explain why water on top of a high mountain boils at a lower temperature than that at the
bottom of the mountain

EXPERIMENTS

1. Describe briefly how Brownian motion occurs


2. Describe an experiment to determine the saturated vapour pressure of a liquid at different
temperature
3. Describe an experiment to determine the temperature dependence of saturated vapour
pressure of water
4. Describe how the saturated vapour pressure of a liquid at various temperature determined
5. Describe an experiment to show that a liquid boil only when its saturated vapour is equal
to the external pressure
6. Describe an experiments to show how the pressure of a vapour in equilibrium with its
liquid depends on temperature
7. Describe with aid of a diagram, an experiments to investigate the variation of s. v. p of
water with temperature
8. Sketch a graph of the variation expected in (7) above and show how the kinetic theory
account qualitatively for the variation
9. Describe briefly an experiment which you can carry out in support of the kinetic theory
of matter

DERIVATIONS AND RELATIONS

1) Sketch a graph of potential energy against separation of two molecules in a substance.


Explain the main features of the kinetic theory expression for the pressure of an ideal gas
2) Derive Dalton’s law using the kinetic theory expression for the pressure of an ideal gas

175 | P a g e
3) Sketch a well graph to show the variation of pressure with volume for an un saturated
vapour below its critical temperature
4) Water vapour and liquid water are contained in air fight vessel. The temperature of the
water is raided until the water evaporated. Draw a sketch graph to show the pressure of
the water vapour changes with temperature and account for its main features
5) Sketch a well labeled graph to show the variation of pressure with volume for an un
saturated vapour below its critical temperature
6) On the same axes, sketch graph of pressure against temperature for a saturated and un
saturated vapour
7) Sketch a graph to show the variation of saturated vapour pressure with temperature of a
liquid.

CALCULATIONS

1) A volume of 4.0x10-3m3 of air is saturated with vapour at 100 0C. This air is cooled to
200C at a pressure of 1.33x105 pa. Calculate the volume of air after cooling if the S. V.
P. of water at 200C is 2.3x103 pa and pressure of dry atmospheric air at 100 0C is 1.0x105
pa
2) A closed vessel contains air saturated with water vapour at 60 0C. The total pressure in the
vessel is 1000mmHg. Calculate the new air pressure if its temperature is reduced to 27 0C.
Taking the S. V. P of water at 600C to be 314mm Hg and 112mm Hg at 270C.
3) A column of air is sealed in a horizontal uniform capillary tube by a water index which is
sufficient to saturate the air. At 20 0C the length of the air column is 15.6cm at
atmospheric pressure of 762.5mmHg. When the air in the capillary tube is heated to 50 oC,
the length of air column becomes 19.1cm at the same atmospheric pressure .if the
saturated vapour pressure of water at 200Cis 17.5mmHg. What is its value at 500C.
4) A uniform capillary tube sealed at one end contains air enclosed by a thin film of water
which keeps it saturated with vapour. The length of air column is 15.6cm at 20 0C and
25cm at 70oC. Assuming S. V. P of water at 70 0C. Take total pressure at 200C to be
88cmHg and 104cm Hg at 700C.
5) Moist air at 500C and pressure of 760mmHg is contained in a sealed vessel .When the
vessel is cooled, saturate starts at 20 0C what will be the total pressure at 10 0C (S. V. P of
water at 200C and 100C are 1705mm Hg and 9.0mm Hg respectively).
6) A column of air 10cm long is trapped in a horizontal uniform capillary tube by a mercury
thread 8cm long. One end of the tube is open to the atmosphere and the other is closed.
Find the volume of air when the tube is held vertically with the open and exposed to the
atmosphere.
7) A volume of 2000cm3 of oxygen at 150C and pressure of 753mmHg has been collected
over water. Find the volume of dry oxygen at s. t. p if the S. V . P of water at 15 0C is
12.78mmHg.

176 | P a g e
8) Two short threads of water confine a sample of air in a uniform capillary tube held
horizontally. At a temperature of 20 oC and an atmospheric pressure of 76cm Hg, the air
bubble is 5.0cm long. When the tube is warmed to 60 0c, the bubble is 6.9cm long. Find
the S. V. P of water at 600c if its value at 200C is 18mmHg. Neglect surface tension
effects.
9) A column of air is trapped in a capillary tube of uniform cross sectional area and closed
at one end by a thread of water as shown

Water thread

Air column

The length of the air column is 10.0cm at 15 0C and 12.6cm at 50 0C. Calculate the S V P
of water at 50 0C. If its value at 15 0C is 1.70x103 pa. (Neglect the weight of the water
thread and assume that the atmospheric pressure is 1.0x105 pa).
10) In a laboratory, Joseph collected 1000cm3 of hydrogen over water at 20 0C. The external
pressure was 70.0cm of mercury. The saturation vapour pressure of water at 20 0C is
1.76cm of mercury. Calculate the corresponding volume of hydrogen at S. T. P.
11) A closed vessel contains a mixture of air and water vapour at 27 0C at a total pressure of
107.0x103Nm-2. The water vapour is just saturated at this temperature. calculate the total
pressure in the vessel if
(i). the temperature is raised to 600C
(ii).the temperature is lowered to 170C
(S. V. P of water 17C0 =1.9x10Nm-2, s. v. p of water at 27C0 =3.7x103 Nm-2).
12)

P Q

Constant temperature

Bath at 600C

Two vessels P and Q each of volume 1.5cm 3, are wined in the middle by a closed tape T,
and placed in a constant temperature bath at 600C as shown in the figure above. P
contains a vacuum while Q contains air and saturate d water vapour. The total pressure in
Q is 200mmHg When T is opened; equilibrium is reached with the water vapour
177 | P a g e
remaining saturated. If the final pressure in the cylinder is 150mm Hg, calculate the S. V.
P of water at 60 0C.
13) A volume of 4.0x10-3 m3 of air is saturated with water vapour at 100 0C. The air is cooled
to 200C at a constant pressure of 1.33x105 pa. Calculate the volume of air after cooling if
the saturated vapour pressure of water at 200C is 2.3x103 pa.
14) The total pressure in a closed vessel containing air and saturated vapour at 35 0C is
1.0x105 pa. if the saturation vapour pressure at 35 0C and 87 0C are 3.99x103 pa and
7.18x104 pa respectively. calculate the total pressure at 87 0C assuming air remains
saturated

15)

P Q

Two cylinders P, and Q each of volume 1.5litres are joined in the middle by a closed tap
T, and placed in a constant temperature bath of 60C 0 as shown above. P contains a
vacuum while Q contains air and saturated water vapour. The total pressure in Q is
200mmHg. When T is opened, equilibrium is reached with water vapour remaining
saturated. if the final pressure in the cylinder is 150mmHg. Calculate the saturation
pressure of water 600C.
16) A horizontal tube of uniform bore, closed at one end has some air trapped by a small
quantity of water. The length of the enclosed air column is 20cm at 12 0C. Find, stating
any assumptions made the length of the air column when the temperature is raised to 38
0
C (S . V. P of water at 12 0C and 38 0C are 105mmHg and 49.5mmHg
respectively .Atmospheric pressure = 75.0 cm Hg).
17) A flask contains a mixture of air and un saturated water vapour at a temperature of 50 0C
and pressure of 800mmHg. The mixture is cooled and when the temperature just reaches
200C, Water begins to condense out of the mixture. Given that saturated vapour pressure
is 18mmHg and 7mmHg at 200Cand 5 0C respectively. Calculate the pressure of the
mixture if it is cooled to 5 0C.
18) A closed vessel contains air saturated with water evaporation 77 0C. If the total pressure
in the vessel is 1000mmHg. Calculate the new pressure in the vessel if the temperature is
reduced to 27C0.(S. V. P of water at 77 0C =314mmHg,S. V. P of water at 27
0
C=27mmHg).

178 | P a g e
19) When hydrogen gas is collected over water, the pressures in the tube at 15 0C and 75 0C
are 65.5cm and 105.6cm of mercury respectively. if the saturated vapour pressure at 150C
is 1.42cm of mercury, find its value at 75 0C.
20) A closed vessel contains a mixture of air and water vapour at 27 0C and the total pressure
is 1.06x105 pa. At this temperature the water is just saturated. if the temperature is raised
to 870C .Calculate the air pressure and the vapour pressure . The temperature is now
lowered to 170C. Calculate the new total pressure
(S. V. P of water at 27C0 = 40X102 pa, at 17 0C it is 2x103 pa)

TOPIC 18: KINETIC THEORY OF IDEAL GASES


DEFINITIONS

1. Define the following terms; an ideal gas, a real gas ,partial pressure, Avogadro’s hypothesis,
critical temperature, boyle temperature ,root mean square speed, mean squares speed ,Co-
volume.

LAWS, PRINCIPLE AND THEOREMS

1. State Dalton’s law of partial pressure


2. State Avogadro’s hypothesis
3. State Graham’s law of diffusion of ideal gases

MERITS, DEMERITS, EXAMPLES, FEATURES AND FACTORS

1. State the conditions under which a real gas behaves like an ideal gas
1
2. State four major assumptions made when deriving P = p < C2> of a gas.
3
3. Modify two of the assumptions stated in (2) to cater for real gases
4. Explain how these modified assumptions in (3) affect the ideal gas equation.
5. State any two ways in which real gases differ from an ideal gas
6. In what ways do a real gas differ from an ideal gas

EXPLANATIONS

1. Account for the pressure exerted on the walls of the container occupied by an ideal gas
using kinetic theory of ideal gases.
2. Explain what is meant by an ideal gas
3. Explain qualitatively how the kinetic theory of gases accounts for the fact that a gas
exerts a pressure
4. Explain why the temperature of a gas falls if it is expanded in a thermally insulated
container.

179 | P a g e
5. Explain why a gas fills any container in which its placed and exerts a pressure on its
walls
6. Explain why the pressure of a gas increases when its temperature is increased
7. Explain why oxygen and Nitrogen are gases found in the atmosphere close to the earth’s
surface
8. Explain why the pressure of a fixed mass of gas in a closed container increased when
temperature of the container is raised.
9. Explain the following observations using the kinetic theory;
10. A gas fills any container in which it is placed and exerts a pressure on its walls
11. The pressure of a fixed mass of a gas rises when its temperature is increased at a constant
volume
12. Explain why the pressure of a fixed mass of gas rises if its temperature is increased.
13. Explain what happens to the pressure of a fixed mass of a gas in a sealed container when
the temperature of the gas is raised
14. Explain why a small increase in pressure will do more work on a gas than on liquid.

DERIVATIONS AND RELATIOSN

1) Distinguish between ideal gases and real gases


2) (i) Sketch a P versus V isothermals for a real gas below and above its critical
temperature.
(ii) Account for the isothermal at a temperature greater than the critical temperature in (i)
above.
(iii) Explain the features of the isothermal of a real gas being compressed below its
critical temperature as shown (i) above.
3) (i) Sketch PV against P isothermal for a real gas below and above its Boyle’s
temperature.
(ii) Account for the isothermals drawn in (i) above.
4) State and derive Dalton’s law of partial pressure
5) Prove that kinetic energy of an ideal gas molecules is directly proportional to its absolute
temperature
6) Sketch PV versus P and P versus V isothermals of an ideal gas.
1
7) (i) Derive the expression P = ρ <C2> for an ideal gas clearly pointing out the
3
assumptions made.
(ii). Use the above expression (i) to derive Avogadro’s Hypothesis
8) Use Kinetic theory expression for pressure of an ideal gas and the interpretation of
temperature as being directly proportional to the mean kinetic energy to deduce;
(i). The pressure law
(ii).Graham’s Law of diffusion.

180 | P a g e
9) What is meant by an ideal gas. The equation of state for n moles of an ideal gas may be
stated as PV = nRT where P- pressure V- Volume and T temperature of the gas. What is
R.

( a
)
10) For a real gas one modification of the ideal gas equation of state is P+ 2 ( V −b ) =
V
nRT. Vander Waal’s equation suggest and explain the physical significance of each of the
constants a and b.
11) Sketch graph of P against V for various values of T indicated by the ideal gas equation
and vander waal’s equation and compare these with the experiment graphs which would
be obtained for a real substance. Indicate the regions of the experimental curves in which;
(i). The ideal gas equation PV = nRT would best apply.
(ii). Vander waal’s equation could apply reasonably well
(iii). There would be little agreement using either equation.
12) (i) State clearly the assumptions made in the simple kinetic theory of gases in the
1
derivation of P = ρ < C2> where the symbols have their usual meaning
3
(ii) Which of the above assumptions in (i) above have to be modified for real gas.
(iii) Sketch the pressure volume variation of a real gas at a temperature above and below
its critical temperature.
1
13) Use the expression P = ρ < C2> for pressure of ideal gas to deduce Dalton’s law of
3
partial pressure. Also deduce Boyle’s and Charles’s law.
14) Using the expression for the Kinetic pressure of an ideal gas, deduce the ideal gas
3
equation if ½ mc2 = KBT.
2

CALCULATIONS ON KINETIC THEORY OF IDEAL GASES


1) Calculate the speed of sand in the atmosphere of Jupiter at temperature of -1300C and
mainly composed of methane gas of mass 6.04g. Assume that the speed of sound is
0.682times the toot mean square of one mole of methane.
2) Calculate the root mean square speed of the Hz molecules at 270C given that the density
of Hz at 1.0 x 105Nm-2 and temperature 00C is 0.09kg m-3.
3) A mole of an ideal gas at 300k is subjected to a pressure of 1.0 x 105 Pa such that its
volume changes to 0.025m3. find;
(i). Molar gas constant
(ii). Boltzmann constant
(iii). The average translational kinetic energy of a single molecule of the gas. Assume
NA = 6.02 x 1023 mol-1
4) Calculate the r.m.s speed of the molecules of oxygen at 76.0cm.Hg pressure and 00C at
which temperature and pressure, the density of oxygen if 1.43kgm-3

181 | P a g e
5) Air consist approximately of 20% oxygen and 80% Nitrogen. The relative molecular
masses of oxygen and Nitrogen are 32 and 28 respectively
Calculate;
(i). The ratio of the mean speed of oxygen to that of Nitrogen in air
(ii). The partial pressures of oxygen to that of Nitrogen in air
(iii). Explain why oxygen and Nitrogen gases found in the atmosphere close to the
earth’s surface.
6) A vessel of volume 1.0 x 10-3m3 contains helium at pressure 2.0 x 105Pa, when the
temperature is 300k.
(i). What is the mass of helium in the vessel
(ii). How many helium atoms are there in the vessel
(iii). Calculate the rms speed of the helium atoms
Assume NA = 6.02 x 1023 mol-1, relative atomic mass of helium is 4. Molar gas constant
R= 8.33Jk-1mol-1.
7) Calculate the root mean square speed of the molecules of an ideal gas at 1270C given that
the density of the gas at a pressure of 1.0 x 105Pa and a temperature of 00C is 1.43kgm-3.
8) Air saturated with water vapour is confined in a container at a temperature of 270C and a
pressure of 1.0 x 105Pa at a temperature of 770C the pressure is 1.6 x 105Pa and the air
remains saturated. Find the saturation vapour pressure of water at 770C. (Saturation
vapour pressure of water at 270C is 3.6 x 105 pa).
9) Two vessels of capacity 1.01 are connected by a tube of negligible volume. Together they
contain 3.42 x 10-4kg of helium at a pressure of 80cm of mercury and temperature 270C.
Calculate the pressure developed in the apparatus if one vessel is cooled to 00C and the
other heated to 1000C, Assuming that the heat capacity of each vessel is unchanged.
10) A container of volume 10.0m3 contains a mixture of 5g of Nitrogen gas and 5g of
hydrogen gas at 293k.
Calculate;
(i). The partial pressure exerted by each gas
(ii).The root mean squares speeds of the gas. (Molar masses of H2 and N2 are 2 and 28
respectively).
11) A sealed container has liquid water, water vapour and air all at 300C. The total pressure
inside the container is 66.3cmHg. When the temperature of the container is raised to
900C, the total pressure becomes 92.7cmHg, Assuming the saturation Vapour pressure of
water at 300C is 3.2cmHg and that the water vapour remains saturated. Calculate
saturated vapour pressure of water at 900C.
12) Helium gas is contained in a cylinder by a gas tight piston which can be assumed to move
without friction. The gas occupies a volume of 1.0 x 10-3m3 at a temperature of 300k and
a pressure of 1.0 x 105Pa.
Calculate;
(i). The no. of helium atoms in the container
(ii).The total kinetic energy of the helium atoms.
182 | P a g e
13) Calculate the speed of sound in the atmosphere of Jupiter knowing that the speed of
sound in a gas is 0.682 times the r.m.s speed of the gas molecules and that the atmosphere
of Jupiter contains mainly methane gas (Temp of Jupiter atmosphere = -1300C molecular
weight of methane = 6.04gm01-1, gas constant R = 8.3 Jmol-1K-1).
14) A gas is confirmed in a container of volume 0.1m3 of a pressure of 1.0 x 105Nm-3 and the
temperature of 300k of the gas is assumed to be ideal. Calculate the density of the gas.
(the relative molecular mass of gas is 32).
15) The masses of H2 and 02 atoms are 1.66 x 10-24kg 2.66 x 10-26kg respectively. What is the
ratio of the root mean square speed of H2 that of 02 molecule of the same temperature.
Tap
16)

Two gas bulbs A and B of volume V and 3V respectively are separated and filled with
gas. They are connected as shown with the tap closed. The pressure of the gas in A and B
are P and 4P respectively when the tap is opened, the common pressure becomes 60Pa.
assuming isothermal conditions, find the value of P.
17) The mean kinetic energy of one mole of helium gas at room temperature is 3.75 x 103J.
Calculate the room temperature.
18) A beam of 2.0 x 1022 Nitrogen atoms each of mass 2.32 x 10-26kg is incident normally on
cubical content of edge 10.0cm the beam is reflected through 1800. If the mean speed of
the atoms is 480ms-1. Find the pressure exerted by the Nitrogen gas.
19) Two containers A and B of volume 3.0 x 103cm3 and 6.0 x 103cm3 respectively contains
helium at a pressure to 373k while container B is cooled to 2273K. Find the final pressure
of the helium gas.
20) Air consists of approximately 20% oxygen and 80% Nitrogen. The relative molecular
masses of 02 and N2 are 32 and 28 respectively. Calculate the ratio of
(i). The mean square speed of 02 to that of N2in air
(ii).The partial pressure of 02 to that of N2 in air.
21) The root mean square of the molecules of a gas is 44.72ms-1. Find the temperature of the
gas if its density is 9.0 x 10-2kgm-3 and the volume is 42.0m3
22) A helium gas occupies a volume of 0.02m3 at a pressure of 200K Pa and temperature of
270C. Calculate;
(i). Mass of the helium gas

183 | P a g e
(ii).The root mean square speed of the molecules of the helium gas.
Take molecules mass of helium = 4g.
23) Two hallow sphere A and B of volume 500cm3 and 250cm3 respectively are connected by
a narrow tube fitted with a tap. Initially the tap is closed and A is filled with an ideal gas
at 100C at a pressure of 3.0 x 105Pa and B is filled with an ideal gas at 1000C at a
pressure of 1.0 x 105Pa. calculate.
(i). The equilibrium pressure when the tap is opened
(ii).The resulting temperature when the tap is opened.
24) Hydrogen has a density of 0.09kgm-3 at 760mmHg and 00C. Find the root mean square
velocity of H2 at 270C. Given that the density of mercury is 13.6 x 103kgm-3.
25) A volume of 0.23m3 contains N2 at a pressure of 0.50x105Pa and temperature 300K
assuming that the gas behaves ideally, calculate the amount in mol of N2 present.
26) Calculate the root mean square speed of N2 molecules at a temperature of 300k. (Molar
mass of N2 = 0.028kgmol-1, molar gas constant = 8.3JK-1mol-1).
27) Two vessels A, B of equal volume are connected by a narrow tube of negligible internal
volume. Initially the whole system is

A B
Filled with 3g of dry air at a pressure of 10 Pa and temperature 300K, the temperature of
5

the vessel B is now raised to 600K, the temperature of A remaining 300K. What is;
(i). The new pressure in the system
(ii).The mass of air in A and in B.
28) An industrial firm supplies compressed air cylinders of volume 0.25m3 filled to a
pressure of 20MPa at 170C. calculate the contents of the cylinder expressed in;
(i). Moles
(ii).(Kilograms) (Take gas constant R = 8.3JK-1mol-1, the molar mass of air is
0.029kgmol-1).
29) N2 gas under an initial pressure of 5.0 x 106Pa at 150C is contained in a cylinder of
volume 0.04m3. after period of three years the pressure has fallen to 2.0 x 106Pa at the
same temperature because of leakage(Assume molar mass of N2 = 0.028kgmol-1, R is
8.3Jmol-1K-1, NA = 6.0 x 1023mol-1) Calculate;
(i). The mass of gas originally present in the cylinder
(ii).The mass of gas which escaped from the cylinder in three years
(iii). The average no. of Nitrogen molecules which escaped from the cylinder
per second. *Take one year to be equal to 3.2 x 107s)
30) A cylinder containing 19kg of compressed air at a pressure 9.5 times that of the
atmosphere is kept in a store at 70C.when it is moved to a workshop where the
temperature is 270C a safety valve on the cylinder operates, releasing some of the air. If

184 | P a g e
the valve allows air to escape when its pressure exceeds 10times that of the atmosphere.
Calculate the mass of air at escapes.

31) A mole of an ideal gas at 300K is subjected to a pressure of 105Pa and its volume is
0.025m3. calculate;
(i). The molar gas constant R
(ii). The Bolts man constant k
(iii). The average translational kinetic energy of a molecule of the gas. (Na = 6.0 x1023
mole – 1).
32) A vessel of volume 1.0 x 10-3m3 contains helium gas at a pressure of 2.0 x 105pa when the
temperature is 300k.
(i). What is the mass of helium in the vessel
(ii). How many helium atoms are there in the vessel
(iii). Calculate the r.m.s speed of the helium atoms
(Relative atomic mass of helium = 4, NA = 6.0 x 1023mol-1, R = 8.3Jmol-1K-1).
33) Helium gas of molar mass = 0.004kg mol-1 is contained in a vessel of volume 8.0 x 10-4 at
a temperature of 300K,the pressure of the gas is 200KPa. Calculate;
(i). The mass of helium present
(ii).The internal energy (The translational kinetic energy of the gas molecules) molar
gas constant = 8.3Jk-1mol-1).
34) A cubical container of volume 0.10m3 contains uranium hexafluoride gas at a pressure of
1.0 x 106Pa and temperature of 300K. Assuming that the gas is ideal determine;
(i). No. of moles of gas present given that the universal gas constant, R = 8.3Jk-1mol-1)
(ii). The mass of gas present given that its relative molecular mass is 352.
(iii). The density of the gas
(iv). The rms speed of the molecules.
35) Air may be taken to consist of 80% Nitrogen molecules and 20% oxygen molecules of
relative molecular masses 28 and 32 resp. Calculate;
(i). The ratio of root mean square of N2 molecules to that of 02 molecules in air.
(ii). The ratio of the partial pressure of Nitrogen and oxygen molecules in air
(iii). The ratio of the root mean square of N2 molecules in air at 100C to that at 1000C.
36) (a) A vessel of volume 1.0 x 10-2m3 contains an ideal gas at temperature of 300K and
pressure of 1.5 x 105Pa. Calculate the mass of gas, given that the density of the gas at
temperature 285K and the pressure 1.0 x 105Pa is 1.2kgm-3.
(b) 750J of heat energy is suddenly released in the gas causing an instantaneous rise of
pressure to 1.8 x 105Pa. assuming ideal gas behavior. Calculate the temperature rise
and hence the specific heat capacity at constant volume of the gas.

37) Calculate the r.m.s of the helium molecules at 20000C. (mass of one mole of helium =
4.00g, R = 8.31Jmol-1k-1).

185 | P a g e
38) B
A

Two bulbs A, of volume 100cm3, and B of volume 50cm3, are connected to a three way
tap T which enables them to be filled with gas or evacuated. The volume of the tubes may
be neglected. Initially A is filled with an ideal gas at 100C to a pressure of 3.0 x 105Pa.
Bulb is filled with an ideal gas at 1000C to a pressure of 1.0 x 105Pa. the two bulbs are
connected with A maintained at 100C and B at 1000C. Calculate the pressure at
equilibrium.

TOPIC 19: THERMODYNAMICS


DEFINITIONS

1. Define the following terms; a dynamic system, internal energy, heat capacity of a gas, molar
heat capacity/Principal heat capacity of a gas at constant volume, molar heat capacity/ principal
heat capacity of a gas at constant pressure ,specific heat capacity at constant volume ,specific
heat capacity at constant pressure ,isothermal change, Adiabatic change ,isobaric process,
isovolumetric process ,a reversible process ,Isothermal reversible process, Isobaric reversible
process ,Isovolumetric reversible process ,a adiabatic reversible process, isothermal reversible
process ,isobaric reversible process ,isovolumetric reversible process, a adiabatic reversible
process .

LAWS, PRINCIPLES AND THEOREMS

1. State the first law of thermal dynamics


2. State the Zeroth’s law of thermo dynamics

EXPLANATIONS

1) Explain why molar principal heat capacity of a solid at constant pressure is less than the
molar principal heat capacity of a gas at constant pressure.
2) Explain the significance of heat supplied to a gas that goes through;
(i). Isothermal change
(ii). Isovolumetric change
(iii). Isobaric change

186 | P a g e
3) Explain why molar heat capacity of a gas at constant pressure is greater than molar heat
capacity of a gas at constant volume.
4) By considering a gas confined in a cylinder by a movable piston use kinetic theory to
explain why a adiabatic expansion of a gas results into cooling
5) Explain why gas areal gas and ideal gas at a common temperature , have different
amount of internal energies
6) Explain why the difference between the principal heats of a solid is small.
7) Explain why internal energy of an ideal gas is independent of its volume yet that for real
gas depends on its volume
8) Explain the meaning of molar heat capacity at constant volume Cv and molar heat
capacity at constant pressure Cp.
9) Explain how an isothermal and a adiabatic expansion can be achieved in practice
10) Explain what happens when a quantity of heat is supplied to a fixed mass of a gas
11) Explain why temperature remains constant during change of phase from solid to liquid.
12) Explain the molar heat capacity of an ideal gas at a constant pressure Cp differs from the
molar heat capacity at constant volume.
13) Explain why the temperature of a gas increases when it is heated
14) Explain why the difference between the specific heat capacity at constant pressure Cp
and that at a constant volume Cv is negligible for solids but not for gases. Hence show
that for one mole of a gas Cp – Cv = R.
15) By considering a gas confined in a cylinder by a movable piston use kinetic theory to
explain why a adiabatic expansion of a gas results in cooling
16) Why is the distinction between specific heat capacity at constant pressure and that at
constant volume important for gases but less important for solids and liquids
17) Explain using kinetic theory what happens to energy that enters the substance as heat in
each of the following
(i). An ideal gas maintained at constant volume
(ii). An ideal gas maintained at constant pressure
(iii). A crystalline solid at a temperature remote from its melting point
(iv). A crystalline solid at its melting point.
18) Explain the following observations;
(i). When pumping up a bicycle tyre the pump barrel gets warm
(ii). When a gas at high pressure in a container is suddenly released the container cools.

MERITS, DEMERITS, FACTORS, FEATURES EXAMPLES AND DIFFERENCES

1) State examples of the following;


(i). Isothermal changes
(ii). adiabatic changes
2) State the conditions of the following to occur
(i). Isothermal process

187 | P a g e
(ii). adiabatic process
3) State the conditions under which a real gas behaves like an ideal gas.
4) State three difference between ideal and real gas
5) State the conditions for achieving a reversible isothermal change.
6) Give on practical example of each of the following;
(i). A process in which heat is supplied to a system without causing an increase in
temperature
(ii).A process in which no heat enters or leaves a system but the temperature changes

DERIVATIONS AND RELATIONS

1) Distinguish between;
(i). Reversible isothermal change and adiabatic expansion of a gas.
(ii).Isovolumetric change and isobaric change.
2) Distinguish between;
(i). A reversible process and an irreversible process giving two example of each.
(ii).An isothermal change and a adiabatic change
3) Derive the relationship between the molar principal heat capacity at constant pressure and
molar principal heat capacity at constant volume for one mole of a gas.
4) Define principle specific heat capacity of a gas at constant pressure
5) Define isothermal and adiabatic process
6) Show that the work done in expanding the gas of volume V1 to volume V2 is given by W
V2

= ∫ pdv where P – Pressure it exerts on the piston of a cylinder.


V1

7) Sketch pressure against volume curves on the same axes for a fixed mass of an ideal gas
undergoing through the following changes.
(i). Isothermal expansion
(ii). Isothermal compression
(iii). adiabatic expansion
(iv). adiabatic compression
(v). Isovolumetric
(vi). Isobaric metric
(vii). Isobaric expansion
(viii). Isobaric compression
8) Derive the relationship between cp and cv
9) Sketch on the same axes diagram and starting from the same point (P1, V1) , P –V sketch
curves for a fixed mass of an ideal gas undergoing the following;
(i). An isothermal process
(ii). An adiabatic process
(iii). An isovolumetric process
(iv). An isobaric process

188 | P a g e
10) A fixed mass of an ideal gas expands from a volume V1 to volume V2
(i). Derive the general expression for the work done by the gas
(ii).Find an expression for work done if the expansion is isothermal.
11) Define isothermal and adiabatic changes and give the equation relating the pressure and
volume of an ideal gas for each type of change.
12) A fixed mass of gas in state (P1V1) under goes and expansion to state (P2V2) obtain an
expression for the work done by the gas.
(i). If the expansion is isothermal
(ii).If the expansion is adiabatic
13) Indicate the different states of a real gas at different temperature on a pressure versus
volume sketch graph.
14) Sketch a pressure versus volume curve for a real gas undergoing compression below its
critical temperature. Explain the main features of the curve
15) Derive an expression for work done during an isothermal expansion.
16) Using the same axis, sketch pressure versus volume graphs for area/gas
(i). Above the critical temperature
(ii). At the critical temperature
(iii). Below the critical temperature indicate in your sketch the different phases of the
gas
17) Distinguish between a reversible isothermal and an adiabatic expansion of a gas starting
from the same point (P1V1), sketch P- V curves for these two processes
18) A fixed mass of a gap in the state (P1 V1) undergoes an isothermal expansion to the state
(P2, V2) obtain an expression for the work done by the gas.
19) Sketch a P – V cure for a real gas below its critical temperature and label the main
features of the curve
20) Derive the relation between the principle molar heat capacities of a gas
21) State the first law of thermodynamics and use it to distinguish between isothermal and
adiabatic changes in a gas.
22) With the aid of a P – V diagram, explain what happens when a real gas is compressed at
different temperatures.

CALCULATIONS ON THERMODYNAMICS

1) Air is contained in a cylinder by a frictionless gas tight piston.


a) Find the work done by the gas as it expands from a volume of 0.015m3 to a
volume of 0.027m3 at constant pressure of 2. X 105Pa
b) Find the final pressure if starting from the same initial conditions as in (a) and
expanding by the same amount the change occurs.
(i). Isothermally
Cp
(ii).Adiabatically (for air = 1.40).
Cv

189 | P a g e
2) A cylinder fitted with a piston which can move without friction contains 0.050moles of a
monatomic ideal gas at a temperature of 270C and pressure of 1.0 x 105Pa, Calculate;
(i). The volume
(ii).The internal energy of the gas

(b) The temperature of the gas in (a) is then raised to 770C, the pressure remaining
constant. Calculate;

(i). The change in internal energy


(ii). The external work done
(iii). The total heat energy supplied.
3) A cylinder with a piston contains 0.5moles of oxygen at 2.0 x 105Pa and 300k,the gas
first expands at constant pressure to twice its original volume, it is then compressed
isothermally back to its volume and finally it is cooled at constant volume to its original
pressure.
a) Show the series of process on a P – V diagram
b) Find ;
(i). The work done
(ii). The heat added
(iii). The change in internal energy, during the initial expansion
c) Calculate the work done by the gas during the entire cycle.
(Molar heat capacity of 02 at constant pressure = 29.4Jmol-1k-1).
4) 1m3 of helium gas at constant at a temperature of 200C expands adiabatically to a volume
of 8m3.
(i). Calculate the change in temperature
(ii).If the same expansion is carried out isothermally what work is done by the gas if
the pressure of the gas was 4.04 x 105Pa.

(iii). (
What is the heat absorbed in the isothermal process. Take =
Cp 5
Cv 3 )
5) The specific heat capacity of Neon at a constant pressure is 1.03 x 103Jkg-1k-1. The

molecular mass of Neon is 20.2. Find the ration


Cp
Cv ( Cp
for Neon. Y = =1.66
Cv )
6) 2 litres of Nitrogen gas at a pressure of 1.0 x 105Pa, and temperature of 270C is heated at
a constant pressure until its volume is doubled. It is then cooled at a constant volume
until its pressure is 2.5 x 104Pa. the gas is then compressed adiabatically to its original
Cp
volume the ratio for Nitrogen is 1.40.
Cv
(i). Show on a labeled P – V diagram the above process
(ii).Calculate the final temperature of the gas
(iii). What is the pressure of the gas at the end of the compression

190 | P a g e
(iv). If the molar heat capacity of Nitrogen at constant pressure is 29.2Jk-1. How
much heat is supplied to the gas during the expansion at constant pressure.
7) One mole of an ideal gas occupies 2.24 x 10-2m3 at a pressure of 1.01 x 105Pa at 00C. If
the molar heat capacity at constant pressure is 28.5Jmol-1K-1. Find the molar heat
capacity at constant volume and explain why it’s different.
8) Air expands from a volume of 0.015m3 at a pressure of 2.0 x 105Pa to a volume of
0.027m3. if the ratio of the principal specific heat capacities for air is 1.40. find the work
done if the expansion is
(i). Isothermal

( Cp
(ii).Adiabatic Assume =1.40
Cv )
5
9) An ideal gas with ϒ = , initially occupying 1.01 at 273k and 1.0 x 105Pa is compressed
3
isothermally to a volume 0.5L. It is then allowed to expand adiabatically to the original
volume.
(i). Sketch the process on a P-V diagram
(ii).Find the final pressure and temperature of the gas
(iii). Calculate the work done by the gas.
10) 1000moles of an ideal gas is initially at a pressure of 1.0x105Pa and a temperature of
250C. It undergoes a reversible adiabatic expansion to twice its volume followed by a
reversible isothermal compression to its original volume.
(i). Sketch a P – V diagram to show the two process
(ii).Calculate the final temperature of the gas
(iii). Find the work done by the gas during the isothermal compression.

( Assum =
Cp 5
Cv 3 )
5
11) An ideal gas with Cv = R per mole and ϒ = 1.40 expands adiabatically from initial
2
pressure of 2.02 x 105Pa and volume 2.0L to a final pressure of 1.01 x 105Pa. Assume
that there are 1.5moles of gas present, calculate;
(i). The initial temperature
(ii).The final volume
(iii). The work done by the gas.
12) A certain ideal gas has ϒ= 1.67,
a) Calculate the molar specific heats Cp and Cv
b) A 0.70m3 of the sample of this gas initially at a pressure of 4.5 x 104 Pa is
compressed adiabatically to a volume of 0.50m3. calculate;
(i). The final pressure
(ii). The ration of final to original temperature
(iii). The work done by the gas.

191 | P a g e
13) An ideal gas in a cylinder is compressed so that its final volume is one fifth of its initial
volume. The gas has ϒ = 1.4 and its initial temperature and pressure are 170C and 1.0 x
105Pa respectively.
(i). Suppose the compression is isothermal. What is the final temperature and
pressure of the gas
(ii).What is the final pressure and temperature of the gas if the compression is
adiabatic
(iii). Draw using the same axes graphs to show how the pressure depends on
the volume for isothermal and adiabatic compression.
(iv). Calculate the work done by the gas in (i).
14) A vessel contains 2.5 x 10-3 m3 of an ideal gas at a pressure of 3.5 x 104Nm-2 and a
temperature of 350C,the gas is compressed isothermally to a volume of 1.0 x 10-3m3. It is

( Cp
then allowed to expand adiabatically to its original volume. Taking =1.40 Find.
Cv )
(i). The final temperature of the gas
(ii).The work done in the isothermal compression
(iii). Final pressure during isothermal compression
(iv). Final pressure during adiabatic expansion.
15) A gas at 27 C, volume of 3000cm3 and a pressure of 80cmHg expands isothermally to
0

double its volume. The gas is then compressed adiabatically to half its original volume.
(i). Show the changes of a P-V sketch

( Cp
(ii).Calculate the final temperature and pressure of the gas. Take =1.40
Cv )
16) A mass of air occupying initially a volume of 2000cm3 at a pressure of 76cmHg and at
temperature of 200C is expanded adiabatically and reversibly to twice its volume and
then compressed isothermally and reversibly to a volume of 3000cm3. Find the final
5
temperature and pressure assuming the ratio of the molar specific heat capacities to be .
3
17) The specific heat capacity of a diatomic gas at constant volume is 0.410Jkg k .
-1 -1

Calculate the specific heat capacity at constant pressure and specific constant (ϒ =1.40).
18) 2.00 moles of 02 at 290K is enclosed in an insulated container by a frictionless piston and
pressure outside the cylinder is 0.4mpa when 1.16kJ of heat is supplied to this gas. Its
temperature increased to 310K and increased by 0.00083m3. Calculate the principal heat
capacity of the gas and the universal gas constant.
19) A gas is enclosed in a cylinder by frictionless Piston of area 100cm2. 250J of heat is
supplied to this gas and it expands against external pressure of 0.1MPa and the piston is
displaced by 15.0cm along the cylinder. Calculate external work done by the gas and
increase in the gas’s internal energy.
20) A cylinder contains 4moles of oxygen gas at a temperature of 270C, the cylinder is
provided with a frictionless piston which maintains a constant pressure of 1.0 x 105Pa.

192 | P a g e
The gas is heated until its temperature increased to 1270C.
(i). Calculate the amount of heat supplied to the gas
(ii).What is the change in the internal energy of the gas
(iii). The work done by the gas.
21) A vessel containing 2.5 x 10-3m3 of an ideal gas at a pressure of 8.5 x 104Nm-2 and
temperature of 350C, the gas is compressed isothermally to a volume of 1.0 x 10-3m3.it is
then allowed to expand adiabatically to the original volume. (Taking ϒ = 1.40)
Calculate;
(i). The final temperature of the gas
(ii).The work done during the isothermal compression of the gas.

22) 4 kilo moles of an ideal gas is carried through a reversible cycle shown in the P – V
diagram below;
23)
(1) (2)
P1

(3)
P2

0 V1 V2 V

In state (1), the gas has a pressure of 1.0 x 105Nm-2 and temperature of 00C, the volume
of the gas in state (2) is four times that in state (1) process (3) to (1) is a reversible
isothermal expansion.
(Taking R = 8.31Jmol-1K-1)
Determine
(i). The temperature of the gas in state (2)
(ii).The pressure of the gas in state (3)
(iii) The work done by the gas in the cycle.
24) An ideal gas of specific heat capacity ratio ϒ = 1.40 is expanded adiabatically and
reversibly from a pressure of 80cmHg, volume 4m3 and a temperature of 110C to a
pressure of 30cmHg. It then under a reversible isothermal compression to its original
pressure. Finally it is expanded isobatically to its original volume
a) Sketch the P-V diagram showing the above process
Find;
(i). The volume at the end of the adiabatic expansion
(ii). The temperature at the end of the isothermal compression
(iii). The work done during the isobaric expression.

193 | P a g e
25) A cylinder with a piston contains one mole of gas at a pressure of 1.0 x 105Pa and
temperature of 300K. The gas is heated at constant pressure until its volume doubles. It is
then compressed isothermally back to its original volume and finally it is cooled at
constant volume to its original state.
(i). Represent the above processes on a P-V diagram
(ii).Calculate the work done by the gas in the cycle.
26) A gas with specific heat capacity ratio ϒ =1.67 is made to expand adiabatically from a
volume of 1.2 x 104cm3 and a pressure of 2.5 x 105Nm-2. If the final pressure is 1.9 x
105Nm-2. Find the final volume of the gas.
27) Air in a cylinder has a volume V = 1000cm3 at an initial pressure 1.0 x 105Nm-2. And
temperature T1 = 300K. A sequence of changes is imposed on the air, assumed to behave
as an ideal gas.

P2 C

(iii) (ii)

P1 A (i) B

0 V1 V2

(i). AB the air is heated to 375k at a constant pressure. Calculate the new volume V2.
(ii).BC the air is compressed isothermally to volume V1. Calculate volume the new
pressure P2.
(iii). CA the air is cooled at constant volume to pressure P1. Calculate the work done
on the gas during the cycle.
28) Nitrogen gas in an expandable container is raised from 00C to 500C at a constant pressure
of 4.0 x 105Pa. the total heat added is 3.0 x 104J.
(i). Find the number of moles of the gas
(ii).Find the change in internal energy of the gas
(iii). Find the work done by the gas (Molar heat capacity of Nitrogen at
constant pressure is 29.1Jmol-1k-1).
29) 4moles gas is initially at a pressure of 8.0 x 104Nm-2 and a temperature of 350C. the gas
under goes a reversible adiabatically expansion to four times of its volume followed by a
reversible isothermal compression to its two process
(i). Sketch a P-V diagram to show the two process
5
(ii).Calculate the final temperature and pressure .(R = 8.31Jk-1mol-1, Cv = R )
2
30) A cylinder contains 4kg of oxygen gas at temperature of 47 C the cylinder is provided
0

with a frictionless piston, which maintains a constant pressure of 1.0 x105Pa,the gas is

194 | P a g e
heated until its temperature is increased to 1470C.(Cp = 29.4Jmole -1 k-1, R = 8.31Jmol -1k-
1
).Calculate;
(i). The amount of heat supplied to the gas
(ii).The change in internal energy of the gas.
31) A vessel contains 1.5 x 10-3m3 of a gas at a pressure of 3.7 x 104Nm-2 and a temperature
of 250C. The gas is compressed 3.7 x 104Nm-2 and a temperature of 250C, the gas is
compressed isothermally to a volume 9.0 x 10-4m3. Then it is allowed to expand
adiabatically to its original volume.
(i). Sketch a P – V graph for the changes, indicating on it the net work done
(ii).Find the final temperature and pressure of the gas
(iii). Find the net work done or by the gas indicating which. (Take ϒ = 1.40).
32) An ideal gas at a pressure of 2.0 x 106Nm-2 occupies a volume of 2000cm3 at 47.50C. The
gas expands adiabatically to a final pressure of 1.1 x 106Nm-2. (Y = 1.40, R = 8.3Jmol-1k-
1
).Find;
(i). The no. of moles of the gas
(ii).The final volume of the gas
(iii). The final temperature of the gas.
33) A vessel containing 1.5 x 10-3m3 of an ideal gas at a pressure of 8.7 x 104Pa and
temperature 250C is compressed isothermally to half its volume and then allowed to
expand adiabatically to its original volume. (ϒ= 1.67)
(i). Calculate the final pressure and temperature of the gas
(ii). Sketch the P –V graph for the whole process
(iii). Calculate the work done on the gas during the isothermal process.
34) The specific heat capacity of 02 at constant volume is 719Jkg-1k-1. If the density of
oxygen at stp is 1.429kgm-3. Calculate the specific heat capacity of oxygen at constant
pressure.
35) An ideal gas at 270C and pressure of 1.0 x 105Pa is compressed reversibly and
isothermally until its volume is halved. It is then expanded reversibly and adiabatically to
twice its original volume. Calculate the final pressure and temperature of the gas if ϒ =
1.40.
36) Calculate the work done against the atmosphere when 1kg of water turns into vapour at
atmospheric pressure of 1.01 x 105Pa.(Density of water vapour = 0.598kgm-3).
37) An ideal gas of volume 100cm3 at s.t.p expands adiabatically until its pressure drops to a
quarter its original value. Find the new volume and expenditure if the ratio of the
principle specific heat capacities is 1.40.
38) A fire extinguisher is filled with 1.0kg of compressed N2 gas at a pressure of 1.2 x 106Pa
and a temperature of 200C. if the gas escapes by expanding adiabatically to pressure of
1.0 x 105Pa when the nozzle of the fire extinguisher is opened. Find the;
(i). Originally volume of the gas
(ii).Temperature of the expanded gas (Take ϒ = 1.4).

195 | P a g e
39) An ideal gas at a pressure of 2.0 x 106Pa occupies a volume of 2.0 x 10-3m3 at 47.50C.
The gas expands adiabatically to a final pressure of 1.10 x 105Pa. the ratio of the specific
heat capacity at a constant pressure to that at a constant volume is 1.40. calculate;
(i). The number of moles of the gas
(ii). The final volume of the gas
(iii). The find temperature of the gas (R = 8.31Jmol-1k-1).

40) A gas initially occupying a volume of 1.0L at 273K and 1.0 x 105Pa is compressed
isothermally to a volume of 0.5L. it is then allowed to expand adiabatically to the
original volume.
(i). Find the final temperature and pressure of the gas
(ii).Indicate the process on a P – V diagram.
41) An ideal gas of volume 1.0L at s.t.p expands at a constant pressure to a volume of 3.0L
calculate;
(i). The work done by the gas
(ii).The final pressure of the gas.
42) A gas at a pressure of 1.0 x 106Pa is compressed adiabatically to half its volume.
Calculate the final pressure of the gas. (Assume the ratio of the principal specific heat
Cp
capture = 1.40).
Cv
43) An ideal gas is trapped in a cylinder by a movable Piston. Initially it occupies a volume
of 8.0 x 10-3m3 and exerts a pressure of 108kPa. The gas under goes an isothermal
expansion until its volume is 27 x 10-3m3. It then compressed adiabatically the original
volume of the gas.
(i). Calculate the final pressure of the gas
(ii).Sketch and label the two stages of the gas on a P – V diagram.
(The ratio of the principal molar heat capacities of the gas = 5:3).
44) The temperature of one mole of helium gas at a pressure of 1.0 x 105Pa increased from
200C to 1000C when the gas is compressed adiabatically. Find the final pressure of the
gas (Take ϒ = 1.67).
45) Nitrogen gas is trapped in a container by a movable piston. If the temperature of the gas
is raised from 00C to 500C at a constant pressure of 4.0 x 105Pa and the total heat added is
3.0 x 104J, calculate the work done by the gas.
Cp
(The molar heat capacity of Nitrogen at constant pressure is 29.1Jmol-1K-1. = 1.40
Cv
46) A mass of an ideal gas of volume 200cm3 at 144k expands adiabatically to temperature
of 137k. Calculate its new volume. (Take ϒ = 1.40).
47) One kilogram of water is converted into steam at a temperature of 100C and a pressure of
1.0 x 105Pa. if the density of steam is 0.58kgm-3 and the specific latent heat of
vaporization of water is 2.3 x 106Jkg-1. Calculate the;

196 | P a g e
(i). External work done
(ii).Internal energy
(1.723 x 105J, 2.128 x106J)

48) Ten moles of a gas, initially at 270C are heated at a constant pressure 1.0 x 105Pa and the
volume increased from 0.250cm3 to 0.375m3. calculate increase in internal energy
(Assume Cp= 28.5Jmol-1k-1).
49) The temperature of one mole of helium gas at a pressure of 760mmHg increased from
280C to 1080C when the gas is compressed adiabatically. Find the final pressure of the
gas. Take ϒ = 1.67).
50) 1 mole of gas has a volume of 2.23 x 10-2m3 at a pressure of 1.01 x 105Pa at 00C, if the
molar heat capacity at constant pressure is 28.5Jmol-1k-1. Calculate the molar heat
capacity at constant volume 20g of this gas at initially 270C is heated at constant volume
20g of this gas at initially 270C is heated at constant pressure of 1.01 x 105Pa so that its
volume increased from 0.250m3 to 0.375m3, calculate;
(i). The external work done
(ii).The increase in internal energy
(iii). The heat supplied if (Mr. of gas) = 2g.
51) A cylinder filled with a piston which can move without friction contain 0.050mole of a
monatomic ideal gas at temperature of 270C and pressure of 1.0 x 105Pa calculate;
(i). The volume
(ii).The internal energy of the gas.
(b) The temperature of the gas in (a) is raised to 770C, the pressure remaining constant
find.
(i). Change in internal energy
(ii). The external work done
(iii). The total heat energy supplied (R = 8.3Jmol-1 k-1).
52) At a temperature of 1000C and a pressure of 1.01 x 105Pa, 1.00kg of steam occupies only
1.04 x 10-3m3. The specific latent heat of vaporization of water at 1000C is 2.26 x 106Jkg-1
for a system consisting of 1.00kg of water changing to steam at 1000C and 1.01 x 105Pa.
find;
(i). The heat supplied to the system
(ii).The work done by the system
(iii). The increase in internal energy of the system.
53) A steel pressure vessel of volume 2.2 x 10-2m3 contains 4.0 x 10-2kg of a gas at a pressure
of 1.0 x 105Pa and temp 300k.An explosion suddenly releases 6.48 x 104J of energy
which raises the pressure instantaneously to 1.0 x 106Pa. Assuming no loss of heat to the
vessel and ideal gas behavior. Calculate;
(i). The maximum temperature attained
(ii).The two principal specific heat capacities of the gas
(iii). What is the velocity of sound in this gas at a temperature of 300K.

197 | P a g e
54) A mass of 0.35kg of ethanol is vaporized at its boiling point of 780C and a pressure of
1.0 x 105Pa at this temperature the specific latent heat of vaporization of ethanol is 0.95 x
106Jkg-1 and densities of the liquid and vapour are 790kgm-3 and 1.6kgm-3 respectively.
Calculate;
(i). The work done by the system
(ii).The change in internal energy of the system.

TOPIC: 20 HEAT TRANSFERS


DEFINITIONS
3. 1. Define the following terms;conduction,coefficientof conductivity, temperature
gradient,convection,forcedconvection, free convection, thermal radiation, black body,
black body Radiation, solar constant, electromagnetic spectrum, emissivity of a body,
global warming, greenhouse effect,relativeintensity,Solar energy, Steady state conditions.

LAWS, PRINCIPLES AND THEOREM


1. State Prevost’s theory of heat exchanges
2. State Stefan –Boltzmann’s law of black body radiation
3. State Wien’s displacement law of black body Radiation
4. State the laws of black body radiation
5. State Wien’s displacement law and Stefan’s law of black body radiation

MERITS, DEMERITS, FEATURES, FACTORS AND EXAMPLE

1) Give any three optical properties of infrared –radiations. Describe a simple experiment to
demonstrate any one of the properties above.
2) State one effect of the following radiations;
(i). X –rays
(ii).Infrared and Radio waves
3) Give one characteristics property of infrared radiation
4) Briefly discuss the advantages of apparatus in an experimental to measure the thermal
conductively of glass.
5) State two factors which determines the rate of heat transfer through a material
6) Mention four precautions taken when determining thermal conductivity of a metal.

DERIVATIONS AND RELATIONS

1) Compare the mechanisms of heat transfer in poor and good solid conductors.
2) Sketch the variation of intensity of radiation emitted with wave length for a black body at
three different temperatures.

198 | P a g e
3) A well consists layers of thickness d1 and d2 with thermal conductivities k1 and k2. if the
surface of the wall are maintained at temperatures 1 and 2, show that the rate of heat
2−1
transfer through the wall is A ( d 1 d2 ) where A – is the area
+
k1 k2
Q ¿
4) The thermal conductivity, k1 of a material is given by the expression =KA(2−1 ¿ x
t
state two conditions that must be satisfied before this expression may be applied
5) Describe the characteristics features of the relative intensity distribution for a black body
6) (a) One end of a long uniform material bar is heated in a steam chest and other end is
kept cool by a current of water. Draw sketch graphs to show the variation of temperature
along the bar, when the bar under steady state conditions has been attained if the bar is;
(i). lagged so that no heat escapes from the sides
(ii).exposed to the surrounding air.
7) With is meant by the thermal conductivity of a material stating its dimensions
8) The ends of a lagged metal bar are pressed against metal tanks at temperatures T 1 and T2
where T2 is greater than T1 but are separated from them by thin layers of dirt. Draw a
diagram to show the variation of temperature with distance from one tank to other
9) Define a black body. Give two examples of a body which approximate to a black body
10) (i) Draw a sketch graphs to illustrate how the distribution of energy radiated changes as
the temperature of the black body is raised.

(ii) As a material is heated, it appears to change colour .Account for this observation

11) With the aid of suitable sketch graphs, explain the temperature distribution a long a
lagged and an un lagged metal rod
12) (i) Draw sketch graphs to show variation of relative intensity of black body radiation
with wave length for three different temperature

(ii) Describe the features of the sketch graphs in (i) above

13) The two end of a metal bar length 1.0m are perfectly lagged up to 20cm from either end.
The ends of the bar are maintained at 100 0C and 0 0C respectively
(i). Sketch a graph of temperature versus distance along the bar
(ii).Explain the features of the graph in (i)above
14) Sketch on the same axes, the variation of relative intensity with wave length for a black
body and a non- black body at the same temperature.
15) Sketch the spherical distribution of a black body radiation for the different temperatures
and describe their main features.
16) With the aid, of a sketch graphs, explain the temperature distribution a long lagged and
un lagged metal rods, heated at one end.

199 | P a g e
17) Sketch the variation of intensity with wave length in a black body for three different
temperatures

EXPLANATIONS
1. Explain why on a winter night one would feel warmer when clouds cover the sky than
when the sky is clear.
2. Explain the mechanism of heat transfer in metals/ solids.
3. Explain why is black body radiation referred to as temperature radiation
4. Explain how Greenhouse effect leads to global warming
5. Explain why a metal is a better conductor of heat than glass
6. Explain the mechanism of heat conduction in poor conductor
7. Explain briefly why is it necessary to make a poor conductor thin with a large cross
sectional area when determining its thermal conductivity
8. Explain using the molecular theory of matter the mechanism of heat conduction in
insulators
9. Briefly account for the fact that metals are better conductors of heat than insulators
10. Explain carefully what is meant by a black body absorber. How can it be realized in
practice
11. Explain what is meant by a black body absorber .How can it be realized in practice
12. Explain how heat is conducted through a glass rod
13. With the material of a poor conductors is made thin and is sand wiched between thick
metal plates and that of a good conductor is made longer than its diameter in
determination of their thermal conductivity
14. Explain how heat is conducted through a glass rod
15. Explain briefly why the Centre of a fire appear white
16. Explain briefly the greenhouse effect and its relation to global warming
17. Explain the greenhouse effect and indicate why it leads to global warming
18. Explain why a baby has to be wrapped well on a cold day
19. Explain the occurrence of land and sea breeze
20. Explain the mechanism of thermal conduction in non –metallic solids
21. Why are metals better thermal conductors than non-metallic solids
22. Explain the temperature distribution a long
23. a perfect lagged metal bar
24. an un lagged metal bar
25. Given an expression for the power radiated by a black body at a given temperature.
Explain the symbols and state the units in the expression. state the units in the expression
EXPERIMENTS
1. Describe how you would determine the coefficient of thermal conductivity of a poor
conductor of heat for example cork, glass, wood

200 | P a g e
2. Describe with the aid of a diagram, how you would measure the thermal conductivity of a
poor conductor, stating the necessary precautions
3. Describe an experiment to determine the thermal conductivity of a poor conductor of
heat
4. How can a black body be realized in practice
5. Describe an experiment to determine the thermal conductivity of a good conductor like
copper.
6. (i) Describe Searle’s method of determining the thermal conductivity of a good
conductors of heat
7. State any two precautions that must be taken in the experiment (i) above
8. With the use of diagram, explain how a thermopile is used to detect radiation
9. Describe briefly how the bolometer can be used to detect thermal radiation
10. Describe an experiment to determine thermal conductivity of a material of very low value
11. Draw a diagram showing the arrangement of apparatus for measurement of thermal
conductivity of a glass rod. Outline the steps taken to perform the experiment
12. With the help of suitable diagrams, explain briefly one way in each case of detecting
ultraviolet and infrared radiation
13. ( i) Draw a labeled diagram of a simple experiment at arrangement you would use to
detect infrared radiation in the beam of sun light
14. What would you observe in the experiment (i) above
15. Describe how infrared radiation can be detected using a bolometer.
16. Describe briefly how a thermopile can be used to detect thermal radiations
17. Describe how an approximate black body can be realized in practice
18. With aid of a labeled diagram, describe how thermopile can be used to detect infrared
radiation
19. (i) Describe Searle’s method of determining the thermal conductivity of a good
conductor of heat
20. Why is the method in (i) above best suited for a good conductor of heat
21. With the aid of a labeled diagram, describe the structure of a sensitive infrared detector
and explain how it works.
22. Briefly describe how a thermopile can be used to detect thermal radiation
CALCULATIONS ON CONDUCTION
1) A cylinder iron vessel with a base of diameter 15cm and thickness 0.30cm has its base
coated with a thin film of soot of thickness 0.10cm. it is then filled with water at 100 0C
and placed on a large block of ice at 0 0C. Calculate the initial rate at which the ice
(Thermal conductivity of soot = 0.12Wm -1k-1)
2) (i) A house has an iron roof of area 5.0cm 2. The temperature on top of the roof is 30 0C
while that inside the room is 200C. If the iron sheets are 1.0mm thick, find the rate of
heat flow into the room.
(ii) If a ceiling material of thermal conductivity 0.6Wm -1k-1 and thickness 1.0mm is
fixed on the outer side of the iron sheets of the house in (i) above. Find the percentage
201 | P a g e
decrease in the rate of heat flow into the room
3) Two perfectly lagged metal bars A and B, each of length 20cm are arranged in parallel,
with their hot ends maintained at 90 0C and their cold ends at 30 0C. If the cross sectional
area of each bar is 2.5cm2, find the net rate of heat flow through the parallel bars Take
thermal conductivity of A =400Wm-1k-1 and that of B =200Wm-1k-1.
4) A double grazed window has two glass sheets each of thickness 3.0cm. They are
separated by a layer of air of thickness 1.0cm if the two inner air glass surfaces have
steady temperatures of 200C and 40C respectively. find the
(i). temperature of the outer air- glass surface
(ii).amount of heat that flows a cross 92m2 cross sectional area in 2 hours
(Conductivity of glass = 0.72Wm-1 k-1and that of air = 0.025Wm-1k-1)

5) The diagram below shows a section of a cavity wall of a house made up of brick, air and
brick. The inside temperature is 200C while the outside temperature is 50C

200C Air 50C


Brick wall
Brick wall

10cm 10cm

The thermal conductivities brick material and air are 0.6Wm -1k-1 and 0.02Wm-1k-1
respectively.
(i). Assuming steady state conditions, sketch a graph to show how temperature
changes with distance between the brick surfaces at 200C and that at 50C
(ii).Calculate the thickness of brick equivalent to 10cm of air.
6) A solid metal sphere is found to cool at a rate of 1.2 0C per minute when it is
temperature is 1270C. At what rate will a sphere of three times the radius cool when at a
temperature of 3270C, if in each case, the temperature is 27 0C. State any assumption
made.
7) A copper rod 2m long and diameter 3cm is lagged .one end is maintained at 300C. The
other end is pressed against a 2cm thick card disc of the same diameter as the rod. The
free end of the disc is maintained at 40C0. If the thermal conduct ties of copper and card
board are 380 and 0.2Wm-1k-1 respectively .Calculate the steady state temperature at the
copper card board junction and the quantity of heat flowing across the junction in 10
minutes.
8) A well lagged composite metal bar of uniform cross section 2cm 2 is made by joining a
40cm rod of copper to a 25cm rod of aluminum. The extreme ends of the bar are
maintained respectively at 100C0 and 0oC
202 | P a g e
Calculate
(i). the temperature of the junction of the two rods
(ii).The heat flowing through each bar in 5 min (Thermal conductivity of copper and
aluminum is 386Wm-1k-1 and 210Wm-1k-1 respectively)
9) Water contained in an aluminum kettle on a stove steadily boiling a way at 100 0C at a
rate of 3.68x10-4 kgs-1. The base has an area of 6.0x10-3m2 and thickness 4mm.
(i). Calculate the rate of heat flow through the base
(ii).The temperature of the lower surface of base. (The thermal conductivity of
Aluminum =210Wm-1k-1, specific latent heat of water is 2.26x106 Jkg-1).
10) A cooking pot has a copper bottom layer 0.005m thick. The inside of the pot is at 1000C
(i). What is the temperature of the copper steel junction.
(ii).At what rate is heated conducted if the area of the bottom of the pot is 0.004m 2.
(Thermal conductivity of copper and steel are 380 and 40 Wm-1k-1 respectively).
11) A boiler with a steel bottom 1.5cm thick rests on a hot stove. The area of the bottom of
the boiler is 1500cm2. The water inside the boiler is at 1000C and 0.75kg of water are
evaporated every 5 minutes. find the temperature of the lower surface of the boiler ,
which is contact with the stove (Thermal conductivity 2.4x10 -2Wm-1k-1) and area 220m2
and un covered glass windows 0.80 cm thick (thermal conductivity 0.84Wm -1k-1) and
total area 32m2. Assuming that the heat loss is only by conduction .Calculate the rate
which heat must be supplied to the house to maintain its temperature at 20 0C if the
outside temperature is -50C.
12) A composite rod is made from uniform rods of copper, aluminum and brass. The rods
which have equal lengths and diameter and placed end to end and lagged as shown
below

Copper Aluminum Brass

1000C 00C

The ends of the composite rod are maintained at 100C 0 and 0 0C respectively. Given that
the coefficient of thermal conductivity of copper and brass and aluminum are in the ratio
18:5:10.

(i). Determine the equilibrium temperature at the copper aluminum and aluminum brass
junctions
(ii).What would these temperatures be if the brass and aluminum rods were exchanged.

203 | P a g e
13) A copper kettle has a circular base of radius 10cm and thickness 3.0mm. The upper
surface of the base is covered with a uniform layer of scale 1mm thick. The kettle
contains water which is brought to its boiling point over an electric heater. In steady
state, 5g of steam are produced each minute. What is the temperature of the lower
surface of the base assuming that the heat conduction from the sides of the kettle can be
ignored. Thermal conduct ties of copper and scale respectively are 390Wm -1k-1 and
13.0Wm-1k-1, specific latent heat of steam is 2.26x106 Jkg-1).
14) Rods of copper, brass, and steel are welded together to form a y shaped figure. The cross
sectional area of each rod is 2cm 2. The ends of the copper rod is maintained at 100
0
Cand the ends of the brass and the steel rod at 0 0C. Assuming that the lengths of the
rods are 46cm, 13cm, and 12cm, respectively
(i). calculate the temperature of the junction
(ii).Find the heat current in the copper rod. Thermal conductivities of copper, brass,
and steel are respectively 385Wm-1 k-1, 109Wm-1k-1 and 50.2Wm-1k-1).
15) An aluminum sauce pan of uniform thickness of 0.4cm and bottom area 0.25m 2 contains
pure water to a depth of 20cm. The water is boiling under standard pressure and losing
mass at a rate of 2gs-1 heat is supplied by a heater at the bottom of the pan. Calculate the
temperature at the lower surface of the pan. Neglect beat from the sides. (Conductivity
of aluminum is 2.3x102 Wm-1k-1).
16) A rectangular room 12m by 10m has vertical walls 4m high to support a flat roof. The
walls and the roof are 25cm thick and are made of a material of thermal conductivity
0.25Wm-1k-1. The doors and windows cover an area 16m 2 and are made of glass of
thickness 5mm and thermal conductivity 1.2Wm-1k-1 if the room is maintained at a
constant temperature above that of its surrounding .Calculate the percentage heat loss by
conduction through the doors and windows. Heat losses through the windows the
windows the floor may be neglected.
17) A concrete floor of a hall has dimensions of 10.0m by 8.0m it is covered with a carpet of
thickness 2.0cm. The temperature inside the hall is 220C while that of the surrounding
just below the concrete is 120C. Thermal conductivities of the concrete and the material
of the carpet are 1Wm-1k-1 and 0.05Wm-1k-1 respectively and the thickness of the
concrete is 10cm. calculate
(i). The temperature at the interface of the concrete and carpet
(ii).The rate at which heat flows through the floor.
18) Water in an aluminum sauce pan of diameter 16cm and thickness 4mm is kept boiling at
1000C on a hot stove. The water boils off at rate of 2.28x10 -4 kgs-1. calculate the
temperature of the underside of the sauce pan assuming it is uniformly heated and
neglecting heat losses from the sides
(Thermal conductivity of aluminum =2.06x102 Wm-1k-1, latent heat of vaporization of
water =2.26x106 J kg-1) (  =100.5C0)
19) 1200C 5mm

204 | P a g e
A

B 10mm

2000C

The metal conductors A and B, each of radius 20cm and thickness 5mm and 10mm
respectively are placed in contact as shown above. The upper surface of A and lower
surfaces of B are maintained at temperatures of 120C 0 and 200C0 respectively (Thermal
conductivities of A and B are 210 Wm-1k-1 and 130Wm-1k-1 respectively)
Calculate;
(i). the temperature of the interface
(ii). the rate of heat flow through A.
20) A window plane consists of a sheet of glass of area 2.0m 2 and thickness 5.0mm. If the
surface temperatures are maintained at 0C0 and 20C0, calculate the rate of flow of heat
through the pane assuming a steady state is maintained. The window is now double
glazed by adding a sheet of glass (similar) so that a layer of air 10mm thick is trapped
between the two planes. Calculate the new heat flow rate due to conduction for the same
temperature difference between the outer surfaces. (Thermal conductivities of glass =
0.80Wm-1k-1, air =0.05Wm-1k-1).
21) A metal boiler is 1.5cm thick. find the difference in temperature between its inner and
outer surface if 32kg of water are evaporated from the boiler per m2 per hour
(L. h. of evaporation of water =2268KJ kg-1, K boiler =63Wm-1k-1).
22) A composite metal bar of uniform cross section is made up of lengths of 25cm of
copper, 10cm nickel, and 15cm aluminum, each being in perfect thermal contact with
the adjoining part. The copper and of the composite bar is maintained at 1000C and the
aluminum end at 00C. The whole bar is lagged so that it may be assumed that no heat
losses occur at the sides. Find the steady temperature of the junction, assuming thermal
conductivity of copper = 385Wm-1k-1 nicked =59Wm-1k-1 aluminum = 209Wm-1k-1).
23) A window of height 1.0m and width 1.5m contains a double glazed unit of two single
glass panes each of thickness 4.0mm separated by an air gap of 2.00mm. Calculate the
rate at which heat is conducted through the window if the temperature of the external
surfaces of glass are 200C and 300C respectively (Thermal conductivities of glass and
air 0.72Wm-1k-1 and 0.025Wm-1k-1).
24) A well 6m x3m consists of two layers A and B of bricks of thermal conductivities
0.6Wm-1k-1 and 0.5Wm-1k-1 respectively. The thickness of each layer is 15.0cm. The
inner surface of the layer A is at a temperature of 20C0 while the outer layer of B is at
a temperature of 10C0 .Calculate the
(i). the temperature of the interface of A and B
(ii).rate of heat flow through the wall.

205 | P a g e
25) A wall of a building consists of two brick layers each of thickness 10.0cm and between
which there is a layer of air 2.0cm thick. find the rate of heat flow through one m 2 of the
wall if the inner and outer temperatures of the building are 25C0 and 15C0 respectively
(Thermal conductivities of brick and air 6.7Wm-1k-1 and 0.024Wm-1k-1 respectively).
26) A concrete floor of a hall has dimensions of 10.0m by 8.0m. It’s covered with a carpet of
thickness of 2.0cm. The temperature inside the hall is 22 0Cwhile that of the surroundings
just below the concrete is 120C. The thermal conductivities of the concrete and the
material of the carpet are 1.0Wm-1k-1 and 0.05Wm-1k-1 respectively and thickness of the
concrete is 10cm. calculate;
(i). the temperature of the interface of the concrete and carpet
(ii).the rate at which the heat flows through the floor.
27) An aluminum plate has a thickness of 5cm. The upper and lower surfaces of the plate
are maintained at a temperature of 1200C and 200C respectively. If the thermal
conductivity of aluminum is 2.37x102 Wm-1k-1. calculate;
(i). the rate of heat flow per cm2 a cross the two surfaces
(ii).the temperature mid- way between the surfaces.
28) A cooking sauce pan made of iron has a base area of 0.05m 2 and thickness of 2.5mm. it has
a thin layer of soot of average thickness 0.5mm on its bottom surface. Water in the
sauce pan is heated until it boils at 1000C .The water boils a way at a rate of 0.60kg per
minute and the side of the soot nearest to the heat source is at 150 0C .Find thermal
conductivity of the soot. (Thermal conductivity of iron =66Wm -1k-1 and its specific latent
heat of vaporization = 2200KJkg-1.
29) Metal rods of copper, brass, and steel are welded together to form a y shaped figure.
The cross sectional area of each rod is 2cm2. The free ends of copper rod is maintained
at 1000C while the free ends of brass and steel rods are maintained at 0 0C. if there is no
heat loss from the surfaces of the rods, and the lengths of the rods are 0.46m, 0.13m and
0.12m respectively
(i). Calculate the temperature at the junction
(ii).find the heat current in the copper rod.(Thermal conductivities of copper, brass
and steel are 385Wm-1k-1, 109Wm-1k-1 and 50.2Wm-1k-1 respectively).
30) Two bricks walls each of thickness 10cm are separated by an air gap of thickness 10cm.
The outer faces of the brick walls are maintained at 200C and 50C respectively
(i). Calculate the temperatures of the inner surfaces of the walls
(ii).Compare the rate of heat loss through the layer of air with that through a single
brick wall.(Thermal conductivities of air is 0.02Wm-1k-1 and that of brick is
0.6Wm-1k-1).
31) The wall of a furnace is constructed with two layers. The inner layer is made of bricks of
thickness 10.0cm and thermal conductivity 0.8Wm-1k-1 and the outer layer is made of
material of thickness 10.0cm and thermal conductivity 1.6Wm-1k-1
(i). Explain why is steady state, the rate of thermal energy transfer must be the
same in both layers
206 | P a g e
(ii).Calculate the rate of heat flow per square metre through the wall
(  =466.67C0, 1066.64Jm-2s-1)
32) (i) A house has an iron roof of area 5.0m2. The temperature on top of the roof is 30C0
while that inside the room is 20o C. If the iron sheets are 1.0mm thick , find the rate of
heat flow to the room
(ii) If a ceiling material of thermal conductivity 0.6Wm -1k-1 and thickness 1.0mm
is fixed on the underside of the iron sheets of the house in (i) above, find the
percentage decease in the rate of heat flow into the room.
33) Assuming the thermal conductivity of air and brick is 0.02Wm -1k-1 and 0.6Wm-1k-1
respectively. calculate the thickness of air equivalent to a thickness of 30cm of brick,
how much heat per minute would flow through them in the steady state when the outside
temperature of brick are 600C and 100C respectively and area of cross section of each is
2.0m2.
34) In a double glazing, two sheets of glass 2 mm thick are separated by 10mm of air. The
temperatures of outside glass surface s are 200C and 50C respectively. Calculate the heat
lost per second per unit are flowing by conduction and the temperature of the interior
glass surfaces assuming a steady state. Thermal conductivity of glass and air are
0.06Wm-1k-1 and 0.02Wm-1k-1.
35) A uniform composite slab is made of two types of materials A and B of thickness 6cm
and 8cm and of thermal conductivity 360 and 120 Wm -1k-1 respectively. if the ends A
and B are maintained at 800C and 200C respectively calculate
(i). the temperature of the junction of the two materials
(ii).rate of heat flow through units area of the slab.
36) The walls of a container used for keeping objects cool consists of two thickness of wool
0.5cm thick separated by a space 1cm wide parked with a poorly conducting material.
calculate rate of flow of heat per unit area into the container if the temperature
difference between the internal and external surface is 200C
(K w =2.4x10-3 wcm-1k-1, Kb =2.4x10-4Wcm-1K-1).
37) An ideally lagged compound bar 25cm long consists of a copper bar 15cm long joined to
an aluminum bar 10cm long and of equal cross section area. The free end of the copper
is maintained at 1000C and the free end of aluminum at 00C. Calculate the temperature
gradient in bar when steady state conditions have been reached.
(Kc= 390Wm-10 C-1, KA= 210Wm-1 0C-1).
38) If a copper kettle has a base of thickness 2.0mm and area 3.0x10 -2m2 estimate the
steady difference in temperature between inner and outer surfaces of the base which
must be maintained to enable enough heat to pass through so that the temperature of
1.0kg of water rises at the rate of 0.25Ks-1.(Kc= 3.8x102Wm-1k-1 s. h . c of water =
4.2x103 Jkg-1k-1 After reaching the temperature of 373k the water is allowed to boil
under the same conditions for 120s and the mass of water remaining in the kettle is
0.948Kg. Deduce a value for the specific latent heat of vaporization of water.

207 | P a g e
39) A cubical container full of hot water at a temperature of 90 0C is completely lagged with
an insulating material of thermal conductivity 6.4x10-2 Wm-1oC-1. The edges of the
container are 1.0m long and the thickness of the lagging is 1.0cm. Estimate the rat of
flow of heat through the lagging if the external temperature of the lagging is 40C 0 state
any assumptions taken.
CALCULATIONS ON CONVECTION
1) A body of surface area 1.8m2 loses heat at a rate of 7.0Wm-1k-1 by convection. The
temperature of the body is 31C0. if the power lost is 126W, what is the temperature of
the surroundings.
2) The temperature of a body falls from 300C top 200C in 5 minutes. The air temperature is
130C find the temperature after 10mm Assuming Newton’s law of cooling.
3) A blackened copper sphere of density 8.9x103 kgm-3, s. h. c. 390J kg-1k-1 and of diameter
10.0cm is cooled in an evacuated enclosure whose walls are kept at 0 0C. How long does
it take for the sphere to cool from 2280C to 2270C taking k =14Wm-1k-1.
CALCULATIONS ON THERMAL RADIATION
1) A blackened metal sphere of diameter 10mm is placed at the focus of a concave mirror
of diameter 0.5m placed on the earth’s surface directed towards the sun. calculate the
maximum temperature the sphere can attain if radius of the sun is 7x10 8m, surface
temperature of sun =6000k, distance from the earth to the sun is 1.5x10 11m 01 = 5.7x10-
8
Wm-2k-4
2) Consider the sun to be a sphere of radius 7.0x108m whose surface temperature is 5900k
calculate;
(i). the solar constant at the surface of the earth if the distance of the earth from the
sun is 1.5x1011m
(ii).The total power received by the earth if its radius is 6.4x106m
(iii). State any assumption made in your calculation.
3) The tungsten filament of a 40W lamp is at a temperature of 217 0C and has an effective
surface area of 0.64 cm2. Assuming that the energy radiated is 31% of that from a black
body in similar conditions and that any effect due to the radiation from the glass
envelope is negligible
(i). Estimate the value of Stefan’s constant
(ii).Calculate the frequency emitted with maximum intensity.
4) The figure below the energy radiated per unit area per second per unit wave length
interval E2 varies with the wave length 2 for radiation from the sun’s surface

Relative
Intensity T = 5800K
E2

208 | P a g e
500nm Wave length
Calculate the wave length zmax at which the corresponding have peaks for
(i). Radiation in the sun’s core where the temperature is approximately 15x106k
(ii).Radiation in interstellar space which corresponds to a temperature of
approximately 2.7k
(iii). Name the part of the electromagnetic spectrum to which the calculated
wave length belongs in each case.
5) A solid copper sphere of diameter 20mm is cooled to a temperature of 500K and is then
placed in an enclosure maintained at 300K.Assuming that all the exchange of heat is by
radiation, calculate the initial rate of loss of temperature of the sphere assumed as a
black body.
6) The element of an electric fire with an output of 1000W is a cylinder of 25 cm long and
1.5cm in diameter. Calculate its temperature when in use if it behaves as a black body
(Wien’s displacement constant =2.9x10-3mK, Stefan’s constant = 5.7x10-8Wm-2k-4,
density of copper =370Jkg-1k-1).
7) A metal sphere of diameter 1x10-2m is cooled to a temperature of 250K and is then
placed in an enclosure maintained at 400K assuming that all interchange of heat is by
radiation; calculate the initial rate of rise of temperature of the sphere. The sphere may
be treated as a black body
(Density of metal = 7.2x103kgm-3, s. h. c. of metal = 3.50x102Jkg-1k-1, Stefan’s
constant= 5.70x10-8Wm-2k-4)
8) The tungsten filament of an electric lamp has a length 0.5m and a diameter of 6.0x10 -5m.
The power rating of the lamp is 60W. Assuming the radiation from the lamp is
equivalent to 80% that of a perfect black body radiator at the same temperature, estimate
the steady temperature of the filament.
9) The intensity of radiant energy from a black body is maximum at the wave length of
1.5x10-6m .calculate the corresponding temperature of the black body ( Wien’s
displacement constant = 2.9x10-3mK)
10) The resistance of a tungsten wire of an electric lamp at 20 0C is 50.At an opening
voltage of 240V the current through the filament is 0.5A .Given that the temperature
coefficient of resistance of tungsten is constant and is equal to 5.0x10-3K-1.
(i). find the temperature of the filament at the above operating voltage
(ii).if the effective surface area of the filament is 0.93cm 3, assuming the filament
radiates as a black body and that any effect due to the radiation from the glass
envelope is negligible calculate the value of the Stefan’s constant.
11) The filament of an electric bulb attains a temperature of 1600K when the power
supplied to it is 25W
(i). find the temperature of the filament if the power supply is increased to 60W
(ii). Find the length of the filament at 1600k if the diameter is 5.0x10-5m
(iii). Calculate the difference in the wave length of the radiation emitted with
maximum intensity at the two temperatures assuming the filament radiates as a
black body.
12) A blackened metal sphere of diameter 10m is placed at the focus of a concave mirror of
diameter 0.5m directed towards the sun if the solar power incident on the mirror is
1400Wm-2, calculate the maximum temperature the sphere can attain.

209 | P a g e
13) Estimate the temperature TE of the earth assuming it is in radioactive equilibrium with
the sun. (Radius of the sun =7x108m, surface temperature of the sun = 6000k, distance
from the earth to the sun = 1.5x1011m, s= 5.7x10-8 Wm-2k-4).
14) A solid copper sphere of diameter 10mm is cooled to a temperature of 150k and then
placed in an enclosure maintained at 290k assuming that all the interchange of heat is by
radiation, calculate the initial rate of rises of temperature of the sphere. The sphere may
be assumed to behave as a black body. (Density of copper = 8.93x10 3kgm-3, s. h. c. of
copper = 370Jkg-1k-1).
15) Radiation from the sun falls normally on a metal foil of area 100cm2 placed on the
ground to give a certain temperature rise. When the radiation is shielded off a current of
0.7A maintained through the foil under a p. d of 8v gives the same temperature rise in
the same time. Assuming that the earth’s atmosphere absorbs 60% of the solar intensity
incident on it estimate the surface temperature of the sun (Radius of sun = 7.0x10 8m
Stefan’s constant = 5.7x10-8Wm-2k-4, distance from the sun to the earth = 1.5x1011m)
16) A blackened platinum strip of area 2x10-5m2 is placed at a distance of 2m from a white
hot iron sphere of radius 5.0x10-3m so that the radiation falls normally on the strip. The
radiation causes the temperature and hence the resistance of the platinum to increase it is
found the same increase in resistance can be produced under similar conditions but in
the absence of radiation when a current of 3.0 mA is passed through the strip, the p. d
between its ends being 24mv. Estimate the temperature of the iron sphere
(Stefan’s constant 01= 5.7x10-8Wm-2k-4)
17) A light bulb radiates as a black body
(i). what quantities would have to be measured to test the validity of this statement
(ii).The filament of the bulb is 0.5m long and is of radius 1.0x10 -4 m .The filament melts
when it is connected a cross 240v and the current through it is 0.40Acalculate the
temperature at which the filament melts
(iii). find the wave length for which the energy radiated is a maximum.
18) The mean distance from the sun to the earth is 1.49x10 11m and that from the sun to
Venus is 1.08x1011m. Given that the solar constant on the earth, s = 1.4KWm-2. find;
(i). the solar constant on Venus
(ii).the equilibrium temperature of Venus assuming Venus absorbs and radiates as a
black body.
19) A blackened platinum strip of area 0.20cm 2 at a distance of 200cm from a white hot iron
sphere of diameter 1.0cm so that the radiation falls normally on the strip. The radiation
causes the temperature and hence the resistance of the platinum to increase. it is found
that the same increase in resistance can be produced under similar conditions but in the
absence of radiation when a current of 3.0mA is passed through the platinum strip the
p.d between its ends being 24mv estimate the temperature of the iron sphere.
20) A 100W electric light bulb has a filament which is 0.69m long and has a diameter of
8.0x10-5m. Estimate the working temperature of the filament if its total emissivity is
0.70, Stefan’s constant = 5.7x10-8Wm-2k-4.
21) The tungsten filament of a 40W lamp is at a temperature of 217 0C and has an effective
surface area of 0.64cm2. Assuming that the energy radiated is 31% of that from a black
body in similar conditions and that any effect due to radiation from the glass envelope is
negligible,
(i). estimate the value of Stefan’s constant

210 | P a g e
(ii).Calculate the frequency emitted with maximum intensity (Wien’s displacement
constant =2.9x10-3mk).
22) Water in aluminum sauce pan of diameter 16cm and thickness 4mm is kept boiling at
1000C on a hot stove .The water boils off at a rate of 2.28x10 -4kgs-1. calculate the
temperature of the underside of the sauce pan assuming it is uniformly heated and
neglecting heat losses from the sides.(Thermal conductivity of aluminum =
2.06x102Wm-1k-1 specific latent heat of vaporization of water =2.26x106 Jkg-1).
23) A metal sphere of radius 1cm is suspended with in an evacuated enclosure whose walls
are at 300K. The emissivity of the metal is 0.40. Find the power in put required to
maintain the sphere at a temperature of 300K if heat conduction along the supports is
neglected.
24) A uniform composite slab is made of two types of material A and B of thickness 6cm
and 3cm and of thermal conductivities 360Wm-1k-1 and 120Wm-1k-1 respectively. if the
ends of A and B are maintained at 800C and 200C respectively calculate;
(i). the temperature of the junction of the two materials
(ii).the rate of flow of heat through an area of 1m2 of the slab.
25) An electric bulb has a filament at a temperature of 2.0x10 3K. if the filament can be
regarded as a black body .calculate;
(i). the frequency of the radiation emitted with the greatest intensity and the total rate of
emission of energy
(ii).the power of the lamp if the filament has a diameter of 0.1mm and length 10cm (Wien’s
displacement law constant = 2.9x10-3mk Stefan’s constant =5.6x10-8Wm-2k-4)
26) A solid copper sphere of diameter 10mm and temperature of 150k is placed in an
enclosure maintained at temperature of 290k. calculate stating any assumptions made the
initial rate of rise of temperature of the sphere.(Density of copper = 8.93x10 3kgm-3, s. h.
c. of copper = 3.7x102Jkg-1k-1)
27) The element of a 1.0W electric fire is 30.0cm long and 1.0cm in diameter. if the
temperature of the surrounding is 20C0, estimate the working temperature of the element
(Stefan’s constant 0 = 5.7x10-8Wm-1k-1)
28) The flux of solar energy on the earth’s surface is 1.36x103 wm-2. calculate
(i). the temperature of the surface of the sun
(ii).the total power emitted by the sun
(iii). the rate of loss of mass by the sun.
29) Assuming that the sun is a sphere of radius 7.0x108m at a temperature of 6000K.
Estimate the temperature of the surface of mars if it’s a distance of 2.28x10 11m from the
sun.
30) Calculate the rate of loss of heat energy of a black body of area 40m 2 at a temperature of
500C. if the radiation it receives from the sun is equivalent to a temperature in space of -
2200C.
31) Consider the sun to be a sphere of radius 7.0x108m whose surface temperature is 5900k
(i). find the solar power incident on an area of 1m 2 at the top of the earth’s atmosphere if this
is a distance of 1.5x1011m from the sun
(ii).Explain why the solar power incident 1m2 of the earth’s surface is less than the value
calculated in (i) above.
32) The total power out of the sun is 4.0x1026W. Given that the mass of the sun is
1.97x1030kg and its density is 1.4x103kgm-3 estimate the temperature of the sun. state any

211 | P a g e
assumptions made.
33) The average distance of Pluto from the sun is about 40 times that of the earth from the
sun. if the sun radiates as a black body at 6000k and is 1.5x10 11m from the earth
calculate the surface temperature of Pluto.
34) The energy intensity received a spherical planet from a star is 1.4x10 3wm-2. The star is
of radius 7.0x105km and is 14.0x107km from the planet
(i). calculate the surface temperature of the star
(ii).state any assumptions you have made above
35) A small blackened solid copper sphere of radius 2cm is placed in an evacuated enclosure
whose walls are kept at 1000C. Find the rate at which energy must be supplied to the
sphere to keep its temperature constant at 1270C.
36) A cube of side 1cm has a grey surface that emits 50% of radiation emitted by a black
body at the same temperature. if the cube’s temperature is 7000C, calculate the power
radiated by the cube.
37) The element of 1kw heater has a surface of 0.006m2. Estimate the working temperature
and state the assumption used in your calculation.
38) Estimate the temperature of the earth assuming it is hemodynamic equilibrium with the
sun.
39) A radiant wall heater is a silica cylinder 1.0m long and radius 0.5mm; it is rated at 1KW,
if the room temperature is 300K. Estimate the working temperature of heater, stating any
assumption.
40) A tungsten filament of an electric lamp has length of 0.5m and a diameter 6.0x10 -5m.
The power rating of the lamp is 60W. Assuming the radiation from the filament is
equivalent to 80% that of the perfect black body radiator at the same temperature.
Estimate the steady temperature of the filament.(Stefan’s constant = 5.7x10-8wm-2k-4)
41) The resistance of tungsten wire of an electric lamp at 20 0C is 50.A at an operating
voltage of 240v, the current through the filament is 0.5A. Given that the temperature
coefficient of the resistance of tungsten is constant and is equal to 5.0x10-3K-1
(i). find the temperature of the filament at above operating voltage if the effective surface
area of the filament is 0.93cm2 assuming that the filament radiated as a black body and
that any effect due radiation from the glass envelope is negligible
(ii).Calculate the value of Stefan’s constant.

TOPIC21: ELECTRON PHYSICS


DEFINITIONS
1. Define the following terms; cathode rays, specific charge of an electron

FACTORS, MERITS DEMERITS, FEATURES AND EXAMPLES


1. State two disadvantages of discharge tubes when used to study cathode rays
2. Give two applications of discharge tubes
3. List four main properties of cathode rays
4. State and justify two properties of cathode rays
5. State at least two ways through which a parallel of air can be ionized.
6. Outline the steps involved in the determination of the charge of the electron by
Millikan’s’ oil drop method
212 | P a g e
7. State three differences between cathode rays and positive rays.

EXPERIMENTS
1. With the aid of a labeled diagram, describe what is observed when a high tension voltage
is applied across a gas tube in which pressure is gradually reduced to very low values
2. Give an account of the stages observed when an electric discharge passes through a gas at
pressure varying from atmospheric to about 0.01mmHg as air is pumped out when the p.d
across the tube is maintained at extra high tension.
3. Describe a simple experiment to show that cathode rays are negatively charged
4. With aid of a diagram describe how cathode rays are produced.
5. A high p.d is applied across two electrodes in air contained in a closed glass tube.
Describe with aid of labeled diagrams what will be observed when the pressure in the
tube is progressively reduced down to very low pressures.
6. Describe an experiment to show that cathode rays travels in straight lines.
7. Describe with the aid of a diagram, the production of cathode rays
8. Describe Thompson’s experiment to determine the specific charge of an electron
9. Describe how to specific charge of an electron can be determined using the fine beam
tube method. Explain the source of error involved during the above experiment
10. Describe briefly the steps involved in the determination of the charge of an electron by
Millikan’s oil drop experiment
11. Describe an experiment to measure the ration of the charge to mass of an electron.

EXPLANATIONS
1. Explain how the sign of the charge of cathode rays may be determined.
2. Explain how Millikan’s experiment for measuring the charge of an electron proves that
charge is quantized.
3. A beam of electrons having a common velocity enters a uniform magnetic field in a
direction normal to the field. Describe and explain the subsequent path of the electrons.
4. Explain whether a similar path would be followed if a uniform electric were substituted
for the magnetic field for a beam of electrons having a common velocity and enters it
5. Explain why; the apparatus in Millikan’s experiment is surrounded with a constant
temperature enclosure and low vapour pressure oil is used
6. Explain what is meant by quantization of charge
7. A cloud of oil droplets is formed between two horizontal parallel metal plates; explain the
following observation.
8. In the absence of an electric field between the plates all the oil droplets fall slowly at a
uniform speeds
9. Explain why a charged oil drop falls at constant speed in the Millikan’s oil drop
experiment when there is no p.d between the plates

213 | P a g e
10. A charged drop of oil is held stationery in the space between two metal plates a cross
which an electric field is applied.
14. Explain with the aid of a diagram how the forces acting enable the drop to remain
stationary.
DERIVATIONS AND RELATIONS
1) (i) A beam of electrons having a common velocity, enters a uniform magnetic field in a
direction normal to the field. Describe and explain the subsequent path of the electrons
(ii) Explain whether a similar path would be followed if a uniform electric field were
substituted for the magnetic field.
2) (i) Write an expression for the force on a particle of charge q moving with a velocity V-in
the uniform magnetic field of flux density.
(ii).State the direction of force above.

3) In Millikan’s experiment, the radius r, of the drop is calculated from r =


√ 9 sv
2 ρg
where s is

the coefficient of air and ρ is density of oil. Identify the symbol V and describe briefly
how it is measured.
4) A charged particle is moving in a circle of radius r in a plane perpendicular to the
direction of a magnetic flux density B show that;
(i). The linear momentum of particle is proportional to r
(ii).Frequency of the particle depends only on the particles specific charge and
magnetic flux density B.
(iii). The kinetic energy of the particle is directly proportional to r2 B2
5) Give the equation for the force on a particular carrying a charge,
(i). In an electric field of intensity E
(ii).While moving with velocity, V at right angles to a magnetic field of flux density B.
(iii). Draw diagrams to indicate the directions of the forces.
6) A beam of electrons is directed into a region of uniform magnetic field of flux density B
along a path at right angles to the direction of B.
(i). What is the path followed by the electrons when they enter the fields.
(ii).Explain why this path is followed.
7) An oil drop of mass 4.0 x 10-15kg. Is held stationary when an elastic field is applied
between the two horizontal plates. If the drop carries 6 electronic charges each of values
1.6 x 19C. Calculate the value of the electric field strength.
8) State what is meant by specific charge and state the unit in which it is measured.

CALCULATIONS ON ELECTION DYNAMICS


1) Electrons are being accelerated from rest by a P.d of 100V Given that the specific change
e
of an electron = 1.76 x 1011ckg-1. Determine the final velocity of the electron.
m
2) A beam of electrons is accelerated through a p.d of 200V and is directed mid -way
between two horizontal plates of length 5.0cm and a separation of 2.0cm and the p.d
214 | P a g e
across the plate is 80V. Calculate the speed of the electrons as they enter the region
between the two plates.
3) Electrodes are mounted at opposite ends of a law pressure discharge tube and a p.d of
1.20KV applied between them assuming that the electrons are accelerated from rest,
calculate the maximum velocity which they could acquire.
(Specific electron charge = 1.76 x 1011ckg-1).
4) An electron starts from rest and it’s accelerated through a p.d of 200V. what is;
(i). The kinetic energy of the electron at the end of the acceleration
(ii).The speed when the electron is at the end of the acceleration.
5) Assuming the electron above now enters a region of uniform magnetic field of flux
density 0.2T and at right angles to the electron path, determine;
(i). Force experienced by the electrons
(ii).The radius of the path followed by the electron
(iii). How long would the electron have to remain in the magnetic field for is to
end up travelling at 900 to its original direction.
6)

5.0cm 2.1cm

2.0cm 20cm
The figure above shows two metal plates 2cm long and plated 5mm apart. A fluorescent
screen is place 20cm from one end of the plate. Calculate the voltage which must be
applied across the plated so as to deflect the electrons through 2.1cm on the screen if an
electron of kinetic energy 3.2 x 10-16J is incident mid-way between the plates.
7) In a cathode ray tube, electrons are accelerated through a voltage of 2600V and focused
into a narrow tube.
(i). Calculate the speed of the electrons in the beam
(ii).The same beam passes along line mid-way between two electron static deflecting
plates 6cm long and 2cm through the largest angle possible for it to be observed
on the screen.
8) A beam of electrons is accelerated through a p.d of 2000V and is directed mid between
two horizontal plates of length 5.0cm and a separation 2.0cm. the p.d across the plates is
80V.
(i). Calculate the speed of the electrons as they enter between the plate
(ii).Explain the motion of the electrons between the plates

215 | P a g e
(iii). Find the speed of the electrons as they emerge from the region between
the plates.
9) An electron having energy of 4.5 x 102ev moves at right angles to a uniform magnetic
field of flux density 1.5 x 10-3T find the;
(i). Radius of the path followed by the electrons
(ii).Period of motion.
10) A beam of cathode rays is directed mid-way between two parallel metal plates of length
40cm and separation 1.0cm. The beam is deflected through 10.0cm on a fluorescent
screen placed 20.0cm beyond the nearest edge of the plates when ap.d of 200v is applied
a cross the plates. If this deflection is annulled by a magnetic field of flux density 1.14 x
10-3T applied normal to the electric field between the plates. Find the change to mass
ratio of the cathode rays.
11) Two large oppositely charged plates are fixed 1.0cm a part as shown in the figure. The
p.d between the plates is 50V.

1.0cm

450
Electron beam
+ + + + + +++++ +

An electron beam enters the region between the plates at an angle of 45 as shown. Find the
maximum speed the electrons must have in order for them not to strike the upper plate.
12) An electron accelerated by a p.d of 1000V passes through uniform electronic field of
intensity E crossed with a uniform magnetic field of flux density 0.3T. If the electron
emerges un deflected. Calculate the electric field intensity E.
13) A stream of singly ionized magnesium atoms is accelerated through a p.d of 50V and
then enters a region of uniform magnetic field of flux density 2.0 x 10-2T. The ions
describe a circular path of radius 0.24m. Calculate the atomic mass of the ions.
14) An electron of energy of 10keV enters mid- way between two horizontal metal plates
each of length 5.0cm and separated by a distance of 2cm. a p.d of 20V is applied a cross
the plates. A fluorescent screen is placed 20cm beyond the plates. Calculate the vertical
deflection of the electron on the screen.
15) An electron is projected with a speed of 3.0x 107 ms-1 in the direction of a uniform
electric field. After travelling a distance of 400m, the electron reverses its direction.
(i). Why does the electron reverses its direction
(ii).Calculate the magnitude of the electric field.
16) A beam of singly ionized carbon atoms is directed into the region where the magnetic and
electric fields are acting perpendicular to each other and to the direction of the beam. The
fields have intensities 0.10T and 1.0 x 104Nc-1 resp. if the beam is able to pass un
deflected determine the velocity of the carbon atoms.

216 | P a g e
17) If the beam in (16) above then enters a region where the magnetic fields alone is acting.
As a result the beam describes an area of radius 0.75m. Calculate the magnetic flux
density in this region and period of the ions. (mass of carbon atom = 2.0 x10-26kg charges
of an electron = 1.6 x 10-19C)
18) An electron beam is accelerated through a p.d of 250V. the electron then enters a region
of uniform magnetic field perpendicular to a beam of an intensity 0.02T
(i). Calculate the speed and kinetic energy of the electrons when they leave the
electron gun
(ii). If the anode and cathode are separated by a distance of 1.6m calculate the
acceleration of the electrons between these electrodes
(iii). An electric field is now applied so that the electro static forces oppose the
magnetic force of the electrons and the beam goes un deflected. Determine the
size of the electric field involved
19) A beam of positive ions is accelerated through a p.d of 1 x 103V into a region of uniform
magnetic field of flux density 0.2T. While in the magnetic field, it moves in a circle of
radius 2.3cm. Derive an expression for the charge to mass ratio of the ions and calculate
its value.
20) The figure below was an experiment to determine the specific charge of an ion. The ion is
projected horizontally along the direction XO and enters a region uniform magnetic field
of flux density 0.4T at X as shown

X
O

G
On leaving the magnetic field, the ions strikes the screen at point G. when an electric
field is an arc of a circle of radius 2 x 10-2m and electric field intensity is 1.408 x 105Vm-
1
, calculate the specific charge of the ions.
21) A particle of charge 3.2 x 1019C is accelerated from rest through a p.d of 104V, it enters
into a region of uniform magnetic field of flux density 0.5T, the particle describes a circle
path of radius 8.94cm. find;
(i). The kinetic energy in entering the magnetic field
(ii).The mass of the particle
22) A stream of singly ionized magnesium atoms is accelerated through a p.d of 50V and
enters a region of uniform magnetic field of flux density 2.0x10-2T. The ions describe a
217 | P a g e
circular path of radius 0.24m. Calculate the atomic mass of the ion.
23) An electron starts from rest and moves in an electric field of intensity 2400Vm-1.
Determine (i) the force on the electron.
(ii) The k.e acquired and the velocity attained if the electron moves through a p.d of 90V.
24) Electrons are accelerated through a p.d of 2.0 x 103V and pass at right angles into a
uniform magnetic field of flux density 1.0 x 102T. Find the radius of their path.
25) An identical beam of electrons accelerated through a p.d of 2.0x103V is projected
perpendicularly into an electric field of 1.0 x 105Vm-1. Calculate deviation D, of the
electron path at a point 5.0 x10-2m perpendicularly into the field as measured from the
point of entry of the beam as shown.
Y

X
Electron beam
5.0 x 10-2m

26) An ion of mass, m kg carries charge Q coulombs. It’s accelerated from rest through a
distance of 3.6 x 10-2m by a uniform electric field of intensity 6.0 x 103Vm-1. It then
enters a uniform magnetic field of flux density 4.8 x 10-4T, whose direction if 600 to the
initial direction of motion. The ions describe a circular orbit or radius 7.2 x 10-1m in a
Q
plane perpendicular to the magnetic field. Calculate the value of for the ion.
m
27) An electron which has been accelerated through a p.d of 103V passes un deflected
through a region of uniform electric and magnetic fields. If the electric field intensity is
e
1.88 x 104Vm-1 and the magnetic flux density is 1.00 x 10-3T, find the ratio of for the
m
electron.
28) Electrons accelerated from rest through a p.d of 3000V, enter a region of uniform
magnetic field, the direction of the field being at right angles to the motion of the
electrons. If the flux density is 0.010T, calculate the radius of the electrons orbit.
29) A magnetron has a potential of 200V between the anode and cathode and anode diameter,
e
of 24mm if 1.76 x 1011Ckg-1 = . Calculate the flux density of the field for which the
m
value just ceases to conduct.
30) An electron moving at 5.0x10-6ms-1 is shot parallel to an electric field of strength 1 x
103Nc-1 arranged so as to retard its motion.
(i). How far will the electron travel in the field before coming momentarily to rest

218 | P a g e
(ii).If the field ends suddenly after 0.75cm, what fraction of its initial energy will the
electron lose in passing through it.
31) If the specific charge of an electron is 1.76 x 1011Ckg-1, calculate orbit radii for electrons
accelerated through a p.d of 2KV in fields of flux density is 1T and 1.8 x 10-5T.
32) An electron moves with a velocity of 2.0 x 107ms-1 along the axis of tube between two
horizontal plates. If the electric field between the plates is 2.0 x 104ms-1, the plate length
is 40mm and the separation of the plates is 20mm, at what angel to the axis will the
electron leave the field.
33) Estimate the velocity of electrons in a copper wire of cross sectional are 2.0 x 10-3mm
when carrying a current is 150mA.
34) Electrodes are mounted at opposite ends of a low pressure discharge tube and a p.d of
1.20KV applied between them assuming that the electrons are accelerated from rest,
calculate the maximum velocity which they could acquire. (Specific electron charge =
1.76 x 1011Ckg-1).
35) In a cathode ray oscilloscope an, electron beam between the Y – deflector plates each of
5cm long and 0.5cm apart. The distance between the Centre of the Y – plates and the
screen is 20cm and the p.d between the anode and the electron gun is 250V. Determine
the deflection in Vm-1 of the electron beam on the screen of the CRO.
36) (a) An electrons starts from rest and its accelerated through a p.d of 2 x 102V. find the;
(i). Kinetic energy of the electron at the end of acceleration
(ii).Speed at which the electron is moving at the end of the acceleration.
(b) Assuming the electron above in (a) above now enters a region of uniform magnetic
field of fux density 0.2T and at right angles to the electron path, determine;
(i). The force experience by the electron
(ii).The radius of curvature of the path produced by the electron
(iii). How long would the electron have to remain in the magnetic field for it to
end up travelling at 900to its original direction.
37) A beam of electrons is accelerated through a p.d 500V and then enters a uniform electric
field of strength 3.0 x 103Vm-1 created by two parallel plates each of length 2.00 x10-2m.
find;
(i). The speed V of the electrons of the length 2.00 x 10-2m. find;
(ii).The time t that each electron spends in the field.
(iii). The angle θ, through which the electrons have deflected by the time they
emerge from the field.
38) Calculate the speed of a proton which has been accelerated through a p.d of 400V. (Mass
of proton = 1.67 x 10-27kg, charge on proton = 1.60 x 10-19C).
39) An electron is moving in a circular path at 3.0x 106ms-1 in a uniform magnetic field of
flux density 2.0 x 10-4T. Find the radius of the path. (mass of electron = 9.1 x 10-31kg,
charge un electron = 1.6 x 10-19C).
40) A beam of electrons travelling with speed 1.2 x 107ms-1 in an evacuated tube is made to
move in a circular path of radius 0.048m by a uniform magnetic field of flux density
219 | P a g e
1.4mT.Calculate the kinetic energy of an electron in the beam.
41) A beam of protons is accelerated from rest through a p.d of 2000V and then enters a
uniform magnetic field which is perpendicular to the direction of the proton beam. If the
flux density is 0.2T. Calculate the radius of the path which the beam describes.
(Proton mass = 1.7 x 10-27kg, e = 1.6 x 10-19C).
42) Electrons accelerated from rest through a p.d of 3.0 x 103V enter a region of uniform
magnetic field, the direction of the field being at right angles to the motion of the
electrons. If the flux density is 0.010T, calculate the radius of the electrons orbit.
43) Two parallel metal sheets of length 10cm are separated by 20mm in a vacuum. A narrow
beam of electrons enters symmetrically between them. When a p.d of 1000V is applied
between the plates the electrons beam just misses one of the plates as it emerges.
Calculate the speed of the electrons as they enter the gap.

MILIKAN’S OIL DROP EXPERIMENT.


1) A spherical oil drop of radius 2.0x 10-4m is held stationery between two parallel metal
plates across which a p.d of 4500V is applied. The separation of the plates is 1.5cm,
calculate the charge on the drop if the density of the oil is 880kgm-3.
2) A charged oil drop of mass 3.27 x 10-15kg is held stationary between two horizontal metal
plates across which a p.d of 1.0 x 103V is applied. If the separation of the plates is 1.5cm.
Find the number of electrons on the drop.
3) An oil drop of radius 7.26 x 10-7m and density 880kgm-3 is held stationary in an electric
field of intensity 1.72 x 104Vm-1. How many electronic charges are on the drop. (Density
of air = 1.29kgm-3).
4) In Millikan’s oil drop experiment, a charged oil drop of radius 9.2 x 10-7m and density
800kgm-3 is held stationary in an electric field of intensity 4.0 x 104Vm-1.
(i). How many electrons charges are on the drop
(ii).Find the electric field intensity that can be applied vertically to move the drop
with velocity 0.005ms-1 upwards. (Density of air = 1.29km-3; coefficient of
viscosity of air = 1.8 x 10-5Nsm-1).
5) An oil drop of mass 7.5 x 10-16kg is held stationary between two horizontal metal plates
4.0cm a part with the upper plate being at a potential of 2000V while the lower one is
earthed.
(i). Calculate the charge of the drop
(ii).Determine the path of the drop if it fall from rest and the plates are held vertically.
6) In a Millikan oil drop experiment, a charged oil drop of density 880kgm-3 is held
stationary between two parallel metal plates 6.0mm apart and maintained at a p.d of
102V. when the electric field is switched off the drop is observed to fall through a
distance of 2.0mm in 35.7 seconds
(i). Find the radius of the drop
(ii).Determine the number of excess electrons caused by the drop if an electric field is
applied so as to keep the drop stationary.

220 | P a g e
(Density of air = 1.2kgm-3, Velocity of air = 1.8 x 105Nsm-3).
7) In an experiment to measure the electron charge by Millikan’s method p.d from 1.5KV
can be applied between two horizontal metal plates 12mm apart. With the electric field
switched off, a drop of oil of mass 10-4kg is observed to fall with a constant velocity of
400Mms-1. When the field is switched on a drop rises with constant velocity of 80Nms-1.
How many electron changes are there on the drop (Assume air resistance is proportional
to the velocity of the drop and air buoyancy may be neglected take g = 10ms-2).
8) In Millikan’s experiment an oil drop of mass 1.92 x 10-14kg is stationary in the space
between two horizontal metal plates which are 2.00 x 10-2m apart. The upper plate is
earthed and lower plate is at a potential of -6000V.
(i). State with reason the sing of the electric charge on the drop
(ii).Neglecting the Buoyancy of air, calculate the magnitude of the change. How
many electron charges are on the drop.
9) Oil droplets are introduced into the space between two flat horizontal plates set 5.00mm
apart. The plate voltage is then adjusted to exactly 780V so that one of the droplets is
stationary. Then the plate voltage is switched off and selected droplets are observed to
fall a measured distance of 1.5mm in 11.25. Given the density of oil used as 900kgm-3
and the viscosity of air 11.25. Given the density of oil used as 900kgm-3 and the viscosity
of air 1.8 x 10-5Nms-1. Calculate the charge of the droplet.
10) In Millikan’s oil drop experiment a charged oil drop of radius 9.2 x10-7m and density
800kgm-3 is held stationary in an electric field of intensity 4.0 x 104Vm-1.
(i). How many electron charges are on the drop
(ii).Find the electric field intensity that can be applied to move the drop with a
velocity of 0.005ms-1 upwards. (Density of air = 1.29kgm-3, coefficient of
viscosity of air = 1.8 x 10-5Nsm-1).
11) In the determination of electron charge by Millikan’s method a potential difference of
1.5kv is applied between horizontal metal plates 12mm apart. With the field switched off,
a drop of oil of mass 1.0 x 10-14kg is observed to fall with a constant velocity 4.0x10-
14ms-1 between the two metal plates 12mm. when a p.d of 1.5kv is applied across the
plates, the drop rises with on the drop. (Assume air resistance is proportional to the
velocity of the drop and neglected air buoyancy).
12) In the measurement of electron change by Millikan’s apparatus a potential difference of
1.6Kv is applied between two horizontal plates 14mm apart. With the p.d switched off, an
oil drop is observed to fall with velocity of 4.0 x 10-4ms-1, when the p.d is switched on the
drop rises with a constant velocity of 8.0x10-5ms-1. If the mass of the oil drop is 1.0 x 10-
14
kg, find the number of electro charges on the drop. (Assume air resistance is
proportional to the velocity of the drop and neglect the up thrust due to the air).
13) Oil droplets are introduced into the space between two flat horizontal plates set 5.0mm a
part. The plate voltage is then adjusted to exactly 780N so that one of the droplets is
stationary. Then the plate voltage is switched off and the selected droplet is observed to

221 | P a g e
fall a measured distance of 1.5mm in 11.2s. Given the density of oil used as 900kgm-3
and the viscosity of air as 1.6 x 10-5Nms-1. Calculate the charge of the droplet.
14) An oil drop of mass 3.25x 10-15kg falls vertically with uniform velocity between vertical
plates which are 2.0cm a part. When a p.d of 1000V is applied between the plates the
drop moves towards the negatively charged plate. Its path being inclined at 450 to the
vertical
(i). Explain why the vertical component of the velocity remains un changed
(ii).Calculate the charge on the drop. If the path of the drop suddenly charges to one
at 260301 to the vertical and subsequently to one at 370 to the vertical what
conclusion can you draw.
15) A charged oil drop falls under gravity with velocity 1.44 x 10-4ms-1 between two
horizontal metal plates, 1.50cm apart when a p.d of 5600V is applied between the plates,
the drop remains stationary. Calculate;
(i). The radius of the drop
(ii).The charge on the drop.(Density of oil = 800kgm-3, Viscosity of air = 1.80 x 10-
5
kgm-1s-1 Neglect the density of air in comparison with that of the oil)
16) An oil drop carrying a charge of 3e falls under gravity in air with a velocity of 4.6 x 10-
4
ms-1 between two parallel plates 5mm apart. When a p.d of 4.6 x 103V is applied between
the plates, the drop rises steadily.
Assuming that the effect of buoyancy on the drop is negligible calculate;
(i). The radius of the oil drop
(ii).The velocity with which the oil drop rises. (Density of oil = 900kgm-3, Viscosity o
air = 1.8 10-5Nsm-2).
17) In Millikan’s oil drop experiment, an oil drop of density 890kgm-3 and radius 2.35 x 10-
4
cm has an excess charge of 10electrons
(i). Find the free fall terminal velocity with the electric field turned off
(ii).What value of the electric field intensity is required to produce zero net force on
the droplet. (Density of air = 1kgm-3, Viscosity of oil = 1.83 x 10-5Nsm-2).
18) In a version of Millikan’s experiment it is found that a charged droplet of mass 1.8 x 10-
5
kg just remains stationery when the p.d between the plates which are 12mm apart, is
150V is the droplet suddenly gains an extra electron. Find;
(i). The initial acceleration of the droplet
(ii).The voltage needed to bring the droplet to rest again.

222 | P a g e
TOPIC 22: POSITIVE AND C.R.O
DEFINITIONS.
1. Define the following terms; positive rays, specific charge of an ion, isotopes, atomic
number ,mass number ,Unified atomic mass unit, Avogadro’s number , faraday’s
constant, time base circuit, fly back time.

FACTORS, MERITS, DEMERITS, EXAMPLES AND FEATURES


1. State four differences between cathode rays and positive rays
2. State the advantages of a CRO over a moving coil voltmeter
3. State two uses of a CRO
4. State four properties of positive rays.

EXPERIMENT
1. Describe a simple form of a mass spectrometer and explain how it is used to distinguish
2. Describe with the aid of diagram, the operation of a Bain bridge spectrometer in
determining the specific charge of ions
3. Describe with the aid of a well labeled diagram, the structure and mode of operation of a
cathode ray oscilloscope (CRO)
4. Describe with the aid of a labeled diagram, the main features of a cathode ray
oscilloscope
5. Draw a labeled diagram showing the essential features of a CRO
6. Describe the Bain bridge mass spectrometer and explain how it can be used to distinguish
between the isotopes.

DERIVATIONS AND RELATIONS

1) The nucleus of 37Cl emits an α - particle followed by two  - particles. Show that the
resultant nuclei are an isotope of chlorine.
2) Use the Avogadro’s constant and Faraday constant to calculate the charge on an ion of a
monatomic element.
3) Describe one use or application of a CRO
4) Briefly explain the use of the time base in a CRO
5) An ion source emits a beam of charged particles of different masses but carrying the
same charge. With the aid of a suitable diagram explain how to select from the beam
particles having the same velocity.

CALCULATIONS
1) The mass of a singly charged Neon isotope 20Ne is 3.3 x 10-26kg. A beam of these ions
enters a uniform transverse magnetic field of 0.3T and describes a circular orbit of radius
0.22m. Determine the;

223 | P a g e
(i). Velocity of the ions
(ii).P.d which has been used to accelerate the ions to this velocity.
2) In an experiment with a Bain bridge mass spectrometer ,tin ions of charge 1.6 x 10-19C
each are produced. They are accelerated by a p.d of 2000V before entering a deflection
chamber two isotopes of tin 118Sn and 120Sn are detected on the photographic plate. A
magnetic field of flux density 0.75T is used in the ion separator. Assuming that 120sn is
at rest before being accelerated, show that the final speed after the accelerated is 1.81 x
105ms-1. Determine also the final speed for 118Sn. (1u = 1.66 x 10-27kg). Determine the
distance of separation between the two points of impact on a photographic plate.
3) The gain control of an oscilloscope is set on 1.0 Vm-1 what is;
(i). The peak value
(ii).Root mean square value of the alternating p.d which produces a vertical line trace
of 2cm long when the time base is off.
4) The gain control of a CRO is set on 0.5Vcm-1 and alternating voltage produces a vertical
trace of 2.0cm long with the time base off. Find the root mean square value of the applied
voltage.
5) A C.R.O has its y sensitivity set to 10Vcm-1. A sinu soidal in put voltage is suitably
applied to give a steady time base switched on so that the electron beam takes 0.01s to
traverse the screen. If the trace seen has a peak to peak height of 4.0cm and contains two
complete cycles, find the;
(i). The root mean square value of the input voltage
(ii).Frequency of the input signal.
6) In an arrangement like that of the cathode ray oscilloscope an electron enters an electric
field of 400Vm-1, directed down ward with initial horizontal velocity of 107ms-1. The
length of the deflecting plates P1 and P2 is 4.0 x 10-2m and the fluorescent screen is 0.25m
away from the nearest edge of the plates;
L = 4.0 x 10-2m P1 Screen

V0 = 107ms-1
P2
0.25m
Determine;
(i). The time spent between the deflecting plates
(ii). The upward component of velocity acquired by the electron
(iii). The displacement of the electron when it hits the screen.
7) A beam of ions passes through a velocity selector in a mass spectrometer that has an
electric field of 1.4 x 105Nc-1.
(i). If the ions emerging from the selector have a velocity of 2.0x105ms-1, what is the
magnetic field in the selector.

224 | P a g e
(ii).The magnetic field in the bending region is 1.0T. What is the radius of the path
followed by a He+ ion with charge e and mass 6.68 x 10-27kg.
8) The gain control of a C.R.O is set on 0.5Vcm-1 and alternating voltage produces a vertical
trace of 2.0cm long with the time base off. Find the root mean square value of the applied
voltage.
9) A beam of electrons is accelerated through a p.d of 2000V and is directed mid – way
between two horizontal plates of length 50cm and a separation of 2.0cm, the p.d across
the plate is 80V.
(i). Calculate the speed of the electrons as they enter the region between the plates
(ii).Explain the motion of the electrons between the plates
(iii). Find the speed of the electrons as they emerge from the region between
the plates.
10) A sinusoidal trace on an oscilloscope screen has wave length of 10cm while the time base
circuit is set of 5Vcm-1. What is the frequency of the applied signal.

TOPIC23: PHOTO ELECTRIC EFFECT


DEFINITIONS.
1. Define the following terms; photoelectric emission, work function, threshold frequency,
threshold wavelength, a photon,an electron volt, stopping potential.

LAWS, PRINCIPLES AND THEOREMS

1. State the laws of photo electric emission


2. State the main characteristics of photo electric emission.
FACTORS, DEMERITS, MERITS, FEATURES AND EXAMPLES.
1. State the conditions under which photo electric emission occurs
2. State the factors affecting photo electric emission
3. Describe one application of a photocell
4. Give any two applications of photo electric effect
5. Mention any three types of photo cells
6. State any 4 uses of photo cells

EXPLANATIONS
1) Explain briefly how the kinetic energy of photo electrons depends on the frequency of the
incident radiation.
2) Explain how the photo electric effect provides evidence for the quantum theory of light.
3) Explain the application of photo electric effect in;
(i). Burglar alarm
(ii). Photo conductive cell

225 | P a g e
4) Explain why light whose frequency is less than threshold frequency cannot cause photo
emission.
5) Explain why the wave theory of light fails to account for photo electric effect.
6) Explain using quantum theory, the experiment observations on photo electric effect.
7) Explain how the photo electric effect provides evidence for the quantum theory of light
8) (i) Write down the Einstein’s photo electric equation
(ii) Explain how the equation above accounts for the emission of electrons from metal
surfaces illuminated by radiation.
9) Explain the failures of the classical wave theory to account for photo electric effect.

EXPERIMENTS

1) Describe a simple experiment to demonstrate photo electric emission


2) Describe an experiment to determine Planck’s constant
3) Describe an experiment to verify Einstein’s equation how Planck’s constant may be
obtained.
4) Describe with the aid of a diagram how the stopping potential of a metal can be
measured.
5) Describe an experiment to verify Einstein equation for photo electric effect and
explain how plank’s constant may be obtained from the experiment.

DERIVATIONS AND RELATIONS

1) Write down Einstein’s equation for the photo electric effect.


2) Sketch using the same axes, graphs showing the dependency of maximum kinetic energy
of photo electrons emitted by two metals having different work functions, on the
frequency of the radiations. How can Planck’s constant be determined from the graph.
3) .(a)

P
1.5V

C
A K
B
M V

P is a vacuum photo cell with anode A and cathode K, made from the same metal of work
function 2.0ev. The cathode is illuminated by monochromatic light of constant intensity
and of wave length 4.4 x 10-7m

226 | P a g e
(i). Describe and explain how the current shown by the micrometer M will vary as the
slider of the potential divider is moved from B to C.
(ii).What will be the reading of the high resistance voltmeter be when photo electric
emission just ceases.

(b) With the slider set midway between B and C describe and explain how the reading of
M would change if;
(i). The intensity of light incident on the photocell was increased
(ii).The wavelength of light incident on the cell was changed to 5.5 x10-7m.
4) A freshly cleaned zinc plate, placed on the cap of a gold lead electro scope with
ultraviolet as shown
U.V radiations
Cleaned
Zinc plate G.L.E

Explain what happens when the G.L.E is;


(i). Negatively charges
(ii).Positively charged
5) Describe the main features of photo electric emission
6) Describe one application of a photocell
7) Describe Einstein’s photo electric equation of quantum theory
8) Describe briefly the classical wave theory of photo electric emission
9) Einstein’s equation for the photo electric emission of electrons from a metal surface
1 2
under radiation of frequency h, can be a written as; hf = me V + φ where the symbols
2
carry usual meaning.
(b) Explain briefly the physical process with which the equation is concerned
10) When electromagnetic radiation falls on a metal surface, electrons may be emitted
(i). State Einstein’s photo electric equation, explain the meaning of each term
(ii).Explain why for a particular metal, electrons are emitted only when the frequency
of the incident radiation is greater than a certain value
(iii). Explain why the maximum speed of the emitted electrons is independent
of the intensity of the incident radiation

227 | P a g e
11) Write down an expression for the energy of a photo explaining the meaning of the
symbols used in your expression and giving the units of the physical quantities involved.

CALCULATIONS.

1) In an experiment of photo electric effect using radiation of wave length 4.0 x 107m the
maximum energy of the ejected electron was observed to be 1.4 x 10-19J.
(i). Calculate the work function
(ii).Calculate the threshold frequency.
2) If the work function of tungsten is 7.2 x10-19J, calculate the maximum energy and speed
of ejected electrons when it’s illuminated by a radiation of 160nm.
3) The minimum frequency of light which will cause photo electric emission from a metal
surface is 5.0x1014Hz. if the surface is illuminated by light of frequency of 6.5 x 1014Hz.
Calculate ;
(i). Work function of the metal surface
(ii).The maximum speed of the electrons
4) The work function of caesium is 1.35eV.
(i). What is the longest wavelength that can cause photo electric emission from a
cesium surface
(ii).What is the maximum velocity with which photo electrons will be emitted from a
caesium surface illuminated with radiations of 400nm
(iii). What p.d will prevent a current from passing through a caesium photocell
illuminated by radiation of wave length 400 nm.
5) In an experiment on the photo electric effect using radiation of wave length 400nm, the
maximum energy is observed to be 1.4 x 10-19J. With the radiation of wave length 300nm,
the maximum energy is 3.0 x 10-19J. Find the value of Planck’s constant.
6) Sodium metal has a work function of 2.3ev. calculate;
(i). Its threshold frequency.
(ii).The maximum velocity of the photo electrons produced when the sodium is
illuminated by light of wave length 5.0x 10-7m
(iii). The stopping potential with light of this wave light.
(Assume h = 6.6 x 10-34Js, C = 3.0 x 108ms-1, 1ev= 1.6 x 10-19J and mass of an
electron).
7) Calculate the stopping potential for a platinum surface irradiated with ultra violet light of
wave length 1.2 x 10-7. The work function of platinum is 6.3ev.
(h = 6.6. x 10-34Js, C = 3.0 x 108ms-1, e = 1.6 x 10-19C).
8) Calcium has a work function of 2.7ev.
(i). What is the work function of calcium expressed in joules
(ii).What is the threshold frequency for calcium
(iii). What is the maximum wave length that will cause emission from calcium.
9) Gold has a work function of 4.9ev.

228 | P a g e
(i). Calculate the maximum kinetic energy in joules of the electrons emitted when
gold is illuminated with ultra violet radiation of frequency 1.7 x 1015Hz.
(ii).What is this energy expresses in ev
(iii). What is the stopping potential for these electrons.
10) Monochromatic radiation of frequency 1.0 x 1015Hz is incident on a clean magnesium
surface for which the work function is 0.59 x 10-18J. calculate;
(i) the maximum kinetic energy of the emitted electrons
(ii) The potential to which the magnetism surface must be raised to prevent
the escape of electrons
(iii) The cut-off wave length.
11) The stopping potential for a copper surface indicated with radiation of wave length 2.5 x
10-7m is 0.25V. Calculate;
(i). The threshold frequency for copper
(ii).The maximum kinetic energy of the electrons emitted by a copper surface.
12) Mono chromatic light of wave length 380nm falls with an intensity of 6.0 x 10-6Wm-2
onto a metallic surface whose work function is 3.2 x 10-19J. calculate;
(i). The number of photo electrons emitted per mm2 of surface if a photon has l in 103
chance of ejecting an electron.
(ii).The maximum kinetic energy of these photo electrons.
13) The work function of tungsten is 4.49ev. Ultraviolet radiation of wave length 250nm falls
on the surface. Calculate;
(i). The threshold wave length for photo emission
(ii).The maximum kinetic energy of the emitted electrons
(iii). The stopping potentials
14) Electromagnetic radiation of frequency 5.0 x 105Hz is indication caesium metal and
photo electrons of maximum kinetic energy of 3.0 x 10-18J are emitted. When the same
radiation is incident on potassium metal photo of kinetic energy of 2.9 x 10-18J are
emitted. Determine the ratio of the work function of caesium and potassium.
15) When light of wave length 5.9x 10-7m is incident on sodium metal electrons of maximum
kinetic energy 1.71x10-20J are emitted. Calculate the maximum kinetic energy of electron
that will be emitted by sodium metal illuminated by light of wave length 4.5 x 10-7m.
16) The work function of a clean metal is 4.5ev. Calculate;
(i). The minimum frequency of radiation that will cause the emission of electrons
from the surface.
(ii).The maximum energy of the electrons emitted when the surface is illuminated
with a radiation of frequency 1.2 x 1015Hz)
17) The work function of potassium metal is 3.56 x 10-19J. If the metal is a radiated with
green light of frequency 6.0 x 1014Hz. Calculate the kinetic energy of emitted photo
electrons.
18) The threshold wave length of a photo emissive surface is 14Hz. calculate;
(i). Threshold frequency
229 | P a g e
(ii).Work function in electron volts
(iii). Minimum speed with which photo electron is emitted is the frequency of
the radiation if 7.5 x 1014Hz.
19) The photo electric work function of potassium is 2.0ev. What p.d will have to be applied
between a potassium surface collecting electrodes in order just to prevent the collection
of electrons when the surface is illuminated with radiation of wave length 350nm.
What could be the kinetic energy and the speed of the most energetic electrons emitted in
this case.
20) If 100mW beam of light of wave length 4.0x10-7m falls on a caesium surface of a
photocell.
(i) What is a photon
(ii) How many photons strike the caesium surface per second
(iii) If 80% of the photons emit photoelectrons from the surface find the resulting
photocurrent.

(iv) Calculate the kinetic energy of each photo electron if the works function of
Caesium is 2.15ev.
21) Violet light of wave length 0.4 μm is incident on a metal surface of threshold wave
Length 0.65 μm. Find the maximum speed of the electron emitted.
22) Ultra –Violet light of a wave length 3.3x10-8m is incident on a metal. Given that the work
function of the metal is 3.5ev, calculate the maximum velocity of the liberated electron.
23) Calculate maximum speed of the photo electrons emitted by a caesium surface radiated
when light of wave length 484nm if the work function of caesium is 3.0 x 10-19J.
24) A metal of work function 2.50ev is irradiated with light of an un known frequency. The
max velocity of the photo electrons is 1.14 x 106ms-1. Calculate the maximum wave
length of the incident radiation.
25) When light of wave length 5.9 x 10-7m, is incident on Na metal, electrons of maximum
kinetic energy 1.7 x 10-20J. Are emitted. Calculate maximum kinetic energy of the
electrons that will be emitted by Na metal illuminated by light of wave length 4.5 x10-7m.
26) When light of wave length 450nm falls on a certain metal electrons of maximum kinetic
energy of 0.76ev are emitted. Find the threshold frequency energy of 0.76ev are emitted.
Find the threshold frequency for the metal.
27) A 100mw beam of light of wave length 4.0 x 10-7m falls on caesium surface of a
photocell.
(i). How many photons strike the caesium surface per second
(ii).If 65% of the photons emit photo electrons, find the resulting photo current
(iii). Calculate the kinetic energy of each photon if the work function of
caesium is 2.20ev.
28) Sodium has a work function of 2.0ev and is illuminated by radiation of wave length
150nm. Calculate the maximum speed of the emitted electrons.
29) Given that sodium has a work function of 3.68 x 10-19J.

230 | P a g e
Calculate;
(i). Its threshold frequency
(ii).The maximum velocity of the electrons emitted by light of wave length 4.0 x 10-
7
m
(iii). The stopping potential with light of this wave length.
30) Electrons emitted from an electron gun are accelerated by a voltage of 500V.calculate;
(i). The energy of the electrons
(ii).The speed of the electrons.
31) An electromagnetic radiation falling on a metal surface has a threshold frequency of 6.0 x
1014Hz.Determine;
(i). Its threshold wave length
(ii).The maximum velocity of the electron emitted under a radiation whose wave
length is 4.0 x 10-7m.
32) The photo electric work function of potassium is 2.0ev.
(i). What is the p.d that will cause the electrode in order to just prevent the collection
of electron when the surface is illuminated with radiation of wave length 350nm.
(ii).What could be the kinetic energy and the speed of the most energetic electrons
emitted in this case.
33) An electron is accelerated from rest through a p.d of 1000V. find;
(i). It kinetic energy in ev
(ii).Its speed.
34) An electron accelerated from rest through a p.d of 50V acquires a speed of 4.2 ×106ms-1.
Which speed is produced by a p.d of 200V.
35) The kinetic energy of α- particle from a radioactive source is 4.0mev. find its speed.
(Charge on electron = 1.6 x 10-19C, mα = 6.4 x 10-27kg)
36) An α- particle accelerated between parallel plates in a vacuum tube acquires a kinetic
energy of 103ev. What is the potential difference between the plates.
37) Light of frequency 6.0 x 1014Hz incident on a metal surface ejects photo electrons having
kinetic energy 2.0 x 10-19J. Calculate the energy needed to remove an electron from the
metal.
38) Light of wave length 0.50nm incident on a metal surface ejects electrons with kinetic
energy up to a maximum of 2.0 x 10-19J. What is the energy required to remove an
electron from the metal.
39) The maximum kinetic energy of photon electrons ejected from a tungsten surface by
monochromatic light of wave length 248nm was found to be 8.6 x 10-20J. find the work
function of tungsten.
40) A source emits mono chromatic light of frequency 5.5 x 1014Hz at a rate of 0.10w of the
photons given out 0.15% fall on the cathode of a photocell which gives a current of
6.0mA in an external circuit. Calculate;
(i). The energy of a photon
(ii). The number of photons leaving the source per second
231 | P a g e
(iii). The percentage of the photons falling on the cathode which produces photo
electrons.
(iv). Calculate the wave length associated with electrons which have been accelerated
from rest through 3000V.

TOPIC24: X-RAYS
DEFINITIONS.
1. Define the following terms; x-rays, quality of x-rays, intensity of x-rays, soft x-rays, hard x-
rays ,a continuous spectrum emission ,a line spectrum emission , x-ray crystallography ,an
energy level, excitation energy ,ionization energy of an tom ,x –ray production ,diffraction of x-
rays.

LAWS, PRINCIPLE AND THEOREM


1. State Bragg’s law of diffraction

Factors, demerits merits, examples and features


1. State any four properties of x-rays
2. Under what conditions does the x-ray diffraction occur.
3. What features of an x-ray make it suitable for continuous production of x-rays
4. What features of an x-ray make it suitable for line spectra production of x-rays
5. State one industrial and one biological use of x-rays
6. Distinguish between continuous and line spectra in an x-ray tube
7. Distinguish between excitation and ionization energies of an atom
8. What are the differences between e-rays and cathode rays
9. Distinguish between x-ray production and photoelectric effect
10. Outline the processes involved in the production of x-rays in a modern x-ray tube
11. How are x-rays differ from beta particles
12. State four properties of electromagnetic radiations
13. Give two ways how x-rays produced can be detected
14. Briefly describe the use of x –rays in medicine, industry and the study of crystal structure
15. Mention any two defectors of x-rays
16. State and explain two applications of x-rays
17. Give any three uses of x-rays

EXPLANATIONS
1) Explain x-ray diffraction by crystals and derive Bragg’s law
2) Explain how intensity and penetrating power of x-rays produced
3) Explain how the intensity and penetrating of x-rays from an x-ray tube would be
affected by changing.

232 | P a g e
(i). The filament current
(ii).The high tension p.d across the tube
4) Explain how line spectra accounts for the existence of discrete energy levels in a
toms
5) Explain the production of the following spectra in an x-ray tube.
(i). Continuous spectra
(ii).Line spectra
6) Explain why the x-ray tube is evacuated
7) Explain the physical process that accounts for
(i). Cut off wave length
(ii).Characteristic lines in an x-ray tube
8) Explain briefly how the intensity and penetrating power of the x-rays produced are
controlled
9) Explain briefly the application of x-rays in the study of crystal structures
10) Explain x-ray diffraction by crystal and derive Bragg’s law.
11) Explain how x-rays are produced in an x-ray tube
12) Explain the emission of x-ray characteristic spectra
13) An x-ray tube produces a spectrum of one or more prominent lines together with a
back ground of continuous radiations having definite minimum wave length. Explain
the occurrence of the;
(i). Prominent lines
(ii).Minimum wave length.
14) Explain the observation of absorption line spectra nature
15) Explain how the radiation from an evacuated x-ray tube is affected by changing;
(i). The filament current
(ii).The filament target p.d
(iii). The target material

EXPERIMENTS
1) (i) Draw a labeled diagram of an x-ray tube
(ii) Use the diagram in (i) above to describe how x-ray are produced
2) Describe with the aid of a diagram the action of an x-ray tube
3) Explain how x-rays are produced in a x-ray tube
4) Draw a labeled diagram showing the main components of an x-ray tube
5) Describe briefly the Bragg’s Diffraction of x-rays by crystals and derive the
Bragg’s law.

DERIVATIONS AND RELATIONS


1) Derive Bragg’s law of x-ray diffraction

233 | P a g e
2) Derive Bragg x –ray diffraction equation
3) When fast moving electrons strike a metal target in an x-ray tube two types of x-ray
spectra are produced
(i). Draw a sketch graph of intensity against wave length of the x-ray
(ii).Account for the occurrence of the two types of spectra.
4) Sketch a graph of intensity versus frequency of radiation produced in an x-ray tube and
explain its features
5) An x-ray tube produces a spectrum of one or more prominent lines together with a black
ground of its radiation having definite minimum wave length. Explain the occurance of
the;
(i). Prominent lines
(ii).Minimum wave length
6) Describe the energy changes which occurs in an x-ray tube in operation
7) Briefly describe the use of x-rays in.
(i). Medicine
(ii).Industry
(iii). The study of crystal structure
8) (i) Draw a labeled diagram of x-ray tube include the electrical circuit
(ii) Explain how x-rays are produced
9) Describe the principles involved in the production of
(i). Continuous x-ray spectrum
(ii).Characteristic x-ray spectrum in an x-ray tube
Use a sketch graph to illustrate your answer.
10) Outline the principles of generation of continuous line spectra of x-rays in the x-ray tube
11) What features of an x-ray tube make it suitable for continuous production of x-rays
12) Sketch a graph of intensity versus wave length of x-rays for an x-ray tube and describe its
main features
13) How do x-rays differ from  - particles
14) Distinguish between x-rays production and the photo electric effect
15) What are the differences between x-rays and cathode rays
16) State briefly how you would control electrically
(i). The intensity
(ii).The penetrating power of the emitted x-rays
17) State one similarity and two differences between optical atomic emission spectra and x-
ray emission spectra produced in this way.

CALCULATIONS ON X-RAY PRODUCTIONS

1) An x-ray tube has an electron beam current of 1.5mA and accelerating voltage of 50KV.
(i). Determine the number of electrons hitting the target per second

234 | P a g e
(ii). If 99% of the incident energy is expressed as heat in the target metal, at what rate
must the target metal be cooled in order to stay at a constant temperature.
(iii). Determine the minimum wave length of the x-rays emitted in the x-ray tube.
(n = 9.38 x 1015s-1, dt = 74.25W, 2min = 2.49 x 1011m)

2) An x-ray tube works at d.c potentials of 50kv. Only 0.4% of energy of the cathode rays is
converted into x-rays and heat is generated in the target at a rate of 600w. determine;
(i). Current passed through the tube
(ii).The velocity of the electrons striking the target
(I = 120 x 10-2A, V = 1.33 x 108ms-1)

3) X-rays are produced by a tube operating at 104v. Calculate their wave length. Assume h =
6.6x10-34Js. E = 1.6 x 10-19c, c = 3.0 x 108ms-1)
(Zmin = 1.24 x 10-10m)

4) X-rays are emitted when a tube operates at 2 x 104v and a current of 0.015A is passing
through it. Calculate;
(i). The number of electrons striking the target per second
(ii).The velocity of the electrons on hitting the targeting
(iii). The power input
(iv). The minimum wavelength of the x-rays emitted’
(9.4 x 10 s , V= 8.4 x 107ms-1, Pin = 300w, Zmin = 6.19 x 1011m)
16 -1

5) In a water cooled x-ray tube a current of 15mA of electrons hits the target which is
maintained at a p.d of 80kv with respect to the filament. Calculate;
(i). The number of electrons striking the anode per second
(ii).The rate of 500C assuming that the kinetic energy of electrons is entirely
converted into heat at the target and that the incoming water is at steady
temperature of 250C, s.h.c of water
dm
(n = 1.125 x 1017s-1, = 1.37 x 10-2kgs-1)
dt

6) When a p.d of 60kv is applied across an x-ray tube, a current of 30MA flows. The anode
is cooled by water flowing at a rate of 0.060kgs-1. If 99% of the power supplied is
converted into heat at the anode. Calculate the rate at which the temperature of the water
rises. (s.h.c of water = 4.2 x 103Jkg-1k-1)

(dθ
dt
=7.07 k s−1 )
7) Calculate the maximum frequency of x-rays emitted by an x-ray tube operating at a
voltage of 34.0k.

235 | P a g e
(fmax = 8.34 x 1015Hz)

8) The p.d between the cathode and anode of an x-ray tube is 5.0x10-4V. If only 0.4% of the
kinetic energy of the electrons is converted into x-ray and the rest is dissipated as heat in
the target at a rate of 600w. Find the.
(i). Current that flows
(ii).Speed of the electrons striking the target
(I = 1.21 x 106A, U = 1.33 x 104ms-1)

9) An x-ray tube is operated at 20kv with an electron current of 16mA in the tube. Estimate
the;
(i). Number of electrons hitting the target per second
(ii).Rate of production of heat, assuming 99.5% of the kinetic energy is converted to
heat.
(n = 1.0 x 1017s-1, 318.4w)

10) An x-ray tube is operated at 50kv and 20mA. If 1% of the total energy supplied is emitted
as x-radiation, calculate;
(i). Maximum frequency of the emitted radiation
(ii).Rate at which heat must be removed from the target in order to keep it at a stead
temperature.
(1.21 x 1019Hz, 990W)

11) An x-ray of wavelength 10-10m is required for study of its diffraction in a crystal. Find
the least accelerating voltage to be applied on an x-ray tube in order to produce these x-
rays
(V = 12375v)

12) (i) In an x-ray tube, 99% of the electrical power supplied to the tube is dissipated as heat.
If the accelerating voltage is 75Kv and power of 742.5w is dissipated as heat. Find the
number of electrons arriving at the target per second.
(6.25 x 1016s-1)
(ii) What would be the effect of increasing the accelerating voltage in (i) above.

13) Electrons of energy 75Kev are stopped by the target of an x-ray tube. Calculate the
minimum wave length of the x-rays produced.
(1.66 x 1011m)

14) An x-ray tube is operated on a p.d of 100kv. Calculate the highest possible frequency of
the protons
(f = 2.41 x 1019Hz)
236 | P a g e
15) Calculate energy of x-rays whose frequency is 3.0 x 1016Hz. (1.98 x 10-17J)
16) Determine the frequency of a type of x-ray whose wave length is 1.0 x 10-10m in vacuum.
(3.0 x 1018Hz)
17) An accelerating potential of 5kv is supplied to an x-ray tube. Determine the velocity with
which the electrons strike the target. (Vmax = 4.193 x 107ms-1)
18) X-rays are emitted when a tube operates at 2 x 104v and a current of 0.015A is passing
through it. Calculate;
(i). The number of electrons striking the target per second
(ii).The velocity of electrons on hitting the target.
(9.37s x 1016 electrons-1, P = 300w)

19) An x-rays tube is operating with an anode potential of 10Kv and a current of 15.0mA.
(i). Calculate the number of electrons hitting the anode per second
(ii).Determine the velocity with which the electrons strike the target.
20) An x-ray tube is operated at 75Kv and 20mA. The target has a mass of 0.3kg and specific
heat capacity 1.46 x 102Jkg-1k-1.
(i). Calculate the shortest wave length of the x-rays emitted by such a tube.
(ii).If only one percent of the electric power supplied is converted into x-rays, and the
rest is dissipated as heat in the target. Calculate the average rate at which the
temperature of the target will rise assuming no thermal losses.
(1.66 x 10-11m, 34.9ks-1)

21) A mono chromatic beam of x-rays is incident on a set of planes in a certain crystal. At
00C first order diffraction is observed at a glancing angle of 30.40. When the temperature
is raised to 4000C, first order diffraction is observed at 300. Calculate the mean coefficient
of linear expansion of the crystal for the temperature range 00C to 4000C
(∝=2.5 x 10−5 K −1 ¿

22) An x-ray tube has an electron beam current of 10mA and the accelerating voltage is
50Kv. Find.
(i). The number of electrons striking the target per second
(ii).The speed of electrons when they hit the target
(iii). The minimum wave length of the x-rays produced.
(6.25 x 1016s-1, 1.33 x 108ms-1, 2.48 x 10-11m)

23) In an x-ray tube operated at 1.5 x 105V, the target is made of a material of specific heat
capacity 2.5 x 102Jkg-1k-1 and has a mass of 0.25kg. one percent of the electric power
supplied is converted into x-rays and the rest is dissipated as heat in the target. If the
temperature of the target rises by 8ks-1, find;

237 | P a g e
(i). The number of electrons which strike the target ever second
(ii).The shortest wave length of x-rays produced.
(2.08 x 1018electrons-1, 8.29 x 10-12m)

24) An x-ray tube with a molybdenum target is operated at 35Kv. The energies needed to
ejected a k and L – electron out of a molybdenum target are 3.22 x 10-15J and 4.2 x 10-16K.
respectively. Determine
(i). The shortest wave length of the continuous x-ray spectrum emitted by tube
(ii).The longest wave length of the k. lines emitted by the molybdenum atom.
(6.18 x 10-11m, 7.10x 10-11m)

25) In an x-ray tube, 90% of the electrical power supplied to the tube is dissipated as heat. If
the accelerating voltage is 75kv and power of 742.5W is dissiplated as heat, find the
number of electrons arriving at the target per second.
(n = 6.25 x 1016s-1)

26) Calculate the wave length of the most energetic x-rays produced by a tube operating at
1.0 x 105V. (h = 6.6x 10-34Js, e = 1.6 x 10-19c, c=3.0 x 108ms-1)
(Zmin = 1.24 x 10-11m)
27) The current in a water cooled x-ray tube operating at 60kv is 30mA. 99% of the energy
supplied to the tube is converted into heat at the target and is removed by water flowing
at a rate of 0.060kgs-1. Calculate;
(i). The rate at which energy is being supplied to the tube
(ii).The increase in temperature of the cooling water (s.h.c of water = 4.2 x 103Jkg-1c-
1
)
(17832Js-1, Dθ = 7.10C)
28) The most energetic x-rays produced of a particular x-ray tube have a wave length of 2.1 x
10-11m. what is the operating p.d of the tube (e = 1.6 x 10-19C, h = 6.64 x 10-34Js, c= 3.0 x
108ms-1)
(59kv, 40mA)

29) An x-ray tube which is 1% efficient produces x-rays energy at a rate of 20Js-1. Calculate
the current in the tube if the operating p.d is 50Kv.
30) Electrons are accelerated from rest through a p.d of 104V in an x-ray tube. Calculate;
(i). The resultant energy of the electrons in ev
(ii).The wave length of the associated electron waves
(iii). The maximum energy and the minimum wave length of the x-radiation generated.
(10,000ev, 1.23 x10-11, 1.60 x 10-15J, 1.24 x 10-10m)

31) An x-ray tube works at a d.c.p.d of 50kv only 0.4% of the energy of the cathode rays is
converted into x-radiation and heat is generated in the target at a rate of 600W. estimate;
238 | P a g e
(i). The current passed through the tube
(ii).The velocity of the electrons striking the target
(12mA, 1.33 x 108ms-1)

32) (a) A 900W x-ray tube operates at a d.c p.d of 30kv. Calculate the minimum wave length
of the x-rays produced.
(b) Calculate the current through the tube
(c) If 99% of the power is dissipated as heat, estimate the number of x-ray photons
produced per second.
(4.1 x 10-11m, 30mA, 1.9 x 1015ms-1)

DIFFRACTION OF X-RAYS

1) A beam of x-rays of frequency 3.5 x 1018Hz is incident on a potassium chloride (KCL)


crystal and the first order Bragg reflection occur at 70410, the density of kcl is
1.98x103kgm-3 and its molecular mass is 74.6. Calculate the value of the Avogradro’s
number using the above data.
(Na = 6.027 x 1023mol-1 d = 3.15 x 10-10m)
2) A beam of x-ray of wave length 1.0 x 10-10m is incident on a set of cubic planes in Nacl
crystal. The first order diffracted beam is obtained for a glancing angel of 10.20. find
(i). The spacing between the consecutive crystal plaens
(ii).The density of Nacl (Relative atomic mass of Na = 23 and cl = 35.5) RMM of
Nacl = 58.5g.
(d= 2.82 x 10-10m, ρ = 2.17 x 103kgm-3)

3) A mono chromatic x-ray beam of wave length 1.0 x 10-10m is incident on a set of planes
in a crystal of spacing 2.8 x 10-10m. what is the order of the diffracted x-rays.
(n = 6)

4) Calculate the atomic spacing of Nacl if the relative atomic mass of Na is 23.0g and that of
chlorine is 35.5g. (Density of the sodium chloride = 2.18 x 103kgm-3
(d = 2.81 x 10-10m)

5) A beam of x-rays of wave length 0.20nm is incident on a crystal at a glancing angel of


300. If the inter planar separation is 0.2nm fin the order of diffraction
(n = 1)

6) X-rays of wave length 1.0 x 10-10m are diffracted from a set of planes of rubidium
chloride. The first diffraction maximum occurs at 8.80. Calculate the inter planar spacing.
7) A second order diffraction image is obtained by reflection of x-rays at atomic planes of
crystal for a glancing angle of 110241. Calculate the atomic spacing of the planes if the

239 | P a g e
wave length of x-ray is 4.0 x 10-11m.
8) A mono chromatic beam of x-rays of wave length 2.0 x 10-10m incident on a set of cubic
planes in a potassium chloride crystal first order diffraction maxima are observed at a
glancing angle of 18.50. Find the density of potassium chloride if its molecular weight is
74.5g. (1.981 x 103 kgm-3)
9) Calculate the glancing angle for the second order diffraction maximum for an x-ray beam
with wave length of 2.0 x 10-11m incident on a crystal with spacing between atomic
planes of the order of 1.0 x 10-10m.
(θ = 11.50)

10) A monochromatic beam of x-rays is incident on a set of parallel atomic planes of sodium
chloride (Nacl) crystal. A first order diffraction maximum is observed at a glancing angle
of 50241. When the same x-radiation is incident on a similar set of atomic planes of a
potassium chloride (Kcl) crystal the second order diffraction maximum is observed at a
glancing angle of 100111. Compare the space between the respective atomic planes for the
two crystal;
d1
(6.45 x 10-34Js, =0.94 ¿
d2
11) The closest spacing between planes of atoms in a crystal of Nacl is 0.282nm. First order
reflection of a monochromatic beam of x-rays occurs at an angle of 150301. How many
order of reflection of these x-rays could be observed from these planes.
(nmax = 3)

12) Calculate the atomic spacing of sodium chloride if the relative atomic mass of sodium is
23.0 and that of chloride is 35.5. (Density of sodium chloride = 2.18 x 103kgm-3)
(d = 2.81 x 10-10m)

13) X-ray of wave length 1.0 x 10-10m are diffracted from a set of planes of rubidium
chloride. The first diffraction maximum occurs at 8.8. calculate the inter planer spacing
(d = 3.27 x 10-10m)

TOPIC25: RADIO ACTIVITY


DEFINITIONS
QN.1.Define the following terms;
1. Radioactivity 6. Positron 12. Isotopes
2. Radioactivity 7. Activity 13. mass number
decay 8. Neutrons 14. Atomic number
3. Alpha particle 9. Decay constant 15. Radioactive
4. Beta particle 10. A Becquerel isotopes
5. Gramma rays 11. Half life 16. Scaler

240 | P a g e
17. A rate meter 23. Nuclear number 28. Binding energy
18. Dead time 24. Electron volt per nucleon
19. Recovery time 25. Mass defect 29. Nuclear fission
20. A quenching 26. Nuclear binding 30. Nuclear fission
agent energy/ Binding 31. Carbon dating
21. Carbon dating energy process
22. Unified atomic 27. Nucleon number
mass unit
32.
FACTORS, MERITS, DEMERITS EXAMPLES AND FEATURES
1. State two precautions that should be taken by personnel working in a radioactive
laboratory
2. Outline the principles of detecting alpha and beta particles
3. State two industrial applications of radio isotopes
4. List two hazards caused radiation
5. In what ways do - rays differ from - particle
6. State the relationship between half life and decay constant
7. State four uses of radioactive isotopes
8. State two industrial uses and two health hazards of radioactivity
9. Write down the mass energy relation
10. Distinguish between nuclear fusion and nuclear fission
11. State the conditions necessary for each of the nuclear reactions in (i) to occur
12. State the similarity between nuclear fusion and nuclear fission
13. Distinguish between nuclear fission and fusion and accounts for the energy released
14. State Einstein’s mass energy relation. what is the physical significance of this relation
15. State four uses of radioactive isotopes.
LAWS, PRINCIPLES AND THEOREM
1. State the exponential law of radioactivity

EXPLANATIONS
1) Explain the meaning of the following terms as applied to a G. M tube
(i). Dead time
(ii).A quenching agent
(iii). Threshold potential difference
2) Explain the application of carbon 14 in carbon dating
3) Explain why neutrons are preferred to charged particles for inducing nuclear reaction
4) Explain how you would use a decay curve for radioactive material to determine its half
life
5) Explain the purpose of each of the following in a G.M tube;
(i). A thin mica window
(ii).Argon gas at low pressure
(iii). Halogen gas mixed with argon gas
(iv). Anode in the form of a wire
6)
Explain why radioactivity is referred to as a spontaneous / random process
7) Explain the term plateau region as used in G.M tube

241 | P a g e
8) Explain why temperature is required during fusion of nuclides
9) Explain briefly how radioisotopes are produced
10) Explain one biological and one industrial application of radio isotopes
11) Explain any changes in mass number and atomic number that may result from the
emission ofα −¿ particles - particles and y- radiation
12) Describe and explain the differences between the tracks formed in a cloud chamber by
alpha and bête particles
13) Explain the meaning of the following terms as applied to a Geiger Muller tube
(i). Threshold potential difference
(ii).Dead time
(iii). A quenching a gent
14) Explain why is radioactive described as random process
EXPLERIMENTS
1. With the aid of a labeled diagram describe the operation of an ionization chamber
2. With the aid of a diagram explain how an ionization chamber works
3. With the aid of a labeled diagram, describe how a cloud chamber can be used to detect
ionization radiation
4. With the aid of a labeled diagram , explain the operation of a cloud diffusion chamber
5. Describe with the aid of a labeled diagram, the structure and action of a difference cloud
chamber
6. (i) With the aid of a labeled diagram describe the working of the Geiger Muller tube
7. How would you use a Geiger Muller tube to determine the half life of a radioactive
sample
8. With the aid of a labeled diagram describe the operation of Wilson’s cloud chamber.
DERIVATIONS AND RELATIVES
1) Sketch the curve of ionization current against applied p. d and explain its main features
2) The following is part of Uranium 238 decay series
238 234 234
U (i)
92 90 T h (ii) 91 Pa
Name the particle emitted at each of the stages (i) and (ii)

3) A source emits two types of radiations simultaneously. The radiation pass through an
absorber of different thickness and are detected
(i). Sketch the graph of intensity of radiation detected against thickness of absorber
(ii).Explain the features of graph in (i)
4) Consider the following nuclear reaction
235
92 U + 10n 133
X A + +610n

(i). Determine the value of x and y


(ii).what is the importance of this reaction
ln 2
5) Show that the half life of a radio isotope is equal to where λ - is the decay constant.
λ
Assume where necessary the radioactive law N=Noe− λt
6) (i) Draw the current voltage characteristic for a Geiger Muller tube

242 | P a g e
(ii) Identity giving reasons, the part of the characteristic over which the tube is
normally operated

T1
7) Show that the half life of a radioactive material is related to the disintegration
2

0.693
constant λ through the expression T 1 /2 = λ

8) Given the radioactive law Nt =No e-2t, obtain the relation between z and half life
9) (i) Sketch the count rate voltage characteristics of a Geiger Muller tube and explain its
main features

(ii) Identify giving reasons, the suitable range in (i) of operation of the tube

10) The nucleus of chlorine 37


17 Cl emits an alpha particle followed two beta particles show
that the

final nucleus is an isotope of chlorine


11) Find the value of x and y in the equation below
X 144 93
92 U + 10n 56 Ba + y Kr + 3 10n + Energy

12) Identify the particle A in the nuclear equation;


235 139 95
92 U + 10n 57 La + 42 Mo + 2A +7 -10 e + Energy

13) Derive the exponential law of radioactivity


(Hint Nt =NO e-2t)

14) (i) The isotope 23892 U emits an - particle and forms an isotope of Thorium (Th) while

the isotope 92U when bombaroled by a neutron forms 144


235
56 U Ba, 90kr and neutrons

(i). Write the nuclear equations for the reactions of 92U and 235
238
92 U
235 238
(ii).How does the reaction of 92 U differ from that of 92U
15) (i) Sketch a graph showing how binding energy per nucleon varies with mass number
and explain its main features
16) Consider the nuclear reaction
2 14
1 H + 7N X + 105B where x is a nuclide

(i). what are Z and A for the nuclide X


(ii).calculate the reaction energy Q
Given mass 21 H =2.01414, 147 N = 14.0031 U
X =6.0151 U, and 105B = 10. 0129U
( Z= 3, A =6, Q = -10.1Mev)
17) (i) Sketch a graph showing how binding energy per nucleon varies with mass number
(ii) Describe the main features of the graph

243 | P a g e
18) Distinguish between nuclear fission and fusion and account for the energy released
19) Sketch a graph of binding energy per nucleon against mass number for naturally
occurring nuclides
20) (i) Sketch a graph showing the variation of binding energy per nucleon with mass
number

(ii) Use the sketch graph, you have drawn in (i) to explain how energy is released
during fission and fusion
21) (i) With the aid of a labeled diagram, describe the operation of a Geiger Muller tube.
(ii) State briefly the steps taken to measure the activity of a source using a Geiger
Muller tube
218
22) The radioactive isotope 84 Po has a half life of 3 min , emitting α - particles according to
the equation
218
84 Po α + xy Pb

(i). what are the value of x, and y


(ii).If N atoms of 21884 Po emit - particles at the rate of 5.12x10 s what will be the
4 -1

1
rate of emission after hours (50s-1)
2
23) An isotope of the element radon has a half life of 4 days. A sample of radon originally
contains 1010 atoms. (1 day = 86x103s) calculate;
(i). the number of radon atoms remaining after 16days
(ii).the rate of decay of the radon sample after 16 days
( 6.3x108, 2.0x10-6s-1 , 1.3x103Bq)
24) The half life of 30 30 15
15 P is 2.5 minutes. calculate the mass of 15 P which has an activity of 10 Bq
(NA =6.0x1023 mol-1)
( 11x10-6 g)
dN
25) The equation for the rate of decay of a radioactive nuclide is - =2N. where N –N0 of
dt
atoms surviving at time
t, z is decay constant
(i). obtain the expression for the value Of N at time, t, given that N =N0 when t =0
T1
(ii).Define the half life, for the decay , and express it in terms of z
2
(iii). Obtain the expression for the number of nuclei that have disintegrated
during the period up to time t
26) The isotope of b is muth mass number 200 has a half life is 5.4x103s. it emits alpha
particles with an energy of 8.2x10-13 J
(i). state the meaning of the term half life
(ii).Determine the decay constant
(iii). Find the initial activity of 1.0x10-6 mole of the isotope
(iv). calculate the initial power output of this quantity of the isotope
(NA = 6.0x1023 mol-1)
(1.3x10-4s-1, 7.7x1013s-1, 63W)

244 | P a g e
HALF LIFE DETERMINATION
1) A mass of 1.0Mg of nuclei 25 11 Na decays by emission of a beta particle. its half life is
60.0s. find;
(i). the number of 25 11 Na atoms present initially
(ii).the initial activity
(iii). The number of 25 11 Na atoms present after 10 minutes.
( N0 = 2.408x10 atoms, Ao =2.78x1014s-1, Nt =2.35x1013 atoms)
16

2) A radioactive isotope 90 38 Sr decay by emission of a beta particle .The half life of the radio
isotope is 28.8 years. Determine the activity of 1g of the isotope
( A =5.10x1012 s-1)
3) Americium ( Am) 241 −6
95 Am is an alpha source of radioactive emission . A mass of 10 g of
this material is estimated to emit a total of 1.0x104 alpha particles per second. Determine
the half life of Americium
T1
( = 1.73X1011S)
2
4) A certain element S has a stable isotope 139 X and a radioactive isotope 138 X of half
life 1.1x104 years whose atoms are 0.1% of those of the stable isotope. Estimate the rate
of decay of 138X with 1kg of 139X
( A = 8152s-1)
5) When 238 206
92 U decays, the end product is 82 Pb . The half life is 1.4x10 s. A rock sample
17

contains 206 238


86 Pb and 92 U in the ratio 1:5 by mass . calculate the;
(i). Number of 206 86 Pb atoms in 1g of the rock sample
(ii).The age of rock ( Assume the radioactive decay law N= Noe-2t)
(4.87x1020 atoms, t= 4.195x1016)
6) An atom of 222Ra emits an alpha particle of energy 5.3Mev. Given that the half life of
222Ra is 3.8 days, use the decay law N= Noe-2t to calculate the;
(i). decay constant
(ii).A mount of energy released by 3.0x10-9 kg of 222 Ra after 3.8 days ( z = 2.11x10-6s-1,
3.45x103J)
7) The silver isotope 108 47 Ag has a half life of 2.4 minutes. Initially a sample contains 2.0x10
6

108
nuclei of 47 Ag . find the number of radioactive nuclei left after 1.2min
(Assume N =N0e-2t)
( N =1.44x106)

8) The radio isotope 60C0 decay by emission of  particle and  - rays its half life is 5.3
years
(i). find the activity of a source containing 0.10g 0f 60C0
(ii).in what ways do - rays differ from - particles
9) A radioactive source contains 1.0mg 0f plutonium of mass number 239 . if the source
emits 2300 -particles per second. calculate the half life of plutonium (Assume the
decay law N= N0e-2t
T1
( = 7.595X1011 seconds)
2

245 | P a g e
10) A radioactive isotope of strontium of mass 5.0mg has half life of 28 years find the mass
of the isotope left after 14 years
(t= 14 years)
11) Suppose the activity of charcoal taken from a fire pit of an ancient home stead is 4.8x10-9
Ci per kg given that half life of 14c is 5760 tears and that the activity of 14c found in
living specimens is 7.0x10-0 ci per kg. calculate the time taken for this to happen
( t= 3136 years)
12) The nuclide 124 55 C s decays with a half life of 30.85. if we have 7.5Ng initially. Find;
(i). How many nucei are present initially
(ii).How many nuclei are present 2.0 min later
(iii). What is the activity 2.0 min later
(iv). What is the activity 2.0min later
(v).After how long will the activity fall to less than one per second.
(2.45 x1015nuclei, 5.51m x 1013Bg, 8.19 x 1014Bq, t = 1526s)
13) The radioisotope 60C0 decays to 60Ni which spontaneously decays to give two y –ray
photons. The half life of 60Co is 5.27years
(i). Find the initial activity of 10g of 60Co
(ii).Estimate the power obtainable from 10g of 60Co
(Mass of 60Co = 59.933814, mass of 60Ni = 59.93079u)
(4.182 x 1014Bq, P= 188W)
14) When 238 206
92 U s decays, the end product is 82 Pb . The hald life is 1.4 x 10 s.
17

Suppose a rock of sample contains206 238


82 Pb . And 92 U s in the ratio 1:5by weight

(i). The number 206 238


82 Pb atoms in a rack sample containg 1.0g of 92 U s

(ii).The age of the rock


(5.84 x 1020, 1.33 x 109year) 4.87 x 1020atom, 4.24 x 1016s)

15) The nucleus 210


84 Po decays to 206Pb by emission of an  - particle. Estimate the kinetic

energy of each  - particle emitted in the decay process


(mass of 210
84 Po = 210.049U, 206Pb = 206.034U and mass of  - particle is 4.0034U)

(11.2Mev)

16) Alpha particles of energy 7.68meV were used to bombard Nitrogen atom. Protons and
oxygen nuclei were produced according to the reaction
14 4 17 1
7 N + 2He 8O+ 1 H

Calculate the total kinetic energy of the proton and the 17O nucleus
(6.26Mev)
17) Radium 226 88 Ro has a half life of 1620years

Determine;
(i). The disintegration constant
(ii).The time it takes for 60% of a given sample to decay
226
(iii). The activity of 1g of 88 Ra given that NA = 6.02 x 1023

246 | P a g e
(1.36 x 10-11s-1, 1709years, 3.5 x1010s-1)
18) The amount of radioactive random -222 was 1g but after three years, 0.9g still remained.
Determine;
(i). The half life of Rodon -222
(ii).The amount removing after 10years
(19.93years, 0.5g)

19) One kilogram of wood from a ship wreck has an activity of 1.2 x 102 counts per second
due to 14C whereas the same amount of wood had an activity of 2.0 x 102 counts per
second. Find the age of the ship wreck.
(Half life of 14C = 5.7 x 103years)
(t = 4.2 x103years)

20) A radioactive isotope of strontium of mass 5.0Ng has a half life of 28years. Find the
amass of the isotope left after 14years (Assume decay law N = Noe-ℵt)
(m = 3.54 x 10-6g)

21) At a certain time, an  particle registers a count rate of 32s-1. Exactly 10days later, the
count rate dropped to 8s-1. Find the decay constant
(0.139 per day)

22) A sample of a radioactive material contains 1018atoms. The half life of the material is
20.0days . calculate;
(i). The fraction remaining after 5.0days
(ii).The activity of the sample after 5.0day

(Nt
No
11
=0.1768 , A=7.092 x 10 Bq
)
23) An isotope of krypton ( 36 Kr ) has a half life of 78minutes. Calculate the activity of 10Ng
87

of 87
36 Kr . (NA = 6.0 x 10 mol )
23 -1

(A = 1.02 x 1013Bq)

24) A sample of radioactive material has an activity of 9.0 x 1012Bq. The remain has a life of
80.0s. How long will it take for the activity to fall to 2.0 x1012 Bq.
(t = 174seconds)

25) The half life of a particular radioactive material is 10minuntes. Determine what fraction

of a sample of material will decay in 30minutes. ()7


8
IONISATION IN RADIATION DETECTORS

247 | P a g e
1) A source of  particles has an initial activity of 2.0 x 105 disintegration s-1. When the  -
particles enter the ionization chamber a saturation current of 2.0 x 10-7A is obtained. If
the energy required producing an ion pair is 32ev, find the energy of one  - particle.
(E = 200Mev)

2) If 32ev is required to produce an ion paid in air, calculate the current produced when a 
- particle per second from the radiation source is stopped inside an ionization chamber.
The energy of  - particles from a radiation source is 4.8Mev.
(I = 2.4 x 10-14A)

3) A radioactive source produces  - particles each of energy 60Mev. If 20% of the  -


particles enter an ionization chamber a current of 0.2NA flows. Find the activity of the -
source if the energy needed to make an ion pair in the chamber is 32Mev.
(3.33 x 1012Bq)

4) A certain -particle truck in a cloud chamber has a length of 40mm. given that the
average energy required to produce an ion pair is 6.0 x 10-16J and that  - particles in air
produces an average of 4.5 x 103such pair per mm of the truck, find the emission energy
Mev.
(6,75Mev)

5) A certain -particle truck in a cloud chamber has a length of 4cm given that the average
energy required to produce an ion paid is 5.8 x 10-18J and that the - particle in air
produces an average of 6.0 x 103pairs per cm of the truck.
(i). Find the initial energy required
(ii).Calculate the saturation current (Is) in the - particles that enter the chamber
tracks 10.5 seconds to such ion pairs.
(iii). Explain why the number of ion pairs is likely to increase towards the end
of the truck of the -particles
(0.87Mev, I = 3.66 x 10-16A)

241
6) A point source of -particles with a tinny mass of nuclei 95 Am is mounted 7.0cm infront
of a G.M tube whose mica window has a receiving area of 3.0cm2.

3cm2

248 | P a g e
G.M tube

7.0cm
A voltmeter connected to GM tube records 5.4 x 104 counts per minute. If the number of
241
95 Am atoms in the sample is 5.8 x 10 , calculate;
15

(i). The number of disintegration per second with in the surface


(ii).The half life of Am – 241
(A = 1.85 x 105s-1, T ½ = 2.18 x 1010s)

7) A Geiger Muller counter placed 2.0cm away from a point source of r – radiation registers
a count rate of 6000s-1. What would the count rate be 1.0m from the source
(Ads = 240s-1)

8) A radioactive sample of uranium 238 is placed 2.54cm away from the G.M tube of
window of effective area 250cm2 and has half life 30 years. The rate meter connected to
the GM tube records 5.6 x 1015 atoms per year. Determine the mass of the sample.
(0.3108g)

9) A radioactive source emittes 2.0 x 105 alpha particles per second. The particles produce a
saturation current of 1.1 x 10-8 A in an ionization chamber. If energy required to produce
an ion pair is 32ev. Determine the energy of Mev of an alpha particle emitted by the
source.
(6.875 x1010, 2.2 x 1012 evs-1, 111MW)

10) At a certain time, an -particle detector registers a contrite of 32s-1. Exactly 10days later,
the count rate dropped to 8s-1. Find the decay constant.

11) A certain  - particle track in a cloud chamber has a length of 37mm. given that the
average energy required to produce an ion pair in air 5.2 x 101-18J and that  - particles in
air produced 5.0 x 103 such pairs per mm of track. Find the initial energy of the  -
particle.

VOLUME OF BLOOD TO OBTAIN ACTIVITY OF A BLOOD SAMPLE


1) A mall volume of a solution containing a radioisotope of sodium had an activity of 16000
disintegration per minute when it was injected into a blood stream of a patient. After
4minues the activity of 1.2cm3 of blood from the patient was found to be 0.4
disintegration per minute. If the half life of sodium is 15minutes.
(i). Calculate the volume of blood in the patient
(ii).State the assumptions taken in the calculations above
(iii). Is blood transfusion necessary? Explain.
(2000 disintegration min-1, V = 600cm3)

249 | P a g e
2) In order to determine the volume of a patient has 6litres of blood containing a radioactive
element whose half life is 20hrs is injected into her blood stream. The patient’s blood
injected with radioactive solution is found to have an activity of 3000 counts per minute
and that of 11.5ml of the blood injected with the radioactive solution is found to be 7.15
counts per minute after 4.5hrs. determine the volume of the patient’s blood
(4128.39ml)

3) In a diagram x-ray examination a 1.2kg portion of a broken leg receives an equivalent


dose of 0.40msv. find;
(i). The equivalent dose in mrem
(ii). The absorbed dose in m Gy
(iii). The number of x-ray photons absorbed if the x-ray photons energy is 50kev
(40mrem, 4.8 x 10-4J, 6.0 x 1010)

4) A radioactive of half life 2.0 hour and activity 2.88 x 107 disintegrations per hour was
injected into the blood stream of a cow. A sample of 2.00cm3 of blood taken from the
cow 16hrs per hour. Find the volume of blood in the cow.
(1.2 x 10-4 cm3)

5) A 6Co source produce an absorbed doze of 400 rad per hour in tissue. The RBE is 0.7
for cobalt y – rays.
(i). How much time is required for an absorbed doze of 300 rad
(ii).How much time is required for a biologically equivalent doze of 300rem.
(45min, 64.3min)
RATES OF WEAR FOR METALS
54 54
1) A steel piston ring contains 15g of radioactive iron 26 Fe , the activity of 26 Fe is 3.7 x 105
disintegration per second. After 100days of its use the crank case oil was found to have a
total activity of 1.23 x 103 disintegration per second. Find the.
(i). Half life of 54
26 Fe

(ii).Average mass of iron worn off the ring per day assuming that all metal removed
from ring accumulates in the oil.
(T ½ = 12.15days, 4.99 x 10-4g per day)

CARBON DATING PROCESS


1) The activity of a dead sample of wood is 10counts per minute while that of a living plant
of the same species is 19counts per minute. If the half life of 14C is 5600 years, find the
age of the wood sample. Assume the decay law.
(t = 5.19 x 103Years)

250 | P a g e
2) 1kg of wood from a ship wreck has an initial activity of 1.2 x 102 counts per second due
to 14C while as the same amount of wood had an activity of 2.0 x 102 counts per second.
Find the age of the ship wreck. (Half life of 14C = 5.7 x 103years)
(t -= 4200years)

3) Living wood has an acidity of 15.3 counts per minute per gram of carbon. A certain
sample of dead wood is found to have an activity of 17.0 counts per minute of 5g.
Calculate the age of the same of dead wood in years. Assume that the half life of 14C is
5568years.
(t = 12082years)

4) A radioactive source has a half life of 20s and an initial activity of 7.0 x 1012Bq.
Calculate its activity after 50s have elapsed. (1.2 x 1012Bq)
5) A sample of radioactive waste has a half life of 80years.how long will it take for its
activity to fall to 20% of its current value.
(186years)

6) Potassium 44 ( 19 K ) has a half life of 20 minutes and decays to form 14


44
20 Ca a stable isotope

of calcium.
(i). How many atoms would there be in a long sample of potassium 44
(ii).What would be the acidity of the sample
(iii). What would the activity be after one hours
(iv). What would the ratio of potassium atoms to calcium atoms be after one
hours (NA = 6.0 x 1023mole-1)
(1.4 x 1020, 7.9 x1016Bq, 9.8 x 1015Bq, 1:7)
7) What mass of radium 227 is 41minutes (NA = 6.02 x 1023mol-1) (1.3 x 10-12g)
8) A radioactive source has a half life of 20days. Calculate the activity of the source after
70days have elapsed if its initial activity is 1010Bq. (8.8 x 108Bq)
9) The activity of a particular radioactive nuclide falls from 1.0 x 1011Bq to 2.0 x 1010Bq in
10hours. Calculate the half life of the nuclide. (4.3hours)
10) Calculate the activity of 2.0Ng of 64 29 Cu . If the half life is 13hours and NA= 6.0 x 10 mol
23 -

1
). (2.8 x 1011Bq)
11) The radioactive isotope of iodine 131I has a half life of 8.0 days and is used as a tracer in
medicine. Calculate.
(i). The no. of atoms of 131I which must be present in the patient when she is tested
to give a disintegration rate of 6.0 x105s-1.
(ii).The no. of atoms of 131I which must have been present in a doze prepared
24hours before.
(6.0 x 1011, 6.5 x 1011)

251 | P a g e
14
12) The activity of a mass of 6C is 5 x 108Bq and the half life is 5570 years. Estimate the no.
of 146C nuclei present. (1.27 x 1020)
13) Wood from a buried ship has a specific activity of 1.2 x 102Bqkg-1 due to 14C where as
comparable living wood has an activity of 2.0 x 102 Bqkg-1. Find the age of the ship if the
half life of 14C is 5.7 x 103years
(4.2 x 103years)

NUCLEAR ENERGY

1) Consider 42He nucleus which has 4 nucleons (2 protons and 2 neutrons). Given that mass
of proton = 1.0073u, mass 42He = 4.0015U, mass of neutrons = 1.0087U. calculate the
4
binding energy of 2He nucleus
(28.3955Mev)

2) Calculate the mass defeat for 56 56


26 Fe given the following information mass of 26 Fe = 55.

93488U. mass of proton = 1.00278U, mass of a neutron = 1.00867U.


(0.3975U)

3) Calculate the binding energy per nucleon of 56


26 Fe use the information mass of the neutron
1 1 0
0 n = 1.00865U, mass of proton 0 p = 1.007277U, mass of electrons −1e =5.4838 x 104U,
mass of 56
26 Fe = 55.9349U, 1U = 931Mev. (8.79Mev)

4) Given mass of 49Be = 9.0150, mass of 136C = 13.0075U, mass of 10n = 1.0087U, and mass
of 11 H = 1.0078U. Determine which of two in Be and C is more stable.
(Be = -6.175633Mev, C = 7.17586Mev)
5) Given the nuclear reaction
210 206 4
84 P 0 82 Pb+ 2 He

Find:
(i). The total energy released in the reaction in joules
(ii).The kinetic energy of the  - particle
(iii). The velocity of the  - particle
(1.34 1 x 10-12J, 1.315 x 10-12J, V = 199 x 107ms-1)

6) Consider the decay process represented by;


226 222 4
88 R 0 86 Rn+ 2 He

252 | P a g e
Calculate the kinetic energy of the alpha particle if the kinetic energy of the Rn – 222 is
neglected.
Given that 226 222 ❑4
88 R 0 = 226.0254U, 86 Rn = 222.0175U, ❑ 2 He = 4.0026

(4.9343Mev)

7) A uranium nucleus 23892 U emitts an alpha particle.

(i). Write down the symbolic equation for the reaction


(ii). Calculate the energy in Mev released in each single disintegrations
(iii). Explain why this energy does not all appear as kinetic energy of the alpha particle.
(iv). Calculate the kinetic energy of the alpha particle assuming the mass below;
238U = 238.125U
234Th = 234.117U
4He = 4.004U
1U = 1.66 x 10-27kg

8) A radioactive isotope of 60Co decays to 60Ni by emission of a beta particle and two
gamma photons. If the half life of cobalt -60 is 5.27years;
(i). Calculate the maximum energy (in Mev) of the radiation is emitted per
disintegration
(ii).Find the power of the radiations emitted by 5g of 60Co, given mass of 60Co =
59.9338U
60Ni = 59.9308U
0
−1e = 0.0005U

(0.0025U, 77.9W)
9) Calculate the energy released why a 10kg of 23592 U under goes fission according to;
235
92 U + 10n 141
56 Ba + 92 1
36 Kr+3 0 n

Mass of 235 141 92 1


92 U = 235.04U, mass of 56 Ba = 140.9U, mass of 36 Kr = 91.91U, mass of 0n

= 1.01U, 1U = 932Mev, NA = 6.02 x 1023 atom


(4.77 x 1027 Mev)

10) Use Einstein’s mass energy relation to calculate the energy in joules that would evolve
from such a reaction when 1 kilo mole of uranium 235 is completely fissioned according
to the equation below.
235 1 139 95 0 1
92 U + 0n 57 La+ 42 Mo + 7 1e + 2 0n

Given mass of 235U = 235.0439U, of 139La = 138.9061U. 95Mo = 94.9058U, 10n =


1.0087U, −10e = 0.00055U
(1.08 x 1015J)

11) A typical nuclear fission event is represented by the equation

253 | P a g e
1 235 92 A 1
0 n+ 92 U 36 Kr + Z Ba + 3 0n
A
(i). Calculate the No. of protons and neutrons in the Z Ba nucleus
(ii).What is the importance of this reaction
(iii). Why are neutrons preferred to charged particles for inducing nuclear
reaction.
(iv). Calculate energy released in one of these events from the following data.
Mass of 10n = 1.67 x 10-2kg, 235 92
92 U = 390.19 x 10-27kg, 36 Kr = 152.57 x10
-27
kg,
A
Ba = 233.92 x 10-27kg.
Z

(v).Estimate the useful power out put of a nuclear powers station which has efficiency
of 33% and uses 235
92 U at a ratio of 4.4 x 10 kgs .
-3 -1

4 2
12) Calculate the energy in Mev liberated when a helium nuclei 2He = 4.004U, 1 H = 2.015 U,
1 U = 1.66 x 10-27kg)
(3.88 x 10-12J or 24.2775Mev)

13) Calculate the energy in Mev released in the fusion reaction


2 2 3 1
1H + 1H 1 H + 1 H + Energy

(Atomic masses 21 H = 2.01402U, 31 H = 3.016049U, 11 H = 1.007825U)


(3.89 x 1012 Mev)

14) Calculate the energy released when a Gallium – 70 ( 31Ga ) under goes a beta decay to
70

produce Germanium 70 ( 32¿ ) atomic mass of ( 31Ga ) = 69.926054U atomic mass of ( 32¿ ) =
70 70 70

69.924254U
(1U = 932 Mev)
(1.6776Mev)

13 13
15) Nitrogen 7 N decays by a positron emission to produce carbon 13 6C according to the
equation
13 13 0
7N 6C + −1e

Calculate (i) the decreased in mass


(ii) The energy released
(1.29 x 10-3U, 1.2 Mev)
16) Calculate the energy released during the decay of 220 216
86 Rn nucleus into 84 Po and an  -

particle
(mass of 216 220 4
84 Po = 219.964176U, 86 Rn = 215.955794U 2 He = 4.001566U, 1U = 931Mev)

17) Calculate the binding energy per nucleon of 56 Fe nucleus.

254 | P a g e
(mass of ion =1.00866 u ,IP =1.007277U ,− ¿ ¿ ¿ 5.4858 X 10−4 U ,IU =931Mev ,56
O

Fe =55.9349 U)
18) Calculate the binding energy per nucleon of an  - particle expressing your answer in
Mev (mass of proton =1.0080U, neutron = 1.0087U, 42He = 4.0026U, 1U = 931Mev)
(7.1687MeV)
19) Consider the nuclear reaction below.
235 1 x 89 1
92 U + 0n 56 Ba + y Kr + 3 0n + Energy

(i). Find the valuces of x 1 and y.


(ii).Cac ulate the energy released by one mole of 235 92 U in the above reaction

20) During the fusion of uranium 235, 200Mev of energy is released. Calculate the energy in
joules released when 1.5kg of uranium takes part a bomb explosion.
21) Alcohol vapour is placed in a cloud chamber to provide carbon nuclei that conserve as a
targets for bombardment by  - particle. What is the k.e of the fragments in the reactions.
12 4 15 1
6 C + 2 He 7 N + 1 H , if the kinetic energy of  - particles 7.68Mev.

(mass of 12 C=12.000U , 4 He=4.0026U , 15 N=15.0001 U , H =1.0078 U)


(2.73Mev)

22) Calculate;
(i). The mass defect
(ii).The binding energy per nucleon for 238
92 U

(A atomic mass of 238 1 1 0


92 U = 238.05076U 0n = 1.00867U, 1 H = 1.00728U, −1e =

0.000554, 1U = 932Mev)
(1.93542U, 7.58Mev)

23) Calculate the energy releases when gallium 70 ( 31Ga ) undergoes  - decay to produce
70

germanium70 70 70
32 ¿. (Atomic mass of 31 Ga = 69.92605U, 32 ¿ 69.924254 , 1U = 932Mev)

(1.68Mev)

24) Radium 224 decays by  - emission to produce radon 220 according to


224 220 4
88 Ra 86 Rn + 2 He

Calculate;
(i). Decrease in mass
(ii).The energy released
(Atomic mass of 224Ra = 224.02022U, 220 4
86 Rn = 220.01140U, 2 He = 4.00260U

(0.00622U, 5.80Mev)

25) Calculate the energy released when 10kg of 235


92 U undergoes fusion according to
235
+ 10n
92
141
56 Ba + 92 1
36 Kr + 3 0n

255 | P a g e
(235 141 92 1
92 U = 235.04U, 56 Ba = 140.91U, 36 Kr = 91.91U, 0n = 1.01U, 1U = 932Mev, NA = 6.02

x 1023MU-1)
(4.77 x 1027Mev)

26) Calculate the energy released when a uranium 236 nucleus under goes fussion according
to;
236 146 87 1
92 U 57 La + 35 Br + 3 0n

(Binding energy per nucleon of 236 146 87


92 U = 7.59Mev of 57 La = 8.41Mev, of 35 Br = 8.59Mev)

(184Mev)

TOPIC26: THE ATOMIC PHYSICS


1) Define the following terms.
1. Abohr atom 6. An energy 10. Band
2. Ground state level. emission
energy. 7. Emission spectra.
3. Excitations spectrum, 11. Absorption
energy. 8. Line spectra.
4. Ionizations emission 12. Electron
energy. spectra. volt.
5. An energy 9. Continuous 13. Ground state
potential. emission energy.
spectra

FACTORS, DEMERITS, MERITS, FEATURES AND EXAMPLES.


1. State Ruther fords’ model of the atom.
2. State Bohr’s postulates of the hydrogen atom.
3. State Rutherford’s model of the atom and discuss the experimented evidence that
supports it.
4. State any two applications of line spectra.
5. Outline the experimental evidence for the existence of a small modes at the centre
of an atom
6. State three failures of Bohr’s model of an atom.

EXPLANATIONS.

256 | P a g e
1. i) Explain the observations made in Rutherford’s ∝ particle-scattering experiment.
i. ii) Why is a vacuum necessary in this experiment?
2. Explain how line spectrum is accounts for the existence of discrete energy levels in an
atom
3. Explain two main failures of Rutherford’s model of the atom.
4. Explain briefly the sources and absorption of infra red radiations.
5. Explain what happens when a beam of ∝ particles is incident on a gold foil.
6. Explain the application of absorption line spectra
7. Explain two main failures of Rutherford’s model of the atom.
8. explain briefly how line spectra are used;
i) In analysis for the identification of elements present.
ii) In astronomy for estimating the component in the line of sight of the velocity
of a star relative to the earth.

DERIVATION AND RELATIONS.


1. Show that when an ∝ particle collides head on with an atom of atomic number ,Z,the
Ze2
closest distance of approach to the nucleus Xo is given by Xo = where e =
πεomv 2
electron charge,εo = permittivity of free space , M= mass of ∝ -particles, V = initial
velocity of the ∝ particle.
−mq 4
2. The total energy E of an electron in an atom may be expressed as E= 2 2 2
8ε n h
i) Identify the quantities,m,q,n, and h
ii) Explain the physical implication of the fact that E is always negative
iii) Draw an energy level diagram for hydrogen to indicate the emission of ultra violet,
visible, and inferred spectra lines.
2) Describe briefly method of detecting infrared radiations
3) The atomic nucleus may be considered to be a sphere of positive charges with a diameter
very much less than that of an atom. Discuss the experimental evidence which supports
this view.
4) In a simple model of the hydrogen atom, an electron of mass M and charge E is
considered to move in a nearly circular orbit about a proton;
i) Write down the expression for the electric force on the electron and show that the
e2
kinetic energy of the electron is where r is the radius of the orbit,εo permittivity
8 ε o πr
of free space
ii) find the total energy of the electrons
nh
iii) Given that the angular momentum of the electron is equal to where n is an integer

( )
4
me 1
and h is Planks’ constant. Show that the total energy of the electron is En = 2 2 2
8 εo h n
5) List the principle results of Rutherford’s experiments on the scattering of alpha particles
by thin gold foils
6) The diagram below shows possible electron orbits in the Bohr model for
hydrogen .Assuming the orbits are circular,

257 | P a g e
n =5

n=4

n=3

n=2

n= 1

i) Show that the total energy of an electron in an orbit of radius r is given by E


2
e
= where E is the electron charge
8 πϵo
nh
ii) if only the orbits allowed are those for which ,rmv = where m is electron

mass, v the electron speed and n an integer and h Plank’s constant. show that
4
−me
the total energy in (i) above can be expressed as Em = 2 2 2
8 ϵo h n
iii) calculate the wave length of the radiation that will be emitted when the
electron makes a transition for n = 4 to n = 3 orbits
(1.89 x 10−6 m
7) What is meant by emission transmission spectra?
8) What is meant by a line spectrum
9) The energy level of the hydrogen is given by
21.7 x 10−9
En = joules where n = 1, 2, 3….
h2
i) Use this result to account for the occurrence of emission and absorption line
spectra
ii) Find the shortest wave length of radiation which can be emitted by the
hydrogen atom. (0.912 x10−7 m)
10) Experiments show that if a fine beam of alpha particles is directed normally on to a thin
gold film;
i) Most of the alpha particles go straight through the film with an diminished
speed.
ii) A small proportion of the alpha particle is strongly deflected.
Explain the two observations.
11) Sketch on the same diagram the patterns of three alpha particles of the same energy
which are directed towards a nucleus so that they are deflected through ;
i) About 100

258 | P a g e
0
ii) 90
0
iii) 180 Respectively.
12) For the deflection of 1800 , describe in qualitative terms how;
i) the kinetic energy
ii) The potential energy of aloha particles varies during its pattern assuming the
nucleus remains stationary.
13) What are the chief characteristics of a line spectrum?
14) Alpha particles in a narrow parallel beam are scattered by a thin metallic foil.
a) State the result of such an experiment and explain why they are evidence for the
nuclear atom.
b) Explain why;
i) Such an experiment is carried out in a vacuum.
ii) The incident alpha particular are confined in a narrow parallel beam.
iii) The foil is thin.
15) i) Describe briefly the Bohr model for the hydrogen atom.
i) State the important assumption that made
16) i) Explain what is meant by electron energy levels in an atom
ii) How does this concept account for the characteristic emission line spectrum of an
element
iii) Describe a simple demonstration of the lines spectrum emission.

18) The table below shows some of the levels for the H 2 atom

Energy level
a -0
b -0.54
c -0.83
d -1.51
e -3.39
f -13.5
i) Define the electron volt
ii) Explain why the energy levels are given negative values.
iii) How might the atoms be changed from state e to c.
iv) State which level corresponds to the ground state
v) Calculate the conisation energy of atomic H 2

Calculations on distance of closest approach.

259 | P a g e
1) In a head on collusion between the ∝ -particle and a gold nucleus ,the minimum distance
of approach is 5.0 x 104 m .Calculate the energy of ∝ particle in Mev ( atomic number of
gold is 79 )
2) A beam of ∝-particles of energy is 4.2. Mev is incident normal to gold foil. What is the
closest distance of approach by the ∝ - particle to the nucleus of the gold atom ( atomic
number of gold is 79 ) (bo =5.4 x10−4m )
3) Consider the electron of Bohr model of H 2 atom , in which it has the orbital radius of 53
pm .Calculate ;
i) kinetic energy ,electrostatic energy ,and hence total energy
ii) The minimum energy that must be given to it to separate it from the proton.
iii) the minimum p.d through which another electron would have to be accelerated so that
they are in elastic collusion and cause ionizations (2.7 x 10−18 j , -2.18 x 10−18 j, 13.6
v)
4) Alpha particles each of kinetic energy of 1.0 x 10−12 J are incident at right angle on a gold
foil. Calculate the distance of closest approach of an alpha particle to the nucleus of a
gold atom if the atomic number of gold is = 79 (d = 3.64 x 10−14 m )
5) An alpha particle with kinetic energy of 5 μ ev is involved in a head on collusion with a
gold foil. it is deflected through 1800 . if the atomic number of gold is 79, calculate the
distance of closest approach between the two. what is the significance of the result ( l ev
=1.6 x10−19 c, εo = 8.85 x 10−12 fm−1 ) ( d=4.55 nx 10−6 m )
6) An alpha particle with velocity 3.5 x 106 m s−1 strikes a block of load atomic number 82.
Calculate the distance of closest possible approach between the alpha particles and any
lead nucleus .( if mass of the alpha particle is = 4 x 1.67 x 10−27 kg ) (d = 3.606 x 10−6 m)
7) An alpha has an initial kinetic energy of 1.60 x 10−13 j and the nucleus has a charge of
+50 e. Calculate the nearest distance of approach of the alpha particle to the nucleus (e =
1.60x10−19c) (bo = 1.44 x 10−13 m

Calculations on energy levels and transitions.


1) The energy levels in a mercury atom are -10.4 eV,-5.5 eV, -3.7 eV and -1.6 eV.
i) Find the ionization energy of mercury in joules. (1.664 x 10−18 J)
ii) What is likely to happen if mercury atom in an unexcited state is bombarded with an
electron of energy 4.0ev, 6.7 eV or 11.0 eV?
2) The figure below shows some of the energy levels of a neon
ϵ ∝ ---------------------------------------------------------------------------------------- -0ev
E 4------------------------------------------------------------------------------------------ -0.81 eV
E 3-------------------------------------------------------------------------------------------- -2.77 eV
E 2 ----------------------------------------------------------------------------------------- -4.87 eV
E 1 ----------------------------------------------------------------------------------------- -21.47ev
i) What is meant by emission line spectra?
ii) Determine the wave length of the radiation emitted in an electron transition from E4 to
E3. In what region of the electro magnetic spectrum does the radiation lie. (2= 6.31 x
−26
10 m)

260 | P a g e
−19
−21.7 x 10
3) The energy levels of the hydrogen atom are given by En = 2 where n =
n
1,2 ,3…
ii) Use this result to account for the occurrence of emission and absorption spectra.
ii) Find the shortest wave length of the radiation which can be emitted by the hydrogen
atom
4) Calculate the ionization potential of hydrogen atom a ground state of -2.18 x 10−18 j.
(136.6 v)
5) A hydrogen atom is in an excited state of energy -10.6 eV .it absorbs a proton of wave
length 1.2 x 10−7 m and it is excited to a higher energy level. What it falls back to its
ground state of a proton of wave length 0.9 x 10−7 m is emitted. Find the energy of the
ground state ( -14.0375 ev)
6) The first ionization energy of a hydrogen atom is 10.2 eV calculate the lowest electron
that can excite a hydrogen atom. (1.86 x10−6 m5−1 )
7) The lowest energy level of an atom is -0.4 ev
i) Find the ionization energy of an atom in joules.
ii) Calculate the wave length of the energy absorbed by the electron when it moves from
the lowest energy level of -1.6 eV
8) Calculate the frequency and wave length resulting from electron transition from;
i) n = 4 to n = 2
ii)E2 to E 1 for a hydrogen atom.
State in each case the region of electro magnetic spectrum where the radiation lies. (h =
6.6 x 10−34 js, c = 3.0 x 10 8 m s−1 )
Given that n 4 = -0.85 eV,n2 = -3.39ev, E2 = -3.39ev E1= -13.6 EV
(4.87 X10−7 , 1.21 x 10−7 m )
9) The figure below shows some of the energy level of mercury atoms
levels
E ∝−−−−−−−−−−−−−−−−−−−−−−−−−−−−−−¿ 0.00
E 4 -----------------------------------------------------------------------------------------1.60
E3 ------------------------------------------------------------------------------------------3.79
E2 ------------------------------------------------------------------------------------------5.50
E1 -------------------------------------------------------------------------------------------10.40
a) i) calculate the ionization energy of mercury atom in joules
ii) find the wave length of the radiation emitted when the electron moves from level 4
to level 2 and state and state which level of the electron magnetic spectrum does the
radiation lie ( 1.66 x 10−8 j , 3.17 x 10−7 m )
b) if a mercury vapor atom in a ground state collides with an electron of energy 10 ev ,how
much energy might be retained by the electron (5.1 ev)

261 | P a g e
10) The diagram below depicts electron orbit in the Bohr model for hydrogen. Assume orbits
4
−me
are and that the total energy of the atom is En = 2 2 2
8 εo h n

n=4

n=3

n=2

n=1

Calculate the wave length of radiation that will be emitted when the electron makes a
transition from n =4 to n = 3 orbit
( 2 = 1.89 x 10−6 m )

11) i)The frequency of a spectra lines emitted by hydrogen can be represented by f = a

( 1
2
n1 n2
1
)
− 2 where a is a constant and n1 =2, n2 =3

ii) State in which part of the spectrum these quanta are found.
(1.89 eV)
12) The three lowest energy level of the electron in the hydrogen atom have energies E1 = -
21.8 X 10−19 J , E2 =-5.45 X 10−19 J, E3 = -2.4 X 10−19 J .The energies are measured so
that electrons would have zero if it were completely free from the atom and at rest.
Calculate;
i) The wave length of radiation emitted for electronic transmission from E3 to E4.
ii)the p.d through which the electron must be accelerated in order for it to ionize a normal
hydrogen atom

13) The diagram below shows some energy level of H2 atom


E (ev)
-0.55________________________________________________________n=5
-0.85________________________________________________________n = 4
-1.5_________________________________________________________n=3
262 | P a g e
-3.4__________________________________________________________n=2
-13.6_________________________________________________________n=1

i) Use the diagram to explain the emission spectrum of H2


ii) Calculate the speed of an electron which could just ionize the H2atom.
iii) Calculate the minimum wave length of the H2 spectrum and state the region of the
electro magnetic spectrum in which it lies. (2.19 x 106 m s−1 , 0.913 x 1 0−7 m)
14) The figure below represents the lowest energy level for mercury.
n = 6 …………………………………………………………………………… -2.71 eV
n = 5……………………………………………………………………………. -3.74 eV
n = 4 ………………………………………………………………………….. -4.98 eV
n = 3 …………………………………………………………………………… -5.55 eV
n = 2 ……………………………………………………………………….…… -5.77 eV
n = 1 ………………………………………………………………………….. -10.44 eV
i) Calculate the energy and wave length of the photon emitted when a mercury atomic
energy changes from E6 to E2.
Ii) Determine which two energy levels in the mercury atom are involved in the emission
of a line whose wave length is 546 nm. (3.06ev, 2 =4.06 x 10−7 m , ¿ n=6 ¿ n=2
15) The figure below shows an energy level of an imaginary element .the atoms can pass
from any level to any other by absorbing or emitting photon.
0 eV ………………………………………………………………………E4
-0.3 eV……………………………………………………………………E3
-1.0 eV……………………………………………………………………E2
-3.0 eV ………………………………………………………………….. E1
i) Calculate the wave length of all the photons emitted by the sample of the atoms in a gas
discharge.
Which of the lines would be in the visible part of the electro magnetic spectrum? (4.14
x10−7 m , 4.60 x 10−7 m visibe spectrum. )
16) Considering the energy of an electron at rest outside an atom to be zero the energies at
the k,l,and m,level of a certain atom are -2.0 x 1104 ev ,−2.0 x 1 03ev and -2.0 x1102 ev . if
this atom is bombard with 40k ev electron , find ;
i) The shortest wave length of the x-rays emitted.
ii) The wave length of the lines in the, K and L series of the radiation characteristics of
the substance. What would be the effect on the characteristic radiation of reducing the
energy of the bombarding electrons to 10 keV? (6.91 x10−11 m ,6.91 x 10−10 m ,19.8 kev ¿
17) Calculate the frequency and wave length resulting from electron transition from E =-0.85
Ev to E = -3.39 ev and state where it lies (f = 6.16 x1014 HZ , 2=4.87 X 10−7 mvisible )
18) Calculate the frequency and wave length resulting from electron transition from E = -3.39
EV to E = -13.8 ev .State the region of electro magnetic spectrum where the radiation lies
. (h =
6.64 x 10−34 JS ,C=3.0 X 1 08 ms−1 ( f =−2.48 x 1 015 HZ , 2=1.21 X 1 0−7 m , U−V region)
19) The three lowest energy levels of the electron in the hydrogen atom have energies E1 =
−9 −19 −9
−2.8 X 10 J , E 2=−5.45 X 10 J , E 3=−2.43 X 1 0 J . The energies are measured so
that the electron would have zero energy if it where completely free from the atom and at
rest. Calculate ;
i) The wave length of radiation emitted for electronic transmission from E3 to E2

263 | P a g e
ii) The p.d through which the electron must be accelerated in order for it to ionize a
normal hydrogen atom
20) The diagram shows the three lowest energy levels of an atom of the target materials of
an x- ray tube as shown
n =3 …………………………………………………………………………11 x 103 ev
n =2 ………………………………………………………………………… -26 x1103 ev
n = 1 ………………………………………………………………………… -98 x 10 3 ev
What is the minimum p.d at which the tube can be operated?
i) If the transition n =3 to n =1 is to be possible.
ii)if n =2 to n =1 is possible
iii) What is the wave length corresponding to the transition n =3 to n = 1 .
( e=1.6 x 10−19 c , h=6.6 x 1 0−34 JS ,C=3.0 X 1 08 ms−1 )
(( 87 kv , 72 kv ,1.4 10 11 m )
21) The ionization potential of hydrogen atom is 13.6 eV. calculate
i)The speed of an electron which just ionize the hydrogen atom.
ii) The minimum wave length which the hydrogen atom can emit.
( 2.19 x 1 06 , 9.14 x 10−8 m)
22) The diagram below shows the lowest energy level of the electron in the hydrogen atom
Energy
N = 6_______________________________________-0.60 x10-19J
5__________________________________________-0.87 x 10-19J
4__________________________________________ -1.36 x 10-19J
3____________________C____D__________________-2.42 x 10-19J
A B
2___________________________________________ -5.43 x 10-19J

1_____________Ground state______________________ -21.7 x10-19J

Calculate the wave length of the lines arising from the transition marked A, B, C, D, on
the figure.
( 661 nm , 489 nm .436 nm , 412 nm )
23) The ionization energy for a H2 atom is 13.6 ev .if the atom is in the first excited state
i) Explain the term ionization energy and excited state
ii) Calculate the wave length of the photon emitted when a H2 atom is returned to the
ground state from the first executed.
iii) Name the part of electro magnetic spectrum to which the wave length belongs.
( 1.22 x 1 0−7 m )
−18
−2.16 x 1 0
24) The energy levels of H2 atom are given by expression En = J where n = is
n2
an integer.
i) What is the ionization energy of the atom?
ii) What is the wave length of the H2 line which arises from the transitions between n =
3, and n= 2 levels.
( 2.16 x 1 0−18 J , 6.6 X 10−7 m)

264 | P a g e
25) The lowest energy level in a helium atom (ground state) is -24.6 eV there are a number of
other energy levels one of which is at -21.4 eV.
i) Define an electron volt.
ii) Explain the significance of the negative signs in the values quoted
iii) What is the energy of a photon emitted when an electron returns to the ground state
from the energy level at -21.4 eV?
iv) Calculate the wave length of the radiation emitted in this transition.
( 5.1 x 10−19 J 3.9 X 10−7 m)
26) Some of the energy levels of the H2 atoms are shown in the diagram below.
Energy (eV)
0.00_____________________________________
-0.54____________________________________
-0.85____________________________________
-1.51____________________________________
-3.39_____________________________________
-13.58_____________________________________ Ground state

i) Why are the energy levels labeled with negative integers?


ii) State which Transition will result in the emission of radiation of wave length 487 nm.
iii) What is likely to happen to a beam of photons of energy?
a) 12.07 eV
b) 5.25 eV, when passed through a vapor of atomic hydrogen
( lev=1.6 x 10−19 J )

TOPIC 7: ELECTRONICS.
DEFINITIONS
Qn 1. Define the following terms,
1. Thermionic 4. Work function 9. Space charge.
emission. 5. Rectification. 10. Rectification.
2. Field emission. 6. A diode 11. Saturation.
3. Secondary 7. A triode 12. Space charge
emission. 8. Transistor. limitation
13. .

MERITS, DEMERITS, EXAMPLES, EXAMPLE AND FEATURES


1) State two advantages of transistor as an amplifier over triode.
2) State 4 ways by which electrons are emitted from the surface of a metal.
265 | P a g e
3) State 2 advantages of a semi conductor device over thermionic valve devices.
4) State 2 advantages of a triode as an amplifier over a transistor.
5) State two uses of diodes.
6) Describe one application of a diode.

EXPERIMENTS.
1. Describe the mechanism of thermionic emission.
2. Describe briefly the mechanism of thermionic emission.

EXPLANATION
1) Explain briefly the mechanism of thermionic emission.

2) i) Explain with the aid of suitable diagrams the time variations of the voltage across the
resistor in the circuit below.
D1

a.c R

D2
ii) With reference to (ii) above, explain the use of a filter circuit
3) Explain the following terms as applied to a vacuum diode
i) Space charge limitations
ii) Saturation
4) Explain the following as applied in thermionic diode
i) Rectification
ii) Space charge.
5) Explain how you would produce;
(i). half wave
(ii). full wave rectification of an alternating current using diode valves
6) Explain with the aid of circuit diagram, how full wave rectification of a sinus alternating
voltage can be achieved using four diodes.
7) Explain the significance of the main features of the characteristics in relation to the use of
the diode as a rectifier.
8) Explain briefly the mechanism of thermionic emission.

DERIVATIONS AND RELATIONS.


1) Sketch the current potential difference characteristics of a thermionic diode for two
different operating temperatures and explain their main features
2) Draw a labeled diagram of the circuit used to determine the anode current and anode
voltage characteristics of a thermionic diode.

266 | P a g e
3) Sketch the characteristics expected in 2) above at constant filament current and
account for its special features.
4) Sketch the current potential difference characteristics of thermionic diode for two
different operating temperatures and explain their main features.
5) Describe the application of a diode.
6) Draw the circuit diagram you would use to obtain the anode current (Ia) anode
voltage (Va) characteristics of a thermionic diode
7) Sketch and explain the main features of a typical Ia –Va characteristics of a vacuum
diode.

8) The figure represents a single stage diode amplifier circuit.

Cg

C3

Rg

RK H.T

(i). State the functions of the capacitor Cg and Ck.


(ii). What are the functions of resistors to Rg and Ck
(iii). Draw the equivalent circuit of the triode as an amplifier and deduce an
expression for the gain

9) a) Draw a labeled circuit diagram to show a triode being used as an amplifier.


10) Derive an expression for the application factory μ in terms of a node resistance Ra and
mutual conductance gm for a triode valve.
11) Draw an equivalent circuit of a triode as single stage and derive an expression for the
gain.
12) Sketch the I – V characteristics curve for a vacuum diode and explain its features.
13) With the aid of a labeled diagram, explain how a triode can be used as a voltage
amplifier.
14) i) Draw a labeled diagram of the circuit used to determine the node current and a node
voltage characteristic of a thermionic diode.
ii) Sketch the characteristics expected in (i) at a constant filament and account
for its special features.
15) Sketch the current potential difference characteristics of a thermionic diode for two
different operating temperatures and explain their main features.
16) i)Draw a labeled diagram of the circuit used to determine the anode current and anode
voltage characteristics of a thermionic diode .
267 | P a g e
ii) Sketch the characteristics expected in i) at a constant filament current and account
for its special features.
17) (i)Draw a labeled circuit to show a triode being used a single stage voltage amplifier.
(ii)With the aid of an equivalent circuit of a triode as an amplifier, obtain an
expression for the voltage gained.
18) (i)Describe the structure of a junction transistor
(ii)Sketch and describe the collector –current against the collector emitter voltage
characteristics of a junction transistor
19) Sketch the current potential characteristics of a thermionic diode for two different
operating temperatures and explain their main features.
20) (i)Draw the circuit diagram you would use to obtain the current voltage
characteristics of a thermion diode.
(ii)Explain the significance of the main features of the characteristics in relation to the
use of the diodes a rectifier.
21) Draw the equivalent circuit of a single stage triode amplifier hence an expression for
the application gained.
22) (i)Draw a labeled diagram to show a triode being used as an amplifier.
(ii)Derive an expression for application factor, μ in terms of anode resistance Ra, and
mutual conductance gm for a triode valve
23) (i)Draw a labeled circuit to show a triode being used as a single stage voltage
amplifier.
(ii)With the aid of an equivalent circuit of the triode, as an amplifier obtain an
expression for the voltage gain.
24) Draw an equivalent circuit of a triode as a single stage amplifier.

CALCULATIONS.
1) (i)A triode with mutual conductance 4.0 MAV −1 and anode resistance of SKCY is
connected to a load resistance of 10KSV. Assuming that the triode is operating under
optimum conditions, estimate the output signal obtained for an alternating input of
25Mv.
(ii)Show how alternating p: d is separated from the direct p:d in the output of a triode
when used as an amplifier (Vo =0.33v)
2) A sinusoidal voltage of amplitude 0.2 v is applied to the grid of a triode of
amplification factor 10. If the anode resistance of the triode is 15Ω, what voltage will
appear across a load of 10kΩ? (1.33v)
3) A triode with mutual conductance of 3.0 x103 AV −1 and a node resistance of 1.0 x 104
Ω. Calculate voltage gain of the amplifier (22.5 )
4) (i)A triode valve with anode resistance of 3.0 x 103Ω is used as an amplifier.
sinusoidal alternating signal of amplitude 0.5 v is applied to the grid of the

268 | P a g e
valve .Find the root mean square value of the output voltage if the amplification
factor is 15 and anode load is 50k Ω
(ii)Draw an equivalent circuit of a triode as a single stage amplifier. (Vrms = 5.003v)
5) A triode valve passes an anode current of 5 Ma at anode voltage of 150v, and grid
voltage of 2v. If the grid voltage is changed to -3.5 v, the anode current decreases to
3.2mA but can restored to 5mA by increasing the anode voltage to 195 v.
Find (i) the mutual conductance.
(ii) The anode resistance.
(iii) The amplification factor.
(1.2 x10−3 Ω−1 , 2.5 x104 Ω , μ = 30 )
6) The current, I, through h a diode is given in the following table for a certain anode
voltage Va.
Va(V 0 80 150
)
I(Ma) 0 2 5
When the diode is connected in series with anode resistant R to a 120 V d.c supply,
current of 2mA flows.
Find (i) the value of R
(ii) the supply voltage necessary to increase the current to 5mA. ( R =20 KΩ ,V
=250 V )
7) A triode valve with an anode resistance of 25 kΩ, is used in a single stage amplifier
with an anode load of 50 k Ω .A sinusoidal alternating signal of amplitude 0.5 k Ω is
applied to the grid of the valve. Find the mean root square value of the out- put
voltage if the amplification factor is 5 ( Vrm =3.5 v )
8) A single stage triode amplifier has an anode load of 20 k Ω .Calculate the amplifier
gain if the mutual conductance and anode resistance of the triode are respectively 2
mΩ and 10 kΩ respectively. (13.3 )

269 | P a g e

You might also like